Comprehensive Peds/OB Questions

Ace your homework & exams now with Quizwiz!

The nurse should encourage the laboring client to begin pushing when

the cervix is completely dilated.

The nurse is teaching breastfeeding to prospective parents in a childbirth education class. Which instruction should the nurse include as content in the class?

Feed your baby every 2 to 3 hours or on demand, whichever comes first.

Which assessment finding should the nursery nurse report to the pediatric healthcare provider? A. blood glucose level of 45mg/dl B. blood pressure of 82/45 mmHG C. Non bulging anterior fontanel D. central cyanosis when crying

D. central cyanosis when crying

What position should the client be placed in for an epidural?

Lateral left with legs flexed and back curved.

In which trimester is UTI most common?

Third trimester

Medical managment of ductus arteriosus

Administration of Indomethacin (to close the ductus arteriosus), surgical correction (surgical opening into the thoracic cavity)

Medical management of Sequestration crisis

Analgesics, volume expanders, transfusions; recurrent episodes treated by splenectomy

What happens to the ductus arteriosus during the first breath?

The ductus arteriosus closes with the increased oxygen content of the newborns blood

Which nursing intervention is helpful in relieving "afterpains" (postpartum uterine contractions)?

Using relaxation breathing techniques.

Palliative procedure for Ventricular Septal Defect (VSD)

*Pulmonary Artery Banding;* can be performed in infants with symptomatic VSDs. The procedure involves impeding the flow of blood from the right ventricle to the pulmonary circulation, thus reducing pulmonary congestion

The nurse is performing an assessment on a 39-week neonate an hour after a spontaneous vaginal delivery. What are common expected newborn findings? Select all that apply.

- Plantar creases up the entire sole - Toes fan outward when the lateral sole surface is stroked - White pearl-like cysts on gum margins

Recommended daily allowance of iron for infants between the ages 7 to 12 months?

11mg/day

client receiving epidural anesthesia begins to experience nausea and becomes pale and clammy. What intervention should the nurse implement first? A. Raise the foot of the bed. B.Assess for vaginal bleeding. C.Evaluate the fetal heart rate. D.Take the client's blood pressure.

A. Raise the foot of the bed.

Client teaching is an important part of the maternity nurse's role. Which factor has the greatest influence on successful teaching of the gravid client? A. The client's readiness to learn. B. The client's educational background. C. The order in which the information is presented. .DThe extent to which the pregnancy was planned.

A. The client's readiness to learn.

Which population is Sickle Cell Disease most common in?

African American population

What is absence of menses by age 16?

Amenorrhea

Classifications of anemia?

Anemia is classified as either hypoproliferative (defective production of erythrocytes) or hemolytic (premature destruction of erythrocytes)

A woman who had a cesarean delivery 5 hours ago now appears anxious and reports shortness of breath. The practical nurse should assess for which priority problem before notifying the registered nurse?

Calf warmth and redness

A pregnant client with mitral stenosis Class III is prescribed complete bedrest. The client asks the nurse, "Why must I stay in bed all the time?" Which response is best for the nurse to provide this client?

Complete bedrest decreases oxygen needs and demands on the heart muscle tissue.

The nurse is monitoring a client who is in active labor with a cervical dilation of 6 cm. Which uterine assessment finding requires an intervention by the nurse?

Contraction duration of 95 seconds

What action should the nurse implement to decrease the client's risk for hemorrhage after c-section. A. Monitor urinary output via an indwelling catheter. B. assess the abdominal dressings for drainage. C. Give the Ringer's lactated infusion at 125ml D. Check the firmness of the uterus every 15mins.

D. Check the firmness of the uterus every 15mins.

In developing a teaching plan for expectant parents the nurse plans to include formation about when the parents can expect the infants fontanels to close. The nurse bases the explanation on knowledge that for the normal newborn, the A. anterior fontanel closes at 2 to 4 months and the posterior by the end of the first week. B. anterior fontanel closes at 5 to 7 months and the posterior by the end of the second week. C. anterior fontanel closes at 8 to 11 months and the posterior by the end of the first month. D. anterior fontanel closes at 12 to 18 months and the posterior by the end of the second month

D. anterior fontanel closes at 12 to 18 months and the posterior by the end of the second month

A 35-year-old primigravida client with severe preeclampsia is receiving magnesium sulfate via continuous IV infusion. Which assessment data indicates to the nurse that the client is experiencing magnesium sulfate toxicity? A.Deep tendon reflexes 2+. B.Blood pressure 140/90. C.Respiratory rate 18/minute. D.Urine output 90 ml/4 hours.

D.Urine output 90 ml/4 hours.

secondary side effect of Terbutaline sulfate (Brethine), a

Drowsiness and bradycardia.

Drug therapy for ALL

Drug therapy for ALL includes doxorubicin or daunorubicin (or daunomycin) cytosine arabinoside

The nurse is planning preconception care for a new female client. Which information should the nurse provide the client?

Encourage healthy lifestyles for families desiring pregnancy. Planning for pregnancy begins with healthy lifestyles in the family (D) which is an intervention in preconception care that targets an overall goal for a client preparing for pregnancy.

riteria for physical maturity score Ballard (Dubowitz) scale.

Full flexion of all extremities in resting supine position.

What is the evaluation of fundal height?

Fundal height is measured to evaluate the gestational age of the fetus.

A pregnant client provides the following obstetric history to the nurse at the first prenatal visit: Elective abortion at age 17; a 5-year-old daughter born at 40 weeks gestation; and 3-year-old twin boys born at 34 weeks gestation. Using the GTPAL system, which option is correct?

G4, T1, P1, A1, L3

The nurse would advise a mother who is concerned because her 10-month-old is lethargic to__________.

Go to the Emergency Department

The nurse instructs a laboring client to use accelerated-blow breathing. The client begins to complain of tingling fingers and dizziness. What action should the nurse take?

Have the client breathe into her cupped hands.

Which of the following is a consequence of hypothermia in a newborn? a) Skin pink and warm b) Holds breath 25 seconds c) Heart rate of 126 d) Respirations of 46

Holds breath 25 seconds Explanation: Apnea is the cessation of breathing for a specific amount of time, and in newborns it usually occurs when the breath is held for 15 seconds. Apnea, cyanosis, respiratory distress, and increased oxygen demand are all consequences of hypothermia.

An obese woman with diabetes has just given birth to a term, large for gestational age (LGA) newborn. Which of the following conditions should the nurse most expect to find in this infant? a) Hypoglycemia b) Hypertension c) Hypotension d) Hyperglycemia

Hypoglycemia Correct Explanation: LGA infants also need to be carefully assessed for hypoglycemia in the early hours of life because large infants require large amounts of nutritional stores to sustain their weight. If the mother had diabetes which was poorly controlled (the cause of the large size), the infant would have had an increased blood glucose level in utero to match the mother's; this caused the infant to produce elevated levels of insulin. After birth, these increased insulin levels will continue for up to 24 hours of life, possibly causing rebound hypoglycemia.

Most common Congenital heart disease?

Increased pulmonary blood flow

A pregnant client comes in for a routine first prenatal examination. According to the last menstrual period, the estimated gestational age is 12 weeks. Where would the nurse expect to palpate the uterine fundus in this client?

Just above the symphysis pubis

S/Sx of Ventricular Septal Defect (VSD)

Loud, harsh systolic murmur and palpable thrill

Onset of ITP

Most common thrombocytopenia of childhood, onset most frequents ages 2 to 10 yrs

The nurse providing culturally sensitive care to a group of new mothers should reinforce information concerning breastfeeding to which client?

Mother of Hispanic descent who refuses to offer colostrum to the newborn

Medical management of sickle cell anemia

Oxygenation, analgesics, volume expanders, transfusion of packed RBCs

What is the fetal presentation?

Part of the fetus that enters the pelvic inlet first. The cephalic presentation is the normal presentation.

A client in active labor is admitted with preeclampsia. Which assessment finding is most significant in planning this client's care?

Patellar reflex 4+. A 4+ reflex in a client with pregnancy-induced hypertension (A) indicates hyperreflexia, which is an indication of an impending seizure.

A client has just given birth at 42 weeks' gestation. What would the nurse expect to find during her assessment of the neonate? a) Lanugo covering the neonate's body b) A sleepy, lethargic neonate c) Vernix caseosa covering the neonate's body d) Peeling and wrinkling of the neonate's epidermis

Peeling and wrinkling of the neonate's epidermis Correct Explanation: Postdate neonates lose the vernix caseosa, and the epidermis may become peeled and wrinkled. A neonate at 42 weeks' gestation is usually very alert and missing lanugo.

Aplastic Crisis

Profound anemia, caused by premature destruction of erythrocytes

S/Sx of Transposition of Great Vessels

Profound cyanosis; cardiomegaly is apparent on X-ray and includes signs of heart failure

A newborn is being monitored for retinopathy of prematurity. Which of the following conditions predisposes an infant to this condition? a) Esophageal atresia b) Down syndrome c) Respiratory distress syndrome d) Hydrocephalus

Respiratory distress syndrome Correct Explanation: Retinopathy of prematurity (ROP) is a complication that can occur when high concentrations of oxygen are given during the course of treatment for respiratory distress syndrome (RDS). ROP is caused by separation and fibrosis of the retinal blood vessels and can often result in blindness.

Epogen and RR

Respiratory rate should decrease rather than increase

characteristic of diaphragmatic hernia.

Scaphoid abdomen and anorexia

Which of the following would be signs of dehydration in a newborn? a) Eight wet diapers a day b) 10% weight gain c) Sunken fontanels d) Frequent feedings

Sunken fontanels Correct Explanation: Sunken fontanels in a newborn are a sign of dehydration. Other signs are sunken eyeballs, decreased urine output, lethargy, decreased skin turgor, decreased body weight, and irritability.

The most common CHD resulting in decreased pulmonary blood flow?

Tetralogy of Fallot

Tetralogy of Fallot

Tetralogy of Fallot involves a combination of four defects: (1) Pulmonary stenosis (2) VSD (3) Right-ventricular hypertrophy (4) Overriding aorta

How would Ventricular Septal Defect (VSD) appear?

The possible scope of the defect ranges from a pinhole-sized opening to absence of the entire septum (single ventricle)

The nurse is assessing the umbilical cord of a newborn. Which finding constitutes a normal finding?

Three vessels: two arteries and one vein

Medical management of Aplastic crisis

Transfusion of packed RBCs

What is the Nitrazine test?

Used to detect the presence of amniotic fluid. Vaginal secretions have a pH of 4.5-5.5 and do not affect the Nitrazine strip or swab. Amniotic fluid has a pH of 7-7.5 and turns the Nitrazine strip blue.

When during pregnancy does ankle edema occur?

Usually occurs in the second and third trimesters. Results from venous stasis and increased venous pressure caused by uterine enlargement.

What routine interventions decrease the risk of postpartum hemorrhage?

Uterine fundal massage and postpartum oxytocin.

When preparing a class on newborn care for expectant parents, what content should the nurse teach concerning the newborn infant born at term gestation?

Vernix is a white, cheesy substance, predominantly located in the skin folds.

What happens when platelet count drops?

When the platelet count drops below 20,000/mm3, clotting is impaired When the platelet counts lower than 10,000/mm3, it is potentially life-threatening

At what time after delivery should RhoGAM be administered in the postpartum period?

Within 72 hours after delivery.

A newborn, whose mother is HIV positive, is scheduled for follow-up assessments. The nurse knows that the most likely presenting symptom for a pediatric client with AIDS is:

a persistent cold.

When explaining "postpartum blues" to a client who is 1 day postpartum, which symptoms should the nurse include in the teaching plan? (Select all that apply.) a.Mood swings. b.Panic attacks. c.Tearfulness d.Decreased need for sleep. e. Disinterest in the infant

a.Mood swings. c.Tearfulness

In developing a teaching plan for expectant parents, the nurse plans to include information about when the parents can expect the infant's fontanels to close. The nurse bases the explanation on knowledge that for the normal newborn, the

anterior fontanel closes at 12 to 18 months and the posterior by the end of the second month.

Epogen and urinary output

is not related to Epogen

A patient who is 32 weeks gestation is experiencing dark red vaginal bleeding and the nurse determines the FHR to be 100 bpm and her abdomen is rigid and board like. What action should the nurse take first? A. Administer O2 per face mask B. Abdominal manipulation C. vaginal manipulation D. Abdominal exam

A. The nurse should immediately notify the healthcare provider and no abdominal or vaginal manipulation or exams should be done. Administer O2 per face mask and monitor for bleeding at IV sites and gums due to the increased risk of DIC

Physiologic jaundice (normal inability of the immature liver to keep up with normal RBC destruction) occurs 2-3 days of life.When does jaundice become pathologic? A. When it occurs before 24 hurs or persists beyond 7 days B. When it occurs before 14 hours or persists beyond 8 day C. When it occurs before 12 hours or persists beyond 3 days D. When it occurs before 10 hours or persists beyond 2 days

A. Physiologic jaundice (normal inability of the immature live to keep up with normal RBC destruction) occurs at 2-3 days of life. It occurs before 24 hours or persists beyond 7 days, it becomes pathologic

The nurse is developing a dietary teaching plan for a patient on a vegetarian diet. The nurse should provide the patient with which examples of protein containing foods? (Select all that apply.) A. Dried beans B. Seeds C. Peanut butter D. Bagel E. Eggs Correct

A. Dried beans B. Seeds C. Peanut butter E. Eggs All of the foods listed except a bagel provide protein. A bagel is an example of a whole grain food, not protein.

Immediately after birth a newborn infant is suctioned, dried and placed under a radiant warmer. The infant has spontaneous respirations and the nurse assesses an apical HR of 80 beats/minute and respirations of 20 breaths/min. What action should be performed next? A. Initiate positive pressure ventilation B . Intervene after one min Apgar is assessed. C. Initiate CPR on the infant D. Assess the infant's blood glucose level

A. Initiate positive pressure ventilation

The nurse is performing a gestational age assessment on a full-term newborn during the first hour of transition using the Ballard (Dubowitz) scale. Based on this assessment, the nurse determines that the neonate has a maturity rating of 40-weeks. What findings should the nurse identify to determine if the neonate is small for gestational age (SGA)? (Select all that apply.) A.Admission weight of 4 pounds, 15 ounces (2244 grams) B.Head to heel length of 17 inches (42.5 cm). C.Frontal occipital circumference of 12.5 inches (31.25 cm). D.Skin smooth with visible veins and abundant vernix. E.Anterior plantar crease and smooth heel surfaces. F. Full flexion of all extremities in resting supine position.

A.Admission weight of 4 pounds, 15 ounces (2244 grams) B.Head to heel length of 17 inches (42.5 cm). C.Frontal occipital circumference of 12.5 inches (31.25 cm).

A client at 28-weeks gestation calls the antepartal clinic and states that she is experiencing a small amount of vaginal bleeding which she describes as bright red. She further states that she is not experiencing any uterine contractions or abdominal pain. What instruction should the nurse provide? A.Come to the clinic today for an ultrasound. B.Go immediately to the emergency room. C.Lie on your left side for about one hour and see if the bleeding stops. D.Bring a urine specimen to the lab tomorrow to determine if you have a urinary tract infection.

A.Come to the clinic today for an ultrasound.

The nurse is caring for a woman with a previously diagnosed heart disease who is in the second stage of labor. Which assessment findings are of greatest concern? A.Edema, basilar rales, and an irregular pulse. B. Increased urinary output and tachycardia. C.Shortness of breath, bradycardia, and hypertension. D.Regular heart rate and hypertension.

A.Edema, basilar rales, and an irregular pulse.

The nurse attempts to help an unmarried teenager deal with her feelings following a spontaneous abortion at 8-weeks gestation. What type of emotional response should the nurse anticipate? A.Grief related to her perceptions about the loss of this child. B.Relief of ambivalent feelings experienced with this pregnancy. C.Shock because she may not have realized that she was pregnant. D. Guilt because she had not followed her healthcare provider's instructions.

A.Grief related to her perceptions about the loss of this child.

Which nursing intervention is helpful in relieving "afterpains" (postpartum uterine contractions)? A.Using relaxation breathing techniques. B.Using a breast pump. C.Massaging the abdomen. D.Giving oxytocic medications.

A.Using relaxation breathing techniques.

The nurse is counseling a couple who has sought information about conceiving. For teaching purposes, the nurse should know that ovulation usually occurs A.two weeks before menstruation. B.immediately after menstruation. immediately before menstruation. C. immediately before menstruation. D. three weeks before menstruation.

A.two weeks before menstruation.

In terms of fine motor development, the infant of 7 months should be able to: a. Transfer objects from one hand to the other. b. Use thumb and index finger in crude pincer grasp. c. Hold crayon and make a mark on paper. d. Release cubes into a cup.

ANS: A By age 7 months infants can transfer objects from one hand to the other, crossing the midline. The crude pincer grasp is apparent at about age 9 months. The child can scribble spontaneously at age 15 months. At age 12 months the child can release cubes into a cup.

The parent of a 3-month-old infant asks the nurse, "At what age do infants usually begin drinking from a cup?" The nurse would reply: a. 5 months. b. 9 months. c. 1 year. d. 2 years.

ANS: A The infant can usually drink from a cup when it is offered at about 5 months.

What represents the major stressor of hospitalization for children from middle infancy throughout the preschool years? a. Separation anxiety b. Loss of control c. Fear of bodily injury d. Fear of pain

ANS: A The major stress for children from infancy through the preschool years is separation anxiety, also called anaclitic depression. This is a major stressor of hospitalization. Loss of control, fear of bodily injury, and fear of pain are all stressors associated with hospitalization. However, separation from family is a primary stressor in this age group.

A 3-month-old infant, born at 38 weeks of gestation, will hold a rattle if it is put in her hands, but she will not voluntarily grasp it. The nurse should interpret this as: a. Normal development. b. Significant developmental lag. c. Slightly delayed development caused by prematurity. d. Suggestive of a neurologic disorder such as cerebral palsy.

ANS: A This indicates normal development. Reflexive grasping occurs during the first 2 to 3 months and then gradually becomes voluntary. No evidence of neurologic dysfunction is present.

A parent of an 18-month-old boy tells the nurse that he says "no" to everything and has rapid mood swings. If he is scolded, he shows anger and then immediately wants to be held. The nurse's best interpretation of this behavior is that: a. This is normal behavior for his age. b. This is unusual behavior for his age. c. He is not effectively coping with stress. d. He is showing he needs more attention.

ANS: A Toddlers use distinct behaviors in the quest for autonomy. They express their will with continued negativity and the use of the word "no." Children at this age also have rapid mood swings. The nurse should reassure the parents that their child is engaged in expected behavior for an 18-month-old.

The nurse discusses child-proofing the home for safety with the mother of a 9-month-old. The statement made by the mother that indicates an unsafe behavior is: a. "I put covers on all of the electrical outlets." b. "In the car, she rides in a front-facing car seat." c. "There are locks on all of the cabinets in the house." d. "I have a gate at the top and bottom of the stairs."

ANS: B A rear-facing infant car seat should be used for infants younger than 1 year of age.

What is descriptive of the play of school-age children? a. Individuality in play is better tolerated than at earlier ages. b. Knowing the rules of a game gives an important sense of belonging. c. They like to invent games, making up the rules as they go. d. Team play helps children learn the universal importance of competition and winning.

ANS: B Play involves increased physical skill, intellectual ability, and fantasy. Children form groups and cliques and develop a sense of belonging to a team or club. At this age children begin to see the need for rules. Conformity and ritual permeate their play. Their games have fixed and unvarying rules, which may be bizarre and extraordinarily rigid. With team play children learn about competition and the importance of winning, an attribute highly valued in the United States.

A 4-year-old boy is hospitalized with a serious bacterial infection. He tells the nurse that he is sick because he was "bad." The nurse's best interpretation of this comment is that it is: a. A sign of stress. b. Common at this age. c. Suggestive of maladaptation. d. Suggestive of excessive discipline at home.

ANS: B Preschoolers cannot understand the cause and effect of illness. Their egocentrism makes them think that they are directly responsible for events, making them feel guilt for things outside of their control. Children of this age show stress by regressing developmentally or acting out. Maladaptation is unlikely. This comment does not imply excessive discipline at home.

Which type of play is most typical of the preschool period? a. Solitary c. Associative b. Parallel d. Team

ANS: C Associative play is group play in similar or identical activities but without rigid organization or rules. Solitary play is that of infants. Parallel play is that of toddlers. School-age children play in teams.

The nurse would expect a 4-month-old to be able to: a. hold a cup. b. stand with assistance. c. lift head and shoulders. d. sit with back straight.

ANS: C Because development is cephalocaudal, of these choices, lifting the head and shoulders is the one that the infant learns to do first. The infant can usually sit with support at about 5 months of age and can sit alone at about 8 months.

Most infants begin to fear strangers at age: a. 2 months c. 6 months b. 4 months d. 12 months

ANS: C Between ages 6 and 8 months fear of strangers and stranger anxiety become prominent and are related to the infant's ability to discriminate between familiar and nonfamiliar people. At age 2 months infants are just beginning to respond differentially to the mother. At age 4 months the infant is beginning the process of separation individuation when the infant begins to recognize self and mother as separate beings. Twelve months is too late and requires referral for evaluation if the child does not fear strangers at this age.

Although a 14-month-old girl received a shock from an electrical outlet recently, her parents find her about to place a paper clip in another outlet. The best interpretation of this behavior is: a. Her cognitive development is delayed. b. This is typical behavior because toddlers are not very developed. c. This is typical behavior because of inability to transfer knowledge to new situations. d. This is not typical behavior because toddlers should know better than to repeat an act that caused pain.

ANS: C During the tertiary circular reactions stage, children have only a rudimentary sense of the classification of objects. The appearance of an object denotes its function for these children. The slot of an outlet is for putting things into. Her cognitive development is appropriate for her age and represents typical behavior for a toddler. Only some awareness exists of a causal relation between events.

A group of boys ages 9 and 10 years have formed a "boys-only" club that is open to neighborhood and school friends who have skateboards. This should be interpreted: a. Behavior that encourages bullying and sexism. b. Behavior that reinforces poor peer relationships. c. Characteristic of social development of this age. d. Characteristic of children who later are at risk for membership in gangs.

ANS: C One of the outstanding characteristics of middle childhood is the creation of formalized groups or clubs. Peer-group identification and association are essential to a child's socialization. Poor relationships with peers and a lack of group identification can contribute to bullying. A boys-only club does not have a direct correlation with later gang activity.

Generally the earliest age at which puberty begins is: a. 13 years in girls, 13 years in boys b. 11 years in girls, 11 years in boys c. 10 years in girls; 12 years in boys d. 12 years in girls, 10 years in boys

ANS: C Puberty signals the beginning of the development of secondary sex characteristics. This begins in girls earlier than in boys. Usually a 2-year difference occurs in the age at onset. Girls and boys do not usually begin puberty at the same age; girls usually begin earlier than boys do.

The most appropriate activity to recommend to parents to promote sensorimotor stimulation for a 1-year-old would be to: a. ride a tricycle. b. spend time in an infant swing. c. play with push-pull toys. d. read large picture books.

ANS: C Push-pull toys are appropriate to promote sensorimotor stimulation for a 1-year-old child.

When assessing development in a 9-month-old infant, the nurse would expect to observe the infant: a. speaking in 2-word sentences. b. grasping objects with palmar grasp. c. creeping along the floor. d. beginning to use a spoon rather sloppily.

ANS: C The 9-month-old tries to creep, has developed pincer movement, and can grasp a spoon without keeping food on it.

The statement made by a parent that indicates correct understanding of infant feeding is: a. "I've been mixing rice cereal and formula in the baby's bottle." b. "I switched the baby to low-fat milk at 9 months." c. "The baby really likes little pieces of chocolate." d. "I give the baby any new foods before he takes his bottle."

ANS: D New solid foods should be introduced before formula or breast milk to encourage the infant to try new foods.

A mother calls the pediatrician's office because her infant is "colicky." The helpful measure the nurse would suggest to the parent is to: a. sing songs to the infant in a soft voice. b. place the infant in a well-lit room. c. walk around and massage the infant's back. d. rock the fussy infant slowly and gently.

ANS: D One technique the nurse can offer parents of a fussy infant is to rock the infant gently and slowly while being careful to avoid sudden movements.

The nurse would advise a parent when introducing solid foods to: a. begin with one tablespoon of food. b. mix foods together. c. eliminate a refused food from the diet. d. introduce each new food 4 to 7 days apart.

ANS: D Only one new food is offered in a 4- to 7-day period to determine tolerance.

The role of the peer group in the life of school-age children is that it: a. Gives them an opportunity to learn dominance and hostility. b. Allows them to remain dependent on their parents for a longer time. c. Decreases their need to learn appropriate sex roles. d. Provides them with security as they gain independence from their parents.

ANS: D Peer-group identification is an important factor in gaining independence from parents. Through peer relationships children learn ways to deal with dominance and hostility. They also learn how to relate to people in positions of leadership and authority and explore ideas and the physical environment. Peer-group identification helps in gaining independence rather than remaining dependent. A child's concept of appropriate sex roles is influenced by relationships with peers.

If a child is on oral iron medication, the family should be taught by the nurse how it should be administered. Out of the following options, what oral iron administration advise is inappropriate? A. Oral iron should be given on empty stomach B. Oral iron should be given with citrus juices C. Oral iron shoule be given with dairy products D. A dropper or straw should be used to avoid discoloring teeth

C. Iron can be fatal in severe overdose and as a result, it should be kept away from children. Also, do not give with dairy products.

Skin traction for fracture reduction should not be removed unless prescribed by healthcare provider. What do the pin sites usually cause in an infant client? A. Hypoglycemia B. Hypocalcemia C. Infection D. Low vitamin K concentration

C. Pin sites can be sources of infection. The nurse should monitor signs of infection and cleanse and dress sites as prescribed.

A woman who is 6 weeks pregnant has the following maternal history: a 2 yr old healthey daughter, a miscarriage at 10 weeks, 3 years ago and an elective abortion at 6 weeks, 5 years ago. Describe gravidity and parity in this case. A. gravida 1, para 4 B. gravida 2, para 4 C. gravida 4, para 1 D. gravida 4, para 2

C. With this pregnancy, the woman is a gravida 4, para 1 ( only 1 delivery after 20 weeks geatation)

Vasodialation below the nerve blok results in pooling in the lower extremities and maternal hypotension. Which is the quanity of IV lactated ringers the client should be hydrated with 20 minutes prior to operation? A. 100-200 cc B. 300-500 cc C. 500-1000 cc D. 600-800 cc

C. Approximately 20 minutes prior to nerve block anesthesia, the client should be hydrated with 500-1000 cc of lactated ringers IV

Changes in fetal heart rate are the first and most important indicator of compromised blood flow to the fetus, and these changes require action! What is the normal FHR in a pregnant woman? A. 150 - 180 bpm B. 160 - 190 bpm C. 110 - 160 bpm D. 120 - 150 bpm

C. Changes in fetal heart rate are the first and most important indicator of compromised blood flow to the fetus, and these changes require action! Remember, the normal FHR is 110 to 160 bpm.

A nurse is preparing to care for a 5-year-old who has been placed in traction following a fracture of the femur. The nurse plans care, knowing that which of the following is the most appropriate activity for this child? A) Large picture books B) A radio C) Crayons and coloring book D) A sports video

C. In the preschooler, play is simple and imaginative and includes activities such as crayons and coloring books, puppets, felt and magnetic boards, and Play-Doh. Large picture books are most appropriate for the infant. A radio and a sports video are most appropriate for the adolescent.

Regardless of who performs the physical assessment , the nurse must know normal versus abnormal variations of the newborn. What is the difference between caput succedaneum and cephalhematoma? A. cephalhematoma crosses suture lines and is usually present at birth B. Cephalhematoma does NOT cross suture lines and manifests a few hours after birth C. Cephalhematoma: edema under scalp D. Caput succedaneum : blood under teh periosteum

B. It is difficult to differentiate between caput succedaneum (edema under the scalp) and cephalhematoma (blood under the peristeum). The caput crosses suture lines and is usually present at birth, while cephalhematoma does not cross suture lines and manifests a few hours after birth.

A nurse consults a pregnant mother and detects late decelerations which indicate uteroplacental insufficiency. What conditions are late decelerations associated with? A. Down Syndrome, AIDS, abruptio placentae B. Postmaturity, preeclampsia, diabetes mellitus, cardiac disease, and abruptio placentae C. Autism, renal insufficiency and cardiac disease D. Kidney failure, cardiac disease, Digialis toxicity

B. Late decelerations indicate unteroplacental insufficiency and are associated with conditions such as postmaturity, preeclampsia, diabetes mellitus, cardiac disease, and abruptio placentae

Physiologic jaundice is the normal inability of the immature liver to keep up with normal RBC destruction. When does jaundice occur in newborns? A. 5-6 days of life B . 2-3 days of life C. 7-8 days of life D. 9-10 days of life

B. Physiologic jaundice (normal inability of the immature liver to keep up with normal RBC destruction) occurs at 2-3 days of life

What describes a toddler's cognitive development at age 20 months? a. Searches for an object only if he or she sees it being hidden b. Realizes that "out of sight" is not out of reach c. Puts objects into a container but cannot take them out d. Understands the passage of time such as "just a minute" and "in an hour"

ANS: B At this age the child is in the final sensorimotor stage. Children will now search for an object in several potential places, even though they saw only the original hiding place. Children have a more developed sense of objective permanence. They will search for objects even if they have not seen them hidden. Putting an object in a container but being able to take it out indicates tertiary circular reactions. An embryonic sense of time exists, although the children may behave appropriately to time-oriented phrases; their sense of timing is exaggerated.

Postpartum blues are usually normal, especially 5 - 7 days after deliverry. In what case is RhoGAM given to a mother after delivery? A. If mother is Rh positive B. If mother is Rh negative C. If the mother has a positive Coombs D. If the mother has a Rh negative fetus

B. Remember RhoGRAM is given to a Rh negative mother who delivers a Rh positive fetus and has a negative direct Coombs. If the mother has a positive Coombs, there is no need to give RhoGRAM since the mother is already sensitized

Nerve lock anesthesia (spinal or epidural) during labor bloks motor as well as nerve fibers. What does result from vasodilation below the level of the block? A> Maternal hypertension B. Maternal hypotension C. Low BP D. High BP

B. Vasodialation below the level of the block results in blood pooling in the lower extremities and maternal hypotension.

In a 24 year old pregnant woman, the amniocentesis is done in early pregnancy. How should the bladder be to help support the uterus and to help push the uterus up in the abdomen for easy access? A. Empty B. Full C. 1/4 empty D. 1/2 full

B. When an amniocentesis is done in early pregnancy, the bladder must be full to help support the urerus and to help push the uterus up in the abdomen for easy access.

Diabetis mellitus (DM) in children was typically diagnosed as insulin dependent diabetes until recently. What diabetes type has been discovered to occur more often in Native Americans, African Americans, and Hispanic children and adolescents? A. Type 1 B. Type 2 C. Type 3 D. Type 4

B. DM in children was typicall diagnosed as insulin dependent until recently. A marked increase in Type 2 DM has occurred recently in the US.

Skin traction for fracture reduction should not be removed unless prescribed by healthcare provider. What fractures have serious consequences interms of growth of the affected limb? A. Greenstick fracture B. Plate fracture C. Torus fracture D. Spiral fracture

B. Fractures involving the epiphyseal plate ( growth plate) can have serious consequences in terms of growth of the affected limb

A nurse should teach the pregnant clients to immediately report any of the following danger signs because early intervention can optimize maternal and fetal outcome. Which are the signs of infection in a pregnant woman? A. FHR 110 - 160 bpm B. Chills C. Persistant vomition D. Visual disturbances

B. Signs of infection in a pregnant woman are Chills, Dysuria, pain in abdomen, fluid discharge from vagina, and increased FHR

A woman of childbearing age present at an emergency room with unilateral and bilateral abdominal pain. What should the nurse correctly suspect in this case? A. Appendicitis B. Ectopic pregnancy C. Entopic pregnancy D. Etiopic pregnancy

B. Suspect ectopic pregnancy in any woman of childbearing age who presents at an emergency room, clinic, or office with unilateral or bilateral abdominal pain. Most are misdiagnosed with appendicitis.

Physiologic jaundice which occurs 2-3 days after birth due to the liver's inability to keep up with RBC destruction. Who is the culprit in this case? A. Conjugated bilirubin B. Unconjugated bilirubin C. Unconjugated penile D. Conjugated penile

B. Typically, NCLEX-RN questions ask about normal problem physiologic jaundice which occurs 2-3 days afte birth due to the livers inability to keep up with RBC destruction and bind bilirubin. Remember, unconjugated bilirubin is the culprit.

Which statement best describes fear in school-age children? a. They are increasingly fearful for body safety. b. Most of the new fears that trouble them are related to school and family. c. They should be encouraged to hide their fears to prevent ridicule by peers. d. Those who have numerous fears need continuous protective behavior by parents to eliminate these fears.

ANS: B During the school-age years children experience a wide variety of fears, but new fears related predominantly to school and family bother children during this time. During the middle-school years children become less fearful of body safety than they were as preschoolers. Parents and other persons involved with children should discuss their fear with them individually or as a group activity. Sometimes school-age children hide their fears to avoid being teased. Hiding the fears does not end them and may lead to phobias.

A 4 week old premature infant has been receiving epoetin alfa for the last 3 weeks. WHich assessment finding indicates to the nurse that the drug is effective. A.slowly increasing urinary output over the last week B.rr changes from 40s to the 60s C. changes in apical HR from the 180 to the 140 D.Change in indirect bilirubin from 12mg/dl to 8mg/dl.

C. changes in apica HR from the 180 to the 140

A client who delivered an infant an hour ago tells the nurse that she feels wet underneath her buttock. The nurse notes that both perineal pads are completely saturated a nd the client is lying in a 6inch diameter pool of blood. A. Cleanse the perineum B. obtain a BP C. palpate the firmness of the fundus D; inspect the perineum for lacerations

C. palpate the firmness of the fundus

A client at 32-weeks gestation is hospitalized with severe pregnancy-induced hypertension (PIH), and magnesium sulfate is prescribed to control the symptoms. Which assessment finding indicates the therapeutic drug level has been achieved? A. 4+ reflexes. B.Urinary output of 50 ml per hour. C.A decrease in respiratory rate from 24 to 16. D. A decreased body temperature.

C.A decrease in respiratory rate from 24 to 16.

A newborn infant is brought to the nursery from the birthing suite. The nurse notices that the infant is breathing satisfactorily but appears dusky. What action should the nurse take first? A.Notify the pediatrician immediately. B.Suction the infant's nares, then the oral cavity. C.Check the infant's oxygen saturation rate. D.Position the infant on the right side.

C.Check the infant's oxygen saturation rate.

A mother who is breastfeeding her baby receives instructions from the nurse. Which instruction is most effective to prevent nipple soreness? A. Wear a cotton bra. B. Increase nursing time gradually. C.Correctly place the infant on the breast. D.Manually express a small amount of milk before nursing.

C.Correctly place the infant on the breast.

At 14-weeks gestation, a client arrives at the Emergency Center complaining of a dull pain in the right lower quadrant of her abdomen. The nurse obtains a blood sample and initiates an IV. Thirty minutes after admission, the client reports feeling a sharp abdominal pain and a shoulder pain. Assessment findings include diaphoresis, a heart rate of 120 beats/minute, and a blood pressure of 86/48. Which action should the nurse implement next?A. Check the hematocrit results. B. Administer pain medication. C.Increase the rate of IV fluids. D.Monitor client for contractions.

C.Increase the rate of IV fluids.

A client at 30-weeks gestation, complaining of pressure over the pubic area, is admitted for observation. She is contracting irregularly and demonstrates underlying uterine irritability. Vaginal examination reveals that her cervix is closed, thick, and high. Based on these data, which intervention should the nurse implement first? A.Provide oral hydration. B.Have a complete blood count (CBC) drawn. C.Obtain a specimen for urine analysis. D.Place the client on strict bedrest.

C.Obtain a specimen for urine analysis.

Twenty minutes after a continuous epidural anesthetic is administered, a laboring client's blood pressure drops from 120/80 to 90/60. What action should the nurse take? A. Notify the healthcare provider or anesthesiologist immediately. B.Continue to assess the blood pressure q5 minutes. C.Place the woman in a lateral position. D.Turn off the continuous epidural.

C.Place the woman in a lateral position.

A couple, concerned because the woman has not been able to conceive, is referred to a healthcare provider for a fertility workup and a hysterosalpingography is scheduled. Which postprocedure complaint indicates that the fallopian tubes are patent? A.Back pain B.Abdominal pain. C.Shoulder pain. D. Leg cramps.

C.Shoulder pain.

The nurse is assessing the umbilical cord of a newborn. Which finding constitutes a normal finding? A.Two vessels: one artery and one vein. B.Two vessels: two arteries and no veins. C.Three vessels: two arteries and one vein. D.Three vessels: two veins and one artery

C.Three vessels: two arteries and one vein.

When preparing a class on newborn care for expectant parents, what content should the nurse teach concerning the newborn infant born at term gestation? A.Milia are red marks made by forceps and will disappear within 7 to 10 days. B.Meconium is the first stool and is usually yellow gold in color. C.Vernix is a white, cheesy substance, predominantly located in the skin folds. D.Pseudostrabismus found in newborns is treated by minor surgery.

C.Vernix is a white, cheesy substance, predominantly located in the skin folds.

The nurse should encourage the laboring client to begin pushing when A.there is only an anterior or posterior lip of cervix left. B.the client describes the need to have a bowel movement. C.the cervix is completely dilated. D.the cervix is completely effaced.

C.the cervix is completely dilated.

The umbilical cord should always be checked at birth. What should the umbilical card contain in a newborn? A. 3 vessels, 2 veins which carry oxygenated blood to the fetus and 1 artery whic carries unoxygenated blood back to placenta B. 4 vessels, 2 veins which carry oxygenated blood to the fetus and 2 arteries which carries unoxygenated blood back to placenta C. 3 vessels, 1 veins which carries oxygenated blood to the fetus and 2 arteries which carries unoxygenated blood back to placenta D. 3 vessels, 1 artery which carries oxygenated blood to the fetus and 2 veins which carries unoxygenated blood back to placenta

C. It should contain 3 vessels, 1 vein which carries oxygenated blood to the fetus nad 2 arteries whic carry unoxygenated blood back to the placenta. This is opposite of normal circulation.

A 26-year-old, gravida 2, para 1 client is admitted to the hospital at 28-weeks gestation in preterm labor. She is given 3 doses of terbutaline sulfate (Brethine) 0.25 mg subcutaneously to stop her labor contractions. The nurse plans to monitor for which primary side effect of terbutaline sulfate? A. Drowsiness and bradycardia. B. Depressed reflexes and increased respirations. C. Tachycardia and a feeling of nervousness. D. A flushed, warm feeling and a dry mouth

C. Tachycardia and a feeling of nervousness.

A new mother who has just had her first baby says to the nurse, "I saw the baby in the recovery room. She sure has a funny looking head." Which response by the nurse is best? A.This is not an unusual shaped head, especially for a first baby. B.It may look funny to you, but newborn babies are often born with heads like your baby's. C. That is normal; the head will return to a round shape within 7 to 10 days. D.Your pelvis was too small, so the baby's head had to adjust to the birth canal.

C. That is normal; the head will return to a round shape within 7 to 10 days.

Fractures in older children are common as they fall during play and are involved in motor vehicle accidents. What fractures in children are related to child abuse? A. Greenstick Fracture B. Growth plate Fracture C. Torus Fracture D. Spiral fracture

D. Spiral fractures (caused by twisting) anf fractures in infants may be related to child abuse

A patient who is 32 weeks gestation has the following symptoms: dark, red vaginal bleeding, 100 bpm FHR, rigid abdomen and severe pain. What is the difference between abruptio placentae and placenta previa? A. Aruptio placentae: painless bright red bleeding occurring in the third trimester B. Abruptio placentae: occurs in the second trimester C. Placenta previa: occurs in the second trimester D. Placenta previa: painless bright red bleeding occurring in the third trimester

D. The nurse must use knowledge base to diffferentiate betewwn abruptia placentae from placenta previa.

Most providers prescribe prenatal vitamins to ensure that the client receives and adequate intake of vitamins. However, only the healthcare provider can prescribe prenatal vitamins. Whis is the quanity of milk a pregnant woman should drink per day for ensuring that the daily calcium needs are met? A. 1/2 quart B. 1/3 quart C. 1/4 quart D. 1 quart

D. It is recommended that pregnant woman drink one quart of milk a day. This will ensure that the daily calcium needs are met and help to alleviate the occurrence of leg cramps

Internal rotation is harder to achieve when the pelvic floor is relaxed by anesthesia resulting in persistent occiput posterior of fetus. What regional blocks often result in assissted delivery due to the inability to push effectively in the 2nd stage? A. Epidermis B. Anal Spincter C. Rectal mucosa D. Caudal

D. Regional blocks, especially epidural and caudal, often result in assissted delivery due to the inability to push effectively in 2nd stage

A 16-year-old is admitted to the hospital for acute appendicitis, and an appendectomy is performed. Which of the following nursing interventions is most appropriate to facilitate normal growth and development? A) Allow the family to bring in the child's favorite computer games B) Encourage the parents to room-in with the child C) Encourage the child to rest and read D) Allow the child to participate in activities with other individuals in the same age group when the condition permits

D. Adolescents often are not sure whether they want their parents with them when they are hospitalized. Because of the importance of the peer group, separation from friends is a source of anxiety. Ideally, the members of the peer group will support their ill friend. Options a, b, and c isolate the child from the peer group.

After each feeding, a 3-day-old newborn is spitting up large amounts of Enfamil® Newborn Formula, a nonfat cow's milk formula. The pediatric healthcare provider changes the neonate's formula to Similac® Soy Isomil® Formula, a soy protein isolate based infant formula. What information should the nurse provide to the mother about the newly prescribed formula? A.The new formula is a coconut milk formula used with babies with impaired fat absorption. B.Enfamil® Formula is a demineralized whey formula that is needed with diarrhea. C. The new formula is a casein protein source that is low in phenylalanine. D. Similac® Soy Isomil® Formula is a soy-based formula that contains sucrose.

D. Similac® Soy Isomil® Formula is a soy-based formula that contains sucrose.

A 22-year-old woman pregnant with a single fetus has a preconception body mass index (BMI) of 24. When she was seen in the clinic at 14 weeks of gestation, she had gained 1.8 kg (4 lbs) since conception. How would the nurse interpret this? A. This weight gain indicates possible gestational hypertension. B.This weight gain indicates that the woman's infant is at risk for intrauterine growth restriction (IUGR). C.This weight gain cannot be evaluated until the woman has been observed for several more weeks. D.The woman's weight gain is appropriate for this stage of pregnancy.

D. The woman's weight gain is appropriate for this stage of pregnancy. During the first trimester, the average total weight gain is only 1 to 2.5 kg.The desirable weight gain during pregnancy varies among women. The primary factor to consider in making a weight gain recommendation is the appropriateness of the prepregnancy weight for the woman's height. A commonly used method of evaluating the appropriateness of weight for height is the BMI. This woman has gained the appropriate amount of weight for her size at this point in her pregnancy. Weight gain should take place throughout the pregnancy. The optimal rate of weight gain depends on the stage of the pregnancy. This woman's BMI is within the normal range.

In developing a teaching plan for expectant parents, the nurse plans to include information about when the parents can expect the infant's fontanels to close. The nurse bases the explanation on knowledge that for the normal newborn, the A. anterior fontanel closes at 2 to 4 months and the posterior by the end of the first week. B.anterior fontanel closes at 5 to 7 months and the posterior by the end of the second week. C.anterior fontanel closes at 8 to 11 months and the posterior by the end of the first month. D. anterior fontanel closes at 12 to 18 months and the posterior by the end of the second month.

D. anterior fontanel closes at 12 to 18 months and the posterior by the end of the second month.

A 40 week gestation primigravida client is being induced with an ocytocin secondary infusion and complains of pain in her lower back. Which intervention should the nurse implement? A. Discontinue the oxytocin infusion B. place the client in a semi-fowler's position C. inform the healthcare provider D. apply firm pressure to sacral area

D. apply firm pressure to sacral area

An expectant father tells the nurse he fears that his wife "is losing her mind." He states she is constantly rubbing her abdomen and talking to the baby, and that she actually reprimands the baby when it moves too much. What recommendation should the nurse make to this expectant father? A. Reassure him that these are normal reactions to pregnancy and suggest that he discuss his concerns with the childbirth education nurse. B.Help him to understand that his wife is experiencing normal symptoms of ambivalence about the pregnancy and no action is needed. C. Ask him to observe his wife's behavior carefully for the next few weeks and report any similar behavior to the nurse at the next prenatal visit. D.Let him know that these behaviors are part of normal maternal/fetal bonding which occur once the mother feels fetal movement.

D.Let him know that these behaviors are part of normal maternal/fetal bonding which occur once the mother feels fetal movement.

A new mother is afraid to touch her baby's head for fear of hurting the "large soft spot." Which explanation should the nurse give to this anxious client? A.Some care is required when touching the large soft area on top of your baby's head until the bones fuse together. B.That's just an 'old wives' tale' so don't worry, you can't harm your baby's head by touching the soft spot. C.The soft spot will disappear within 6 weeks and is very unlikely to cause any problems for your baby. D.There's a strong, tough membrane there to protect the baby so you need not be afraid to wash or comb his/her hair.

D.There's a strong, tough membrane there to protect the baby so you need not be afraid to wash or comb his/her hair.

A client with no prenatal care arrives at the labor unit screaming, "The baby is coming!" The nurse performs a vaginal examination that reveals the cervix is 3 centimeters dilated and 75% effaced. What additional information is most important for the nurse to obtain?

Date of last normal menstrual period. Evaluating the gestation of the pregnancy (C) takes priority. If the fetus is preterm and the fetal heart pattern is reassuring, the healthcare provider may attempt to prolong the pregnancy and administer corticosteroids to mature the lungs of the fetus.

Reason of prevalent decrease in iron-deficiency anemia?

Family participation in the Women, Infants, and Children (WIC) program, which provides iron-fortified formula for the first year of life

Why should salicylate drugs be avoided for pain use?

Family should be counseled never to depend on salicylate drugs (aspirin) for pain because salicylates inhibit platelet function; salicylate substitutes such as acetaminophen (Tylenol) should be used instead

A nurse is caring for a newborn whose screening test result indicates the possibility of sickle cell anemia (SCA) or sickle cell trait. The nurse would expect the test result to be confirmed by which of the following lab tests: a) Peripheral blood smear b) Hemoglobin electrophoresis c) Erythrocyte sedimentation rate d) Reticulocyte count

Hemoglobin electrophoresis If the screening test result indicates the possibility of SCA or sickle cell trait, hemoglobin (Hgb) electrophoresis is performed promptly to confirm the diagnosis. While Hgb electrophoresis is the only definitive test for diagnosis of the disease, other laboratory testing that assists in the assessment of the disease include reticulocyte count (greatly elevated), peripheral blood smears (presence of sickle-shaped cells and target cells), and erythrocyte sedimentation rate (elevated).

What are the neonatal signs of respiratory distress and hypoxemia?

Nasal flaring, severe retractions, grunting, cyanosis,. Bradycardia or apnea for more than 15 seconds.

How is reflex inability scored for the Apgar?

No response to stimulation 0 points. Grimace/feeble cry when stimulated 1 point. Cry or pull away score 2 points.

Transposition of Great Vessels

Pulmonary artery arises from the left ventricle and the aorta arises from the right ventricle. (The pulmonary artery is in the left atrium and the aorta is in the right atrium)

What is RICE?

R-rest I-ice C-compression E-elevate When external bleeding occurs, the affected area should be elevated while pressure and ice is applied.

SGA neonate's parameters

Skin smooth with visible veins and abundant vernix. E. Anterior plantar crease and smooth heel surfaces.

A nurse is caring for a preterm newborn who has developed rapid, irregular respirations with periods of apnea. Which of the following additional signs should the nurse consider as indications of respiratory distress syndrome (RDS) in the newborn? a) Inspiratory grunt b) Deep inspiration c) Sternal retraction d) Expiratory lag

Sternal retraction Correct Explanation: The nurse should consider sternal retraction as a sign of respiratory distress syndrome in the preterm newborn. Deep inspiration is not seen during respiratory distress; rather a shallow and rapid respiration is seen. There is an inspiratory lag, instead of an expiratory lag, during respiratory distress. There is a grunting heard when the air is breathed out, which is during expiration and not during inspiration.

In caring for a child with sickle cell disease, the highest priority goal is which of the following? a) The family caregivers' anxiety will be reduced. b) The child's skin integrity will be maintained. c) The family will verbalize understanding of of the disease crisis. d) The child's fluid intake will improve

The child's fluid intake will improve. Explanation: The highest priority goals for this child are maintaining comfort and relieving pain. The child is prone to dehydration because of the kidneys' inability to concentrate urine, so increasing fluid intake is the next highest priority. Other goals include improving physical mobility, maintaining skin integrity, reducing the caregivers' anxiety, and increasing the caregivers' knowledge about the causes of crisis episodes, but these goals are not the highest priority.

A nurse notes that a term patient in labor is having late decelerations. Why should the nurse notify the physician? 1. Utero-placental insufficiency 2. Fetal distress 3. Impending delivery 4. Cord Compression

1. Utero-placental insufficiency Rationale: Late decelerations are caused by uteroplacental insufficiency. Cord compression causes variable decelerations. Impending delivery is signaled by crowning of the fetal head.

How many chromosomes present in the ovum and sperm?

23 each

The nurse is caring for 4 hospitalized clients. Which client should the nurse see first?

Client with suspected ectopic pregnancy who has abdominal and shoulder pain

During the first prenatal assessment, the client reports the last normal menstrual period starting on March 1 and ending on March 5, but also slight spotting on March 23. The client had unprotected intercourse on March 15. Using Naegele's rule, what is the estimated date of birth (EDB)?

December 8

Aplastic Anemia

Failure of the cell-generating capacity of the bone marrow and defective, underdeveloped or absent blood elements resulting in severe anemia, leukopenia, and thrombocytopenia (pancytopenia)

Ductus Arteriosus

Fetal artery that connects the pulmonary artery to the aorta

Who are the potential carriers for Hemophilia?

Hemophilia is transmitted by female carriers and affects boys in 1 per 5000 male births

Reason why an infant shouldn't drink cow's milk?

Infants who drink cow's milk have a 50% chance of increased fecal loss of blood.

Medical management for ITP (1)

It is a self-limiting illness and about three fourths of affected children recover without complications within 3 months

Purpose of the placenta for the fetus?

Source of oxygen during fetal development -pg. 977 Foundations of Nursing

A client postpartum 3 days scheduled for discharge today was given education about diaper changes yesterday. The client says to the nurse, "I'm so glad you are here. I think my baby has a dirty diaper. I can't change it as well as you can. Will you change my baby's diaper for me?" What is the nurse's best response?

Suggest that the mother change the diaper as the nurse watches

A 20-year-old client gave birth to a baby boy during the 43rd week of gestation. Which of the following might the nurse observe in the newborn during routine assessment? a) The newborn may have short nails and hair. b) The newborn may look wrinkled and old at birth. c) The testes in the child may be undescended. d) The infant may have excess of lanugo and vernix caseosa.

The newborn may look wrinkled and old at birth. Correct Explanation: Postterm babies are those born past 42 weeks of gestation. These babies often appear wrinkled and old at birth. They often have long fingernails and hair, dry parched skin, and no vernix caseosa. Both the quantity of lanugo and the amount of vernix decrease with gestational age. Undescended testes are usually not seen in postterm newborns; however, they are highly prevalent in preterm infants

What does a non-stress test do?

The non-stress test checks for increases in the baby's heart beat with fetal movements.

The nurse clarifies that the family-centered care approach terminates which policies? (Select all that apply.) a. Rigid visiting hours b. Freedom to choose which medications to take c. Exclusion of family during procedures d. Discouraging family to stay overnight e. Restricting parents from reading the chart

a. Rigid visiting hours c. Exclusion of family during procedures d. Discouraging family to stay overnight e. Restricting parents from reading the chart Family-centered care terminates all the restrictive policies of traditional hospitals. Medication orders should still be followed. p. 934

The nurse welcomes the presence of the family in a pediatric unit because it reduces the stressors of hospitalization. Which are common stressors for the hospitalized child? (Select all that apply.) a. Separation b. Lack of love c. Fear of pain d. Unfamiliar food e. Loss of control

a. Separation c. Fear of pain e. Loss of control Parents lend stability and comfort for the child and restore his or her sense of control. p. 934

6. A pregnant woman reports that she is still playing tennis at 32 weeks of gestation. The nurse would be most concerned that during and after tennis matches this woman consumes: a. Several glasses of fluid. b. Extra protein sources such as peanut butter. c. Salty foods to replace lost sodium. d. Easily digested sources of carbohydrate.

a. Several glasses of fluid.

A 42-week gestational client is receiving an intravenous infusion of oxytocin (Pitocin) to augment early labor. The nurse should discontinue the oxytocin infusion for which pattern of contractions? a. Transition labor with contractions every 2 minutes, lasting 90 seconds each a. Early labor with contractions every 5 minutes, lasting 40 seconds each c. Active labor with contractions every 31 minutes, lasting 60 seconds each d. Active labor with contractions every 2 to 3 minutes, lasting 70 to 80 seconds each

a. Transition labor with contractions every 2 minutes, lasting 90 seconds each Contractions pattern (A) describes hyperstimulation and an inadequate resting time between contractions to allow for placental perfusion. The oxytocin infusion should be discontinued.

A multigravida client at 41-weeks gestation presents in the labor and delivery unit after a non-stress test indicated that the fetus is experiencing some difficulties in utero. Which diagnostic test should the nurse prepare the client for additional information about fetal status? a.Biophysical profile (BPP). t b.Ultrasound for fetal anomalies. c.Maternal serum alpha-fetoprotein (AF) screening d.Percutaneous umbilical blood sampling (PUBS).

a.Biophysical profile (BPP).

On admission to the prenatal clinic, a 23-year-old woman tells the nurse that her last menstrual period began on February 15, and that previously her periods were regular. Her pregnancy test is positive. This client's expected date of delivery (EDD) is a.November 22. b.November 8. c.December 22. d.October 22.

a.November 22.

A 42-week gestational client is receiving an intravenous infusion of oxytocin (Pitocin) to augment early labor. The nurse should discontinue the oxytocin infusion for which pattern of contractions? a.Transition labor with contractions every 2 minutes, lasting 90 seconds each. b.Early labor with contractions every 5 minutes, lasting 40 seconds each. c.Active labor with contractions every 31 minutes, lasting 60 seconds each. d.Active labor with contractions every 2 to 3 minutes, lasting 70 to 80 seconds each.

a.Transition labor with contractions every 2 minutes, lasting 90 seconds each.

the nurse is assessing a 3-day old infant with a cephalohematoma in the newborn nursery. Which assessment finding should the nurse report to the healthcare provider? a.Yellowish tinge to the skin. b. Babinski reflex present bilaterally. c.Pink papular rash on the face. d.Moro reflex noted after a loud noise.

a.Yellowish tinge to the skin.

A pregnant client with mitral stenosis Class III is prescribed complete bedrest. The client asks the nurse, "Why must I stay in bed all the time?" Which response is best for the nurse to provide this client? a. Complete bedrest decreases oxygen needs and demands on the heart muscle b. We want your baby to be healthy, and this is the only way we can make sure that will happen again c. I know you're upset. Would you like to talk about somethings you could so while in bed? d. Labor is difficult and you need to use this time to rest before you have to assume all child-caring duties

a. Complete bedrest decreases oxygen needs and demands on the heart muscle tissue To help preserve cardiac reserves, the woman may need to restrict her activities and complete bedrest is often prescribes (A).

A healthcare provider informs the charge nurse of a labor and delivery unit that a client is coming to the unit with suspected abruptio placentae. What findings should the charge nurse expect the client to demonstrate? (Select all that apply) a. Dark, red vaginal bleeding b. Lower back pain c. Premature rupture of membranes d. Increased uterine irritability e. Bilateral pitting edema f. A rigid abdomen

a. Dark, red vaginal bleeding d. Increased uterine irritability f. A rigid abdomen The symptoms of abruptio placentae include dark red vaginal bleeding (A), increased uterine irritability (D), and a rigid abdomen (F).

A woman with Type 2 diabetes mellitus becomes pregnant, and her oral hypoglycemic agents are discontinued. Which intervention is most important for the nurse to implement? a. Describe diet changes that can improve the management of her diabetes. b.Inform the client that oral hypoglycemic agents are teratogenic during pregnancy. c. Demonstrate self-administration of insulin. d. Evaluate the client's ability to do glucose monitoring.

a. Describe diet changes that can improve the management of her diabetes.

17. Which statement regarding acronyms in nutrition is accurate? a. Dietary reference intakes (DRIs) consist of recommended dietary allowances (RDAs), adequate intakes (AIs), and upper limits (ULs). b. RDAs are the same as ULs except with better data. c. AIs offer guidelines for avoiding excessive amounts of nutrients. d. They all refer to green leafy vegetables, whole grains, and fruit.

a. Dietary reference intakes (DRIs) consist of recommended dietary allowances (RDAs), adequate intakes (AIs), and upper limits (ULs).

3. Which nutrient's recommended dietary allowance (RDA) is higher during lactation than during pregnancy? a. Energy (kcal) b. Iron c. Vitamin A d. Folic acid

a. Energy (kcal)

37. Identify a goal of a patient with the following nursing diagnosis: Imbalanced Nutrition: Less Than Body Requirements related to diet choices inadequate to meet nutrient requirements of pregnancy. a. Gain a total of 30 lb. b. Take daily supplements consistently. c. Decrease intake of snack foods. d. Increase intake of complex carbohydrates.

a. Gain a total of 30 lb.

35. A pregnant patient would like to know a good food source of calcium other than dairy products. Your best answer is: a. Legumes b. Yellow vegetables c. Lean meat d. Whole grains

a. Legumes

When explaining "postpartum blues" to a client who is 1 day postpartum, which symptoms should the nurse include in the teaching plan? (Select all that apply.) a. Mood swings b. Panic attacks c. Tearfulness d. Decreased need for sleep e. Disinterest in the infant

a. Mood swings c. Tearfulness "Postpartum blues" is a common emotional response related to the rapid decrease in placental hormones after delivery and include mood swings (A), tearfulness (C), feeling low, emotional, and fatigued.

When the nurse is inserting a feeding tube in an 8-month-old, what safety reminder device (SRD) should the nurse most likely use? a. Mummy b. Clove hitch c. Jacket device d. Elbow device

a. Mummy The mummy restraint controls the arms and the body of the infant. p. 960

A client receiving epidural anesthesia begins to experience nausea and becomes pale and clammy. What intervention should the nurse implement first? a. Raise the foot of the bed b. Assess for vaginal bleeding c. Evaluate the fetal heart rate d. Take the client's blood pressure

a. Raise the foot of the bed These symptoms are suggestive of hypotension which is a side effect of epidural anesthesia. Raising the foot of the bed (A) will increase venous return and provide blood to the vital areas. Increasing the IV fluid rate using a balanced non-dextrose solution and ensuring that the silent is in a lateral position are also appropriate interventions.

The nurse is providing discharge teaching for a client who is 24 hours postpartum. The nurse explains to the client that her vaginal discharge will change from red to pink and then to white. The client asks, "What if I start having red bleeding after it changes?" What should the nurse instruct the client to do? a. Reduce activity level and notify the healthcare provider b. Go to bed and assume a knee-chest position c. Massage the uterus and go to the emergency room d. Do not worry as this is a normal occurrence

a. Reduce activity level and notify the healthcare provider Lochia should progress in stages from rubra (red) to serosa (pinkish) to alba (whitish), and not return to red. The return to rubra usually indicates subinvolution of infection.

List 3 defects resulting in obstruction to systemic blood flow.

(1) Aortic Stenosis (2) Pulmonary Stenosis (3)Coarctation of the Aorta

A child has been diagnosed with Reye's syndrome. The nurse understands that a major symptom associated with Reye's syndrome is:

*1. Persistent vomiting* 2. Protein in the urine 3. Symptoms of hyperglycemia 4. A history of a Staphylococcus infection *Rationale:* Persistent vomiting is a major symptom that is associated with increased intracranial pressure (ICP). Options 2, 3, and 4 are incorrect. Protein is not present in the urine. Reye's syndrome is related to a history of viral infections, and hypoglycemia is a symptom of this disease.

After a tonsillectomy, a child is brought to the pediatric unit. The nurse places the child in which appropriate position?

*1. Prone* 2. Supine 3. Trendelenburg's 4. High Fowler's *Rationale:* The child should be placed in a prone or side-lying position after tonsillectomy to facilitate drainage. Options 2, 3, and 4 will not achieve this goal.

When assessing neonates in the nursery, the practical nurse should report which findings to the registered nurse? Select all that apply.

- Chest wall retractions - Head circumference of 30 cm - Jaundice of the head and sclera - No voiding in 24 hours

The nurse checks the chart of a client who gave birth 4 hours ago. Which contributing factor indicates that the client has a high risk of early postpartum hemorrhage? Select all that apply.

- Grand multiparity - Infant birth weight of 9 lb, 2 oz (4139 g) - Third stage of labor lasting 1 hour

A client has just recieved syntheric prostaglandins for the induction of labor. The nurse plans to monitor the client for which of the following side effects? 1. Nausea and uterine tetany 2. Hypertension and vaginal bleeding 3. Urinary retention and severe headache 4. Bradycardia and hypothermia

1. Two side effects of prostaglandin administration are nausea and uterine tetany

Antibiotics are prescribed for a child following a myringotomy with insertion of tympanostomy tubes, and the nurse provides instructions to the parents regarding the administration of the antibiotics. Which statement, if made by a parent, would indicate that the instructions were understood?

*1. "We will administer the antibiotics until they are gone."* 2. "We will administer the antibiotics if the child has a fever." 3. "We will administer the antibiotics until the child feels better." 4. "We will begin to taper the antibiotics after 3 days of a full course." *Rationale:* Antibiotics need to be taken as prescribed, and the full course needs to be completed. It is important that parents are instructed regarding the administration of antibiotics. Options 2, 3, and 4 are incorrect. Antibiotics are not tapered but administered until they are completed.

22. With regard to nutritional needs during lactation, a maternity nurse should be aware that: a. The mother's intake of vitamin C, zinc, and protein now can be lower than during pregnancy. b. Caffeine consumed by the mother accumulates in the infant, who may be unusually active and wakeful. c. Critical iron and folic acid levels must be maintained. d. Lactating women can go back to their prepregnant calorie intake.

b. Caffeine consumed by the mother accumulates in the infant, who may be unusually active and wakeful.

What activity by an infant would cause a false elevation of the tympanic temperature? a. Having a bowel movement b. Crying vigorously c. Having just eaten d. Having been in a cold room

b. Crying vigorously Crying increases the temperature; eating and bowel movements do not. A cold room would lower the temperature. p. 939

A client in active labor complains of cramps in her leg. What intervention should the nurse implement? a. Ask the client if she takes a daily calcium tablet b. Extend the leg and dorsiflex the foot c. Lower the leg off the side of the bed d. Elevate the leg above the heart

b. Extend the leg and dorsiflex the foot Dorsiflexing the foot by puching the sole of the foot forward or by stnading (if the client is capable) (B), and putting the heel of the foot on the floor is the best means of relieving leg cramps.

Which maternal behavior is the nurse most likely to see when a new mother receives her infant for the first time? a. She eagerly reaches for the infant, undresses the infants, and examines the infant completely b. Her arms and hands receive the infant and she then traces the infant's profile with her fingertips c. Her arms and hands receive the infant and she then cuddles the infant to her own body d. She eagerly reaches for the infant and then holds the infant close to her own body

b. Her arms and hands receive the infant and she then traces the infant's profile with her fingertips Attachment/bonding theory indicates that most mothers will demonstrate behaviors described in (B) during the first visit with the newborn, which may be at delivery of later.

33. The most important reason for evaluating the pattern of weight gain in pregnancy is to: a. Prevent excessive adipose tissue deposits b. Identify potential nutritional problems or complications of pregnancy c. Assess the need to limit caloric intake in obese women d. Determine cultural influences on the woman's diet

b. Identify potential nutritional problems or complications of pregnancy

16. Maternal nutritional status is an especially significant factor of the many factors that influence the outcome of pregnancy because: a. It is very difficult to adjust because of people's ingrained eating habits. b. It is an important preventive measure for a variety of problems. c. Women love obsessing about their weight and diets. d. A woman's preconception weight becomes irrelevant.

b. It is an important preventive measure for a variety of problems.

A client who is attending antepartum classes asks the nurse why her healthcare provider has prescribed iron tablets. The nurse's response is based on what knowledge? a. Supplementary iron is more efficiently utilized during pregnancy b. It it difficult to consume 18 mg of additional iron by diet alone c. Iron absorption is decreased in the GI tract during pregnancy d. Iron is needed to prevent megaloblastic anemia in the last trimester

b. It is difficult to consume 18 mg of additional iron by diet alone Consuming enough iron-containing foods to facilitate adequate fetal storage of iron and to meet the demands of pregnancy is difficult (B) so iron supplements are often recommended.

A school-aged child is admitted to the hospital with a vaso-occlusive sickle cell crisis. Which measure in his care should be given priority? a) Encouraging him to take deep breaths hourly b) Maintaining a fluid intravenous line c) Beginning active range-of-motion exercises d) Seeing that he ingests a protein-rich diet

b. Maintaining a fluid intravenous line Explanation: Dehydration increases sickling of cells, so maintaining fluid balance is important.

The nurse identifies crepitus when examining the chest of a newborn who was delivered vaginally. Which further assessment should the nurse perform? a. Elicit positive scarf sign on the affected side b. Observe for an asymmetrical Moro (startle) reflex c. Watch for swelling of fingers on the affected side d. Note paralysis of affected extremity and muscles

b. Observe for an asymmetrical Moro (startle) reflex The most common neonatal birth trauma due to vaginal delivery is fracture of the clavicle. Although an infant may be asymptomatic, a fracture clavicle should be suspected is an infant has limited use of the affected arm malposition of the arm, an asymmetric Moro reflex (B), crepitus over the clavicle, focal swelling or tenderness, or cries when the arm is moved.

How should an infant be positioned after a feeding? a. On the stomach b. On the right side c. On the left side d. On the back

b. On the right side After feeding, the infant is positioned on the right side to direct the food into the stomach. p. 958

Where is the typical IV insertion site in an infant younger than 9 months of age? a. Radial vein b. Scalp vein c. Femoral vein d. Brachial vein

b. Scalp vein A superficial scalp vein is the injection site for administering IV medication to infants younger than 9 months of age. p. 967

What is a disadvantage of using a mist tent with a toddler? a. The nurse must remove the restless child. b. The wet bedding and clothing must be changed frequently. c. The mist tent must be opened at least once every hour. d. All objects must be kept outside of the tent.

b. The wet bedding and clothing must be changed frequently. Frequent linen and clothing changes will be necessary because of the heavy humidity in the tent. The nurse can open the tent to soothe the restless child instead of removing the child. The tent does not have to be opened every hour. Toys can be placed inside the tent. p. 964

Why must the pediatric nurse be cautious about medicating infants and young children? a. They are less susceptible to medication effects than adults. b. They are more susceptible to medication effects than adults. c. They are equally susceptible to medication effects as adults. d. They are more susceptible to drug interactions than adults.

b. They are more susceptible to medication effects than adults. Newborns and young children are more susceptible to the toxic effects of certain medications than adults. p. 965

What is the purpose of a mist tent? a. To provide a constant oxygen supply b. To liquefy respiratory secretions c. To aid in lowering temperature d. To improve the infant's hydration

b. To liquefy respiratory secretions The purpose of the mist tent is to liquefy respiratory secretions. A constant oxygen supply can be given by methods other than a mist tent. A mist tent does not lower temperature or improve hydration. p. 963

A 3-year-old female is brought to the ER by her parents and presents with bruising and mucous membrane bleeding from the nose and mouth. The nurse knows that these symptoms are indicative of: a) Hemophilia b) von Willebrand disease c) Chronic iron deficiency anemia d) Disseminated intravascular coagulation

b. von Willebrand disease Explanation: The primary clinical manifestations of von Willebrand disease are bruising and mucous membrane bleeding from the nose, mouth, and gastrointestinal tract; bleeding may be severe and lead to anemia and shock. Deep bleeding into joints and muscles, like that seen in hemophilia, is rare, except with type III von Willebrand disease.

A client who has an autosomal dominant inherited disorder is exploring family planning options and the risk of transmission of the disorder to an infant. The nurse's response should be based on what information? a.Males inherit the disorder with a greater frequency than females. b.Each pregnancy carries a 50% chance of inheriting the disorder. c.The disorder occurs in 25% of pregnancies. d.All children will be carriers of the disorder.

b.Each pregnancy carries a 50% chance of inheriting the disorder.

A primigravida at 40-weeks gestation is receiving oxytocin (Pitocin) to augment labor. Which adverse effect should the nurse monitor for during the infusion of Pitocin? a.Dehydration. b.Hyperstimulation. c. Galactorrhea. d.Fetal tachycardia.

b.Hyperstimulation.

A nurse caring for an infant with bronchiolitis is monitoring for signs of dehydration. The nurse monitors which of the following as the reliable method of determining fluid loss?

1. Intake 2. Output 3. Skin turgor *4. Body weight* *Rationale:* Body weight is the most reliable method of measurement of body fluid loss or gain. One kilogram of weight change represents 1 L of fluid loss or gain. Although options 1, 2, and 3 may be used to determine fluid status, they are not the most reliable determinants.

A nurse employed in the emergency department is collecting data on a 7-year-old child with a fractured arm. The child is hesitant to answer questions that the nurse is asking and consistently looks at the parents in a fearful manner. The nurse suspects physical abuse and continues with the data collection procedures. Which of the following findings would most likely assist in verifying the suspicion?

1. Poor hygiene *2. Bald spots on the scalp* 3. Lacerations in the anal area 4. Swelling of the genitals *Rationale:* Bald spots on the scalp are most likely to be associated with physical abuse. The most likely findings in sexual abuse include difficulty walking or sitting; torn, stained, or bloody underclothing; pain; swelling or itching of the genitals; and bruises, bleeding, or lacerations in the genital or anal area. Poor hygiene may be indicative of physical neglect.

The nurse is counseling a woman who wants to become pregnant. The woman tells the nurse that she has a 36-day menstrual cycle and the first day of her menstrual period was January *. The nurse correctly calculates that the woman's next fertile period is a. January 14-15 b. January 22-23 c. January 30-31 d. February 6-7

c. January 30-31 This woman can expect her next period to begin 36 days from the first day of her last menstrual period - the cycle begins at the first day of the cycle and continues to the first day of the next cycle. Her next period would, therefore, begin on February 13. Ovulation occurs 14 days before the first day of the menstrual period. Therefore, ovulation for this woman would occur January 31 (C).

The nurse is counseling a woman who wants to become pregnant. The woman tells the nurse that she has a 36-day menstrual cycle and the first day of her last menstrual period was January 8. The nurse correctly calculates that the woman's next fertile period is a.January 14-15. b.January 22-23. c.January 30-31. d.February 6-7.

c. January 30-31.

The nurse assesses a client admitted to the labor and delivery unit and obtains the following data: dark red vaginal bleeding, uterus slightly tense between contractions, BP 110/68, FHR 110 beats/minute, cervix 1 cm dilated and uneffaced. Based on these assessment findings, what intervention should the nurse implement? a. Insert an internal fetal monitor b. Assess for cervical changes q1h c. Monitor bleeding from IV sites d. Perform Leopold's maneuvers

c. Monitor bleeding from IV sites Monitoring bleeding from peripheral sites (C) is the priority intervention. This client is presenting with signs of placental abruption. Disseminated intravascular coagulation (DIC) is a complication of placental abruption, characterized by abnormal bleeding.

A client at 30-weeks gestation, complaining of pressure over the pubic area, is admitted for observation. She is contracting irregularly and demonstrates underlying uterine irritability. Vaginal examination reveals that her cervix is closed, thick, and high. Based on these data, which intervention should the nurse implement first? a. Provide oral hydration b. Have a complete blood count (CBC) drawn c. Obtain a specimen for urine analysis d. Place the client on strict bedrest

c. Obtain a specimen for urine analysis Obtaining a urine analysis (C) should be done first because preterm clients with uterine irritability and contractions are often suffering from a urinary tract infection, and this should be ruled out first.

What is one way to enhance the nutrition of the hospitalized toddler? a. Reward with sweets for eating meals b. Discourage participation in noneating activities c. Offer nutritious fluids frequently d. Leave nutritious finger foods out for the child to eat

c. Offer nutritious fluids frequently Using nutritious liquids may satisfy the nutritional needs when a toddler is "too busy" to eat. Toddlers should not be left to eat unsupervised because of the danger of aspiration. Junk food should not be used as rewards. Activities are important and should not be discouraged. p. 964

A client is admitted with the diagnosis of total placenta previa. Which finding is most important for the nurse to report to the healthcare provider immediately? a. Heart rate of 100 beats/minute b. Variable fetal heart rate c. Onset of uterine contractions d. Burning urination

c. Onset of uterine contractions Total (complete) placenta previa involves the placenta covering the entire cerviccal os (opening). The onset of uterine contractions (C) places the client at risk for dilation and placental separation, which causes painless hemorrhaging.

What should be the focus of a practice where the pediatric nurse uses a developmental approach? a. Stimulation of the child to reach expected norms b. Age-centered care plans c. Strengths and abilities of the child d. Characteristics for the particular age

c. Strengths and abilities of the child A developmental approach emphasizes the child's strengths and abilities and considers individuality. It builds on what the child can do instead of focusing on what the child cannot do. p. 936

25. The labor and delivery nurse is preparing a bariatric patient for an elective cesarean birth. Which piece of "specialized" equipment is unnecessary when providing care for this pregnant woman. a. Extra long surgical instruments b. Wide surgical table c. Temporal thermometer d. Increased diameter blood pressure cuff

c. Temporal thermometer

What should be included in the teaching plan for the parents of a 3-year-old child who has been prescribed an opioid analgesic? a. The opioid is likely to cause significant respiratory depression. b. The medicine is prescribed with the knowledge that addiction may occur. c. The opioid is very effective as a pain control method. d. The opioid is only to be given in cases of severe pain.

c. The opioid is very effective as a pain control method. It is an effective type of analgesia. When administered to children, opioid analgesics do not have any greater respiratory depression than when given to an adult, and the risk of addiction is virtually nonexistent in children. p. 953

What is the preferred IM injection site for a 2-year-old? a. Deltoid muscle b. Upper thigh c. Vastus lateralis d. Gluteus

c. Vastus lateralis The preferred site for an IM injection for a 2-year-old is the vastus lateralis. p. 966, Table 30-10

After observing parental behavior that leads the nurse to suspect child abuse, when should the nurse report the abuse? a. If the parent confesses to child abuse b. If the child admits to being abused c. Whenever maltreatment of a child is suspected d. When the type of abuse can be determined

c. Whenever maltreatment of a child is suspected Mandatory reporting of child abuse is required when the health care provider has reason to suspect the child has been abused. p. 948

A newborn, whose mother is HIV positive, is scheduled for follow-up assessments. The nurse knows that the most likely presenting symptom for a pediatric client with AIDS is: a. shortness of breath b. joint pain c. a persistent cold d. organomegaly

c. a persistent cold Respiratory tract infections commonly occur in the pediatric population. However, the child iwth AIDS has a decreased ability to defend the body against these infections and often the presenting symptom of a child with AIDS is a persistent cold (C).

The nurse instructs a laboring client to use accelerated blow breathing. The client begins to complain of tingling finger and dizziness. What action should the nurse take? a. administer o2 by face mask b. notify the HCP for the client's syndrome c. have the client breathe into her cupped hands d. check the client's BP and fetal HR/

c. have the client breathe into her cupped hands.

The nurse should encourage the laboring client to begin pushing when a. there is only an anterior or posterior lip of cervix left b. the client describes the need to have a bowel movement c. the cervix is completely dilated d. the cervix is completely effaced

c. the cervix is completely dilated Pushing begins with the second stage of labor, i.e., when the cervix is completely dilated (A, B, and D), the cervix can become edematous and may never completely dilate, necessitating an operative delivery. Many primigravidas begin active labor 100% effaced and then proceed to dilate.

A 24-hour-old newborn has a pink papular rash with vesicles superimposed on the thorax, back, and abdomen. What action should the nurse implement? a.Notify the healthcare provider. b.Move the newborn to an isolation nursery. c.Document the finding in the infant's record. d.Obtain a culture of the vesicles.

c.Document the finding in the infant's record.

The nurse is teaching care of the newborn to a group of prospective parents and describes the need for administering antibiotic ointment into the eyes of the newborn. Which infectious organism will this treatment prevent from harming the infant? a.Herpes. b.Staphylococcus c.Gonorrhea. d. Syphilis.

c.Gonorrhea.

A client is admitted with the diagnosis of total placenta previa. Which finding is most important for the nurse to report to the healthcare provider immediately? a.Heart rate of 100 beats/minute. b.Variable fetal heart rate. c.Onset of uterine contractions. d.Burning on urination.

c.Onset of uterine contractions.

The nurse observes a new mother is rooming-in and caring for her newborn infant. Which observation indicates the need for further teaching? a.Cuddles the baby close to her. b.Rocks and soothes the infant in her arms. c.Places the infant prone in the bassinet. d. Wraps the baby in a warm blanket after bathing.

c.Places the infant prone in the bassinet.

11. A woman has come to the clinic for preconception counseling because she wants to start trying to get pregnant in 3 months. She can expect the following advice: a. "Discontinue all contraception now." b. "Lose weight so that you can gain more during pregnancy." c. "You may take any medications you have been taking regularly." d. "Make sure that you include adequate folic acid in your diet."

d. "Make sure that you include adequate folic acid in your diet."

A pregnant woman comes to the prenatal clinic for an initial visit. In reviewing her childbearing history, the client indicated that she has delivered premature twins, one full-term baby, and has had no abortions. Which GTPAL should the nurse document in this client's record? a. 3-1-2-0-3 b. 4-1-2-0-3 c. 2-1-2-1-2 d. 3-1-1-0-3

d. 3-1-1-0-3 (D) describes the correct GTPAL. The client has been pregnant 3 times including the current pregnancy (G-3). She had one full-term infant (T-1). She also had a preterm (P-1) twin pregnancy (a multifetal gestation is considered one birth when calculating parity). There were no abortions (A-0), so this client has a total of 3 living children.

You are assessing children in an ambulatory clinic. Which child would be most likely to have iron-deficiency anemia? a) A 7-month-old boy who has started table food b) A 3-month-old boy who is totally breastfed c) An 8-year-old girl who carries her lunch to school d) A 15-year-old girl who has heavy menstrual periods

d. A 15-year-old girl who has heavy menstrual periods Explanation: Adolescents with heavy menstrual flows lose enough blood each month to cause iron-deficiency anemia.

When measuring the head circumference of an infant, where should the nurse place the tape measure? a. Across the eyebrows and around the occipital lobe b. Over the zygomatic arches and around the parietal areas c. Around forehead and around the crown of the head d. Above the eyebrows and pinnas, and around the occipital lobe

d. Above the eyebrows and pinnas, and around the occipital lobe Head circumference is measured in children up to 36 months above the eyebrows and pinnas, and around the occipital lobe. p. 937

The nurse is caring for a 10-year-old boy with hemophilia. He asks the nurse for suggestions about appropriate physical activities. Which activity would the nurse most likely recommend? a) Wrestling b) Football c) Soccer d) Basebal

d. Baseball Explanation: Children with hemophilia should stay active. Good physical activities would be swimming, baseball, basketball, and bicycling (with a helmet). He would still need to be careful about falls and sliding into base. Intense contact sports like football, wrestling, and soccer should be avoided

One hour after giving birth to an 8-pound infant, a client's lochia rubra has increased from small to large and her fundus is boggy despite massage. The client's pulse is 84 beats/minute and blood pressure is 156/96. The healthcare provider prescribes Methergine 0.2 mg IM x 1. What action should the nurse take immediately? a. Give the medication as prescribed and monitor for efficacy b. Encourage the client to breastfeed rather than bottle feed c. Have the client empty her bladder and massage the fundus d. Call the healthcare provider to question the prescription

d. Call the healthcare provider to question the prescription Methergine is contraindicated for clients with elevated blood pressure, so the nurse should contact the healthcare provider and question the prescription (D).

Which assessment finding should the nursery nurse report to the pediatric healthcare provider? a. Blood glucose level of 45 mg/dl b. Blood pressure of 82/45 mmHg c. Non-bulging anterior fontanel d. Central cyanosis when crying

d. Central cyanosis when crying An infant who demonstrates central cyanosis when crying (D) is manifesting poor adaptation to extrauterine life which should be reported to the healthcare provider for determination of a possible underlying cardiovascular problem.

A client at 32-weeks gestation comes to the prenatal clinic with complaints of pedal edema, dyspnea, fatigue, and a moist cough. Which question is most important for the nurse to ask this client? a. Which symptom did you experience first? b. Are you eating large amounts of salty foods? c. Have you visited a foreign country recently? d. Do you have a history of rheumatic fever?

d. Do you have a history of rheumatic fever? Clients with a history of rheumatic fever (D) may develop mitral valve prolapse, which increases the risk for cardiac decompensation due to the increased blood volume that occurs during pregnancy, so obtaining information about the client's health history is priority.

28. Nutrition is one of the most significant factors influencing the outcome of a pregnancy. It is an alterable and important preventive measure for various potential problems, such as low birth weight and prematurity. While completing the physical assessment of the pregnant client, the nurse can evaluate the client's nutritional status by observing a number of physical signs. Which sign would indicate that the client has unmet nutritional needs? a. Normal heart rate, rhythm, and blood pressure b. Bright, clear, shiny eyes c. Alert, responsive, and good endurance d. Edema, tender calves, and tingling

d. Edema, tender calves, and tingling

31. A pregnant woman's diet may not meet her need for folates. A good source of this nutrient is: a. Chicken b. Cheese c. Potatoes d. Green leafy vegetables

d. Green leafy vegetables

36. To determine the cultural influence on a patient's diet, the nurse should first: a. Evaluate the patient's weight gain during pregnancy b. Assess the socioeconomic status of the patient c. Discuss the four food groups with the patient d. Identify the food preferences and methods of food preparation common to that culture

d. Identify the food preferences and methods of food preparation common to that culture

What observation in an emergency department should lead a nurse to suspect child abuse in a child with a fractured arm? a. Lack of parental concern for the severity of the injury b. The child not answering questions concerning the injury c. Parents not asking about the child's condition d. Inconsistency between the injury and the parents' explanation of it

d. Inconsistency between the injury and the parents' explanation of it Special attention must be paid to injuries that are inconsistent with the parents' explanation. p. 948

Why does obtaining the respirations of an infant require a modified approach from that of an adult? a. Infants breathe through their noses b. Infants have very rapid respirations c. Infants' respirations are thoracic in nature d. Infants' respiratory movements are abdominal

d. Infants' respiratory movements are abdominal In children under 6 or 7 years of age, respiratory movements are abdominal or diaphragmatic. Abdominal movements must be observed when counting respirations. p. 940

The child with Thalassemia may be given which of the following classifications of medications to prevent one of the complications frequently seen with the treatment of this disorder. a) Potassium supplements b) Vitamin supplements c) Factor VIII preparations d) Iron-chelating drugs

d. Iron-chelating drugs Explanation: Frequent transfusions can lead to complications and additional concerns for the child, including the possibility of iron overload. For these children, iron-chelating drugs such as deferoxamine mesylate (Desferal) may be given. Vitamin and potassium supplements would not be given to treat the iron overload. Factor VIII preparations are given to the child with hemophilia.

A client who is in the second trimester of pregnancy tells the nurse that she wants to use herbal therapy. Which response is best for the nurse to provide? a. Herbs are a corner stone of good health to include in your treatment b. Touch is also therapeutic in relieving discomfort and anxiety c. Your healthcare provider should direct treatment options for herbal therapy d. It is important that you want to take part in your care

d. It is important that you want to take part in your care The emphasis of alternative and complementary therapies, such as herbal therapy, is that the client is viewed as a whole being, capable of decision-making and an integral part of the health care team, so (D) recognizes the client's request.

To prevent further sickle cell crisis, you would advise the parents of a child with sickle cell anemia to a) encourage the child to participate in school activities, such as long-distance running. b) prevent the child from drinking an excess amount of fluids per day. c) notify a health care provider if the child develops an upper respiratory infection. d) administer an iron supplement daily

notify a health care provider if the child develops an upper respiratory infection. Explanation: Reduction of oxygen and dehydration lead to increased sickling of cells. Early prevention of these with respiratory illness is important

The nurse is educating a new mother about infectious diseases. The mother demonstrates understanding of teaching by identifying which disorders as being contagious? (Select all that apply) 1.) Impetigo 2.) S. aureus infection 3.) Infantile eczema (atopic dermatitis) 4.) Pediculosis 5.) Cystic Fibrosis

1.) Impetigo 2.) S. aureus infection 4.) Pediculosis Pg. 1042-1049 -Foundations of Nursing

A client who gave birth to a healthy 8 pound infant 3 hours ago is admitted to the postpartum unit. Which nursing plan is best in assisting this mother to bond with her newborn infant? a. Encourage the mother to provide total care for her infant b. Provide privacy so the mother can develop a relationship with the infant c. Encourage the father to provide most of the infant's care during hospitalization d. Meet the mother's physical needs and demonstrate warmth toward the infant

d. Meet the mother's physical needs and demonstrate warmth toward the infant It is most important to meet the mother's requirement for attention to her needs so that she can begin infant care-taking (D). Nurse theorist Reva Rubin describes the initial postpartal period as the "taking-in phase," which is characterized by maternal reliance on others to satisfy the needs for comfort, rest, nourishment, and closeness to families and the newborn.

What is the correct order for assessing vital signs in an infant to ensure the accuracy of measurements? a. Respiration, temperature, pulse b. Pulse, respiration, temperature c. Temperature, pulse, respiration d. Respiration, pulse, temperature

d. Respiration, pulse, temperature The respiration is taken first on an infant before the child is disturbed, pulses are assessed next, and last the temperature is obtained. p. 939

The nurse is calculating the estimated date of confinement (EDC) using Nagele's rule for a client whose last menstrual period started on December 1. Which date is most accurate? a. August 1 b. August 10 c. September 3 d. September 8

d. September 8 Calculation of a client's EDC provides baseline data to monitor fetal gestation. Nagele's rule uses the formula: subtract 3 months and add 7 days to the first day of the last normal menstrual period, so December 1 minus 3 months + 7 days is September 8 (D).

A woman who had a miscarriage 6 months ago becomes pregnant. Which instruction is most important is most important for the nurse to provide this client? a. Elevate lower legs while resting b. Increase caloric intake by 200 to 300 calories per day c. Increase water intake to 8 full glasses per day d. Take prescribed multivitamin and mineral supplements

d. Take prescribed multivitamin and mineral supplements A client who has had a spontaneous abortion or still birth in the last 1.5 years should take multivitamin and mineral supplements (D) and maintain a balanced diet because the previous pregnancy may have left her nutritionally depleted.

1. A 22-year-old woman pregnant with a single fetus has a preconception body mass index (BMI) of 24. When she was seen in the clinic at 14 weeks of gestation, she had gained 1.8 kg (4 lb) since conception. How would the nurse interpret this? a. This weight gain indicates possible gestational hypertension. b. This weight gain indicates that the woman's infant is at risk for intrauterine growth restriction (IUGR). c. This weight gain cannot be evaluated until the woman has been observed for several more weeks. d. The woman's weight gain is appropriate for this stage of pregnancy.

d. The woman's weight gain is appropriate for this stage of pregnancy.

To prevent further sickle cell crisis, you would advise the parents of a child with sickle cell anemia to a) encourage the child to participate in school activities, such as long-distance running. b) administer an iron supplement daily. c) prevent the child from drinking an excess amount of fluids per day. d) notify a health care provider if the child develops an upper respiratory infection.

d. notify a health care provider if the child develops an upper respiratory infection. Explanation: Reduction of oxygen and dehydration lead to increased sickling of cells. Early prevention of these with respiratory illness is important

When discussing growth and development with the parents of a child, the nurse explains that nutrition is the single most important influence on: a. cognitive development. b. secondary sexual characteristics. c. the production of blood cells. d. the growth of bones and muscle.

d. the growth of bones and muscle. Nutrition is probably the single most important influence on growth. p. 945

A primigravida client who is 5 cm dilated, 90% effaced, and at 0 station is requesting an epidural for pain relief. Which assessment finding is most important for the nurse to report to the healthcare provider? a.Cervical dilation of 5 cm with 90% effacement. b.White blood cell count of 12,000/mm3. c.Hemoglobin of 12 mg/dl and hematocrit of 38%. d.A platelet count of 67,000/mm3.

d.A platelet count of 67,000/mm3.

Which assessment finding should the nursery nurse report to the pediatric healthcare provider? a.Blood glucose level of 45 mg/dl. b. Blood pressure of 82/45 mmHg. c.Non-bulging anterior fontanel. d.Central cyanosis when crying.

d.Central cyanosis when crying.

What action should the nurse implement to decrease the client's risk for hemorrhage after a cesarean section? a.Monitor urinary output via an indwelling catheter. b.Assess the abdominal dressings for drainage. c.Give the Ringer's Lactated infusion at 125 ml/hr. d.Check the firmness of the uterus every 15 minutes.

d.Check the firmness of the uterus every 15 minutes.

the nurse is planning preconception care for a new female client. Which information should the nurse provide the client? a.Discuss various contraceptive methods to use until pregnancy is desired. b.Provide written or verbal information about prenatal care. c.Ask the client about risk factors associated with complications of pregnancy. d.Encourage healthy lifestyles for families desiring pregnancy.

d.Encourage healthy lifestyles for families desiring pregnancy.

A client who is in the second trimester of pregnancy tells the nurse that she wants to use herbal therapy. Which response is best for the nurse to provide? a.Herbs are a cornerstone of good health to include in your treatment. b.Touch is also therapeutic in relieving discomfort and anxiety. c.Your healthcare provider should direct treatment options for herbal therapy. d.It is important that you want to take part in your care.

d.It is important that you want to take part in your care.

A 30-year-old multiparous woman who has a 3-year-old boy and an newborn girl tells the nurse, "My son is so jealous of my daughter, I don't know how I'll ever manage both children when I get home." How should the nurse respond? a.Tell the older child that he is a big boy now and should love his new sister. b.Ask friends and relatives not to bring gifts to the older sibling because you do not want to spoil him. c.Let the older child stay with his grandparents for the first six weeks to allow him to adjust to the newborn. d.Regression in behaviors in the older child is a typical reaction so he needs attention at this time.

d.Regression in behaviors in the older child is a typical reaction so he needs attention at this time.

The nurse is calculating the estimated date of confinement (EDC) using Nägele's rule for a client whose last menstrual period started on December 1. Which date is most accurate? a.August 1. b.August 10. c.September 3. d.September 8.

d.September 8.

While breastfeeding, a new mother strokes the top of her baby's head and asks the nurse about the baby's swollen scalp. The nurse responds that the swelling is caput succedaneum. Which additional information should the nurse provide this new mother? a.The infant should be positioned to reduce the swelling. b.The swelling is a subperiosteal collection of blood. c.The pediatrician will aspirate the blood if it gets larger. d.The scalp edema will subside in a few days after birth.

d.The scalp edema will subside in a few days after birth.

A child with hypoplastic anemia develops hemosiderosis. The therapy for this is a) ferrous sulfate. b) prednisone. c) deferoxamine. d) aspirin.

deferoxamine. Explanation: Hemosiderosis is deposition of iron into tissue. A chelating agent, such as deferoxamine, removes it from tissue.

The mother of a child with diabetes asks the nurse in charge of the family-centered pediatric unit if she might see her child's laboratory reports. What response by the nurse is the most appropriate? a. "Although the actual reports are not shared, I can tell you the blood sugar is 200 mg." b. "I'll write them down for you and bring them to your room." c. "Come to the conference room where we can have privacy while you look at them." d. "I'll notify the physician that you wish to see the reports."

c. "Come to the conference room where we can have privacy while you look at them." With a family-centered care approach, hospitals welcome parents, and parents have access to information 24 hours a day. p. 934

What should be done before initiating a gavage feeding? a. Hold the feeding tube under water to check for bubbling b. Check for gastric distention c. Aspirate stomach contents d. Ensure the sterility of feeding equipment

c. Aspirate stomach contents Aspirating stomach contents and aspirating a small amount of air while listening for stomach gurgling are the best ways to ensure correct tube placement. Holding the feeding tube under water to check for bubbling is not an effective method to check tube placement. Gastric distention would be important following the feeding. A gavage feeding is not a sterile procedure. p. 959

When the pediatric nurse is attempting to establish a trusting relationship with a child, what is the most important and lasting thing to do? a. Convey respect b. Talk with the child c. Be honest d. Talk with family

c. Be honest To establish a trusting relationship, the most important thing is to be honest. p. 933

2. Which meal would provide the most absorbable iron? a. Toasted cheese sandwich, celery sticks, tomato slices, and a grape drink b. Oatmeal, whole wheat toast, jelly, and low-fat milk c. Black bean soup, wheat crackers, orange sections, and prunes d. Red beans and rice, cornbread, mixed greens, and decaffeinated tea

c. Black bean soup, wheat crackers, orange sections, and prunes

A mother who is breastfeeding her baby receives instructions from the nurse. Which instruction is most effective to prevent nipple soreness? a. Wear a cotton bra b. Increase nursing time gradually c. Correctly place the infant on the breast d. Manually express a small amount of milk before nursing

c. Correctly place the infant on the breast The most common cause of nipple soreness is incorrect positioning (C) of the infant on the breast, e.g., grasping too little of the areola or grasping on the nipple.

The nurse is teaching care of the newborn to a group of prospective parents and describes the need for administering antibiotic ointment into the eyes of the newborn. Which infectious organism will this treatment prevent from harming the infant? a. Herpes b. Staphylococcus c. Gonorrhea d. Syphilis

c. Gonorrhea Erythromycin ointment is instilled into the lower conjunctive of each eye within 2 hours after birth to prevent ophthalmica neonatorum, an infection caused by gonorrhea, and inclusion conjunctivitis, an infection caused by chlamydia (C). The infant may be exposed to these bacteria when passing the birth canal.

When attempting to provide information to the parents of a child undergoing surgery, the nurse notes that the parents appear confused and do not seem to remember what they are being told. What is the most probable cause of the parents' forgetfulness? a. Noisy environment b. Serious nature of surgery c. Increased level of parents' anxiety d. Developmental age of the child

c. Increased level of parents' anxiety Anxiety of the parents may result in confusion and forgetfulness. It is not known if the environment is noisy, if the surgery is serious in nature, or what is the developmental age of the child. p. 956

32. When providing care to the prenatal patient, the nurse understands that pica is defined as: a. Intolerance of milk products b. Iron deficiency anemia c. Ingestion of nonfood substances d. Episodes of anorexia and vomiting

c. Ingestion of nonfood substances

When communicating with parents suspected of child abuse, what should the nurse be sure to do? a. Tell them the law requires reporting of the incident b. Be sympathetic to their needs c. Interact with them in a nonjudgmental manner d. Suggest psychiatric counseling

c. Interact with them in a nonjudgmental manner The nurse should maintain a nonjudgmental attitude toward the parents. The nurse does not have to tell the parents that she is reporting them. The nurse does not have to be sympathetic, she only has to be professional at all times. It is not the place of the nurse to suggest counseling. p. 949

The child has anemia and iron supplements will be administered by his parents at home. Which of the following statements by the child's parents indicates that further education is required? a) "He may develop diarrhea." b) "If the iron is mixed in a drink, then he should drink it with a straw." c) "His urine may look dark." d) "It's better if I give the iron with orange juice." e) "I can give the iron mixed with chocolate milk."

• "I can give the iron mixed with chocolate milk." • "He may develop diarrhea." Explanation: Iron supplements should not be mixed in milk because it reduces absorption. Iron supplements may make the child constipated. All of the other options are correct

Choice Multiple question - Select all answer choices that apply. The nurse is reviewing the laboratory test results of a child with thalassemia. Which result would the nurse expect to find with the hemoglobin electrophoresis? Select all that apply. a) Hemoglobin A2 b) Hemoglobin A c) Hemoglobin F d) Hemoglobin S

• Hemoglobin F • Hemoglobin A2 Explanation: In thalassemia, the hemoglobin electrophoresis would reveal the presence of hemoglobin F and A2 only. Hemoglobin S would be found with sickle cell disease.

The nurse caring for newborns on an obstetrical ward assesses a SGA newborn. What characteristics are typical for this classification of newborn? Select all that apply. a) Diminished muscle tissue b) Tight and moist skin c) Sparse or absent hair d) Narrow skull sutures e) Poor skin turgor f) Increased fatty tissue

• Poor skin turgor • Sparse or absent hair • Diminished muscle tissue Explanation: Characteristics of the SGA newborn include poor skin turgor, loose and dry skin, sparse or absent hair, wide skull sutures caused by inadequate bone growth, and diminished muscle and fatty tissue. Weight, length, and head circumference are below normal expectations as defined on growth charts.

The nurse is counseling a couple who has sought information about conceiving. For teaching purposes, the nurse should know that ovulation usually occurs

two weeks before menstruation

When interviewing parents of an infant wit hypertrophic pyloric stenosis, the nurse expects the parents to report which symptom? 1.) Diarrhea 2.) Projectile vomiting 3.) Poor appetite 4.) Constipation

2.) Projectile vomiting Pg. 1012 -Foundations of Nursing

Which of the following neonates is at highest risk for cold stress syndrome? 1. Infant of diabetic mother 2. Infant with Rh incompatibility 3. Postdates neonate 4. Down syndrome neonate

3. Postdate babies are at high risk for cold stress syndrome because while still in utero they often metabolize the brown adipose tissue for nourishment when the placental function deteriorates

What is the normal postpartum pulse?

50 BPM. Pulse >1000 BPM indicates excessive blood loss or infection.

Medical management of Ventricular Septal Defect (VSD)

50% of these close spontaneously; surgical repair

A client with prior traumatic delivery and history fo D&C may experience miscarriage or preterm. What is the most common cause of miscarriages? A. Incompetent cervix B. Incompetent pelvis C. Incompetent uterus D. Incompetent vagina

A. Clients with prior traumatic delivery, history of D&C, multiple abortions, or daughters of DES motheres may experience miscarriage or preterm labor related to incompetent cervix. The cervix may be surgically repaired prior to pregnancy, or during gestation

An adolescent girl tells the nurse that she is very suicidal. The nurse asks her if she has a specific plan. Asking this should be considered: a. An appropriate part of the assessment. b. Not a critical part of the assessment. c. Suggesting that the adolescent needs a plan. d. Encouraging the adolescent to devise a plan.

ANS: A Routine health assessments of adolescents should include questions that assess the presence of suicidal ideation or intent. Questions such as, "Have you ever developed a plan to hurt yourself or kill yourself" should be part of that assessment. Threats of suicide should always be taken seriously and evaluated. Suggesting that the adolescent needs a plan and encouraging them to devise this plan are inappropriate statements by the nurse.

The nurse is aware that the age at which the posterior fontanelle closes is _____ months. a. 2 to 3 b. 3 to 6 c. 6 to 9 d. 9 to 12

ANS: A The posterior fontanelle closes between 2 and 3 months of age.

In terms of gross motor development, what would the nurse expect a 5-month-old infant to do? Choose all that apply. a. Roll from abdomen to back b. Put feet in mouth when supine c. Roll from back to abdomen d. Sit erect without support e. Move from prone to sitting position f. Adjust posture to reach an object

ANS: A, B Rolling from abdomen to back and placing the feet in the mouth when supine are developmentally appropriate for a 5-month-old infant.

The nurse cautions that children who have unmet hunger needs will likely display which characteristic(s)? Select all that apply. a. Irritability b. Ineffective feeding patterns c. No predictable sleep-wake cycle d. Distrust e. Effective parent bonding

ANS: A, B, C, D Children who experience frequent hunger do not have effective parental bonding. All other options are probable outcomes for a child who has unmet hunger needs.

A parent brings a 6-month-old infant to the pediatric clinic for her well-baby examination. Her birth weight was 8 pounds, 2 ounces. The nurse weighing the infant today would expect her weight to be at least _____ pounds. a. 12 b. 16 c. 20 d. 24

ANS: B Birth weight is usually doubled by 6 months of age.

Which statement accurately describes physical development during the school-age years? a. The child's weight almost triples. b. A child grows an average of 2 inches per year. c. Few physical differences are apparent among children at the end of middle childhood. d. Fat gradually increases, which contributes to the child's heavier appearance.

ANS: B In middle childhood growth in height and weight occur at a slower pace. Between the ages of 6 to 12 years, children grow 2 inches per year. In middle childhood children's weight will almost double; they gain 3 kg/year. At the end of middle childhood girls grow taller and gain more weight than boys. Children take on a slimmer look with longer legs in middle childhood.

A vaginally delivered infant of an HIV positive mother is admitted to the newborn nursery. What intervention should the nurse perform first? A.Bathe the infant with an antimicrobial soap. B.Measure the head and chest circumference. C. Obtain the infant's footprints. D. Administer vitamin K (AquaMEPHYTON).

A.Bathe the infant with an antimicrobial soap.

A pregnant client with mitral stenosis Class III is prescribed complete bedrest. The client asks the nurse, "Why must I stay in bed all the time?" Which response is best for the nurse to provide this client? A.Complete bedrest decreases oxygen needs and demands on the heart muscle tissue. B.We want your baby to be healthy, and this is the only way we can make sure that will happen. C.I know you're upset. Would you like to talk about some things you could do while in bed? D.Labor is difficult and you need to use this time to rest before you have to assume all child-caring duties.

A.Complete bedrest decreases oxygen needs and demands on the heart muscle tissue.

What would the nurse expect of a healthy 3-year-old child? a. Jump rope b. Ride a two-wheel bicycle c. Skip on alternate feet d. Balance on one foot for a few seconds

ANS: D 3-year-olds are able to accomplish the gross motor skill of balancing on one foot. Jumping rope, riding a two-wheel bike, and skipping on alternative feet are gross motor skills of 5-year-old children

When does the third stage of labor end?

After delivery of the placenta

A nurse is caring for a postpartum client who has breast engorgement following breastfeeding. Which instructions should the nurse reinforce regarding relief of breast engorgement?

Allow newborn to nurse for at least 10-15 minutes on each breast

When assessing a client who is at 12-weeks gestation, the nurse recommends that she and her husband consider attending childbirth preparation classes. When is the best time for the couple to attend these classes?

At 30-weeks gestation.

What is the significance of blood in the amniotic fluid?

Bloody amniotic fluid may indicate abruptio placentae or fetal trauma. An unpleasant odor to amniotic fluid is associated with infection.

The nurse explains that by the age of 6 months an iron-rich formula should be offered because the infant has: a. limited ability to produce red blood cells. b. ineffective digestive enzymes. c. exhausted maternal iron stores. d. need of the iron to support dentition.

C

Sperm lives approximately 3 days and eggs live abut 24 hours. Which is the time interval a coulple should avoid unprotected intecourse after the ovulation? A. 24 hours B. 48 hours C. 72 hours D. 128 hours

C. From ovulation to the begging of the next menstrual cycle is usually exactly 14 days. In other words, ovulation occurs 14 days before the next menstrual period

At the examination of a expecting woman, the deceleration patterns are associated with decreased or absent variability and tachycardia. What should be done immediately in this case? A. Position the mother in High Fowlers position B. Position the mother in knee chest position C. Immediate intervention and fetal assessment D. Spontaneous abortion

C. When deceleration patterns (late or variable) are associated with decreased or absent variability and tachycardia, the situation is OMINOUS (potentially disastrous) and requires immediate intervention and fetal assessment

Diagnostic tests used for Congenital heart disease.

Cardiac catheterization, or MRI, angiography, ABG, electrocardiogram, echocardiography, fluoroscopy

The nurse in a women's health clinic is returning client phone calls. Which client would be the priority to call first?

Client who gave birth vaginally a few days ago who states, "They want to hurt my baby."

An 11-year-old male is diagnosed with mild hemophilia. Upon assessment, the nurse documents the following factor level for this category of hemophilia: a) Factor level less than 1% b) Factor level of 1% to 5% c) Factor level of 5% to 50% d) Factor level greater than 50%

Factor level of 5% to 50% Explanation: Mild hemophilia is characterized by a factor level of 5% to 50%. People with mild hemophilia experience prolonged bleeding only when injured. Thus, their condition may not be diagnosed unless they have trauma or surgery.

A client with diabetes delivers a full-term neonate who weights 10 lb, 1 oz (4.6 kg). While caring for this large-for-gestational age (LGA) neonate, the nurse palpates the clavicles for which reason? a) Clavicles are commonly absent in neonates of mothers with diabetes. b) Neonates of mothers with diabetes have brittle bones. c) LGA neonates have glucose deposits on their clavicles. d) One of the neonate's clavicles may have been broken during delivery.

One of the neonate's clavicles may have been broken during delivery. Correct Explanation: Because of the neonate's large size, clavicular fractures are common during delivery. The nurse should assess all LGA neonates for this occurrence. None of the other options are true.

A client is admitted with the diagnosis of total placenta previa. Which finding is most important for the nurse to report to the healthcare provider immediately. A.heart rate of 100 beats min B. variable fetal HR C. Onset of uterine contractions D. Burning on urination

Onset of uterine contractions.

Medical management of Transposition of Great Vessels

Open heart surgery involving switching the great vessels to their proper positions

S/Sx of Coarctation of the Aorta

Hypertension, lower extremity bounding pulses, HF signs, leg cramping on exertion, epistaxis (BP in the arms is 20 mmHg higher than the legs)

A client receiving epidural anesthesia begins to experience nausea and becomes pale and clammy. What intervention should the nurse implement first?

Raise the foot of the bed.

When a safety reminder device (SRD) is used to protect a child, what is a responsibility of the nurse? a. Apply it loosely b. Remove it every 2 hours c. Place it over clothing d. Apply only one type

b. Remove it every 2 hours Any SRD should be removed every 2 hours. p. 960

How does true labor differ from false labor?

True labor produces regular contractions, pelvic pain, progressive fetal dissent, blood show, and progressive cervical effacement and dilation.

What is the luteal phase of the menstrual cycle?

The luteal phase begins with ovulation. Body temperature drops and then rises by 0.5 degrees after ovulation. The corpus luteum is formed from follicle cells that remain in the ovary following ovulation.

What is the postpartum period?

The postpartum period starts immediately after delivery and is usually completed by week 6 after delivery.

While collecting data from pregnant clients in the obstetric clinic, the nurse should alert the health care provider to see which client first?

Third-trimester client with right upper quadrant pain and nausea

26. To help a woman reduce the severity of nausea caused by morning sickness, the nurse might suggest that she: a. Try a tart food or drink such as lemonade or salty foods such as potato chips. b. Drink plenty of fluids early in the day. c. Brush her teeth immediately after eating. d. Never snack before bedtime.

a. Try a tart food or drink such as lemonade or salty foods such as potato chips.

Client teaching is an important part of the maternity nurse's role. Which factor has the greatest influence on successful teaching on the gravid client? a. The client's readiness to learn b. The client's educational background c. The order in which the information is presented d. The extent to which the pregnancy was planned

a. the client's readiness to learn When teaching any client, readiness to learn (A) is the most important criterion. For example, the client with severe morning sickness in the first trimester may not be "ready to learn" about ways to relieve morning sickness.

13. Women with an inadequate weight gain during pregnancy are at higher risk of giving birth to an infant with: a. Spina bifida. b. Intrauterine growth restriction. c. Diabetes mellitus. d. Down syndrome.

b. Intrauterine growth restriction

In von Willebrand's disease, girls exhibit unusually heavy menstrual flow. a) False b) True

b. True Explanation: von Willebrand's disease, an inherited autosomal dominant disorder, affects both sexes and is often referred to as angiohemophilia. Along with a factor VIII defect, there is also an inability of the platelets to aggregate and the blood vessels to constrict to aid in coagulation. Bleeding time is prolonged, with most hemorrhages occurring from mucous membrane sites. Epistaxis is a major problem, because all children tend to rub or pick at their nose as a nervous mechanism. In girls, menstrual flow is unusually heavy and may cause embarrassment from stained clothing.

You care for a 4-year-old with sickle cell anemia. A physical finding you might expect to see in him is a) increased growth of long bones. b) slightly yellow sclerae. c) enlarged mandibular growth. d) depigmented areas on the abdomen

b. slightly yellow sclerae. Explanation: Many children with sickle cell anemia develop mild scleral yellowing from excess bilirubin from breakdown of damaged cells.

tells the nurse that she want to have an uncomplicated pregnancy and a healthy baby. What information should the nurse share with the client? a.Your current dose of Insulin should be maintained throughout your pregnancy. b.Maintain blood sugar levels in a constant range within normal limits during pregnancy. c.The course and outcome of your pregnancy is not an achievable goal with diabetes. d.Expect an increase in insulin dosages by 5 units/week during the first trimester.

b.Maintain blood sugar levels in a constant range within normal limits during pregnancy.

The nurse observes a new mother avoiding eye contact with her newborn. Which action should the nurse take? a.Ask the mother why she won't look at the infant. b.Observe the mother for other attachment behaviors. c.Examine the newborn's eyes for the ability to focus. d.Recognize this as a common reaction in new mothers.

b.Observe the mother for other attachment behaviors.

The nurse clarifies that child abuse and neglect are complicated and preventable problems falling under which broader term? a. Child abandonment b. Child mismanagement c. Child maltreatment d. Child torment

c. Child maltreatment Child maltreatment is a broad term used to describe neglect and abuse of children. p. 947

What is the special category that encompasses children who have congenital abnormalities, malignancies, gastrointestinal (GI) diseases, or central nervous system (CNS) anomalies? a. Very dependent children b. Children requiring special education c. Children with special needs d. Children requiring long-term care

c. Children with special needs The definition of children with special needs includes congenital abnormalities, malignancies, GI diseases, and CNS anomalies. p. 934

4. A pregnant woman's diet consists almost entirely of whole grain breads and cereals, fruits, and vegetables. The nurse would be most concerned about this woman's intake of: a. Calcium. b. Protein. c. Vitamin B12. d. Folic acid.

c. Vitamin B12.

A client who delivered an infant an hour ago tells the nurse that she feels wet underneath her buttock. The nurse notes that both perineal pads are completely saturated and the client is lying in a 6-inch diameter pool of blood. Which action should the nurse implement next? a.Cleanse the perineum. b.Obtain a blood pressure. c.Palpate the firmness of the fundus d. Inspect the perineum for lacerations.

c.Palpate the firmness of the fundus

The initial prenatal laboratory screening results of a client at 12 weeks gestation indicate a rubella titer status of nonimmune. What will the nurse anticipate as the plan of care for this client?

Administer MMR vaccine immediately postpartum

A client with gestational htn is an active labor and receiving an infusion of magnesium sulfate. Which drug should the nurse available for signs of potential toxicity? A. oxytocin B. calcium gluconate C. terbutaline D. naloxone 9

B. calcium gluconate

What are the physiological maternal blood pressure changes of pregnancy?

Blood pressure slightly decreases in the 2nd trimester.

side effects of magnesium sulfate

Depressed reflexes and increased respirations. A flushed, warm feeling and a dry mouth.

A client at 32-weeks gestation comes to the prenatal clinic with complaints of pedal edema, dyspnea, fatigue, and a moist cough. Which question is most important for the nurse to ask this client?

Do you have a history of rheumatic fever? Clients with a history of rheumatic fever (D) may develop mitral valve prolapse, which increases the risk for cardiac decompensation due to the increased blood volume that occurs during pregnancy, so obtaining information about this client's health history is a priority

What is the neonatal breastfeeding schedule?

Every 2 to 3 hours.

A neonate requires respiratory resuscitation. Which is the proper head position of the neonate for rescue breathing?

Explain: The neonate should be placed on the back with the neck slightly extended. This is a neutral or "sniffing" position. A blanket or towel roll can be placed under the shoulders, elevating them 0.75-1.0 in (19-25.4 mm) off the mattress. This is particularly useful if the infant has a large occiput from molding or edema. The nurse must watch that the infant's head does not shift to an improper position during caregiving activities.

Failure of closure of the ductus arteriosus results in?

Failure of the ductus arteriosus to close within the first few weeks of life allows oxygenated blood to shunt from the high-pressure aorta to the low-pressure pulmonary artery, which causes the blood to become deoxygenated -pg. 980 Foundations of Nursing

The primary intervention for beta-thalassemia is a chronic transfusion program of packed white blood cells with iron chelation. a) True b) False

False Explanation: The primary intervention for beta-thalassemia is a chronic transfusion program of packed red blood cells with iron chelation. Such a program facilitates adequate oxygenation of body tissues and practically eliminates all symptoms of thalassemia.

If the nurse manages a new infant with low blood sugar, which of the following would be an appropriate intervention to prevent hypoglycemia? a) Check the heart rate. b) Hold all feedings. c) Feed the infant. d) Give antibiotics.

Feed the infant. Correct Explanation: The infant could be fed early either breast milk or formula to prevent low blood sugar. If unable to feed well, the infant can receive intravenous fluids. The other choices do not raise blood sugar.

The registered nurse is preparing to administer oxytocin to induce labor in a client. The practical nurse assists the registered nurse and recognizes that the oxytocin infusion can lead to which of the following conditions?

Fetal distress and cesarean birth

How long does a full term healthy infant have sufficient iron stores for?

First 4 to 6 months of life

What are the assessment for premature rupture of membranes?

Fluid pooling into a vaginal vault; Nitrazine test is positive. Assess the amount, color, consistency, and odor of fluid. Elevated temperature may indicate the presence of infection. Fetal tachycardia is a sign of infection.

Which of the following would you expect to assess in an infant with hypoglycemia? a) Prolonged jaundice b) Limpness or jitteriness c) Pain along the sixth cranial nerve d) Excessive hunger

Limpness or jitteriness Correct Explanation: Hypoglycemia (reduced glucose serum level) usually presents with jitteriness.

Which nursing intervention is most helpful in relieving postpartum uterine contractions or "afterpains?"

Lying prone with a pillow on the abdomen. Lying prone (A) keeps the fundus contracted and is especially useful with multiparas, who commonly experience afterpains due to lack of uterine tone.

When explaining "postpartum blues" to a client who is 1 day postpartum, which symptoms should the nurse include in the teaching plan? (Select all that apply.) A. Mood swings. B. Panic attacks. C. Tearfulness. D. Decreased need for sleep. E. Disinterest in the infant.

Mood swings. Tearfulness.

20. Which minerals and vitamins usually are recommended to supplement a pregnant woman's diet? a. Fat-soluble vitamins A and D b. Water-soluble vitamins C and B6 c. Iron and folate d. Calcium and zinc

c. Iron and folate

15. Pregnant adolescents are at high risk for _____ because of lower body mass indices (BMIs) and "fad" dieting. a. Obesity b. Diabetes c. Low-birth-weight babies d. High-birth-weight babies

c. Low-birth-weight babies

What was one of the major strides in pediatric care made by Dr. Abraham Jacobi? a. Pediatric wards in hospitals b. Free inoculations against smallpox c. Milk stations in the city of New York d. Serving nutritious foods in orphanages

c. Milk stations in the city of New York Dr. Abraham Jacobi, referred to as the father of pediatrics, initiated the establishment of milk stations in New York demonstrating how to sanitize milk for children. p. 932

Type of Leukemia most common in children?

Acute Lymphoid Leukemia (ALL)

List 3 mixed defects.

(1) Transposition of the Great Vessels (2) Truncus Arteriosus (3) Hypoplastic Left Heart Syndrome

Types of Sickle Cell Crisis

(1) Vasoocclusive crisis (2) Sequestration crisis (3) Aplastic crisis

pyloric stenosis in the infant.

Projectile vomiting and cyanosis.

Medical management for ITP (2)

Short course of corticosteroids, transfusion of packed RBCs, IVGG (intravenous gamma globulin), anti-D antibody, splenectomy

At what time should a woman take her temperature when using the basal body temperature method of contraception?

Every morning upon waking and before activity.

Medical management of iron-deficiency anemia

Iron replacement therapy, nutritional counseling, Tx of underlying condition

Onset age in which leukemia peaks?

It peaks between ages 2 and 6 years. And is more common in boys than it is in girls.

Sequestration Crisis

Pooling of blood in the spleen and the liver, resulting in hepatosplenomegaly; sometimes progresses to CV collapse and death

Medical management of Coarctation of the Aorta

Surgical correction or graft placement

A nurse is preparing to administer epinephrine intravenously to a preterm newborn. The newborn weighs 1,500 g and the physician orders 0.1 mL/kg. How much would the nurse administer? a) 0.25 mL b) 0.20 mL c) 0.15 mL d) 0.1 mL

0.15 mL Correct Explanation: The newborn weighs 1,500 g, which is equivalent to 1.5 kg. Calculating the dose based on 0.1 mL/kg, the nurse would administer 0.15 mL.

A primigravida client who is 5 cm dilated, 90% effaced, and at 0 station is requesting an epidural for pain relief. Which assessment finding is most important for the nurse to report to the healthcare provider?

A platelet count of 67,000/mm3.

When should urinalysis be done during pregnancy?

A urine specimen for glucose and protein determinations should be obtained at every prenatal visit.

When during pregnancy is gonorrhea screening done?

A vaginal test is done during the initial prenatal examination to screen for gonorrhea. The test may be repeated during the third trimester in high-risk clients.

Atrial Septal Defect

Abnormal opening in the atrial septum, enables oxygenated blood to flow from the higher pressure left atrium to the lower pressure right atrium, which causes blood to become deoxygenated

Congenital Heart Disease (CHD)

Abnormality of the heart present at birth

What is the function of amniotic fluid?

Protects the fetus from injury, provides an even temperature for the fetus, and helps dilate the cervix in labor.

Which action should the nurse implement when preparing to measure the fundal height of a pregnant client?

Have the client empty her bladder.

Which vaccine can safely be given in the second or third trimester of pregnancy?

Tetanus-diphtheria-pertussis vaccine

What happens to the cells in Leukemia?

The uncontrollable proliferation of blast cells (immature WBCs), which accumulate in the marrow and cause crowding and depression of other healthy cells (mature WBCs, RBCs, and platelets)

A nurse is caring for a pregnant client who has hyperemesis gravidarum. Which assessment findings should the nurse anticipate? Select all that apply.

- Positive urine ketones (moderate) - Pulse 106/min

What are the assessments for newborns with diabetes?

-Excessive size and weight -Edema or puffiness in the face and cheeks -Signs of hypoglycemia -Twitching/difficulty feeding, lethargy -Hyperbilirubinemia.

The nurse is preparing a presentation for a local parent group about nutritional measures to prevent anemia. The group of parents have children between the ages of 4 and 8 years of age. The nurse would recommend a daily iron intake of which amount? a) 10 mg b) 12 mg c) 6 mg d) 15 mg

10 mg Explanation: The recommended daily dietary iron intake for children 1 to 10 years of age is 10 mg. The recommended daily dietary iron intake for children 0 to 6 months of age is 6 mg. The recommended daily dietary iron intake for boys 11 to 18 years of age is 12 mg. The recommended daily dietary iron intake for girls 11 to 18 years of age is 15 mg.

What percentage of neonates requires some type of assistance to transition to extrauterine life? a) 25% b) 10% c) 50% d) 5%

10% Correct Explanation: Most newborns transition to extrauterine life smoothly. About 10% of newborns need some type of assistance at birth.

A baby is born with erythroblastosis fetalis. Which of the following signs/symptoms would the nurse expect to see? 1. Ruddy complexion 2. Anasarca 3. Alopecia 4. Erythema toxicum

2. Babies born with erythroblastosis fetalis often ore in severe congestive heart failure and, therfore, exhibit anasarca

New technique surgeons use for ductus arteriosus

3 small incision are made on the left side of the chest, and then a thoracoscope and instruments are used to place a clip on the ductus arteriosus

A baby is grunting in the neonatal nursery. Which of the following actions by the nurse is appropriate? 1. Place pacifier in the baby's mouth 2. Check the babies diaper 3. Have the mother feed the baby 4. Assess the respiratory rate

4. Grunting is often accompanied by tachypnea, another sign of respiratory distress

Which assessment below would increase your suspicion that iron-deficiency anemia may be present in a child? a) A 15-year-old girl constantly sucks ice cubes b) An 8-year-old girl is shy and does not participate in class c) A 3-month-old boy sucks his thumb d) A 7-month old boy does not say whole words yet

A 15-year-old girl constantly sucks ice cubes Explanation: Iron-deficiency anemia is associated with pica, or the eating of nonfood substances

The nurse is preparing a client with a term pregnancy who is in active labor for an amniotomy. What equipment should the nurse have available at the client's bedside?

A sterile glove An amnihook. Lubricant

39. Most women with uncomplicated pregnancies can use the nurse as their primary source for nutritional information. The nurse or midwife should refer a client to a registered dietitian for in-depth nutritional counseling in the following situations (Select all that apply). a. Preexisting or gestational illness such as diabetes b. Ethnic or cultural food patterns c. Obesity d. Vegetarian diet e. Allergy to tree nuts

A, B, C, D

Corticosteroids are used shor term in low doses suring exacerbations. What side effect do corticosteroids have on long term? A. Adverse effects on growth B. Adverse effects on bone structure C. Hypoglycemia D. Hypocalcemia

A. Corticosteroids are used short term in low doses during exacerbations. Long term use is avoided due to side effects and their adverse effect on growth

Cord abnormalities usually indicate cardivascular or renal anomalies. What happens if fetal structures of foramen ovale, ductus arteriosus and ductus venous do no close postnatal? A. Cardia pulmonary compromise B. Renal compromise C. Gastro intestinal compromise D. Neurological compromise

A. Postnatally, the fetal structures of foramen ovale, ductus arteriosus and ductus venosus shoul close. If they do not, cardiac and pulmonary compromise will develop

The nurse is teaching a woman how to use her basal body temperature (BBT) pattern as a tool to assist her in conceiving a child. Which temperature pattern indicates the occurrence of ovulation, and therefore, the best time for intercourse to ensure conception? A. Between the time the temperature falls and rises. B. Between 36 and 48 hours after the temperature rises. C. When the temperature falls and remains low for 36 hours. D. Within 72 hours before the temperature falls.

A. Between the time the temperature falls and rises.

The nurse explains that the second process of self-mobility an infant learns is seen at the age of 9 months, when the infant begins to ___________.

ANS: creep At 7 months the infant begins to crawl, using arms and dragging trunk and legs. At 9 months the infant begins to creep, holding his or her trunk above the floor. The next self-mobility activity is cruising, where the child walks from one piece of furniture to the next before it begins to walk independently.

In terms of cognitive development the 5-year-old child would be expected to: a. Use magical thinking. b. Think abstractly. c. Understand conservation of matter. d. Be unable to comprehend another person's perspective.

ANS: A Magical thinking is believing that thoughts can cause events. Abstract thought does not develop until school-age years. The concept of conservation is the cognitive task of school-age children ages 5 to 7 years. Five-year-olds cannot understand another's perspective.

The parent of a 4-year-old son tells the nurse that the child believes "monsters and boogeyman" are in his bedroom at night. The nurse's best suggestion for coping with this problem is to: a. Insist that the child sleep with his parents until the fearful phase passes. b. Suggest involving the child to find a practical solution such as a night light. c. Help the child understand that these fears are illogical. d. Tell the child frequently that monsters and boogeyman do not exist.

ANS: B A night light shows a child that imaginary creatures do not lurk in the darkness. Letting the child sleep with parents will not get rid of the fears. A 4-year-old is in the preconceptual age and cannot understand logical thought.

The nurse knows that an infant's birth weight should be tripled by: a. 9 months. b. 1 year. c. 18 months. d. 2 years.

ANS: B The infant usually triples his or her birth weight by about 12 months of age.

According to Erikson, the psychosocial task of adolescence is developing: a. Intimacy. c. Initiative. b. Identity. d. Independence.

ANS: B Traditional psychosocial theory holds that the developmental crises of adolescence lead to the formation of a sense of identity. Intimacy is the developmental stage for early adulthood. Initiative is the developmental stage for early childhood. Independence is not one of Erikson's developmental stages.

The nurse is aware that the earliest age at which an infant is able to sit steadily alone is _____ months. a. 4 b. 5 c. 8 d. 15

ANS: C The infant can sit alone without support at about 8 months of age.

The abnormal finding in an evaluation of growth and development for a 6-month-old infant would be: a. weight gain of 4 to 7 ounces per week. b. length increase of 1 inch in 2 months. c. head lag present. d. can sit alone for a few seconds.

ANS: C The infant should be holding the head up well by 5 months of age. If head lag is present at 6 months, the child should undergo further evaluation.

The first day of a womans last normal menstrual period was October 17. By using Nagele's rule, what is the EDB? A. July 10 B. July 24 C. June 10 D. June 24

B. If the first day of a womans last normal menstrual period was October 17, her EDB using Nagele's rule is July 24. Count back 3 months and add 7 days ( Always give February 28 days)

A full term infant is transferred to the nursery from labor and delivery. Which information is most important for the nurse to receive when planning immediate care for the newborn? a. Length of labor and method of delivery b. Infant's condition at birth and treatment received. C. Feeding method chosen by the parents. D. History of drugs given to the mother during labor.

B. Infant's condition at birth and treatment received.

30-year-old gravida 2, para 1 client is admitted to the hospital at 26-weeks' gestation in preterm labor. She is given a dose of terbutaline sulfate (Brethine) 0.25 mg subcutaneous. Which assessment is the highest priority for the nurse to monitor during the administration of this drug? A. Maternal blood pressure and respirations. B.Maternal and fetal heart rates. C.Hourly urinary output. D.Deep tendon reflexes.

B.Maternal and fetal heart rates.

Idiopathic Thrombocytopenia Purpura (ITP)

Characterized by a marked decrease in the amount of circulating platelets with resultant bleeding beneath the skin

A mother who is breastfeeding her baby receives instructions from the nurse. Which instruction is most effective to prevent nipple soreness?

Correctly place the infant on the breast.

What is fetal expulsion?

Expulsion is the birth of the body of the infant.

Most prevalent blood disorder in infancy and early childhood.

Iron-Deficiency Anemia

What factor is deficient in Hemophilia?

Is caused by a deficiency in factor VIII, which is a necessary component of blood coagulation

Just after delivery, a new mother tells the nurse, "I was unsuccessful breastfeeding my first child, but I would like to try with this baby." Which intervention is best for the nurse to implement first?

Provide assistance to the mother to begin breastfeeding as soon as possible after delivery.

A woman gives birth to a newborn at 38 weeks' gestation. The nurse classifies this newborn as which of the following? a) Late preterm b) Term c) Preterm d) Postterm

Term Correct Explanation: A term newborn is one born from the first day of the 38th week of gestation through 42 weeks. A postterm newborn is one born after completion of 42 weeks' gestation. A preterm newborn is one born before completion of 37 weeks' gestation. A late preterm newborn is one who is bone between 34 and 36-6/7 weeks' gestation

A client's gestational age is 38 weeks and 6 days. If the baby is born today, which of the following terms accurately describes the gestational age of the newborn? a) Term. b) Premature. c) Postterm. d) Preterm.

Term. Correct Explanation: A term infant is born after the beginning of week 38 and before week 42 of pregnancy. Premature or preterm refers to the birth prior to 37 completed weeks. Postterm refers to birth beyond 42 weeks.

A nurse is caring for a client following a forceps-assisted vaginal birth. The client reports severe vaginal pain and fullness. On assessment the nurse notices a firm, midline uterine fundus. Lochia rubra is light. Which diagnosis should the nurse anticipate?

Vaginal hematoma

A disfiguring facial wound would have the most significant developmental impact on which child? a. 4-year-old b. 6-year-old c. 10-year-old d. 14-year-old

d. 14-year-old The adolescent fears a change in body image associated with surgery. p. 955, Table 30-8

10. A woman in week 34 of pregnancy reports that she is very uncomfortable because of heartburn. The nurse would suggest that the woman: a. Substitute other calcium sources for milk in her diet. b. Lie down after each meal. c. Reduce the amount of fiber she consumes. d. Eat five small meals daily.

d. Eat five small meals daily.

The parents ask about preparation of their toddler for hospital admission. When does the nurse suggest that the parents tell their toddler of the admission? a. A week prior b. 2 weeks prior c. The day of admission d. Only two or three days before

d. Only two or three days before The nurse should suggest the toddler be told only days before. School-age children can be given more time to prepare. Adolescents should be told as far in advance as possible. p. 951

21. Which vitamins or minerals can lead to congenital malformations of the fetus if taken in excess by the mother? a. Zinc b. Vitamin D c. Folic acid d. Vitamin A

d. Vitamin A

The nurse is returning the results of a urine pregnancy test to a client currently taking several medications. Which of the following prescriptions are contraindicated in pregnancy? Select all that apply.

- Doxycycline - Isotretinoin - Lisinopril

A couple is excited about finding out the sex of their baby during ultrasound at 14 weeks gestation. What is the nurse's best response?

"If the baby is in the right position, the genitalia may be visualized."

The umbilical cord is being clamped by the obstetrician.. Which of the following physiological changes is taking place at this time? 1. The babys' blood bypasses its pulmonary system 2. The babys oxygen level begins to drop 3. Bacteria begin to invade the babys bowel 4. Bilirubin rises in the babys bloodstream

2. This is the correct answer. When the cord is clamped, the blood is no longer being oxygenated through the placenta. The babys oxygen levels, therefore, begin to drop

A nurse has just recieved report on 4 neonates in the newborn nursery. Which of the babies should the nurse assess first? 1. Neonate whose mother is HIV positive 2. Neonates whose mother is group B streptococcus positive 3. Neonates whose mother's labor was 12 hours long 4. Neonates whose mother gained 45 pounds during her pregnancy

2. This is the correct response. Babies who are born to mothers who are GBS positive are at high risk for sepsis. The incidence of sepsis is reduced, however, when the mother receives IV antibiotics during labor

Assessment of a newborn at 40 weeks' gestation reveals that he is a low birth weight newborn. Which of the following weights would the nurse identify as being low birth weight? a) 4400 g b) 1400 g c) 2400 g d) 3400 g

2400 g Correct Explanation: A birth weight of less than 2500 g is categorized as a low birth weight in infants. The normal birth weight of term infants ranges from 3000-4000 g. Hence infants with a birth weight of 3500 g or 4500 g will not be categorized as low birth weight infants. Infants having birth weights lower than 1500g are termed as very low birth weight infants, and not merely low birth weight.

The practical nurse is collecting data on several clients in the antepartum unit. Which client should the practical nurse report to the registered nurse for further assessment?

25 weeks gestation, hemoglobin is 9 g/dL

Average expected weight gain during pregnancy?

25-35 lbs

The HCP has diagnosed depression in a 16-year-old patient. Which statement by the parents indicates that they have a proper understanding of the diagnosis? 1.) "My child will need to be placed in an inpatient mental health facility." 2.) "My child will have to take antidepressive medications or life." 3.) "The recovery process for my child is likely to be a slow, lengthy process." 4.) "Depression is nothing to worry about in a child of this age."

3.) "The recovery process for my child is likely to be a slow, lengthy process." Pg. 1061 -Foundations of Nursing

In terms of gross motor development, what would the nurse expect a 5-month-old infant to do? a. Roll from abdomen to back. b. Roll from back to abdomen. c. Sit erect without support. d. Move from prone to sitting position.

ANS: A Rolling from abdomen to back is developmentally appropriate for a 5-month-old infant. The ability to roll from back to abdomen usually occurs at 6 months old. Sitting erect without support is a developmental milestone usually achieved by 8 months. The 10-month-old infant can usually move from a prone to a sitting position.

The parents of a 2-year-old tell the nurse that they are concerned because the toddler has started to use "baby talk" since the arrival of their new baby. The nurse should recommend that the parents: a. Ignore the "baby talk." b. Explain to the toddler that "baby talk" is for babies. c. Tell the toddler frequently, "You are a big kid now." d. Encourage the toddler to practice more advanced patterns of speech.

ANS: A The baby talk is a sign of regression in the toddler. It should be ignored, while praising the child for developmentally appropriate behaviors. Regression is children's way of saying that they are expressing stress. The parents should not introduce new expectations and should allow the child to master the developmental tasks without criticism.

The total bilirubin level of a 36-hour, breastfeeding newborn is 14 mg/dl. Based on this finding, which intervention should the nurse implement? A.Provide phototherapy for 30 minutes q8h. B. Feed the newborn sterile water hourly. C.Encourage the mother to breastfeed frequently. D.Assess the newborn's blood glucose level.

C.Encourage the mother to breastfeed frequently.

5. A pregnant woman experiencing nausea and vomiting should: a. Drink a glass of water with a fat-free carbohydrate before getting out of bed in the morning. b. Eat small, frequent meals (every 2 to 3 hours). c. Increase her intake of high-fat foods to keep the stomach full and coated. d. Limit fluid intake throughout the day.

b. Eat small, frequent meals (every 2 to 3 hours).

Causes of Aplastic Anemia

Arises as a result of neoplastic disease of the bone marrow, or more commonly from destruction of the bone marrow by exposure to toxic chemicals, ionizing radiation, or certain antibiotics or other medications

What produces the clinical manifestations of acute leukemia?

As nonfunctional blast cells infiltrate the lymph nodes, the liver, the kidneys, and the spleen, they cause these organs to enlarge, which produces the clinical manifestations of acute leukemia

Fetal well being is determined by assessing fundal height, fetal heart tones/rate, fetal movement and uterine activity (contractions). What do changes in fetal heart rate indicate? A. Leukorrhea B. Compromised blood flow to the fetus C. Fluid discharge from vagina D. Change in fetal movement

B. Changes in fetal heart rate are the first and most important indicator of compromised blood flow to the fetus, and these changes require action

The menstrual phase varies in length for most women. How many days usually are from ovulation to the beginning of the next menstrual cycle? A. 12 days B. 14 days C. 16 days D. 18 days

B. From ovulation to the beginning of the next menstrual cycle is usually exactly 14 days. In other words, ovulation occurs 14 days before the next menstrual period

An infant with hypothyroidism is often described as a "good, quite baby" by the parents. What early disease detection is essential in preventing mental retardation in infants? A. Hyperthyroidism B. Phenylketonuria C. Diabetes Mellitus D. Ketoacidosis

B. Early detection of hypothyroidism and phenylketonuria is essential in preventing mental retardation in infants. Knowledge of normal groth and development is important, sin a lack of attaintment can be used to detect the existence of these metabolic/endocrine disorders and attainment can be used for evaluating the treatments effecct

The screening for neural tube defects is highly associated woth both false positives and false negatives. Through what does the screening for neural tube defects in some states? A. Spinal Bifida B. Maternal serum AFP levels C. MSAFP D. distribution curves of maternal serum APP

B. In some states, the screening for neural tube defects through either maternal seru AFP levels or amniotic fluid AFP leves is mandated by state law. This screening test is highly associated with both false positives and false negatives

Women with an inadequate weight gain during pregnancy are at higher risk of giving birth to an infant with: A. spina bifida. B. intrauterine growth restriction. Correct C. diabetes mellitus. D. Down syndrome.

B. intrauterine growth restriction. Spina bifida is not associated with inadequate maternal weight gain. An adequate amount of folic acid has been shown to reduce the incidence of this condition. Both normal-weight and underweight women with inadequate weight gain have an increased risk of giving birth to an infant with intrauterine growth restriction. Diabetes mellitus is not related to inadequate weight gain. A gestational diabetic mother is more likely to give birth to a large-for-gestational age infant. Down syndrome is the result of a trisomy 21, not inadequate maternal weight gain.

The nurse should explain to a 30 year old gravid client that alpha fetoprotein testing is recommended for which purpose? A.detect cardiovascular disorders B.screen for neural tube defects c .monitor the placental functioning d. assess for maternal pre-eclampsia

B.screen for neural tube defects

A 22 year old primigravida at 12 weeks gestation has a high Hgb of 9.6 g/dl and a Hct of 31% and she has gained 3 pounds during the first trimester, even if the gain of 3.5 to 5 pounds during the first trimester is recommended. Taking into consideration that the client is anemic, what supplements should be recommended to her? A. Potassium B. Magnesium C. Iron D. Calcium

C. For the anemic pregnant client, supplemental iron and a diet high in iron is needed

At approximately 28 - 32 weeks gestation, the maximum plasma volume increase of 25 - 40% occurs, resulting in normal hemodilution of pregnancy and Hct values of 32 - 42%. What does Hct in reality represent, even if its values may look "good"? A. Pregnancy induced hyperglycemia B. Pregnancy induced hypoglycemia C. Pregnancy induced hypertension D. pregnancy induced hypotension

C. High Hct values may look "good" but in reality represent pregnancy induced hypertension and depleted vascular space

The nurse is teaching breastfeeding to prospective parents in a childbirth education class. Which instruction should the nurse include as content in the class? A.Begin as soon as your baby is born to establish a four-hour feeding schedule. B.Resting helps with milk production. Ask that your baby be fed at night in the nursery. C.Feed your baby every 2 to 3 hours or on demand, whichever comes first. D. Do not allow your baby to nurse any longer than the prescribed number of minutes.

C.Feed your baby every 2 to 3 hours or on demand, whichever comes first.

Pitocin should be given with caution to clients with hypertension. What drug shouldn't be given to clients with hypertension due to its vasoconstrictive action? A. Analgesics B. Meperidine C. Codeine D. Methergine

D. Methergine is NOT given to clients with hypertension due to its vasoconstrictive action. Pitocinis given with caution to those with hypertension

A nurse should teach newbie parents to take both axillary and rectal temperature of the child. How long should the thermometer be held in place if it is done recatally? A. 2 minutes B. 3 minutes C. 4 minutes D. 5 minutes

D. Rectal temperature: the thermometer should be used with blunt end. Insert thermometer 1/4 to 1/2 inch and hold in place for 5 minutes. Hold feet and legs firmly

When assessing a client who is at 12-weeks gestation, the nurse recommends that she and her husband consider attending childbirth preparation classes. When is the best time for the couple to attend these classes? A.At 16-weeks gestation. B.At 20-weeks gestation. C.At 24-weeks gestation. D.At 30-weeks gestation.

D.At 30-weeks gestation.

Question: A nursing instructor describes what happens to the red blood cell after it disintegrates and how bilirubin is formed. Place the events in the order that the instructor would discuss from first to last.

Degradation of heme portion Conversion to protoporphyrin Break down into indirect bilirubin Conversion to direct bilirubin Excretion in bile Explanation: As the heme portion is degraded, it is converted into protoporphyrin. Protoporphyrin is then further broken down into indirect bilirubin. Indirect bilirubin is fat soluble and cannot be excreted by the kidneys in this state. It is therefore converted by the liver enzyme glucuronyl transferase into direct bilirubin, which is water soluble. This is then excreted in bile.

A client asks the nurse what surfactant is. Which explanation would the nurse give as the main role of surfactant in the neonate? a) Helps the lungs remain expanded after the initiation of breathing b) Promotes clearing of mucus from the respiratory tract c) Assists with ciliary body maturation in the upper airways d) Helps maintain a rhythmic breathing pattern

Helps the lungs remain expanded after the initiation of breathing Correct Explanation: Surfactant works by reducing surface tension in the lung, which allows the lung to remain slightly expanded, decreasing the amount of work required for inspiration. Surfactant hasn't been shown to influence ciliary body maturation, clearing of the respiratory tract, or regulation of the neonate's breathing pattern.

A 30-year-old gravida 2, para 1 client is admitted to the hospital at 26- weeks gestation in preterm labor. She is started on an IV of ritodrine hydrochloride (Yutopar). What are the highest priority readings that the nurse should monitor frequently during the administration of this drug?

Maternal and fetal heart rates. Monitoring the maternal and fetal heart rates (B) is most important when ritodrine is being administered. Ritodrine is a sympathomimetic agent that stimulates both beta 1 and beta 2 receptors. Stimulation of beta 1 receptors causes tachycardia (side effect of the drug) and stimulation of beta 2 receptors causes uterine relaxation (desired effect of the drug)

Medical management of atrial septal defect

Minor cases are not treated they are monitored, surgical correction for major cases. Small defects w/ a purse-string suture Large defects w/ a polyester fiber (Dacron) patch

The mother of a 3-year-old expresses concern about her daughter's slowed growth rate. What would be the most informative response by the nurse? a. "Three-year-olds have typically finished a growth spurt, and you may notice a decreased rate in your daughter's growth." b. "Children's growth is hereditary. She may be of small stature like you." c. "The growth of a 3-year-old is associated with their nutrition. How is she eating?" d. "Your daughter is healthy and happy. Don't worry about her growth right now."

a. "Three-year-olds have typically finished a growth spurt, and you may notice a decreased rate in your daughter's growth." Three-year-olds slow down in their growth in a natural cycle. p. 936

When assessing a child for a possible hematologic disorder, which of the following would the nurse need to keep in mind as most important? a) Sequelae are rare with chronic problems. b) A child's nutritional status is key. c) Demographic data is of little relevance. d) Multiple body sites can be affected.

Multiple body sites can be affected. The nurse needs to keep in mind that hematologic alterations can affect multiple body sites, so assessment needs to address all body systems. A child's nutritional status may be helpful in assessing certain hematologic disorders such as iron deficiency anemia, but this information is not the most important to remember. Sequelae commonly occur with hematologic alterations, especially chronic conditions such as hemophilia or sickle cell disease. The child's demographic data are important, because some hematologic diagnoses are more commonly associated with a certain age group, sex, race, or geographic location.

The nurse is caring for a woman with a previously diagnosed heart disease who is in the second stage of labor. Which assessment findings are of greatest concern? a. edema, basilar rales, and an irregular pulse b. Increased urinary output, and tachycardia c. Shortness of breath, bradycardia, and hypertension d. Regular heart rate, and hypertension

a. Edema, basilar rales, and an irregular pulse Edema, basilar rales, and an irregular pulse (A) indicate cardiac decompensation and require immediate intervention.

After teaching a group of students about hemophilia, the instructor determines that the students have understood the information when they identify hemophilia A as involving a problem with which of the following? a) Factor VIII b) Plasmin c) Platelets d) Factor IX

a. Factor VIII Explanation: In hemophilia A, the problem is with factor VIII, and in hemophilia B it is factor IX. Platelets are problematic in idiopathic thrombocytopenia purpura. Plasmin is involved in the pathophysiologic events of disseminated intravascular coagulation

The nurse is providing discharge teaching for a client who is 24 hours postpartum. The nurse explains to the client that her vaginal discharge will change from red to pink and then to white. The client asks, "What if I start having red bleeding after it changes?" What should the nurse instruct the client to do?

Reduce activity level and notify the healthcare provider.

A nurse is measuring the uterine fundal height of a client who is at 36 weeks gestation in supine position. The client suddenly reports dizziness and the nurse observes pallor and damp, cool skin. What should the nurse do first?

Reposition client into a lateral position

What happens to the foramen ovale during the first breath?

The foramen ovale closes as the pressure in the left atrium exceeds the pressure in the right atrium

Question: The blood cell becomes an erythrocyte. Rank the following steps in the proper order of occurrence.

The bone marrow releases a stem cell. Thrombopoietin acts on the cell. The myeloid cell becomes a megakaryocyte. Erythropoietin helps the cell turn into a red blood cell. Explanation: The bone marrow releases a stem cell. Thrombopoietin acts on the cell to help turn it into a myeloid cell. Erythropoietin acts on the cell and it turns into a megakaryocyte. The megakaryocyte becomes an erythrocyte (red blood cell).

What hormones are secreted by the corpus luteum?

The corpus luteum secretes estrogen and progesterone during the remaining 14 days of the cycle.

What are the interventions for the contraction stress test?

The external fetal monitor is applied to the client and a 20-minute baseline strip is recorded. The uterus is stimulated with (Pitocin) or by having the client nipple stimulate until 3 contractions with a duration of 40 seconds or more in a 10-minute period have been achieved.

38. In teaching the pregnant adolescent about nutrition, the nurse should: a. Emphasize the need to eliminate common teen snack foods because they are too high in fat and sodium. b. Determine the weight gain needed to meet adolescent growth and add 35 lb. c. Suggest that she not eat at fast-food restaurants to avoid foods of poor nutritional value. d. Realize that most adolescents are unwilling to make dietary changes during pregnancy.

b. Determine the weight gain needed to meet adolescent growth and add 35 lb.

A nurse is reviewing the medical records of several children who have undergone lead screening. The nurse would identify the child with which lead level as requiring no further action? a) 8 mcg/dL b) 20 mcg/dL c) 14 mcg/dL d) 26 mcg/dL

a. 8 mcg/dL Explanation: A blood lead level less than 10 mcg/dL requires no action. A level of 14 mcg/dL would need to be confirmed with a repeat test in 1 month along with parental education for decreased lead exposure and then a repeat test in 3 months. Levels of 20 mcg/dL and 26 mcg/dL need to be confirmed with a repeat test in 1 week along with parental education and a referral to the local health department for investigation of the home for lead reduction.

In hemophilia A, the classic form, only females manifest a bleeding disorder. a) False b) True

a. False Explanation: The classic form of hemophilia is caused by deficiency of the coagulation component factor VIII, the antihemophilic factor, and transmitted as a sex-linked recessive trait. In the United States, the incidence is approximately 1 in 10,000 white males. A female carrier may have slightly lowered but sufficient levels of the factor VIII component so that she does not manifest a bleeding disorder. Males with the disease also have varying levels of factor VIII; their bleeding tendency varies accordingly, from mild to severe.

18. With regard to protein in the diet of pregnant women, nurses should be aware that: a. Many protein-rich foods are also good sources of calcium, iron, and B vitamins. b. Many women need to increase their protein intake during pregnancy. c. As with carbohydrates and fat, no specific recommendations exist for the amount of protein in the diet. d. High-protein supplements can be used without risk by women on macrobiotic diets.

a. Many protein-rich foods are also good sources of calcium, iron, and B vitamins.

7. Which statement made by a lactating woman would lead the nurse to believe that the woman might have lactose intolerance? a. "I always have heartburn after I drink milk." b. "If I drink more than a cup of milk, I usually have abdominal cramps and bloating." c. "Drinking milk usually makes me break out in hives." d. "Sometimes I notice that I have bad breath after I drink a cup of milk."

b. "If I drink more than a cup of milk, I usually have abdominal cramps and bloating."

12. To prevent gastrointestinal upset, clients should be instructed to take iron supplements: a. On a full stomach. b. At bedtime. c. After eating a meal. d. With milk.

b. At bedtime.

hich maternal behavior is the nurse most likely to see when a new mother receives her infant for the first time? a. She eagerly reaches for the infant, undresses the infant, and examines the infant completely. b. Her arms and hands receive the infant and she then traces the infant's profile with her fingertips. c. Her arms and hands receive the infant and she then cuddles the infant to her own body. She eagerly reaches for the infant and then holds the infant close to her own body.

b. Her arms and hands receive the infant and she then traces the infant's profile with her fingertips.

The total bilirubin level of a 36-hour, breastfeeding newborns is 14 mg/dl. Based on this finding, which intervention should the nurse implement? a. Provide phototherapy for 30 minutes q8h b. Feed the newborn sterile water hourly c. Encourage the mother to breastfeed frequently d. Assess the newborn's blood glucose level

c. Encourage the mother to breastfeed frequently The normal total bilirubin level is 6 to 12 mg/dl after Day 1 of life. This infant's bilirubin is beginning to climb and the infant should be monitored to prevent further complications. Breast milk provides calories and enhances GI motility, which will assist the bowel in eliminating bilirubin (C).

A 23-year-old client who is receiving Medicaid benefits is pregnant with her first child. Based on knowledge of the statistics related to infant mortality, which plan should the nurse implement with this client? a.Refer the client to a social worker to arrange for home care. b.Recommend perinatal care from an obstetrician, not a nurse-midwife. c.Teach the client why keeping prenatal care appointments is important. d.Advise the client that neonatal intensive care may be needed.

c.Teach the client why keeping prenatal care appointments is important.

24. When counseling a client about getting enough iron in her diet, the maternity nurse should tell her that: a. Milk, coffee, and tea aid iron absorption if consumed at the same time as iron. b. Iron absorption is inhibited by a diet rich in vitamin C. c. Iron supplements are permissible for children in small doses. d. Constipation is common with iron supplements.

d. Constipation is common with iron supplements.

When using anticipatory guidance to prepare a 5-year-old for an IM injection, what statement by the nurse would be most appropriate? a. "Ethan, I'm going to give you a shot." b. "Ethan, the doctor wants you to have some medicine, and it will hurt." c. "Ethan, some medicine can only be given with a needle." d. "Ethan, I am going to give you some medicine that will sting, but only for a little while."

d. "Ethan, I am going to give you some medicine that will sting, but only for a little while." Anticipatory guidance is the psychological preparation of a patient for a stressful event by explaining what will happen and the probable outcome. p. 937

A 35-year-old primigravida client with severe preeclampsia is receiving magnesium sulfate via continuous IV infusion. Which assessment data indicates to the nurse that the client is experiencing magnesium sulfate toxicity? a. Deep tendon reflexes 2+ b. Blood pressure 140/90 c. Respiratory rate 18/minute d. Urine output 90 ml/4 hours

d. Urine output 90 ml/4 hours Urine outputs of less than 100 ml/4 hours (D), absent DTRs, and a respiratory rate of less than 12 breaths/minute are cardinal signs of magnesium sulfate toxicity

Infants of drug-dependent women tend to be large for gestational age. a) False b) True

False Infants of drug-dependent women tend to be small for gestational age.

Nursing Interventions for a child with a platelet count lower than 100,000/mm3

Family must be taught to protect the child, to restrict the child's activity, and to help the child avoid injury

After teaching the parents of a child diagnosed with pernicious anemia about the disorder and treatment, the nurse determines that the teaching was successful when the parents state which of the following? a) "We'll need to plan for a bone marrow transplant soon." b) "He needs to eat more green leafy vegetables to cure the anemia." c) "He'll need to have those vitamin shots for the rest of his life." d) "He might get constipated from the supplement."

"He'll need to have those vitamin shots for the rest of his life." Explanation: Monthly injections of vitamin B12 are required for life. Although diet is important, diet alone will not cure the anemia. Iron used to treat iron-deficiency anemia can lead to constipation. Bone marrow transplant is used to treat aplastic anemia.

The nurse is reinforcing teaching to a client, gravida 1 para 0, at 8 weeks gestation about expected weight gain in pregnancy. The client's prepregnancy BMI is 21 kg/m2. Which of the following statements made by the client indicates an understanding about weight gain?

"I should gain about 30 lb (13.6 kg) during pregnancy."

The nurse is reinforcing instructions to a parent about how to care for a newly circumcised newborn. Which statement by the parent indicates a need for further teaching?

"I will clean the area with alcohol-based wipes or soap water."

The nurse is reinforcing instructions to a postpartum client about cord care for the newborn. Which statement by the client indicates a need for further teaching?

"I will secure the diaper over the cord to protect it."

The nurse is reinforcing teaching about infant safety to a class of expectant parents. Which statement by a participant indicates a need for further instruction?

"I will tie bumper pads to the sides of the crib to protect my baby's head."

The mother of Mary Jo, a 10-year-old who had a febrile reaction following a transfusion, asks the nurse: "Why did this happen to my child?" What is the nurse's best response? a) "Mary Jo's blood was not compatible with the blood product, causing red blood cell destruction." b) "The donor blood contained plasma proteins or other antigens to which Mary Joe was hypersensitive." c) "Mary Jo's blood has developed antibodies to leukocyte, platelet, or plasma protein antigens in the donor blood." d) "Too much of the blood product was transfused at too rapid a rate."

"Mary Jo's blood has developed antibodies to leukocyte, platelet, or plasma protein antigens in the donor blood." Explanation: A febrile reaction is not associated with hemolysis and generally occurs when the recipient has developed antibodies to leukocyte, platelet, or plasma protein antigens in the donor blood. In a hemolytic reaction, the blood product is not compatible with the recipient's blood. An allergic reaction is a nonhemolytic reaction that occurs when the donor blood contains plasma proteins or antigens to which the recipient is hypersensitive.

The nurse is reinforcing teaching with a group of caregivers of children diagnosed with iron deficiency anemia. One of the caregivers tells the group, "I give my child ferrous sulfate." Which of the following statements made by the caregivers is correct regarding giving ferrous sulfate? a) "My husband gives our daughter orange juice when she takes her ferrous sulfate, so she gets Vitamin C." b) "When I give my son ferrous sulfate I know he also needs potassium supplements." c) "I always give the ferrous sulfate with meals." d) "We watch closely for any diarrhea since that usually happens when he takes ferrous sulfate."

"My husband gives our daughter orange juice when she takes her ferrous sulfate, so she gets Vitamin C." Explanation: When ferrous sulfate is administered, it should be given between meals with juice (preferably orange juice, because vitamin C aids in iron absorption). For best re sults, iron should not be given with meals. Ferrous sulfate can cause constipation or turn the child's stools black

A nurse is providing dietary interventions for a 5-year-old with an iron deficiency. Which of the following responses indicates a need for further teaching? a) "He will enjoy tuna casserole and eggs" b) "There are many iron fortified cereals that he likes" c) "Red meat is a good option; he loves the hamburgers from the drive-thru." d) "I must encourage a variety of iron-rich foods that he likes"

"Red meat is a good option; he loves the hamburgers from the drive-thru." Explanation: While iron from red meat is the easiest for the body to absorb, the nurse must limit fast food consumption from the drive thru as they are also high in fat, fillers, and sodium. The other statements are correct

A nurse is assigned to care for a child with a diagnosis of Wilms' tumor. In planning care for the child, the nurse understands that this tumor is:

1. An abdominal tumor *2. A renal tumor* 3. A brain tumor 4. A bone tumor *Rationale:* Wilms' tumor, or nephroblastoma, is the most common renal tumor in children. Arising from the renal parenchyma of the kidney, this tumor grows very rapidly. It may be present unilaterally and localized, or bilaterally, and sometimes with metastasis to other organs.

A child is to receive oral iron therapy in liquid form three times a day. After teaching the parents about administering the iron, which statement indicates a need for additional teaching? a) "She can drink the medicine from a medicine cup." b) "We will have her drink water or juice with the medicine." c) "Her bowel movements will probably turn very dark." d) "We'll try to give the medicine to her in between milk servings."

"She can drink the medicine from a medicine cup." Explanation: Liquid iron can stain the teeth; therefore, the parents should give the liquid iron through a straw or syringe, placing it toward the back of the child's mouth. Iron turns stools dark. To maximize absorption, it is best to give the iron with water or juice between meals.

The nurse weighs the new infant and calculates his measurements. The new mom asks, "Did my baby grow well? The doctor said he was LGA: What does that mean?" What is the best explanation? a) "That means your baby is over the 90th percentile for weight." b) "That means your baby is in the 5th percentile for weight." c) "That means that your baby is lazy sometimes." d) "That means your baby is average for gestational age."

"That means your baby is over the 90th percentile for weight." Correct Explanation: LGA stands for large for gestational age. These infants are over the 90th percentile for weight. The other choices are not over the 90th percentile for weight or describe a different characteristic.

An emergency department nurse is gathering initial data on a child suspected of epiglottitis. The nurse's priority would be to:

*1. Assess for a patent airway.* 2. Prepare the child for an x-ray. 3. Prepare the child for tracheotomy. 4. Assist the health care provider with intubation. *Rationale:* When epiglottitis is suspected, the priorities are to maintain a patent airway and to next obtain an x-ray to confirm the diagnosis. If epiglottitis is present, the child is taken promptly to the operating room for tracheal intubation. Although options 2, 3, and 4 may be components of care, they are not the priority.

The nurse is caring for a 2-year-old with sickle cell anemia and describing the acute and chronic manifestations of sickle cell anemia to his mother. Which statement by the mother indicates a need for further teaching? a) "Aplastic crisis is a life-threatening acute manifestation of sickle cell anemia." b) "Delayed growth and development and delayed puberty are chronic manifestations." c) "The acute manifestations, like splenic sequestration, are most often life-threatening." d) "Bone infarction, dactylitis, and recurrent pain episodes are acute manifestations.

"The acute manifestations, like splenic sequestration, are most often life-threatening." Splenic sequestration is a life-threatening acute manifestation of sickle cell anemia, but some of the chronic manifestations of the disease, such as pulmonary hypertension and restrictive lung disease, are also often life-threatening. Aplastic crisis is a life-threatening acute manifestation. Bone infarction, dactylitis, and recurrent pain episodes are acute manifestations; delayed growth and development and chronic puberty are chronic manifestations.

The nurse is caring for a child with aplastic anemia. The nurse is reviewing the child's blood work and notes the granulocyte count about 500, platelets over 20,000, and the reticulocyte count is over 1%. The parents ask if these values have any significance. The nurse is correct in responding: a) "The doctor will discuss these findings with you when he comes to the hospital." b) "I'm really not allowed to discuss these findings with you. c) "These labs are just common labs for children with this disease." d) "These values will help us monitor the disease."

"These values will help us monitor the disease." Explanation: This response answers the parent's questions. In the nonsevere form, the granulocyte count remains about 500, the platelets are over 20,000, and the reticulocyte count is over 1%. The other responses do not address what the parents are asking and would block therapeutic communication.

The nurse is providing family education for the prevention or early recognition of vaso-occlusive events in sickle cell anemia. Which response by a family member indicates a need for further teaching? a) "We must be compliant with vaccinations and prophylactic penicillin." b) "We must watch for unusual headache, loss of feeling, or sudden weakness." c) "We should call the doctor for any fever over 100°F." d) "We need to seek medical attention for abdominal pain."

"We should call the doctor for any fever over 100°F." Explanation: The nurse must emphasize that ANY febrile illness requires immediate attention. Fever causes dehydration, which can trigger problems in a child with sickle cell anemia. Seeking medical attention for abdominal pain; watching for unusual headache, loss of feeling, or sudden weakness; and compliance with vaccinations are appropriate.

A 5-year-old child has been transferred to the pediatric unit after a cardiac catheterization. Which of the following interventions has the highest priority in the care of this child immediately following the procedure?

*1. Assess for any bleeding on the dressing.* 2. Position the child's leg so that it is straight. 3. Assess the strength and presence of the distal pulses. 4. Take the vital signs including blood pressure and oxygen saturation. *Rationale:* Bleeding is a primary concern following this procedure. Although options 2, 3, and 4 are correct interventions, they are not the priority.

The nurse is providing family education for the prevention or early recognition of vaso-occlusive events in sickle cell anemia. Which response by a family member indicates a need for further teaching? a) "We must be compliant with vaccinations and prophylactic penicillin." b) "We need to seek medical attention for abdominal pain." c) "We must watch for unusual headache, loss of feeling, or sudden weakness." d) "We should call the doctor for any fever over 100°F."

"We should call the doctor for any fever over 100°F." Explanation: The nurse must emphasize that ANY febrile illness requires immediate attention. Fever causes dehydration, which can trigger problems in a child with sickle cell anemia. Seeking medical attention for abdominal pain; watching for unusual headache, loss of feeling, or sudden weakness; and compliance with vaccinations are appropriate.

A nurse is caring for a 7-year-old boy with hemophilia who requires an infusion of factor VIII. He is fearful about the process and is resisting treatment. How should the nurse respond? a) "Will you help me apply this band-aid?" b) "Please be brave; we need to stop the bleeding" c) "Would you help me dilute this and mix it up?" d) "Would you like to administer the infusion?"

"Would you help me dilute this and mix it up?" Explanation: The best response for a 7-year-old is to use distraction and involve him in the infusion process in a developmentally appropriate manner. A 7-year-old is old enough to assist with the dilution and mixing of the factor. Asking for help with the band-aid would be best for a younger child. Teens should be taught to administer their own factor infusions. Telling him to be brave is not helpful and does not teach.

The nurse is evaluating a parent's understanding of post-circumcision care for a newborn. Circumcision was performed using the clamp method. Which statement by the parent demonstrates a need for further teaching?

"Yellow exudate on the glans penis indicates infection."

A client is at 20 weeks gestation. The client reports having to "run to the bathroom all the time," "it hurts to pee," and my urine "smells bad." Which statement by the nurse is the most appropriate?

"You may need to be checked for a urinary tract infection."

The nurse is providing teaching about iron supplement administration to the parents of a 10-month-old child. It is critical that the nurse emphasize which of the following teaching points to the parents? a) "Place the liquid behind the teeth; the pigment can cause staining" b) "Please give him plenty of fluids and encourage fiber" c) "You must precisely measure the amount of iron" d) "Your child may become constipated from the iron"

"You must precisely measure the amount of iron" Explanation: The priority is to emphasize to the parents that they precisely measure the amount of iron to be administered in order to avoid overdosing. The other instructions are accurate, but the priority is to emphasize precise measurement.

List the 3 common defects involving increased pulmonary blood flow.

(1) Patent Ductus Arteriosus (2) Atrial Septal Defect (3)Ventricular Septal Defect (VSD)

List the 3 defects involving decreased pulmonary blood flow.

(1) Pulmonary Stenosis (2) Pulmonary Atresia (3)Tetralogy of Fallot

A mother of a child brings the child to a clinic and reports that the child has a fever and has developed a rash on the neck and trunk. Roseola is diagnosed, and the mother is concerned that her other children will contract the disease. Which instruction should the nurse give to the mother to prevent the transmission of the disease?

*1. "Disease transmission is unknown."* 2. "The disease is transmitted through the urine and feces, so the other children should use a separate bathroom." 3. "The disease is transmitted through the respiratory tract, so the child should be isolated from the other children as much as possible." 4. "The disease is transmitted by contact with body fluids, so any items contaminated with body fluids need to discarded in a separate receptacle." *Rationale:* The method of transmission of roseola is unknown. Options 2, 3, and 4 are not correct transmission routes of roseola.

A nurse is collecting data on an infant with a diagnosis of suspected Hirschsprung's disease. Which of the following questions to the mother will most specifically elicit information regarding this disorder?

*1. "Does your infant have foul-smelling, ribbon-like stools?"* 2. "Is your infant constantly vomiting?" 3. "Does your infant constantly spit up feedings?" 4. "Does your infant have diarrhea?" *Rationale:* Chronic constipation, beginning in the first month of life and resulting in pellet-like or ribbon-like stools that are foul smelling, is a clinical manifestation of Hirschsprung's disease. Delayed passage or absence of meconium stool in the neonatal period is the cardinal sign. Bowel obstruction, especially in the neonatal period, abdominal pain and distention, and failure to thrive are also clinical manifestations. Options 2, 3, and 4 are not specific clinical manifestations of this disorder.

A nurse provides instructions to the mother of an infant with cleft palate regarding feeding. Which statement by the mother indicates a need for further instructions?

*1. "I need to use a nipple with a small hole to prevent choking."* 2. "I need to stimulate sucking by rubbing the nipple on the lower lip." 3. "I need to allow my infant time to swallow." 4. "I need to allow my infant to rest frequently to provide time for swallowing what has been placed in the mouth." *Rationale:* The mother should be taught the ESSR method of feeding the child with a cleft palate: ENLARGE the nipple by cross-cutting a hole so that food is delivered to the back of the throat without sucking; STIMULATE sucking by rubbing the nipple on the lower lip; SWALLOW; then REST to allow the infant to finish swallowing what has been placed in the mouth.

A pregnant client at 30 weeks gestation comes to the prenatal clinic. Which vaccines may be administered safely at this prenatal visit? Select all that apply.

- Influenza injection - Tetanus, diphtheria, and pertussis

A nurse is reinforcing information on formula preparation for a client with a newborn. Which statements by the client indicate proper understanding? Select all that apply.

- "I must wash the top of the concentrated formula can before opening it." - "Prepared formula should be kept in the refrigerator and discarded after 48 hours."

A client denies illicit drug use but has some suspicious behaviors. The client's neonate has a low birth weight. What other signs would lead the nurse to suspect neonatal abstinence syndrome? Select all that apply.

- Irritability and restlessness - Poor feeding and loose stools - Stuffy nose and frequent sneezing

What are the assessments for stage 4 of labor?

-Blood pressure returns to prelabor level. -Fundus remains contracted in the midline. -1 or 2 finger breadths below the umbilicus. -Monitor the lochia discharge. -Lochia may be moderate in amount and red in color in stage 4.

A woman is being interviewed by a triage nurse at a medical doctor's office. Which of the following signs/symptoms by the client would warrant the nurse to suggest that a pregnancy test be done? (Select all that apply) 1. Amenorrhea 2. Fever 3. Fatigue 4. Nausea 5. Dysuria

1,3,4 1.Pregnancy is the most common cause of amenorrhea 3. A common complaint of women in early pregnancy is fatigue 4. A common complaint of women is early pregnancy is nausea

The macrosomic baby in the nursery is suspected of having a fractured clavicle from a traumatic delivery. Which of the following signs/symptoms would the nurse expect to see? ( Select all the apply) 1. Pain with movement 2. Hard lump at the fracture site 3. Malpositioning of the arm 4. Asymmetrical Moro reflex 5. Marked localized ecchymosis

1,2,3,4 1.The baby will complain of pain at the site. 2.If not in the immediate period after the injury, within a few days there will be a palpable lump on the bone at the site of the break 3.Because of the break, the baby is likely to position the arm in an aytpical posture 4.Because of the injury to the bone, the baby is unable to respond with symmetrical arm movement

The nurse is assessing a newly admitted patient. What clinical manifestations would probably indicate that the child has autism? (Select all that apply) 1.) Avoidance of body contact with other people 2.) Speech and language delays 3.) Deficits in social development 4.) Good eye contact with the nurse during the assessment 5.) Increased sensitivity to stimuli

1.) Avoidance of body contact with other people 2.) Speech and language delays 3.) Deficits in social development 5.) Increased sensitivity to stimuli Pg. 1058 -Foundations of Nursing

In an assessment of a newborn, what findings are suggestive of hydrocephalus? 1.) Bulging fontanelle, dilated scalp veins 2.) Depressed fontanelle, decreased BP 3.) Constant low-pitched cry, restlessness 4.) Closed fontanelle, high-pitched cry

1.) Bulging fontanelle, dilated scalp veins Pg. 1032 -Foundations of Nursing

Recommended daily allowance of iron for toddlers?

7mg/day

A nurse is reviewing her assignments. Which patient should she assess first? 1. A 12-hour infant who is small for gestational age. 2. Four hour infant with a cardiac defect. 3. 9 hour old infant who has not voided 4. 3 day old infant waiting for discharge

2. Four hour infant with a cardiac defect Rationale: The infant with a cardiac defect is at the most risk for complications and should be assessed first.

A pregnant woman comes to the prenatal clinic for an initial visit. In reviewing her childbearing history, the client indicates that she has delivered premature twins, one full-term baby, and has had no abortions. Which GTPAL should the nurse document in this client's record

3-1-1-0-3. describes the correct GTPAL. The client has been pregnant 3 times including the current pregnancy (G-3). She had one full-term infant (T-1). She also had a preterm (P-1) twin pregnancy (a multifetal gestation is considered one birth when calculating parity). There were no abortions (A-0), so this client has a total of 3 living children

A client with type 1 diabetes mellitus is 6 weeks pregnant. Her fasting glucose and hemoglobin A1c are noted to be 168 mg/dL and 12%, respectively. Which of the following nursing diagnoses is appropriate for the nurse to make at this time? 1. Altered matenal skin integrity 2. Deficient maternal fluid volume 3. Risk for fetal injury 4. Fetal urinary retention

3. A nursing diagnosis of risk for fetal injury is an appropriate nursing diagnosis.

The nurse is caring for a patient with nephrotic syndrome. What assessment finding does the nurse expect to find? 1.) Gross hematuria, albuminuria, temp. 101 F to 103 F 2.) Elevated BP, weight loss, hematuria 3.) Albuminuria, edema, puffiness of face 4.) Edema, albuminuria, hypotension

3.) Albuminuria, edema, puffiness of face Pg. 1017 -Foundations of Nursing

The child has been diagnosed with severe iron deficiency anemia. The child requires 5 mg/kg of elemental iron per day in three equally divided doses. The child weighs 47.3 pounds. How many milligrams of elemental iron should the child receive with each dose? Round to the nearest whole number

36 mg Explanation: 47.3 pounds x 1 kg/2.2 pounds = 21.5 kg 21.5 kg x 5 mg/1 kg = 107.5 mg/day 107.5 mg/3 doses = 35.8333 mg/dose Rounded to the nearest whole number = 36 mg

A fetus is in the LOA position in utero. Which of the following findings would the nurse observe when doing Leopold's maneuvers? 1. Hard, round object in the fundal region 2. Flat object above the symphysis pubis 3. Soft, round object on the left side of the uterus 4. Small objects on the right side of the uterus

4. A nurse could conclude that a fetus is in the LOA when feeling small objects- the fetal arms and legs- on the right side of the uterus

A patient is admitted to the labor and delivery unit. The patient says, "my water just broke." Which action should the nurse take first? 1. Notify physician 2. Check pH of amniotic fluid 3. Document characteristics of amniotic fluid. 4. Check fetal heart rate.

4. Check fetal heart rate Rationale: Normal baseline FHR 120-160 bpm. Tachycardia is above 160 bpm is an early sign of fetal hypoxia.

A nurse is caring for a patient undergoing augmentation of labor. The nurse notes 3 consecutive late decelerations on the fetal monitor. Which action should the nurse take? 1. Increase infusion of lactated ringer's solution 2. O2 mask at 10 L 3. Place patient on left side 4. Turn off infusion of oxytocin (Pitocin)

4. Turn off infusion of oxytocin (Pitocin) Rationale: Fetal hypoxia is treated with measures to increase oxygen flow to fetus, such as increasing fluids, oxygen, turning off pitocin, and placing the mother in the left lying position.

A 14-year-old has a positive test result for human immunodeficiency virus (HIV) abs expresses a desire to kill herself. What would be the most appropriate initial response of the nurse? 1.) Tell her that they are very close to discovering a cure for HIV. 2.) Encourage her to talk to her pastor 3.) Arrange a visit from another adolescent who s HIV positive. 4.) Immediately report the threat to a mental health care provider.

4.) Immediately report the threat to a mental health care provider. Pg. 1062 -Foundations of Nursing

Hypoglycemia in a mature infant is defined as a blood glucose level below which of the following? a) 30 mg/100 mL whole blood b) 100 mg/100 mL whole blood c) 40 mg/100 mL whole blood d) 80 mg/100 mL whole blood

40 mg/100 mL whole blood Explanation: Because newborns do not manifest symptoms of a reduced glucose level until it decreases well below adult levels, a finding below 40 mg/100 mL whole blood is considered hypoglycemia

Iron-Deficiency Anemia

A decrease in RBC volume, a decrease in Hemoglobin, or both

A client at 32-weeks gestation is hospitalized with severe pregnancy- induced hypertension (PIH), and magnesium sulfate is prescribed to control the symptoms. Which assessment finding indicates the therapeutic drug level has been achieved?

A decrease in respiratory rate from 24 to 16. Magnesium sulfate, a CNS depressant, helps prevent seizures. A decreased respiratory rate (C) indicates that the drug is effective. (Respiratory rate below 12 indicates toxic effects.)

When examining a client after delivery, the nurse finds the fundus soft, boggy, and displaced above and to the right of the umbilicus. After perfoming fundal massage and having the client empy her bladder when should the nurse recheck fundus? A. q 15 minutes *4 (1 hour) B. q 45 minutes *2 (1.5 hour) C. q 30 minutes *4 (2 hours) D. q 30 minutes *2 (1 hour)

A. The nurse should recheck fundus q 15 minutes *4 (1 hour); q 30 minutes *2 hours

A 28 year old porgnant woman has the following symptoms: visual disturbance, persistant vomiting, swelling of face, fingers or sacrum and severe continuous headache. What do these symptoms most probably indicate? A. Preeclampsia/eclampsia B. Dysuria C. Chills D. Fluid discharge from the vagina

A. Visual distubance, persistant vomiting, swelling of face, fingers or sacrum and severe continuous headache in pregnant woman possible indications of preeclampsia/ eclampsia

A 7lb 8oz baby would need 50 calories x7lbs=350 calories plus 25 calories= 375 calories per day. Taking into consideration that most infant formulas contain 20 calories/ounce, how many ounces of formula are needed per day? A. 18.75 B. 14.75 C. 13.75 D. 16.75

A. Dividing 375 by 20 =18.75 ounces of formula needed per day for a 7lb 8oz baby

In a 24 year old pregnant woman, the amniocenteses is done in late pregnancy. How should the bladder be to avoid puncturing the bladder? A. Empty B. Full C. 1/4 empty D. 1/2 full

A. When an amniocentesis is done in late pregnancy, the bladder must be empty to avoid puncturing the bladder

The nurse is aware that the earliest age at which the infant should be able to walk independently is _____ months. a. 8 to 10 b. 12 to 15 c. 15 to 18 d. 18 to 21

ANS: B For the majority of children, the milestone of walking alone is achieved between 12 and 15 months.

Which characteristic best describes the gross motor skills of a 24-month-old child? a. Skips b. Rides tricycle c. Broad jumps d. Walks up and down stairs

ANS: D The 24-month-old child can go up and down stairs alone with two feet on each step. Skipping and the ability to broad jump are skills acquired at age 3. Tricycle riding is achieved at age 4.

Ventricular Septal Defect (VSD)

Abnormal interventricular septum opening, causes oxygenated blood flow from the higher-pressure left ventricle to the lower-pressure right ventricle, which causes the blood to become deoxygenated

An 8-month-old girl appears pale, irritable, and anorexic. On blood testing, the red blood cells are hypochromic and microcytic. The hemoglobin level is less than 5 g/100 mL, and the serum iron level is high. Which of the following symptoms should the nurse most expect as a result of excessive iron deposits? a) An enlarged heart b) An enlarged spleen c) An enlarged thyroid gland d) Enlarged lymph nodes

An enlarged spleen Explanation: The child with thalassemia major may have both an enlarged spleen and liver due to excessive iron deposits and fibrotic scarring in the liver and the spleen's increased attempts to destroy defective RBCs.

All infants need to be observed for hypoglycemia during the newborn period. Based on the facts obtained from pregnancy histories, which infant would be most likely to develop hypoglycemia? a) An infant whose mother craved chocolate during pregnancy b) An infant whose labor began with ruptured membranes c) An infant who has marked acrocyanosis of his hands and feet d) An infant who had difficulty establishing respirations at birth

An infant who had difficulty establishing respirations at birth Explanation: Newborns use a great many calories in their effort to achieve effective respirations. Infants who had difficulty establishing respirations need to be assessed for hypoglycemia.

S/Sx of Leukemia

Anemia, pallor, fatigue, leukopenia, fever, infection, thrombocytopenia, bleeding, bruising, and petechiae. Hepatosplenomegaly and enlargement of the lymph nodes are often present at diagnosis.

The nurse examines a 26-week-old premature infant. The skin temperature is lowered. What could be a consequence of the infant being cold? a) Sleepiness b) Tachycardia c) Apnea d) Crying

Apnea Explanation: A premature infant has thin skin, an immature central nervous system (CNS), lack of brown fat stores, and an increased weight-to-surface area ratio. These are predisposing thermoregulation problems that can lead to hypothermia. As a result, the infant may become apneic, have respiratory distress, increase his or her oxygen need, or be cyanotic. The other choices are not specific to increased oxygen demand or respiratory distress.

A woman who gave birth 48 hours ago is bottle-feeding her infant. During assessment, the nurse determines that both breasts are swollen, warm, and tender upon palpation. What action should the nurse take?

Apply cold compresses to both breasts for comfort.

The nurse is assessing a client who is having a non-stress test (NST) at 41- weeks gestation. The nurse determines that the client is not having contractions, the fetal heart rate (FHR) baseline is 144 bpm, and no FHR accelerations are occurring. What action should the nurse take?

Ask the client if she has felt any fetal movement. The client should be asked if she has felt the fetus move (D). An NST is used to determine fetal well-being, and is often implemented when postmaturity is suspected. A "reactive" NST occurs if the FHR accelerates 15 bpm for 15 seconds in response to the fetus' own movement, and is "nonreactive" if no FHR acceleration occurs in response to fetal movement. The client should empty her bladder before starting the test, but bladder distention does not impede fetal movement

What assessments should be made for heavy third trimester vaginal bleeding?

Assess maternal vital signs and fetal heart rate.

What is the Apgar scoring system?

Assesses 5 items to be scored and add the points to determine the newborn's total score. The newborn's Apgar score is assessed and recorded at 1 minute and at 5 minutes after birth.

The nurse teaches parents how to help their children learn impulse control and cooperative behaviors. This would occur during which of the stages of development defined by Erikson? A.Trust versus mistrust B.Initiative versus guilt C.Industry versus inferiority D.Autonomy vs. Shame and doubt

B) Initiative vs Guilt. The stage of initiative versus guilt occurs from ages 3 to 6 years, during which children develop direction and purpose. Teaching impulse control and cooperative behaviors during this stage help the child to avoid risks of altered growth and development. In the autonomy versus sense of shame and doubt stage, toddlers learn to achieve self-control and willpower. Trust versus mistrust is the first stage, during which children develop faith and optimism. During the industry versus inferiority stage, children develop a sense of competency.

A nurse caring for a 7lb 8oz baby feeds him with 18.75 ounces of infant formual needed per day. If every infant formula contains 20 calories/ounce, which is the total amunt of calories a baby needs per day? A. 175 calories per day B. 375 calories per day C. 575 calories per day D. 275 calories per day

B. A 7lb 8oz baby would need 50 calories x 7lbs = 350 calories plus 25 calories = 375 calories per day. Most infant formulas contain 20 calories /ounce. Dividing 375 by 20 = 18.75 ounces of formula needed per day

The umbilical cord in a newborn should contain 3 vessels, 1 vein which carries oxygenated blood to the fetus and 2 arteries which carry unoxygenated blood back to the placenta. What do cord abnormalities usually indicate? A. Neurologic anomalies B. Renal anomalies C. Congenital vertebral anomaly D. Chromosome anomaly

B. Cord abnormalities usually indicate cardiovascular or renal anomalies.

The early decelerations in fetal heart rate monitoring are the transient decrease in heart rate which coincides with the onset of the uterine contraction. Between what cm do the early decelerations caused by head compression and fetal descent usually occur in the 2nd stage? A. 2 and 6 cm B. 4 and 7 cm C. 3 and 8 cm D. 7 and 10 cm

B. Early decelerations, caused by head compression and fetal descent, usually occur between 4 and 7 cm in the 2nd stage. Check for labor progress if early decelerations are noted

In the first 12 hours after delivery, the 22 year old client shows signs of hemorrhage. What is one of the common reasons for uterine atony and/or hemorrhage in the first 24 hours after delivery? A. Empty bladder B. Full bladder C. Hypoglycemia D. Low blood pressure

B. FULL BLADDER is one of the most common reasons for uterine atony and/or hemorrhage in the first 24 hours after delivery

if the structures of the foramen ovale, ductus arteriosus and ductus venosus don't close postnatally, cardiac and pulmonary compromise will develop. What should be suctioned by the nurse firstly? A. Nose B. Mouth C. Lungs D. Kidney

B. Suctioning the mouth first and then the nose. Stimulating the nares can initiate inspiration which could cause aspiration of mucus in oral pharynx

A new mother asks the nurse, "How do I know that my daughter is getting enough breast milk?" Which explanation should the nurse provide? A.Weigh the baby daily, and if she is gaining weight, she is eating enough. B.Your milk is sufficient if the baby is voiding pale straw-colored urine 6 to 10 times a day. C.Offer the baby extra bottle milk after her feeding, and see if she is still hungry. D.If you're concerned, you might consider bottle feeding so that you can monitor her intake.

B.Your milk is sufficient if the baby is voiding pale straw-colored urine 6 to 10 times a day.

Which meal should the nurse recommend for a pregnant client at 13 weeks gestation?

Baked chicken, turnip greens, peanut butter cookie, and grape juice

A vaginally delivered infant of an HIV positive mother is admitted to the newborn nursery. What intervention should the nurse perform first?

Bathe the infant with an antimicrobial soap

Reasons adolescents are at increased risk for iron-deficiency anemia?

Because of their rapid growth rate, combined with poor eating habits. Inadequate intake of dietary iron

What is preterm labor?

Before 37 weeks

The nurse is teaching a woman how to use her basal body temperature (BBT) pattern as a tool to assist her in conceiving a child. Which temperature pattern indicates the occurrence of ovulation, and therefore, the best time for intercourse to ensure conception?

Between the time the temperature falls and rises.

Diagnostic tests for ITP

Bleeding time, clot reaction time, tourniquet test (BP cuff is inflated to 100 mmHg and left in place for 5mins and yields a positive result: the appearance of more than 15 petechiae)

In understanding the cardiovascular and hematologic systems of the body it is important to know that the blood is made up of plasma, red blood cells, white blood cells, and platelets. These blood cells are formed in which of the following? a) Capillaries b) Lymph nodes c) Arteries d) Bone marrow

Bone marrow Explanation: Blood is a fluid composed of many elements, including plasma, red blood cells, white blood cells, and platelets. Each of these elements has a different function. These blood cells are formed in the bone marrow. Arteries carry blood away from the heart to the body, and veins collect the blood and return it to the heart. Capillaries are the exchange vessels for the materials that flow through the body. The lymph nodes filter the lymph.

Exhibit Of the abdominal lines shown in the exhibit, where would the nurse expect the fundal height of a 20-week gestation client to be felt? Click on the exhibit button for additional information.

C

A mother of a 3-year-old tells a clinic nurse that the child is rebelling constantly and having temper tantrums. The nurse most appropriately tells the mother to: A) Punish the child every time the child says "no", to change the behavior B) Allow the behavior because this is normal at this age period C) Set limits on the child's behavior D) Ignore the child when this behavior occurs

C) Set limits on the child's behavior-According to Erikson, the child focuses on independence between ages 1 and 3 years. Gaining independence often means that the child has to rebel against the parents' wishes. Saying things like "no" or "mine" and having temper tantrums are common during this period of development. Being consistent and setting limits on the child's behavior are the necessary elements.

Oxytocin should be administered after the placenta is delivered because the drug will cause the uterus to contract. What can happen if the drug is administered before the placenta is delivered? A. Will predispose the client to nausea B. Will predispose the client to amnesia C. Will predispose the client to hemorrhage D. Will predispose the client to hypocalcemia

C. If the oxytocic durg is administered before the placenta is delivered, it may result in a retained placenta, which predisposed the client to hemorrhage and infection

There has been an increase in the number of children diagnosed with Type 2 diabetes with the increasing rate of obesity in children thought to be a contributing factor. What other factors are thought to be contributing in the increase of Type 2 cases? A. Hypotension B. Hypokalemia C. Lack of physical activity D. Hyperkalemia

C. There has been an increase in the number of children diagnosed with Type 2 diabetes. The increasing rate of obesity is children is thought to be a contributing factor. Other contributing factors include lack of physical activity, and a family history of Type 2 diabetes.

When it comes to X-linked recessive linked recessive trait, the trait is carried on the x chromosome, therefore, usually affects male offspring. What is the chance for a pregnant woman carrier her offspring to get the disease? A. Male child: 75% of having the disease B. Female child: 50% of having the disease C. Male child: 50% of having the disease D. Female child: 25% of having disease

C. With each pregnancy of a woman who is a carrier there is a 25% chance of having a child with hemophilia. If the child is male, he has a 50% chance of having hemophilia. If the child is female, she has a 50% chance of being a carrier.

A full-term infant is admitted to the newborn nursery and, after careful assessment, the nurse suspects that the infant may have an esophageal atresia. Which symptoms is this newborn likely to have exhibited?

Choking, coughing, and cyanosis. includes the "3 Cs" of esophageal atresia caused by the overflow of secretions into the trachea.

Individuals with hemophilia B have a deficiency in factor IX, which can cause excessive blood loss. What is another name for this clotting factor? a) Stuart factor b) Christmas factor c) Proconvertin d) Antihemophilic factor

Christmas factor Explanation: Factor IX is also known as plasma thromboplastin component or Christmas factor; factor X is Stuart factor; factor VIII is antihemophilic factor; and factor VII is proconvertin.

The clinic nurse is collecting data on a pregnant client in the first trimester. Which finding is most concerning and warrants priority intervention?

Client is taking lisinopril to control hypertension

Complete surgical correction of Tetralogy of Fallot includes?

Closure of the VSD, a pulmonic valvotomy, and repair of the overriding aorta

Which of the following is an example of developmental care in the NICU? a) Giving medications b) Holding the infant c) Giving a bath d) Cluster care and activities

Cluster care and activities Correct Explanation: Clustering care and activities in the NICU decreases stress and helps developmentally support premature and sick infants. Developmental care can decrease assistance needed and length of hospital stay. The other choices are part of basic infant care.

A client at 28-weeks gestation calls the antepartal clinic and states that she is experiencing a small amount of vaginal bleeding which she describes as bright red. She further states that she is not experiencing any uterine contractions or abdominal pain. What instruction should the nurse provide?

Come to the clinic today for an ultrasound. Third trimester painless bleeding is characteristic of a placenta previa. Bright red bleeding may be intermittent, occur in gushes, or be continuous. Rarely is the first incidence life-threatening, nor cause for hypovolemic shock. Diagnosis is confirmed by transabdominal ultrasound (A). Bleeding that has a sudden onset and is accompanied by intense uterine pain indicates abruptio placenta, which IS life- threatening to the mother and fetus--then (B) would be appropriate.

Complications associated with bleeding most often involve joints and muscles. Adjunct measures to control bleeding include: a) Compression b) Lowering extremities c) Heat d) Exercise

Compression Complications associated with bleeding most often involve joints and muscles. Adjunct measures include rest, ice, compression, and elevation (RICE). In addition, corticosteroids such as prednisone may be used to reduce inflammation in the joint.

When discussing heat loss in newborns, placing a newborn on a cold scale would be an example of what type of heat loss? a) Conduction b) Convection c) Evaporation d) Radiation

Conduction Correct Explanation: A conduction heat loss results from direct contact with an object that is cooler.

What is the role of amniotic fluid?

Consists of 800-1200 mL by the end of pregnancy. Amniotic fluid surrounds, cushions, and protects the fetus and allows for fetal movement. The amniotic fluid maintains the body temperature of the fetus.

Exhibit The nurse is observing a pregnant client receiving an oxytocin infusion for induction of labor. The baseline fetal heart rate is 140/min; the strip is shown in the exhibit. What is the nurse's best course of action? Click on the exhibit button for additional information.

Continue to monitor the client

After giving birth to a full-term neonate, the client tells the practical nurse, "I have been taking hydrocodone on a regular basis for several years." The practical nurse collaborates with the registered nurse to include which intervention in the neonate's plan of care?

Feed newborn while swaddled

The healthcare provider prescribes terbutaline (Brethine) for a client in preterm labor. Before initiating this prescription, it is most important for the nurse to assess the client for which condition?

Gestational diabetes.

What are the indications for rubella vaccine after pregnancy?

Give subcutaneously before hospital discharge to a non-immune postpartum client with a rubella titer is less than 1:8.

Early detection of rheumatoid arthritis can decrease the amount of bone and joint destruction and often the disease will go into remission. What activity recommendations should the nurse provide a client with rheumatoid arthritis? A. Exercise of painful, swollen joints to strengthen them B. Exercise joint to the point of pain so that the pain lessens C. Make Jerky movements during the exercise so that the pain lessens D. Perform exercises slowly and smoothly

D. A nurse should advise the client to perform exercises slowly and smoothly so that no extra pain occurs

A nurse must use knowledge base to differentiate between abruptio placentae from plaventa previa. What assessments should be done in case of a patient suspected of abruptio placentae or placenta previa. A. abdominal or vaginal manipulation B. Leopold's maneuvers C. internal monitoring D. Monitor for bleeding at IV sites and gums due to increased risk of DIC

D. Patients with abruptio placentae or placventa previa should have No abdominal or vaginal manipulation. No Leopold's maneuvers. No vaginal exams. No rectal exams, enemas, or suppositories. No internal monitoring

When assessing a client who is at 12 week gestation, the nurse recommends that she and her husband consider attending childbirth preparation classes. When is the best time for the couple to attend these classes? A. at 16 weeks gestation B.at 20 weeks gestation C. at 24 weeks gestation D. at 30 weeks gestation

D. At 30 weeks gestation.

An off-duty nurse finds a woman in a supermarket parking lot delivering an infant while her husband is screaming for someone to help his wife. Which intervention has the highest priority? A.Use a thread to tie off the umbilical cord. B.Provide as much privacy as possible for the woman. C.Reassure the husband and try to keep him calm. D.Put the newborn to breast

D.Put the newborn to breast

A woman with Type 2 diabetes mellitus becomes pregnant, and her oral hypoglycemic agents are discontinued. Which intervention is most important for the nurse to implement?

Describe diet changes that can improve the management of her diabetes.

The nurse is caring for a 10-year-old girl with iron toxicity. Which of the following would the nurse expect the physician to order? a) Edentate calcium disodium b) Succimer c) Desferal d) Dimercaprol

Desferal Desferal is indicated for iron toxicity. It binds with iron, which is removed via the kidneys. Dimercaprol is indicated for blood lead levels greater than 45 mcg/dL. It removes lead from soft tissues and bone, allowing for its excretion via the renal system. Edentate calcium disodium is indicated for blood lead levels greater than 45 mcg/dL. The medication removes lead from soft tissues and bone, allowing for its excretion via the renal system. Succimer is indicated for blood lead levels greater than 45 mcg/dL; it removes lead from soft tissues and bone, allowing for its excretion via the renal system.

The nurse is caring for a large-for-gestational-age newborn (also known as macrosomia). What maternal condition is the usual cause of this condition? a) Diabetes b) Celiac disease c) Alcohol use d) Hypertension

Diabetes Correct Explanation: In the condition known as macrosomia, a newborn is born large for gestational age (LGA). These newborns are those with birth weights that exceed the 90th percentile of newborns of the same gestational age. They are born most often to mothers with diabetes.

What is lochia?

Discharge from the uterus that consists of blood from the vessels of the placental site and debris from the decidua, lasts 2 to 3 weeks after delivery.

A nurse caring for an 8-year-old patient with a bleeding disorder documents the following nursing diagnosis: ineffective tissue perfusion related to intravascular thrombosis and hemorrhage. This diagnosis is most appropriate for a patient with: a) Hemophilia b) Iron deficiency anemia c) Disseminated intravascular coagulation d) von Willebrand disease

Disseminated intravascular coagulation Explanation: Disseminated intravascular coagulation (DIC) is an acquired coagulopathy that, paradoxically, is characterized by both thrombosis and hemorrhage. The outcome for this patient is: The child will maintain adequate tissue perfusion of all body systems affected by DIC and regain adequate laboratory values for hemostasis

Exhibit A nurse auscultates a loud cardiac murmur on a newborn with suspected trisomy 21 (Down syndrome). A genetic screen and an echocardiogram are scheduled that day. The neonate's vital signs are shown in the exhibit. What would be an appropriate action for the nurse to complete next? Click on the exhibit button for additional information.

Document the finding

S/Sx of ITP

Early signs: Bleeding, ecchymoses, pinpoint petechial rash Other manifestations: bleeding gums/lips, epistaxis, hemarthosis, melena, hematemesis, menorrhagia, and most serious complication is intracranial hemorrhage (bleeding into the subdural, subarachnoid, or intracerebellar space)

The nurse is caring for a woman with a previously diagnosed heart disease who is in the second stage of labor. Which assessment findings are of greatest concern?

Edema, basilar rales, and an irregular pulse.

The total bilirubin level of a 36-hour, breastfeeding newborn is 14 mg/dl. Based on this finding, which intervention should the nurse implement?

Encourage the mother to breastfeed frequently.

A client at 32-weeks gestation is diagnosed with preeclampsia. Which assessment finding is most indicative of an impending convulsion?

Epigastric pain. Epigastric pain (C) is indicative of an edematous liver or pancreas which is an early warning sign of an impending convulsion (eclampsia) and requires immediate attention.

The primary intervention for beta-thalassemia is a chronic transfusion program of packed white blood cells with iron chelation. a) False b) True

False Explanation: The primary intervention for beta-thalassemia is a chronic transfusion program of packed red blood cells with iron chelation. Such a program facilitates adequate oxygenation of body tissues and practically eliminates all symptoms of thalassemia.

A 28-year-old client in active labor complains of cramps in her leg. What intervention should the nurse implement?

Extend the leg and dorsiflex the foot.

A client in active labor complains of cramps in her leg. What intervention should the nurse implement?

Extend the leg and dorsiflex the foot.

A toddler who is beginning to walk has fallen and hit his head on the corner of a low table. The caregiver has been unable to stop the bleeding and brings the child to the pediatric clinic. The nurse is gathering data during the admission process and notes several bruises and swollen joints. A diagnosis of hemophilia is confirmed. This child most likely has a deficiency of which of the following blood factors? a) Factor X b) Factor V c) Factor XIII d) Factor VIII

Factor VIII Explanation: The most common types of hemophilia are factor VIII deficiency and factor IX deficiency, which are inherited as sex-linked recessive traits, with transmission to male offspring by carrier females.

An obviously pregnant client walks into the emergency department screaming, "I am about to give birth!" What questions are essential to ask in preparation for and performance of possible neonatal resuscitation? Select all that apply.

Fetal distress and cesarean birth

Which of the following would the nurse be least likely to assess in a child with a hematologic disorder? a) Anemia b) Fever c) Abnormal hemostasis d) Neutropenia

Fever Explanation: Pediatric hematologic alterations usually are characterized by atypical hemostasis, anemia, and/or neutropenia. Fever suggests infection which may or may not be present with a hematologic disorder.

The practical nurse is monitoring a client 12 hours after the prolonged vaginal delivery of a term infant. Which finding should be reported to the registered nurse?

Foul-smelling lochia

The nurse is teaching care of the newborn to a group of prospective parents and describes the need for administering antibiotic ointment into the eyes of the newborn. Which infectious organism will this treatment prevent from harming the infant?

Gonorrhea. chlamydia

The nurse attempts to help an unmarried teenager deal with her feelings following a spontaneous abortion at 8-weeks gestation. What type of emotional response should the nurse anticipate?

Grief related to her perceptions about the loss of this child.

Leukemia

Group of malignant diseases of the bone marrow and lymphatic system

A newborn that has a surfactant deficiency will have which assessment noted on a physical exam: a) Pink skin b) Regular respirations c) Hypertension d) Grunting

Grunting Correct Explanation: Infants that are deficient in lung surfactant will show signs of respiratory distress: grunting, retracting, tachypnea, cyanosis, poor perfusion, hypotension, and skin mottling.

What type of pelvis is the normal female pelvis?

Gynecoid pelvis.

Doctors recommendations before considering a splenectomy for a child w/ ITP?

HCP generally recommends waiting until the child is more than 5yrs of age because of the increased risk of bacterial infection. And HCP orders pneumococcal and meningococcal vaccines before splenectomy

A boy with hemophilia A is scheduled for surgery. Which of the following precautions would you institute with him? a) Do not allow a dressing to be applied postoperatively. b) Caution him not to brush his teeth before surgery. c) Handle him gently when transferring him to a stretcher. d) Mark his chart for him to receive no analgesia

Handle him gently when transferring him to a stretcher. Explanation: Gentle handling can reduce bruising. Analgesia will be needed postoperatively; IM injections are contraindicated because of potential bleeding.

A client has delivered a small for gestation age (SGA) newborn. Which of the following would the nurse expect to assess? a) Head larger than body b) Brown lanugo body hair c) Round flushed face d) Protuberant abdomen

Head larger than body Explanation: Head being larger than the rest of the body is the characteristic feature of small for gestational age infants. Small for gestational age infants weigh below the 10th percentile on the intrauterine growth chart for gestational age. The heads of SGA infants appear larger in proportion to their body. They have an angular and pinched face and not a rounded and flushed face. Round flushed face and protuberant abdomen are the characteristic features of large for gestational age (LGA) infants. Preterm infants, and not SGA infants, are covered with brown lanugo hair all over the body.

What can cause paralysis in a patient w/ Hemophilia?

Hematomas in the spinal cord can cause paralysis

A client at 20 weeks gestation states that she started consuming an increased amount of cornstarch about 3 weeks ago. Based on this assessment, the nurse should anticipate that the health care provider will order which laboratory test(s)?

Hemoglobin and hematocrit levels

S/Sx of iron-deficiency anemia

Hgb values 6 to 10 g/dL: Irritability and weakness, decreased play activity, fatigue Hgb value falls below 5 g/dL: anorexia, skin pallor, pale mucous membranes, glossitis (inflammation of the tongue), concave or "spoon" fingernails, inability to concentrate, tachycardia, systolic murmurs

Immediate care is most dependent on the infant's current status (i.e., Apgar scores at 1 and 5 minutes) and any treatment or resuscitation that was indicated. The transitional care nurse needs the information listed in the choices

Increase the rate of IV fluids.

A primigravida at 40-weeks gestation is receiving oxytocin (Pitocin) to augment labor. Which adverse effect should the nurse monitor for during the infusion of Pitocin?

Hyperstimulation. Pitocin causes the uterine myofibril to contract, so unless the infusion is closely monitored, the client is at risk for hyperstimulation (B) which can lead to tetanic contractions, uterine rupture, and fetal distress or demise

A woman with diabetes has just given birth. While caring for this neonate, the nurse is aware that he's at risk for which complication? a) Anemia b) Hypoglycemia c) Nitrogen loss d) Thrombosis

Hypoglycemia Correct Explanation: Neonates of mothers with diabetes are at risk for hypoglycemia due to increased insulin levels. During gestation, an increased amount of glucose is transferred to the fetus through the placenta. The neonate's liver can't initially adjust to the changing glucose levels after birth. This inability may result in an overabundance of insulin in the neonate, causing hypoglycemia. Neonates of mothers with diabetes aren't at increased risk for anemia, nitrogen loss, or thrombosis.

A nurse is preparing a 7-year-old girl for bone marrow aspiration. Which of the following sites should she prepare? a) Sternum b) Femur c) Anterior tibia d) Iliac crest

Iliac crest Bone marrow aspiration provides samples of bone marrow so the type and quantity of cells being produced can be determined. In children, the aspiration sites used are the iliac crests or spines (rather than the sternum, which is commonly used in adults) because performing the test at these sites is usually less frightening for children; these sites also have the largest marrow compartments during childhood. In neonates, the anterior tibia can be used as an additional site.

What happens after a confirmed diagnosis of ALL?

Immediately after confirmed dx, inductions therapy begins and lasts 4 to 6 weeks and principal dugs that serve for induction in ALL are corticosteroids (especially prednisone), vincristine, and L-asparaginase, with or without doxorubicin

A nurse completes the initial assessment of a newborn. According to the due date on the antenatal record, the baby is 12 days postmature. Which of the following physical findings does not confirm that this newborn is 12 days postmature? a) Increased amounts of vernix. b) Absence of lanugo. c) Meconium aspiration. d) Hypoglycemia.

Increased amounts of vernix. Correct Explanation: Vernix caseosa is a whitish substance that serves as a protective covering over the fetal body throughout the pregnancy. Vernix usually disappears by term gestation. It is highly unusual for a 12-day postmature baby to have increased amounts of vernix. A discrepancy between EDC and gestational age by physical examination must have occurred. Meconium aspiration is a sign of fetal distress but does not coincide with gestation. The presence of lanugo is greatest at 28 to 30 weeks and begins to disappear as term gestation approaches. Therefore, an absence of lanugo on assessment would be expected with a postmature infant. Hypoglycemia can occur at any gestation.

The child with Thalassemia may be given which of the following classifications of medications to prevent one of the complications frequently seen with the treatment of this disorder. a) Iron-chelating drugs b) Factor VIII preparations c) Potassium supplements d) Vitamin supplements

Iron-chelating drugs Explanation: Frequent transfusions can lead to complications and additional concerns for the child, including the possibility of iron overload. For these children, iron-chelating drugs such as deferoxamine mesylate (Desferal) may be given. Vitamin and potassium supplements would not be given to treat the iron overload. Factor VIII preparations are given to the child with hemophilia.

client who is in the second trimester of pregnancy tells the nurse that she wants to use herbal therapy. Which response is best for the nurse to provide?

It is important that you want to take part in your care. The emphasis of alternative and complementary therapies, such as herbal therapy, is that the client is viewed as a whole being, capable of decision-making and an integral part of the health care team, so (D) recognizes the client's request

A newborn is designated as very low birthweight. The nurse understands that this newborn's weight is: a) Approximately 2,500 g b) More than 4,000 g c) Less than 1,500 g d) Less than 1,000 g

Less than 1,500 g Correct Explanation: A very-low-birthweight newborn weighs less than 1,500 g. A large-for-gestational-age newborn typically weighs more than 4,000 g. A small-for-gestational-age newborn or a low-birthweight newborn typically weighs about 2,500 g. An extremely-low-birthweight newborn weighs less than 1,500 g.

An expectant father tells the nurse he fears that his wife "is losing her mind." He states she is constantly rubbing her abdomen and talking to the baby, and that she actually reprimands the baby when it moves too much. What recommendation should the nurse make to this expectant father?

Let him know that these behaviors are part of normal maternal/fetal bonding which occur once the mother feels fetal movement. These behaviors are positive signs of maternal/fetal bonding

The nurse is evaluating the complete blood count of a 7-year-old child with a suspected hematological disorder. Which of the following findings would be associated with an elevated mean corpuscular volume (MCV)? a) Hemoglobin (Hgb) of 11.2 g/dL b) Platelet count of 250,000 c) Macrocytic red blood cells (RBCs) d) Decreased white blood cells (WBCs)

Macrocytic red blood cells (RBCs) Explanation: When the MCV is elevated, the RBCs are larger and referred to as macrocytic. The WBC count does not affect the MCV. The platelet count and Hgb are within normal ranges for a 7-year-old child.

What are the disadvantages of epidural anesthesia?

Maternal hypotension and 10-20 minutes to take effect.

When reviewing the medical record of a newborn who is large for gestational age (LGA), which of the following factors would the nurse identify as having increased the newborn's risk for being LGA? a) Fetal exposure to low estrogen levels b) Low weight gain during pregnancy c) Maternal pregravid obesity d) Low maternal birth weight

Maternal pregravid obesity Correct Explanation: The nurse should identify maternal pregravid obesity as a risk factor for the development of LGA newborns. The other risk factors for the development of LGA newborns include fetal exposure to high estrogen, excess weight gain during pregnancy, gestational diabetes and high maternal birth weight.

A pregnant client tells the nurse that the first day of her last menstrual period ␣␣␣␣␣␣␣␣␣␣␣␣␣␣␣␣␣␣␣␣␣␣␣␣␣␣␣␣␣␣␣␣␣␣␣␣␣␣␣␣␣␣␣␣␣␣␣␣␣␣␣␣␣␣␣␣␣␣␣␣␣␣␣␣␣␣␣␣␣␣␣␣␣␣ delivery?

May 29, 2007. ␣␣␣␣␣␣␣␣ rule is used to calculate the expected date of delivery, and is obtained by subtracting 3 months and adding 7 days beginning from the first day of the last normal menstrual period.

Exhibit The nurse is monitoring a newborn in the well-baby nursery. The nurse observes skin discoloration as shown in the exhibit. What would be an appropriate action for the nurse to complete? Click on the exhibit button for additional information.

Measure and document the size and location of the markings

A 38-week primigravida who works as a secretary and sits at a computer 8 hours each day tells the nurse that her feet have begun to swell. Which instruction would be most effective in preventing pooling of blood in the lower extremities?

Move about every hour

During a prenatal visit, the nurse discusses with a client the effects of smoking on the fetus. When compared with nonsmokers, mothers who smoke during pregnancy tend to produce infants who have A. lower Apgar scores. B. lower birth weights. C. respiratory distress. D. a higher rate of congenital anomalies.

Move about every hour.

Question: A group of nursing students are reviewing the process of blood cell formation. The students demonstrate understanding of this process when they place the following events in the proper sequence

Multipotent stem cell Myeloid progenitor Megakaryocyte/erythroid progenitor Megakaryocyte Platelets *There is a picture that shows the process on page 912* Explanation: The process begins with the multipotent stem cell that then becomes a myeloid progenitor, then a megakaryocyte/erythroid progenitor, a megakaryocyte, and finally platelets

Coarctation of the Aorta

Narrowing of the lumen of the Aorta, resulting in increased pressure proximal to the defect (head and upper extremities) and decreased pressure distal to the defect (body and lower extremities)

When a newborn takes his first breath, what happens?

Newborns lungs expand, and the fluid within them is absorbed into the pulmonary circulation. As a result, pulmonary and right-sided heart pressures fall and, with the removal of the placenta, systemic pressures rise

A 6-year-old boy visits the doctor's office with his mother. He has a rash on his buttocks, posterior thighs, and the extensor surface of his arms and legs. His joints are tender and swollen. The physician diagnoses him with Henoch-Schönlein syndrome. The nurse should expect which of the following laboratory results in this case? a) Elevated platelet count b) Decreased platelet count c) Decreased white blood cell count d) Normal platelet count

Normal platelet count Explanation: In Henoch-Schönlein syndrome, laboratory studies show a normal platelet count. Sedimentation rate, WBC count, and eosinophil count are elevated

On admission to the prenatal clinic, a 23-year-old woman tells the nurse that her last menstrual period began on February 15, and that previously her periods were regular. Her pregnancy test is positive. This client's expected date of delivery (EDD) is

November 22. Nagele's ␣␣␣␣␣␣ rule for estimating the due date by counting back 3 months from the first day of the last menstrual period (January, December, November) and adding 7 days (15+7=22).

The nurse identifies crepitus when examining the chest of a newborn who was delivered vaginally. Which further assessment should the nurse perform?

Observe for an asymmetrical Moro (startle) reflex. The most common neonatal birth trauma due to a vaginal delivery is fracture of the clavicle. Although an infant may be asymptomatic, a fractured clavicle should be suspected if an infant has limited use of the affected arm, malposition of the arm, an asymmetric Moro reflex (B), crepitus over the clavicle, focal swelling or tenderness, or cries when the arm is moved.

The nurse observes a new mother avoiding eye contact with her newborn. Which action should the nurse take?

Observe the mother for other attachment behaviors.

To assess for other attachment behaviors, continued observation of the new mother's interactions with her infant

Observe the mother for other attachment behaviors. helps the nurse determine problems in attachment. Ask the mother why she won't look at the infant may cause undue confusion, stress, or impact the mother's self-confidence.

Diagnostic tests for Hemophilia

PT, INR, bleeding time, normal platelet count, PTT, levels of factors VIII and IX *(PT-prothrombin time; INR-international normalized ratio; PTT- partial thromboplastin time)*

S/Sx of Hemophilia

Pain and limiting movement as a sign of internal bleeding in the joints (hemarthrosis) and the muscles.

Medical management of Tetralogy of Fallot

Palliative procedure, most commonly a Blalock-Taussig shunt (creates an artificial connection between the pulmonary artery and the aorta and thus redirects blood flow back to the lungs to allow for oxygenation

S/Sx of Sickle Cell Anemia

Pallor, irritability, fatigue, jaundice, pain, atelectasis, cardiomegaly and HF (in response to hypoxia and decreased cardiac output), persistent priapism

Patients w/ Hemophilia who are at risk for HIV?

Patients who received blood transfusions before the use of the current purification techniques (before 1985) were at risk for exposure to HIV. In the 1990s the use of recombinant factor products decreased the risk

A pregnant client comes to the labor and delivery unit stating the water just broke at home. On assessment of the client's perineal area, the nurse visualizes a loop of umbilical cord protruding from the vagina. Which nursing intervention would be appropriate?

Position the client on hands and knees

The nurse is assessing a child and notices pinpoint hemorrhages appearing on several different areas of the body. The hemorrhages do not blanch on pressure. The nurse documents this finding as which of the following? a) Poikilocytosis b) Purpura c) Ecchymosis d) Petechiae

Petechiae Explanation: Petechiae are pinpoint hemorrhages that occur anywhere on the body and do not blanch with pressure. Purpura are larger areas of hemorrhage in which blood collects under the tissues and appear purple in color. Ecchymosis refers to areas of bruising. Poikilocytosis refers to the variation in the size and shape of the red blood cells commonly found in children with thalassemia.

A neonate is born with exstrophy of the bladder. Which intervention would be appropriate for the nurse to complete first?

Place a protective film over the bladder

The nurse monitoring a newborn after birth observes a bluish discoloration of the hands and feet. The trunk has a pink color. What is the nurse's initial action?

Place infant skin-to-skin with mother

What is the fundal height measurement method?

Place the client in the supine position. Place the end of the tape measure at the symphysis pubis and measure to the top of the uterine fundus.

When preparing to resuscitate a preterm newborn, which of the following would the nurse do first? a) Use positive-pressure ventilation. b) Administer epinephrine. c) Hyperextend the newborn's neck. d) Place the newborn's head in a neutral position.

Place the newborn's head in a neutral position. Correct Explanation: When preparing to resuscitate a preterm newborn, the nurse should position the head in a neutral position to open the airway. Hyperextending the newborn's neck would most likely close off the airway and is inappropriate. Positive-pressure ventilation is used if the newborn is apneic or gasping or the pulse rate is less than 100 beats per minute. Epinephrine is given if the heart rate is less than 60 beats per minute after 30 seconds of compression and ventilation.

Twenty minutes after a continuous epidural anesthetic is administered, a laboring client's blood pressure drops from 120/80 to 90/60. What action should the nurse take?

Place the woman in a lateral position

What fetal maneuver will encourage fetal rotation from occiput posterior to occiput anterior?

Placing the mother on all fours.

A preterm infant is placed on ventilatory assistance for respiratory distress syndrome. In light of her lung pathology, which additional ventilatory measure would you anticipate planning? a) Administration of chilled oxygen to reduce lung spasm b) Positive end-expiratory pressure to increase oxygenation c) Increased inspiratory pressure; decreased expiratory pressure d) Administration of dry oxygen to avoid over-humidification

Positive end-expiratory pressure to increase oxygenation Correct Explanation: Positive end-expiratory pressure, like expiratory grunting, prevents alveoli from fully closing on expiration and reduces the respiratory effort needed for inspiration.

What is premature rupture of membranes?

Premature rupture of the membranes is spontaneous rupture of the amniotic membrane before the onset of labor. When the rupture of membranes is before term infection becomes a risk.

An 18-year-old client has given birth to a very-low-birth-weight preterm infant. Which of the following should the nurse consider to prevent the newborn from losing body temperature? a) Hold the newborn close, rocking gently. b) Provide isolette or radiant warmer care to the newborn. c) Give the newborn a warm water bath. d) Administer vitamin K to the newborn.

Provide isolette or radiant warmer care to the newborn. Correct Explanation: The nurse should place the infant in an isolette to simulate the uterine environment as closely as possible and to keep the infant warm. The isolette maintains even levels of temperature, humidity, and oxygen. A hood covers it, and nurses can give care through portholes. Holding and frequent handling of the newborn should be avoided to prevent loss of energy. Minimal handling helps the neonate to conserve energy. Administration of vitamin K to the infant is necessary to prevent bleeding in the infant because the newborn is unable to produce its own vitamin K during the early stages of life. It does not help in providing warmth to the baby. The infant is not given baths until later because this often results in loss of body temperature.

An off-duty nurse finds a woman in a supermarket parking lot delivering an infant while her husband is screaming for someone to help his wife. Which intervention has the highest priority?

Put the newborn to breast. Putting the newborn to breast (D) will help contract the uterus and prevent a postpartum hemorrhage--this intervention has the highest priority. It is not necessary to tie off the umbilical cord

The nurse enters the room and notices that the infant is in the crib against the window. What type of heat loss may this infant suffer? a) Radiation b) Convection c) Evaporation d) Conduction

Radiation Correct Explanation: Radiation heat loss results from the transfer of heat in an environment from warmer to cooler objects that are not in direct contact with each other.

The nurse is reinforcing discharge instructions to a postpartum client. Which instruction should the nurse include to promote newborn safety?

Remove pillows and loose blankets from the infant's crib

What would be appropriate for the nurse to document in a child suffering from meconium aspiration syndrome? a) Respirations as increased and high b) Skin as pink c) Chest expansion as normal d) Heart rate as normal

Respirations as increased and high Correct Explanation: Infants with meconium aspiration syndrome may show signs of respiratory distress (tachypnea, cyanosis, retractions, chest retractions). The other choices document normal findings.

What are the assessments for meconium aspiration syndrome?

Respiratory distress at birth; tachypnea, cyanosis, retractions, nasal flaring, grunting, crackles, and rhonchi. Nails, skin, and umbilical cord may be stained yellow-green.

Exhibit A client at 38 weeks gestation is in labor and receiving an oxytocin infusion. The continuous fetal heart rate (FHR) monitor displays the strip shown in the exhibit. Which action by the nurse is most appropriate? Click on the exhibit button for additional information.

Review medication administration record

When developing the postoperative plan of care for a child with sickle cell anemia who has undergone a splenectomy, which of the following would the nurse identify as the priority? a) Impaired skin integrity b) Risk for delayed growth and development c) Risk for infection d) Deficient fluid volume

Risk for infection Explanation: Removal of the spleen places the child at significant risk for infection. Although the child's skin integrity is disrupted due to the surgery, this is not the priority nursing diagnosis. Loss of fluids occurs during surgery and adequate hydration is important to prevent a sickle cell crisis, but this diagnosis is not the priority in the postoperative period. Although the child is at risk for delayed growth and development, the priority postoperatively is to prevent infection

Absence of menses for 6 months or more in a client with prior normal menses is known as?

Secondary amenorrhea

A client at 39 weeks gestation with preeclampsia has a blood pressure of 170/100 mm Hg, 2+ proteinuria, and moderate peripheral edema. Immediately after hospital admission, she develops seizures and uterine contractions. Magnesium sulfate is prescribed. Which finding indicates that the drug has achieved the desired therapeutic effect?

Seizure activity stops

The nurse is administering meperidine as ordered for pain management for a 10-year-old boy in sickle cell crisis. The nurse would be alert for which of the following? a) Priapism b) Leg ulcers c) Behavioral addiction d) Seizures

Seizures Explanation: Repeated use of meperidine for pain management during sickle cell crisis increases the risk of seizures when used in children with sickle cell anemia. Behavioral addiction is rarely a concern in the child with sickle cell anemia if the narcotic is used for the alleviation of severe pain. Priapism is a complication of sickle cell anemia unrelated to meperidine administration. Leg ulcers are a complication of sickle cell anemia unrelated to meperidine administration.

The nurse is calculating the estimated date of confinement (EDC) using ␣␣␣␣␣␣␣␣ rule for a client whose last menstrual period started on December 1. Which date is most accurate?

September 8.

Hemophilia

Serious, lifelong bleeding disease inherited as an X-linked recessive disorder

The nurse is caring for a 3-year-old boy with suspected iron-deficiency anemia. Which test would the nurse expect to be ordered to confirm the diagnosis? a) Hemoglobin electrophoresis b) Reticulocyte count c) Iron test d) Serum ferritin

Serum ferritin Serum ferritin is a measure of ferritin (the major iron storage protein) in the blood. It is the most sensitive test for determination of iron-deficiency anemia. Hemoglobin electrophoresis is indicated for sickle cell anemia and thalassemia and measures the percentage of normal and abnormal hemoglobin in the blood. Reticulocyte count measures the number of immature red blood cells (RBCs) in the blood and indicates the bone marrow's ability to respond to anemia with production of RBCs. The iron test evaluates iron metabolism

A client in the postpartum unit has a temperature of 100.9 F (38.3 C) and tachycardia on the second day following a cesarean delivery. Examination shows uterine tenderness, fundus +2 above the umbilicus, moderate lochia rubra with a foul smell, and chills. Which prescription should the nurse implement first?

Serum laboratory draws for blood culture and sensitivity

What disorder of pregnancy is characterized by BP >160/110, proteinuria >5000, or 3-4+ on dipstick?

Severe preeclampsia

S/Sx of atrial septal defect

Smaller defects <5cm in diameter are often asymptomatic. Respiratory infections, difficulty breathing, harsh systolic murmur heard at the third intercostal space

The nurse is examining the hands of a child with suspected iron deficiency anemia. Which of the following would the nurse expect to find? a) Spooning of nails b) Absence of bruising c) Capillary refill in less than 2 seconds d) Pink palms and nail beds

Spooning of nails Explanation: A convex shape of the fingernails termed 'spooning' can occur with iron deficiency anemia. Capillary refill in less than 2 seconds, pink palms and nail beds, and absence of bruising are normal findings.

What is stage 1 of labor?

Stage 1 is the longest stage of labor. Cervical dilation is 1-4 cm. Uterine contractions occur every 15 to 30 minutes, are 15 to 30 seconds in duration and are of mild intensity.

Meconium is the first stool passed in a newborn. What would be the correct documentation of the meconium? a) Soft brown b) Sticky forest green c) Seedy yellow d) Formed green

Sticky forest green Correct Explanation: Meconium is usually a sticky, forest-green liquid. It contains bile acids, salts, and mucus. The other choices describe stool at various stages after the passage of meconium.

A pregnant client has labor induced with oxytocin infusion. The practical nurse notes that the fetal heart rate (FHR) tracing shows a change in the baseline rate from 145/min to 170/min and minimal variability. What is the nurse's first action?

Stop oxytocin infusion

When a child is beginning antidepressant medications, it is important for parents to be alert for signs of ________________ ________________ for the first 2 weeks of treatment.

Suicidal thoughts Pg. 1061 -Foundations of Nursing

What is peripartum supine hypotension?

Supine hypotension occurs when the venous return to the heart is impaired by the weight of the uterus on the vena cava.

The parents of a child with a bleeding disorder ask the nurse about appropriate activities and sports that they should encourage the child to participate in. Which of the following would be the safest for the nurse to suggest? a) Swimming b) Gymnastics c) Rugby d) Soccer

Swimming Explanation: Swimming, a noncontact sport or activity, would be the safest for the nurse to recommend. Soccer and gymnastics may be appropriate; however, these are considered riskier. Rugby would not be recommended because the risks outweigh the benefits.

A 26-year-old, gravida 2, para 1 client is admitted to the hospital at 28-weeks gestation in preterm labor. She is given 3 doses of terbutaline sulfate (Brethine) 0.25 mg subcutaneously to stop her labor contractions. The nurse plans to monitor for which primary side effect of terbutaline sulfate?

Tachycardia and a feeling of nervousness.

primary side effect. Terbutaline sulfate (Brethine), a

Tachycardia and a feeling of nervousness.

A woman who had a miscarriage 6 months ago becomes pregnant. Which instruction is most important for the nurse to provide this client?

Take prescribed multivitamin and mineral supplements. A client who has had a spontaneous abortion or still birth in the last 11⁄2 years should take multivitamin and mineral supplements (D) and maintain a balanced diet because the previous pregnancy may have left her nutritionally depleted

A newborn girl who was born at 38 weeks of gestation weighs 2000 g and is below the 10th percentile in weight. The nurse recognizes that this girl will most likely be classified as which of the following? a) Term, small for gestational age, and very-low-birth-weight infant b) Term, small for gestational age, and low-birth-weight infant c) Late preterm and appropriate for gestational age d) Late preterm, large for gestational age, and low-birth-weight infant

Term, small for gestational age, and low-birth-weight infant Correct Explanation: Infants born before term (before the beginning of the 38th week of pregnancy) are classified as preterm infants, regardless of their birth weight. Term infants are those born after the beginning of week 38 and before week 42 of pregnancy. Infants who fall between the 10th and 90th percentiles of weight for their gestational age, whether they are preterm, term, or postterm, are considered appropriate for gestational age (AGA). Infants who fall below the 10th percentile of weight for their age are considered small for gestational age (SGA). Those who fall above the 90th percentile in weight are considered large for gestational age (LGA). Still another term used is low-birth-weight (LBW; one weighing under 2500 g at birth). Those weighing 1000 to 1500 g are very-low-birth-weight (VLB). Those born weighing 500 to 1000 g are considered extremely very-low-birth-weight infants (EVLB).

The nurse is collecting data from the caregivers of a child brought to the clinic setting. The parents tell the nurse that the child's skin seems to be an unusual color. The nurse notes that the child's skin appears bronze-colored and jaundiced. This observation alerts the nurse to the likelihood that this child has which of the following disorders? a) Hemophilia b) Kawasaki disease c) Thalassemia d) Sickle cell disease

Thalassemia Explanation: In the child with Thalassemia the skin may appear bronze-colored or jaundiced. The child with hemophilia may have bruised areas on the skin. The skin color in children with sickle cell disease may be pale in color, and with Kawasaki disease the child may have a rash on the trunk and extremities

A new mother who has just had her first baby says to the nurse, "I saw the baby in the recovery room. She sure has a funny looking head." Which response by the nurse is best?

That is normal;; the head will return to a round shape within 7 to 10 days.

For the child diagnosed with iron deficiency anemia which of the following would the nurse anticipate would be done in treating this disorder? a) The child would be given corticosteroids via a metered-dose inhaler. b) The child would be given enteric coated aspirin with milk. c) The child would be given ferrous sulfate with orange juice between meals. d) The child would be given a high dose of intravenous immunoglobulin.

The child would be given ferrous sulfate with orange juice between meals. Explanation: Treatment consists of improved nutrition, with ferrous sulfate administered between meals with juice (preferably orange juice, because vitamin C aids in iron absorption). For best results, iron should not be given with meals.

The nurse is preparing to assess a client visiting the women's health clinic. The client's obstetric history is documented as G5T1P2A1L2. Which interpretation of this notation is correct?

The client had 1 birth at 37 wk 0 d gestation or beyond

A patient is placed on bed rest at home for mild preeclampsia at 38 weeks gestation. Which of the following must the nurse teach the patient regarding her condition? 1. Eat a sodium restricted diet 2. Check her temperature 4 times daily 3. Report swollen hand and face 4. Limit fluids to 1 liter per day

The client should call her primary caregiver to report swollen hand and face.

Client teaching is an important part of the maternity nurse's role. Which factor has the greatest influence on successful teaching of the gravid client?

The client's readiness to learn.

A 20-year-old client gave birth to a baby boy during the 43rd week of gestation. Which of the following might the nurse observe in the newborn during routine assessment? a) The newborn may have short nails and hair. b) The infant may have excess of lanugo and vernix caseosa. c) The testes in the child may be undescended. d) The newborn may look wrinkled and old at birth.

The newborn may look wrinkled and old at birth. Correct Explanation: Postterm babies are those born past 42 weeks of gestation. These babies often appear wrinkled and old at birth. They often have long fingernails and hair, dry parched skin, and no vernix caseosa. Both the quantity of lanugo and the amount of vernix decrease with gestational age. Undescended testes are usually not seen in postterm newborns; however, they are highly prevalent in preterm infants.

The nurse observes a neonate delivered at 28 weeks' gestation. Which finding would the nurse expect to see? a) The pinna of the ear is soft and flat and stays folded. b) The neonate has 7 to 10 mm of breast tissue. c) The skin is pale, and no vessels show through it. d) Creases appear on the interior two-thirds of the sole.

The pinna of the ear is soft and flat and stays folded. Correct Explanation: The ear has a soft pinna that's flat and stays folded. Pale skin with no vessels showing through and 7 to 10 mm of breast tissue are characteristic of a neonate at 40 weeks' gestation. Creases on the anterior two-thirds of the sole are characteristic of a neonate at 36 weeks' gestation.

While breastfeeding, a new mother strokes the top of her baby's head and asks the nurse about the baby's swollen scalp. The nurse responds that the swelling is caput succedaneum. Which additional information should the nurse provide this new mother?

The scalp edema will subside in a few days after birth. Caput succedaneum is edema of the fetal scalp that crosses over the suture lines and is caused by pressure on the fetal head against the cervix during labor;; it subside in a few days after birth without treatment

A new mother is afraid to touch her baby's head for fear of hurting the "large soft spot." Which explanation should the nurse give to this anxious client?

There's a strong, tough membrane there to protect the baby so you need not be afraid to wash or comb his/her hair. (D) provides correct information and attempts to alleviate anxiety related to knowledge deficit. The anterior fontanel or "large soft spot" has a strong epidermal membrane present, which can be touched

How do contractions change during the transition phase of labor?

They increase in frequency, intensity, and duration.

A couple has been trying to conceive for nine months without success. Which information obtained from the clients is most likely to have an impact on the couple's ability to conceive a child?

They use lubricants with each sexual encounter to decrease friction. The use of lubricants (D) has the potential to affect fertility because some lubricants interfere with sperm motility.

A preterm newborn has just received synthetic surfactant through an endotracheal tube by a syringe. Which of the following interventions should the nurse implement at this point? a) Tip the infant into an upright position b) Immediately suction the infant's airway c) Take a blood sample d) Place the infant supine in a radiant heat warmer

Tip the infant into an upright position Explanation: It's important the infant is tipped to an upright position following administration of surfactant and the infant's airway is not suctioned for as long a period as possible after administration of surfactant to help it reach lower lung areas and avoid suctioning the drug away. A blood sample may be taken to rule out a streptococcal infection, which mimics the signs of RDS, but this would have been done before administration of surfactant. The infant should not be placed supine in a radiant heat warmer at this time but should be held in an upright position.

Patient teaching of iron Administration

To avoid staining the teeth with liquid preparation of iron, administer medication with a syringe placed towards back of mouth in infants and older children may use a straw

During labor, the nurse determines that a full-term client is demonstrating late decelerations. In which sequence should the nurse implement these nursing actions? (Arrange in order.) A. Reposition the client. B. Call the healthcare provider. C. Increase IV fluid. D. Provide oxygen via face mask.

To stabilize the fetus, intrauterine resuscitation is the first priority, and to enhance fetal blood supply, the laboring client should be repositioned (1) to displace the gravid uterus and improve fetal perfusion. Secondly, the IV fluids should be increased (2) to expand the maternal circulating blood volume. Next, to optimize oxygenation of the circulatory blood volume, oxygen via face mask (3) should be applied to the mother. Then, the primary healthcare provider should be notified (4) for additional interventions to resolve the fetal stress.

A preterm infant is transferred to a distant hospital for care. When her parents visit her, which of the following would be most important for you to urge them to do? a) Call the baby by her name. b) Touch and, if possible, hold her. c) Stand so the baby can see them. d) Bring a piece of clothing for her.

Touch and, if possible, hold her. Correct Explanation: Preterm infants may be hospitalized for an extended time, so parents need to be encouraged to touch and interact with the infant to begin bonding.

Medical management of Hemophilia

Treat small cuts with first aid, deep cuts may require replacing missing clotting factor

Iron-deficiency anemia could be virtually eliminated if all infants were breastfed and those infants who are formula-fed were fed iron-fortified formula for the full first year. a) False b) True

True

Preterm infant deaths account for 80% to 90% of infant mortality in the first year of life. a) False b) True

True

The nurse is assisting with a vaginal delivery of a full-term infant. Which assessment finding of the newborn is most important for the nurse to follow-up?

Tuft of hair at the base of the spine

A 35-year-old primigravida client with severe preeclampsia is receiving magnesium sulfate via continuous IV infusion. Which assessment data indicates to the nurse that the client is experiencing magnesium sulfate toxicity?

Urine output 90 ml/4 hours. Urine outputs of less than 100 ml/4 hours (D), absent DTRs, and a respiratory rate of less than 12 breaths/minute are cardinal signs of magnesium sulfate toxicity.

34. If a patient's normal prepregnancy diet contains 45 g of protein daily, how many more grams of protein should she consume per day during pregnancy? a. 5 b. 10 c. 25 d. 30

c. 25

A client delivers a newborn baby at term. The nurse records the weight of the baby as 1.2 kg, interpreting this to indicate that the newborn is of: a) Normal birth weight b) Very low birth weight c) Extremely low birth weight d) Low birth weight

Very low birth weight Correct Explanation: A birth weight of 1.2 kg would be classified as very low birth weight. A normal birth weight at term ranges between 2,500 g and 4,000 g. Typically it is between 3,000 g and 4,000 g. A birth weight below 2,500 g is termed a low birth weight. A birth weight between 1,000 g and 1,500 g is termed a very low birth weight. A birth weight less than 1,000 g is termed an extremely low birth weight

What are some interventions for varicose veins during pregnancy?

Wearing support hose. Elevating the feet when sitting. Lying with the feet and hips elevated. Avoiding long periods of standing or sitting. Avoid leg crossing. Avoid contracting articles of clothing.

Which maternal behavior is the nurse most likely to see when a new mother receives her infant for the first time?

Which maternal behavior is the nurse most likely to see when a new mother receives her infant for the first time?

What is the best time to bathe an infant? a. At bedtime b. Early in the morning c. After a feeding d. Before a feeding

d. Before a feeding Bathing is usually done before a feeding to reduce the possibility of vomiting, regurgitation, or stimulation. p. 957

A premature infant develops respiratory distress syndrome. With this condition, circulatory impairment is likely to occur because with increased lung tension, the a) foramen ovale closes prematurely. b) ductus arteriosus remains open. c) pulmonary artery closes. d) aorta or aortic valve strictures.

ductus arteriosus remains open. Explanation: Excess pressure in the alveoli stimulates the ductus arteriosus to remain open, compromising efficient cardiovascular function.

A common symptom that would alert you that a preterm infant is developing respiratory distress syndrome is a) inspiratory "crowing." b) expiratory grunting. c) inspiratory stridor. d) expiratory wheezing.

expiratory grunting. Correct Explanation: Expiratory grunting is a physiologic measure to ensure alveoli do not fully close on expiration (so they require less energy expenditure to reopen).

Epogen and bilirubin

is usually related to resolution of hyperbilirubinemia, treated with phototherapy or increased oral intake in the infant.

When planning care for a child with idiopathic thrombocytopenic purpura, you plan to teach her a) not to pick or irritate her nose. b) to apply a soothing cream to lesions. c) to use mainly cold water to wash. d) what foods are high in folic acid.

not to pick or irritate her nose. Explanation: Without adequate platelets, children bleed easily from lesions

The pediatric nurse, along with the primary caregiver(s), has a special duty to ________ the child and the family

teach The pediatric nurse is in a position to assess, instruct, and support children and their families about developmental progress, nutrition, and possible undiagnosed anomalies. p. 935

A 3-year-old female is brought to the ER by her parents and presents with bruising and mucous membrane bleeding from the nose and mouth. The nurse knows that these symptoms are indicative of: a) Disseminated intravascular coagulation b) von Willebrand disease c) Hemophilia d) Chronic iron deficiency anemia

von Willebrand disease Explanation: The primary clinical manifestations of von Willebrand disease are bruising and mucous membrane bleeding from the nose, mouth, and gastrointestinal tract; bleeding may be severe and lead to anemia and shock. Deep bleeding into joints and muscles, like that seen in hemophilia, is rare, except with type III von Willebrand disease.

During a prenatal visit, the nurse discusses with a client the effects of smoking on the fetus. When compared with nonsmokers, mothers who smoke during pregnancy tend to produce infants who have

weights.

A 10-month-old has been admitted to the hospital with severe hemolytic anemia and chronic hypoxia. The nurse notes icteral sclerae, jaundice of the skin, and frontal and maxillary bossing. The nurse interprets these findings as most likely indicating which of the following? a) Sickle cell anemia b) β-Thalassemia major c) Hemophilia d) von Willebrand disease

β-Thalassemia major Explanation: Severe hemolytic anemia and chronic hypoxia, icteral sclerae, jaundice of the skin, and frontal and maxillary bossing are signs and symptoms of β-thalassemia major. Hemophilia is manifested by clotting dysfunctions. von Willebrand disease is manifested by abnormal clotting. Sickle cell anemia involves abnormal hemoglobin that leads to significant anemia and acute and chronic symptoms.

The young girl has been diagnosed with a hematologic disorder. Her erythrocyte count is below normal. The mean corpuscular volume is below normal. The girl's mean corpuscular hemoglobin (Hbg) concentration is below normal. Which of the following statements by the girl's nurse is true regarding this girl? Select all that apply. a) "She's anemic." b) "Her red blood cells are smaller than normal." c) "The amount of hemoglobin in her red blood cells is very dilute." d) "Her red blood cells are macrocytic." e) "Her red blood cells are hypochromi"

• "Her red blood cells are smaller than normal." • "The amount of hemoglobin in her red blood cells is very dilute." • "She's anemic." • "Her red blood cells are hypochromi" Explanation: This girl's erythrocyte count is below normal, which indicates she is anemic. The mean corpuscular Hbg concentration is below normal which indicates that her cells are hypochromic with a diluted amount of Hbg available. The mean corpuscular volume of the erythrocytes are decreased which indicates her cells are microcytic or smaller than normal

A nurse is reviewing laboratory test results from several children, looking specifically at their thrombocyte levels. The nurse would identify that the child with which platelet level might be at risk for bleeding? Select all that apply. a) 80,000 per cubic millimeter b) 175,000 per cubic millimeter c) 110,000 per cubic millimeter d) 234,000 per cubic millimeter e) 287,000 per cubic millimeter

• 80,000 per cubic millimeter • 110,000 per cubic millimeter Explanation: Normal thrombocyte level ranges from 150,000 to 300,000 per cubic millimeter. Therefore, a child with a thrombocyte level of 80,000 and 110,000 per cubic millimeter would be at risk for bleeding.

Choice Multiple question - Select all answer choices that apply. The nurse is working with a child who is in sickle cell crisis. Treatment and nursing care for this child includes which of the following. Select all that apply. a) Promoting exercise and activity b) Administering analgesics c) Administering oxygen d) Maintaining fluid intake e) Preventing injury and bleeding episodes

• Administering oxygen • Administering analgesics • Maintaining fluid intake Explanation: Treatment for a crisis is supportive for each presenting symptom, and bed rest is indicated. Oxygen may be administered. Analgesics are given for pain. Dehydration and acidosis are vigorously treated. Prognosis is guarded, depending on the severity of the disease.

A client just delivered a preterm baby in the 30th week of gestation. The nurse knows that which nursing measures will be performed for this infant? Select all that apply. a) Estimate the urinary flow by weighing the diaper. b) Dress the baby in a stockinette cap. c) Dress the baby to keep the body warm. d) Place the baby under isolette care. e) Carry and handle the baby frequently.

• Dress the baby in a stockinette cap. • Place the baby under isolette care. • Estimate the urinary flow by weighing the diaper. Explanation: The nurse should dress the baby in a stockinette cap, place the baby under isolette care, and estimate the urinary flow by weighing the diaper. Controlling the temperature in high-risk newborns is often difficult; therefore, special care should be taken to keep these babies warm by dressing then in a stockinette cap and recording their temperature often. Isolette care simulates the uterine environment as closely as possible, thus maintaining even levels of temperature, humidity, and oxygen for the child. The isolette is transparent, so the newborn is visible at all times. The kidneys of preterm infants are not fully developed; hence, they may have difficulty eliminating wastes. The nurse should determine accurate output by weighing the diaper before and after the infant urinates. The diaper's weight difference in grams is approximately equal to the number of milliliters voided. Frequently carrying and handling the baby should be avoided so that the infant can conserve energy. Generally, preterm newborns in the high-risk category are not dressed, so the attending nurse can observe their breathing.

The nurse in the NICU is caring for preterm newborns. Which of the following are recommended guidelines for care of these newborns? Select all that apply. a) Take the newborn's temperature often. b) Discourage contact with parents to maintain asepsis. c) Dress the newborn in ways to preserve warmth. d) Supply oxygen for the newborn, if necessary. e) Handle the newborn as much as possible. f) Give the newborn a warm bath immediately.

• Dress the newborn in ways to preserve warmth. • Take the newborn's temperature often. • Supply oxygen for the newborn, if necessary. Explanation: Controlling the temperature of preterm newborns is often difficult; therefore, special care should be taken to keep these babies warm. Nurses should dress them in a stockinette cap, take their temperature often, and supply oxygen, if necessary. To conserve the energy of small newborns, the nurse should handle them as little as possible. Usually, they will not give them a bath immediately. Parents should be encouraged to bond with their infants.

A nurse is teaching a school-aged child with iron-deficiency anemia and her parents about dietary measures to increase iron intake. The nurse determines that the teaching was successful when they state which food is high in iron? Select all that apply. a) Eggs b) Fortified cereal c) Milk d) Citrus fruits e) Green leafy vegetables

• Eggs • Fortified cereal • Green leafy vegetables Explanation: Foods high in iron include meat, cheese, eggs, green leafy vegetables, and fortified cereal. Citrus fruits and milk are not iron-rich foods

A nurse is caring for a large for gestational age newborn. Which of the following signs would lead the nurse to suspect that the newborn is experiencing hypoglycemia? Select all that apply. a) Bulging fontanels b) Lethargy and stupor c) Appearance of central cyanosis d) Respiratory difficulty e) High-pitched shrill cry

• Lethargy and stupor • Respiratory difficulty • Appearance of central cyanosis Explanation: The features indicating hypoglycemia in LGA infants include lethargy, stupor and fretfulness, respiratory difficulty and central cyanosis. The other features include poor feeding in a previously well feeding infant and weak whimpering cry. High-pitched shrill cry and bulging fontanels are seen in increased intracranial pressure following head trauma in LGA infants.

A 5-year-old boy is diagnosed with congenital aplastic anemia. Which of the following symptoms should the nurse expect in this child? (Select all that apply.) a) Bradycardia b) Cyanosis c) Pallor d) Bradypnea e) Easy bruising f) Fatigue

• Pallor • Fatigue • Easy bruising • Cyanosis Explanation: When symptoms begin, a child appears pale, fatigues easily, and has anorexia from the lowered RBC count and tissue hypoxia. Because of reduced platelet formation (thrombocytopenia), the child bruises easily or develops petechiae (pinpoint, macular, purplish-red spots caused by intradermal or submucous hemorrhage). A child may have excessive nosebleeds or gastrointestinal bleeding. As a result of a decrease in WBCs (neutropenia) a child may contract an increased number of infections and respond poorly to antibiotic therapy. Observe closely for signs of cardiac decompensation such as tachycardia (not bradycardia), tachypnea (not bradypnea), shortness of breath, or cyanosis from the long-term increased workload of all these effects on the heart.

Which of the following would the nurse expect to find in a newborn who is considered small for gestational age? Select all that apply. a) Dry or thin umbilical cord b) Sunken abdomen c) Narrow skull sutures d) Poor muscle tone over buttocks e) Increased subcutaneous fat stores

• Sunken abdomen • Poor muscle tone over buttocks • Dry or thin umbilical cord Explanation: A small-for-gestational-age newborn typically has a sunken abdomen, wide skull sutures, decreased subcutaneous fat stores, poor muscle tone over buttocks and cheeks, and a thin umbilical cord.

Which of the following would be most effective in reducing pain in the preterm newborn? Select all that apply. a) Removing tape quickly from the skin b) Swaddling the newborn closely c) Offering a pacifier prior to a procedure d) Encouraging kangaroo care during procedures e) Increasing the volume on device alarms f) Using cool blankets to soothe the newborn

• Swaddling the newborn closely • Encouraging kangaroo care during procedures • Offering a pacifier prior to a procedure Explanation: Interventions to reduce pain in the preterm newborn include swaddling the newborn closely to establish physical boundaries, using gentle handling, rocking, caressing, and cuddling, encouraging kangaroo care during procedures, and offering a pacifier for nonnutritive sucking prior to a procedure. Tape should be used minimally and should be removed gently to prevent skin tearing. Environmental stimuli need to be reduced, such as by turning down the volumes on alarms. Warm rather than cool blankets facilitate relaxation.

A 17-month-old child would be expected to be in what stage according to Piaget? a. Trust b. Preoperations c. Secondary circular reaction d. Tertiary circular reaction

ANS: D The 17-month-old is in the fifth stage of the sensorimotor phase: tertiary circular reactions. The child uses active experimentation to achieve previously unattainable goals. Trust is Erikson's first stage. Preoperations is the stage of cognitive development usually present in older toddlers and preschoolers. Secondary circular reactions last from about ages 4 to 8 months.

Which suggestions should the nurse include when teaching about appropriate weight gain in pregnancy? (Select all that apply.) A. Underweight women should gain 12.5 to 18 kg. Correct B. Obese women should gain at least 7 to 11.5 kg. Correct C. Adolescents are encouraged to strive for weight gains at the upper end of the recommended scale. Correct D. In twin gestations, the weight gain recommended for a single fetus pregnancy should simply be doubled. E. Normal weight women should gain 11.5 to 16 kg.

A. Underweight women should gain 12.5 to 18 kg. B. Obese women should gain at least 7 to 11.5 kg. C. Adolescents are encouraged to strive for weight gains at the upper end of the recommended scale. E. Normal weight women should gain 11.5 to 16 kg. Underweight women need to gain the most. Obese women need to gain weight during pregnancy to equal the weight of the products of conception. Adolescents are still growing; therefore, their bodies naturally compete for nutrients with the fetus. Women bearing twins need to gain more weight (usually 16 to 20 kg) but not necessarily twice as much. Normal weight women should gain 11.5 to 16kg.

Which nursing intervention is most helpful in relieving postpartum uterine contractions or "afterpains?" A. Using relaxation breathing techniques B. Using a breast pump. C. Massaging the abdomen. Incorrect D. Giving oxytocic medications.

A. Using relaxation breathing techniques

With regard to protein in the diet of pregnant women, nurses should be aware that: A. many protein-rich foods are also good sources of calcium, iron, and b vitamins. Correct B. many women need to increase their protein intake during pregnancy. C. as with carbohydrates and fat, no specific recommendations exist for the amount of protein in the diet. D. high-protein supplements can be used without risk by women on macrobiotic diets.

A. many protein-rich foods are also good sources of calcium, iron, and b vitamins. Good protein sources such as meat, milk, eggs, and cheese have a lot of calcium and iron. Most women already eat a high-protein diet and do not need to increase their intake. Protein is sufficiently important that specific servings of meat and dairy are recommended. High-protein supplements are not recommended because they have been associated with an increased incidence of preterm births.

A child with croup is placed in a cool-mist tent. The mother asks if the child may have her security blanket inside the tent. The appropriate response is:

*1. "The child may have the security blanket inside the tent."* 2. "Objects from home are not allowed to be brought to the hospital." 3. "The blanket is not allowed because it will promote the growth of bacteria." 4. "The blanket is not allowed but the child may have a toy from the hospital playroom." *Rationale:* Familiar objects provide a sense of security for children in the strange hospital environment. The child is allowed to have a favorite toy or blanket while in the mist tent. Options 2, 3, and 4 are inappropriate statements.

A school nurse is preparing a physical education plan for a child with Down syndrome. Before preparing the plan, the nurse obtains a copy of an x-ray report of the child's:

*1. Cervical spine* 2. Hands 3. Heart 4. Chest and lungs *Rationale:* Children with Down syndrome frequently have instability of the space between the first two cervical vertebrae. They require diagnostic studies (an x-ray of the cervical spine) to determine if this is present before participating in activities that put pressure on the head and neck, which could cause spinal cord compression. Options 2, 3, and 4 are not necessary.

A nurse is providing discharge instructions to the mother of a child who had a myringotomy with insertion of tympanostomy tubes. The nurse instructs the mother that if the tubes fall out, she should:

*1. Contact the health care provider.* 2. Bring the child to the emergency department immediately. 3. Replace them immediately. 4. Immediately immerse the tubes in half-strength hydrogen peroxide. *Rationale:* The size and appearance of the tympanostomy tubes should be described to the parents following surgery. They should be reassured that if the tubes fall out, it is not an emergency, but the health care provider should be notified. Therefore options 2, 3, and 4 are incorrect.

A nurse is caring for an infant with a diagnosis of Hirschsprung's disease. The nurse should check for which clinical findings that are consistent with Hirschsprung's disease? *Select all that apply.*

*1. Fever* *2. Constipation* *3. Failure to thrive* 4. Intolerance to wheat *5. Abdominal distention* *6. Explosive, watery diarrhea* *Rationale:* Clinical manifestations of Hirschsprung's disease during infancy include failure to thrive, constipation, abdominal distention, episodes of diarrhea and vomiting, signs of enterocolitis, explosive and watery diarrhea, and fever. The infant appears significantly ill. Intolerance to wheat occurs in celiac disease.

A nursing student is asked to discuss the topic of clubfoot at a clinical conference. The student plans to tell the group that clubfoot:

*1. Is a congenital anomaly* 2. Always occurs bilaterally 3. Affects girls more often than boys 4. Is a rare deformity of the skeletal system *Rationale:* Clubfoot, one of the most common deformities of the skeletal system, is a congenital anomaly characterized by a foot that has been twisted inward or outward. The condition generally affects both feet, and boys are affected twice as often as girls.

A mother arrives at the emergency department with her child and a diagnosis of epiglottitis is documented. Which of the health care provider's prescriptions would be important for the nurse to question?

*1. Obtain a throat culture.* 2. Obtain axillary temperatures. 3. Administer humidified oxygen. 4. Administer antipyretics for fever. *Rationale:* The throat of a child with suspected epiglottitis should not be examined or cultured, because any stimulation with a tongue depressor or culture swab could cause laryngospasm and complete airway obstruction. Humidified oxygen and antipyretics are components of the treatment. Axillary rather than oral temperatures should be taken.

A 5-year-old child is admitted to the hospital for heart surgery to repair tetralogy of Fallot. The nurse notes that the child has clubbed fingers, and the nurse knows that this symptom is likely a result of:

*1. Peripheral hypoxia* 2. Chronic hypertension 3. Delayed physical growth 4. Destruction of bone marrow *Rationale:* Clubbing, a thickening and flattening of the tips of the fingers and toes, is thought to occur because of a chronic tissue hypoxemia and polycythemia. Options 2, 3, and 4 are not causes of clubbing.

A nurse is reviewing a health care provider's prescription for a child with sickle cell anemia who was admitted to the hospital for the treatment of vaso-occlusive crisis. Which prescriptions documented in the child's record should the nurse question? *Select all that apply.*

*1. Restrict fluid intake.* 2. Position for comfort. 3. Avoid strain on painful joints. 4. Apply nasal oxygen at 2 L per minute. 5. Provide a high-calorie, high-protein diet. *6. Administer meperidine (Demerol) 25 mg for pain.* *Rationale:* Sickle cell anemia is one of a group of diseases called hemoglobinopathies in which hemoglobin A is partly or completely replaced by abnormal sickle hemoglobin S. It is caused by the inheritance of a gene for a structurally abnormal portion of the hemoglobin chain. Hemoglobin S is sensitive to changes in the oxygen content of the red blood cell, and insufficient oxygen causes the cells to assume a sickle shape; the cells become rigid and clumped together, thus obstructing capillary blood flow. Oral and intravenous fluids are important parts of treatment. Meperidine (Demerol) is not recommended for the child with sickle cell disease because of the risk for normeperidine-induced seizures. Normeperidine, which is a metabolite of meperidine, is a central nervous system stimulant that produces anxiety, tremors, myoclonus, and generalized seizures when it accumulates with repetitive dosing. Therefore, the nurse would question the prescriptions for restricted fluids and meperidine for pain control. Positioning for comfort, avoiding strain in painful joints, oxygen, and a high-calorie, high-protein diet are important parts of the treatment plan.

A nurse is planning care for a child with hemolytic-uremic syndrome (HUS). The child has been anuric and will be receiving peritoneal dialysis treatment. The nurse plans to:

*1. Restrict fluids, as prescribed.* 2. Administer analgesics, as prescribed. 3. Care for the arteriovenous (AV) fistula. 4. Encourage the intake of foods that are high in potassium. *Rationale:* HUS is thought to be associated with bacterial toxins, chemicals, and viruses that cause acute renal failure in children. Clinical features of the disease include acquired hemolytic anemia, thrombocytopenia, renal injury, and central nervous system symptoms. A child with HUS who is undergoing peritoneal dialysis for the treatment of anuria will be on fluid restrictions. Pain is not associated with HUS, and potassium would be restricted rather than encouraged if the child was anuric. Peritoneal dialysis does not require an AV fistula (only hemodialysis does).

A nurse reinforces home-care instructions to the parents of a child with celiac disease. Which of the following food items would the nurse advise the parents to include in the child's diet?

*1. Rice* 2. Oatmeal 3. Rye toast 4. Wheat bread *Rationale:* Dietary management is the mainstay of treatment for celiac disease. All wheat, rye, barley, and oats should be eliminated from the diet and replaced with corn and rice. Vitamin supplements, especially fat-soluble vitamins and folate, may be required during the early period of treatment to correct deficiencies. These restrictions are likely to be lifelong, although small amounts of grains may be tolerated after the gastrointestinal ulcerations have healed.

A nurse is caring for a 2-year-old child diagnosed with croup. The nurse collects data on the child, knowing that which of the following are characteristic of this illness? *Select all that apply.*

*1. The cough is harsh and metallic.* *2. Inspiratory stridor may be present.* *3. Symptoms usually worsen at night and are better during the day.* 4. Symptoms usually worsen during the day and are relieved during sleep. *5. It is usually preceded by several days of upper respiratory infection symptoms.* *Rationale:* Croup often begins at night and may be preceded by several days of upper respiratory infection symptoms. It is characterized by a sudden onset of a harsh, metallic cough, sore throat, and inspiratory stridor. Symptoms usually worsen at night and are better in the day.

The nurse should implement which of the following in the care of a child who is having a seizure? *Select all that apply.*

*1. Time the seizure.* 2. Restrain the child. *3. Stay with the child.* 4. Insert an oral airway. 5. Place the child in a supine position. *6. Loosen clothing around the child's neck.* *Rationale:* During a seizure, the child is placed on his or her side in a lateral position. Positioning on the side will prevent aspiration because saliva will drain out of the corner of the child's mouth. The child is not restrained because this could cause injury to the child. The nurse would loosen clothing around the child's neck and ensure a patent airway. Nothing is placed into the child's mouth during a seizure because this action may cause injury to the child's mouth, gums, or teeth. The nurse would stay with the child to reduce the risk of injury and allow for observation and timing of the seizure.

A nurse is providing teaching regarding the prevention of Lyme disease to a group of teenagers going on a hike in a wooded area. Which of the following points should the nurse include in the session? *Select all that apply.*

*1. Tuck pant legs into socks.* *2. Wear closed shoes when hiking.* *3. Apply insect repellent containing DEET.* *4. Cover the ground with a blanket when sitting.* 5. Remove attached ticks by grasping with thumb and forefinger. 6. Wear long sleeves and long pants in dark colors when in high-risk areas. *Rationale:* Measures to prevent tick bites focus on covering the body as completely as possible and spraying insect repellent containing DEET on the skin and clothing. Long sleeves and pants tucked into the socks along with closed shoes will offer some protection. Light-colored clothing should be worn so that ticks would be easily visible. Hikers should not sit directly on the ground and should cover the ground with an item such as a blanket. Ticks should be removed with tweezers.

After a tonsillectomy, the child begins to vomit bright red blood. The initial nursing action would be to:

*1. Turn the child to the side.* 2. Notify the RN or health care provider (HCP). 3. Administer the prescribed antiemetic. 4. Maintain nothing-by-mouth (NPO) status. *Rationale:* After a tonsillectomy, if bleeding occurs, the child is turned to the side, and the RN or HCP is notified. An NPO status would be maintained, and an antiemetic may be prescribed; however, the initial nursing action would be to turn the child to the side.

A nurse is collecting data from a child with a diagnosis of diabetes insipidus. Which clinical finding is consistent with this diagnosis?

*1. Urinary output is increased.* 2. Urinary output is decreased. 3. Serum sodium is decreased. 4. Urine specific gravity is increased. *Rationale:* A child with a diagnosis of diabetes insipidus experiences increased urinary output, increased serum sodium, and decreased urine specific gravity. Decreased urinary output, decreased serum sodium, and increased urine specific gravity are consistent with a diagnosis of syndrome of inappropriate antidiuretic hormone (SIADH).

A nurse is caring for a 1-year-old child following a cleft palate repair. Which solution should the nurse use after feedings to cleanse the child's mouth?

*1. Water* 2. Diluted hydrogen peroxide 3. A soft lemon glycerin swab 4. Half-strength povidone-iodine (Betadine) solution *Rationale:* Following a cleft palate repair, the mouth is rinsed with water after feedings to clean the palate repair site. Rinsing food and residual sugars from the suture line reduces the risk of infection. Options 2, 3, and 4 are incorrect procedures, and the solutions identified in these options should not be used.

A nurse is preparing to administer digoxin (Lanoxin) to an infant with congestive heart failure (CHF). Before administering the medication, the nurse double-checks the dose, counts the apical heart rate for 1 full minute, and obtains a rate of 88 beats per minute. Based on this finding, which of the following is the appropriate nursing action?

*1. Withhold the medication.* 2. Administer the medication. 3. Double-check the apical heart rate and administer the medication. 4. Check the blood pressure and respirations and administer the medication. *Rationale:* Digoxin is effective within a narrow therapeutic range (0.5 to 2 ng/mL). Safety in dosing is achieved by double-checking the dose and counting the apical heart rate for 1 full minute. If the heart rate is less than 100 beats per minute in an infant, the nurse would withhold the dose and notify the registered nurse and health care provider. Options 2, 3, and 4 are incorrect actions.

After a tonsillectomy, which of the following fluid or food items would be appropriate to offer to the child?

*1. Yellow Jell-O* 2. Cold ginger ale 3. Vanilla pudding 4. Cherry Popsicle *Rationale:* After a tonsillectomy, clear, cool liquids should be administered. Citrus, carbonated, and extremely hot or cold liquids need to be avoided, because they may irritate the throat. Milk and milk products (pudding) are avoided, because they coat the throat and cause the child to clear the throat, thus increasing the risk of bleeding. Red liquids need to be avoided, because they give the appearance of blood if the child vomits.

A 28-year-old client is admitted to the labor and delivery unit for severe preeclampsia. She is started on IV magnesium sulfate. Which signs indicate that the client has developed magnesium sulfate toxicity? Select all that apply.

- 0/4 patellar reflex - Respirations are 10/min

A client is at 24 weeks gestation and preeclampsia-eclampsia syndrome is suspected. Which of the following are significant signs/symptoms criteria related to this syndrome? Select all that apply.

- 300 mg/24 hr (0.3 g/day) protein in urine - Headache, blurry vision

A male infant is born at 28 weeks gestation. What assessment findings would the nurse expect the newborn to exhibit? Select all that apply.

- Abundant lanugo on the shoulders and back - Eyelids that are fully open - Smooth, pink skin with visible veins

A nurse is caring for a client following delivery of a stillborn infant. Which actions should the nurse plan to take? Select all that apply.

- Bathe the infant and apply lotion and powder - Encourage the parents to hold the infant - Perform handprints and footprints

A nurse is reinforcing teaching about breastfeeding. Which statement by the client indicates correct understanding of the teaching?

"If I need to reposition the baby's latch, I will use my finger to break the suction first."

Which findings in a newborn are considered abnormal and should be reported to the registered nurse? Select all that apply.

- Decreased muscle tone - Sacral dimple - Single artery in the umbilical cord

A 14-year-old client confides to the school nurse that she is about 22 weeks pregnant and has not had prenatal care. Which topics are most important and priorities for the nurse to discuss with the client in anticipation of referral for prenatal care? Select all that apply.

- Family and social support - Nutrition and prenatal vitamins - Sexual abuse

A client at 34 weeks gestation has constipation. The client has been taking 325 mg ferrous sulfate tid for anemia since the last appointment 4 weeks ago. Which instructions should the nurse reinforce for this client? Select all that apply.

- Increase intake of fruits and vegetables - Moderate-intensity regular exercise

Describe acute form ITP and chronic form ITP

Acute form of ITP usually follows a viral infection (such as respiratory infection, rubella, rubeola, mumps or chickenpox; or after infection with parvovirus B19) and is self-limiting; the chronic form has periods of remission

A nurse has reinforced home care instructions to the mother of a child who is being discharged after cardiac surgery. Which statement, if made by the mother, indicates the need for further instructions?

1. "A balance of rest and exercise is important." *2. "I can apply lotion or powder to the incision if it is itchy."* 3. "Activities during which the child could fall need to be avoided for 2 to 4 weeks." 4. "Large crowds of people need to be avoided for at least 2 weeks after this surgery." *Rationale:* The mother should be instructed that lotions and powders should not be applied to the incision site because these items can affect the skin integrity and the healing process. Options 1, 3, and 4 are accurate instructions regarding home care after cardiac surgery.

A nurse provides home care instructions to the mother of a child recovering from Reye's syndrome. Which statement by the mother indicates a need for further instruction?

1. "I need to check for jaundiced skin and eyes every day." 2. "I need to have my child nap during the day to provide rest." 3. "I need to decrease the stimuli at home to prevent intracranial pressure." *4. "I need to give frequent, small, nutritious meals if my child starts to vomit."* *Rationale:* The vomiting that occurs in Reye's syndrome is caused by cerebral edema and is a symptom of increased intracranial pressure. Small, frequent meals will not affect the amount of vomiting, and the health care provider is notified if vomiting occurs. Options 1, 2, and 3 are all correct statements. Decreasing stimuli and providing rest decrease stress on the brain tissue. Checking for jaundice will assist in identifying the presence of liver complications, which are characteristic of Reye's syndrome.

A client has been prescribed valproic acid (Depakene) for the treatment of generalized seizures, and the nurse teaches the child about the potential side effects of the medication. Which statement by the client would indicate that further teaching is required?

1. "I need to take the pills whole and not crush them." 2. "I need to take the medication with food so that I won't get an upset stomach." *3. "I am so glad that I won't lose any of my hair. I was worried what my friends would think."* 4. "I know that I might gain weight with the medication so I need to be careful to not eat a lot of sweets and to eat more fruits and vegetables." *Rationale:* Side effects of valproic acid include nausea and vomiting, tremors, weight gain, and hair loss. It is important to take the medication whole and not crush or cut the medication.

The school nurse is visiting a kindergarten classroom to teach the students the importance of handwashing. During the teaching session the nurse notes that one girl is scratching her head. On inspection the nurse determines that the child has pediculosis capitis. When teaching the mother about care of this condition, which statement by the mother indicates that she needs further teaching regarding this condition?

1. "I will put all the stuffed animals in a sealed plastic bag for 14 days." *2. "I will call a carpet cleaning service to clean all my carpets in the house."* 3. "My two daughters should not share their hairbrushes or hair ribbons." 4. "I will machine wash all the washable clothing, towels, and bed linens in hot water." *Rationale:* Teaching about measures to prevent the spread of pediculosis capitis includes washing items in hot water, vacuuming carpets, discouraging sharing of personal items, and sealing items in plastic bags that cannot be vacuumed. Option 2 is too costly for many families and is unnecessary. Option 2 indicates the mother does not understand the measures that will prevent the spread of the parasite.

A nurse is reinforcing instructions to the mother of an 8-year-old child who had a tonsillectomy. The mother tells the nurse that the child loves tacos and asks when the child can safely eat one. The nurse makes which response to the mother?

1. "In 1 week". *2. "In 3 weeks".* 3. "Two days following surgery". 4. "When the health care provider says it's OK". *Rationale:* Rough, scratchy foods or spicy foods are to be avoided for 3 weeks. Citrus juices, which irritate the throat, need to be avoided for 10 days. Red liquids are avoided because they will give the appearance of blood if the child vomits. A full liquid diet is allowed on the second postoperative day, and soft foods are allowed as the child tolerates them.

A nurse assists with providing an instructional session to parents regarding impetigo. Which statement by a parent indicates the need for further instruction?

1. "It is extremely contagious." 2. "It is most common during humid weather." *3. "Lesions are most often located on the arms and chest."* 4. "It begins in an area of broken skin, such as an insect bite." *Rationale:* Impetigo is most common during the hot and humid summer months. It begins in an area of broken skin, such as an insect bite. It may be caused by Staphylococcus aureus, group A β-hemolytic streptococci, or a combination of these bacteria. It is extremely contagious. Lesions are most often located around the mouth and nose, but they may be present on the extremities.

A student nurse examines an Asian-American infant's eyes and notes that the infant's eyes are crossed. Which statement by the student to the nurse indicates an understanding of this finding?

1. "It probably is strabismus because the baby's mother has abused tranquilizers." *2. "It probably isn't strabismus but appears that way because of the child's ethnic background."* 3. "You will want to call the pediatrician immediately because this could lead to a detached retina." 4. "Strabismus isn't life threatening, but it requires surgery in the first two months to prevent the crossed eyes from being a lifelong condition." *Rationale:* Asian-American, American-Indian, and Alaskan-Native infants often have a pseudostrabismus because of a flattened nasal bridge. It needs to be distinguished from a true strabismus in the assessment. Options 1, 3, and 4 are inaccurate statements.

A nurse is caring for a hospitalized child newly diagnosed with type 1 diabetes mellitus. At 11:00 AM, the child suddenly complains of weakness, headache, and blurred vision. The nurse should immediately:

1. Give the child ½ cup of orange juice to drink. *2. Obtain a blood glucose reading.* 3. Call the dietary department and ask that the lunch tray be delivered early. 4. Contact the health care provider. *Rationale:* The signs of hypoglycemia and hyperglycemia may be difficult to distinguish. Weakness, headache, and blurred vision can occur in either blood glucose alteration. A blood glucose reading will assist in confirming the diagnosis so that the appropriate action can be taken. Option 1 would be implemented if the child had hypoglycemia. Option 3 is inappropriate because the child should eat meals at basically the same time each day to achieve the best diabetic control. Contacting the health care provider would not be the immediate action; however, the nurse should inform the registered nurse of the situation.

When checking a child's glossopharyngeal nerve function, the nurse would perform which data collection technique?

1. Have child shrug the shoulders while applying mild pressure. 2. Have child follow a light in the six cardinal positions of gaze. *3. Test sense of sour or bitter taste on the posterior segment of the tongue.* 4. Test sense of sweet or salty taste on the anterior section of the tongue. *Rationale:* To test glossopharyngeal nerve function, the nurse would test the sense of sour or bitter taste on the posterior segment of the tongue. Option 1 is the data collection technique for checking the accessory nerve. Option 2 is the technique for checking the oculomotor nerve. Option 4 is the data collection technique for checking the facial nerve.

A mother of a 6-year-old child with type 1 diabetes mellitus calls the clinic nurse and tells the nurse that the child has been sick. The mother reports that she checked the child's urine and it showed positive ketones. Which of the following would the nurse instruct the mother to do?

1. Hold the next dose of insulin. 2. Come to the clinic immediately. *3. Encourage the child to drink liquids.* 4. Administer an additional dose of regular insulin. *Rationale:* When the child is sick, the mother should test for urinary ketones with each voiding. If ketones are present, liquids are essential to help with clearing them. The child should be encouraged to drink liquids. It is not necessary to bring the child to the clinic immediately, and insulin doses should not be adjusted or changed.

A nurse is assisting with performing admission data collection on a 2-year-old child who has been diagnosed with nephrotic syndrome. The nurse collects data knowing that a common characteristic associated with nephrotic syndrome is:

1. Hypotension *2. Generalized edema* 3. Increased urinary output 4. Frank, bright red blood in the urine *Rationale:* Nephrotic syndrome is defined as massive proteinuria, hypoalbuminemia, and edema. The urine is dark, foamy, and frothy, but microscopic hematuria may be present. Frank, bright red blood in the urine does not occur. Urine output is decreased, and the blood pressure is normal or slightly decreased.

A nurse is assigned to care for a child who is suspected of having glomerulonephritis. The nurse reviews the child's record and notes that which finding is associated with the diagnosis of glomerulonephritis?

1. Hypotension *2. Red-brown urine* 3. Low urinary specific gravity 4. A low blood urea nitrogen (BUN) level *Rationale:* Gross hematuria resulting in dark, smoky, cola-colored or red-brown urine is a classic symptom of glomerulonephritis, and hypertension is also common. A mid to high urinary specific gravity is associated with glomerulonephritis. BUN levels may be elevated.

A 2-year-old child is diagnosed with constipation. Which of the following describes a characteristic of this disorder?

1. Incomplete development of the anus 2. Invagination of a section of the intestine into the distal bowel *3. The infrequent and difficult passage of dry stools* 4. The presence of fecal incontinence *Rationale:* Constipation can affect any child at any time, although its incidence peaks at ages 2 to 3 years. Option 4 describes encopresis, which can develop as a result of constipation and is one of the major concerns regarding constipation. Encopresis generally affects preschool and school-age children. Option 1 describes imperforate anus, which is diagnosed in the neonatal period. Option 2 describes intussusception, which is the most common cause of bowel obstruction in children ages 3 months to 6 years.

A nurse is monitoring an infant for signs of increased intracranial pressure (ICP) and notes that the anterior fontanel bulges when the infant is sleeping. Based on this finding, which of the following is the priority nursing action?

1. Increase oral fluids. 2. Document the finding. *3. Notify the registered nurse.* 4. Place the infant supine in a side-lying position. *Rationale:* The anterior fontanel is diamond-shaped and located on the top of the head. It should be soft and flat in a normal infant, and it normally closes by 12 to 18 months of age. A larger-than-normal fontanel may be a sign of increased ICP within the skull. Although the anterior fontanel may bulge slightly when the infant cries, bulging at rest may indicate increased ICP. Options 1 and 4 are inaccurate interventions. Although the nurse would document the finding, the first action is to report the finding to the registered nurse, who will then contact the health care provider.

A nurse receives a call from the mother whose child has a foreign body in the eye. The object is clearly visible and not embedded. When the mother asks for the most effective way to get it out, the nurse responds:

1. Irrigate the eye with natural tears. 2. Irrigate the eye with running tap water. 3. Let the object just "work its way out" of the eye. *4. Touch the object gently with a cotton swab, and lift it out.* *Rationale:* The most effective method that would cause the least amount of trauma would be to lift the foreign body from the eye. It should not be allowed to remain and "work its way out." Irrigating the eye may cause the foreign body to move and cause trauma in another area of the eye.

Which of the following are characteristics of scabies? *Select all that apply.*

1. It is caused by a fungal infection. *2. It appears as burrows or fine, grayish-red lines.* *3. It is transmitted by close personal contact with an infected person.* *4. It is endemic among schoolchildren and institutionalized populations.* 5. Meticulous skin care and the application of antifungal cream are components of treatment. *6. Household members and contacts of the infected child need to be treated at the same time that the child is being treated.* *Rationale:* Scabies usually appears as burrows or fine, grayish-red lines. It is not caused by a fungal infection, and it is treated with the application of a topical scabicide. It is transmitted by close personal contact with an infected person, and it is endemic among schoolchildren and institutionalized populations. Household members and contacts of the infected child need to be treated at the same time that the child is being treated.

Oral iron is prescribed for a child with an iron deficiency anemia, and the nurse provides instructions to the mother regarding the administration of the iron. The nurse instructs the mother to administer the iron:

1. Just before a meal 2. Just after a meal *3. Between meals* 4. With a fruit low in vitamin C *Rationale:* The mother should be instructed to administer oral iron supplements between meals. The iron should be given with a citrus fruit or juice high in vitamin C because vitamin C increases the absorption of iron by the body.

A 1-year-old child is admitted to the hospital for control of tonic-clonic seizures. The nurse would do which of the following in order to protect the child from injury? *Select all that apply.*

1. Keep a padded tongue blade at the bedside for use during a seizure. *2. Remove toys that have bright, blinking lights on them.* *3. Keep side rails and other hard objects padded.* *4. Turn the client to the side during a seizure.* 5. Restrict the client's fluid intake. *Rationale:* Attempting to place something in a child's mouth during a seizure is not helpful even if it is padded. The mouth is usually clenched, and one would have to use force to open the mouth. One must attempt to keep the airway clear and can do that by positioning (option 4). Option 2 may be helpful in preventing a seizure, and option 3 safeguards the client's physical safety. Option 5 is not necessary.

An adolescent with diabetes mellitus becomes flushed and complains of hunger and dizziness. A blood glucose level is drawn and the results indicate a glucose level of 60 mg/dL. The appropriate intervention is to:

1. Keep the child NPO. 2. Contact the health care provider. *3. Give the child a glass of fruit juice.* 4. Let the child rest until the dizziness subsides. *Rationale:* A blood glucose less than 70 mg/dL indicates hypoglycemia. When signs of hypoglycemia occur, the child needs an immediate source of glucose. Options 1, 2, and 4 do not address the hypoglycemic condition.

The child with cryptorchidism is being discharged after orchiopexy, which was performed on an outpatient basis. The nurse informs the parents about which priority care measure?

1. Measuring intake and output 2. Administering anticholinergics *3. Preventing infection at the surgical site* 4. Applying cold, wet compresses to the surgical site *Rationale:* The most common complications associated with orchiopexy are bleeding and infection. The parents are instructed in postoperative home care measures, including the prevention of infection, pain control, and activity restrictions. The measurement of intake and output is not required. Anticholinergics are prescribed for the relief of bladder spasms; they are not necessary after orchiopexy. Cold, wet compresses are not prescribed. The moisture from a wet compress presents a potential for infection.

Which test would the nurse anticipate for a teenage client who has been treated for vaginal candida repeatedly in the last 6 months to assist in the identification of the underlying chronic pathology?

1. Pap smear 2. Blood culture 3. Throat culture *4. Blood glucose level* *Rationale:* A blood glucose level is an indicator of diabetes mellitus. In females, monilial infections of the genitourinary tract are a common manifestation of diabetes mellitus. Pap smears are specific for detecting cancer of the cervix. A throat culture may show a candidal infection, but this test is unrelated to an undiagnosed underlying chronic disease. An infection of the blood (diagnosed by a blood culture) is indicative of an acute systemic disease.

A 4-year-old child is hospitalized for severe gastroenteritis. The child is crying and clinging to the mother. The mother becomes very upset and is afraid to leave the child. Which of the following nursing interventions would be most appropriate to alleviate the child's fears and the mother's anxiety?

1. Reassure the mother that the child will be fine after she leaves. 2. Give the mother the telephone number of the pediatric unit, and tell the mother to call at any time. *3. Ask the mother if she would like to stay overnight with the child.* 4. Tell the mother to bring the child's favorite toys the next time she comes to the hospital to visit. *Rationale:* Although a 4-year-old may already be spending some time away from his or her parents at a day care center or preschool, illness adds a stressor that makes separation more difficult. The only option that addresses the mother's anxiety, while at the same time alleviating the fears of the child is option 3. Options 1, 2, and 4 do not address the fears and anxieties of the mother and child.

A child is scheduled to receive a measles, mumps, and rubella (MMR) vaccine. The nurse who is preparing to administer the vaccine reviews the child's record. Which finding should make the nurse question the health care provider's prescription?

1. Recent recovery from a cold 2. A history of frequent respiratory infections *3. A history of an anaphylactic reaction to neomycin* 4. A local reaction at the site of a previous MMR vaccine injection *Rationale:* The MMR vaccine contains minute amounts of neomycin. A history of an anaphylactic reaction to neomycin is considered a contraindication to the MMR vaccine. The general contraindication to all immunizations is a severe febrile illness. The presence of a minor illness such as the common cold is not a contraindication. In addition, a history of frequent respiratory infections is not a contraindication to receiving a vaccine. A local reaction to an immunization is treated with cool packs for the first 24 hours after injection, and this is followed by warm or cool compresses if the inflammation persists.

A nurse caring for a child with nephrotic syndrome reviews the medication record. The nurse notes that prazosin hydrochloride (Minipress) is prescribed for the child. The nurse determines that this medication has been prescribed to:

1. Reduce proteinuria. 2. Decrease inflammation. 3. Suppress the autoimmune response. *4. Control hypertension.* *Rationale:* Prazosin hydrochloride (Minipress) may be used to control hypertension. The child also may be placed on diuretic therapy until protein loss is controlled. Corticosteroids, such as prednisone, may be prescribed to decrease inflammation. Corticosteroids also suppress the autoimmune response and stimulate vascular reabsorption of edema. Cyclophosphamide is an alkylating agent and may be used in maintaining remission.

A nurse is monitoring for bleeding in a child after surgery for the removal of a brain tumor. The nurse checks the head dressing for the presence of blood and notes a colorless drainage on the back of the dressing. Which nursing action is appropriate?

1. Reinforce the dressing. *2. Notify the registered nurse (RN).* 3. Document the findings and continue to monitor. 4. Circle the area of drainage and continue to monitor. *Rationale:* Colorless drainage on the dressing would indicate the presence of cerebrospinal fluid and should be reported to the RN immediately; the RN would then contact the health care provider. The colorless drainage should also be checked for evidence of cerebrospinal fluid; one method is to check for the presence of glucose using a dipstick. Options 1, 3, and 4 are incorrect and delay required immediate interventions.

A nurse is assisting in developing a plan of care for a 10-year-old child diagnosed with acute glomerulonephritis. Following review of the plan of care, the nurse determines that which of the following is the priority for the child?

1. Restricting oral fluids 2. Allowing the child to play with the other children in the playroom *3. Promoting bedrest* 4. Encouraging visits from friends *Rationale:* Bedrest is required during the acute phase, and activity is gradually increased as the condition improves. Providing for quiet play according to the developmental stage of the child is important. Fluids should not be forced or restricted. Visitors should be limited to allow for adequate rest.

A nurse has provided dietary instructions to the mother of a child with celiac disease. The nurse determines that further instructions are needed if the mother states that she will include which of the following in the child's nutritional plan?

1. Rice 2. Corn *3. Oatmeal* 4. Vitamin supplements *Rationale:* Dietary management is the mainstay of treatment for the child with celiac disease. All wheat, rye, barley, and oats should be eliminated from the diet and replaced with corn and rice. Vitamin supplements, especially fat-soluble vitamins and folate, may be needed in the early period of treatment to correct deficiencies.

A nurse is caring for an infant with congenital heart disease. Which of the following signs, if noted in the infant, would alert the nurse to the early development of congestive heart failure (CHF)?

1. Strong sucking reflex 2. Slow and shallow breathing 3. Pallor *4. Diaphoresis during feeding* *Rationale:* The early symptoms of CHF include tachypnea, poor feeding, and diaphoresis during feeding. Tachycardia would occur during feeding. Pallor may be noted in the infant with CHF, but it is not an early symptom. A strong sucking reflex is unrelated to the development of CHF.

The appropriate child position after a tonsillectomy is which of the following?

1. Supine position *2. Side-lying position* 3. High Fowler's position 4. Trendelenburg's position *Rationale:* The child should be placed in a semi-prone or side-lying position after tonsillectomy to facilitate drainage. Options 1, 3, and 4 will not achieve this goal.

A nursing instructor assigns a student nurse to present a clinical conference to the student group about brain tumors in children. The student prepares for the conference and plans to include which of the following in the presentation?

1. Surgery is not normally performed because of the risk of functional deficits occurring as a result of the surgery. 2. Head shaving is no longer required before removal of the brain tumor. 3. The common site of metastasis is the kidneys. *4. The significant symptoms are headaches and morning vomiting.* *Rationale:* The hallmark symptoms of children with brain tumors are headache and morning vomiting related to the child getting out of bed. Initial intervention is "debulking" or operating to remove as much of the tumor as possible while minimally disturbing the surrounding brain tissue, so that the child's neurological functioning is preserved as much as possible. Before surgery, the child's head will be shaved, although every effort is made to shave as much hair as is necessary only. Depending on the type of tumor, a myelogram may be done to determine metastatic disease in the spinal cord.

A nurse is caring for a child who sustained a head injury in an automobile accident and is monitoring the child for signs of increased intracranial pressure (ICP). The nurse plans to monitor for the earliest sign of increased ICP by assessing for:

1. Tachycardia *2. Changes in level of consciousness (LOC)* 3. Posturing 4. Apnea *Rationale:* An altered level of consciousness is an early sign of increased ICP. Late signs of increased ICP include tachycardia leading to bradycardia, apnea, systolic hypertension, widening pulse pressure, and posturing.

A 12-year-old child is seen in the clinic, and a diagnosis of Hodgkin's disease is suspected. Several diagnostic studies are performed to determine the presence of this disease. When evaluating the diagnostic results, the nurse would expect to note which of the following if this child had Hodgkin's disease?

1. The presence of blast cells in the bone marrow *2. The presence of Reed-Sternberg cells* 3. The presence of Epstein-Barr virus 4. Elevated creatinine level *Rationale:* Hodgkin's disease is a neoplasm of lymphatic tissue. The presence of giant, multinucleated cells (Reed-Sternberg cells) is the hallmark of this disease. The presence of blast cells in the bone marrow is indicative of leukemia. Infectious mononucleosis and the Epstein-Barr virus have been associated with Hodgkin's disease but would not determine the presence of Hodgkin's. An elevated creatinine level is indicative of a renal system disorder.

The nurse is caring for a 4-month-old patient with severe infantile diarrhea. What condition is this patient most likely to experience as a result? 1.) Metabolic acidosis 2.) Metabolic alkalosis 3.) Respiratory acidosis 4.) Respiratory alkalosis

1.) Metabolic acidosis Pg. 1009 -Foundations of Nursing

A full term infant admitted to the newborn nursery has a blood glucose level of 35 mg/dL. The nurse should monitor this baby carefully for which of the following? 1. Jaundice 2. Jitters 3. Erythema toxicum 4. Subconcunctival hemorrhages

2. Babies who are hypoglycemic will often develop jitters

When caring for a 9-month-old patient who underwent surgery to repair a cleft palate, what us the priority intervention? 1.) Referral to a parent support group 2.) Maintaining adequate nutrition 3.) Keeping an IV line open 4.) Keeping the patient sedated

2.) Maintaining adequate nutrition Pg. 1006 -Foundations of Nursing

The practical nurse is collecting data on several clients waiting to be seen in the prenatal clinic. Which client situation is most important to report to the registered nurse?

32 weeks gestation client taking ibuprofen for moderate back pain

What should the teaching plan include about infant fall precautions? Select all that apply. a. Remove all unsteady furniture. b. Keep crib rails up and in locked position. c. Steady infant with hand when on changing table. d. Use tray attachment on high chair as restraint. e. Keep infant seat on the floor.

ANS: A, B, C, E The tray attachment to a high chair is an inadequate restraint. All other options are good precautions to prevent an infant from a fall.

A nurse is admitting a post-date client at 43 weeks gestation to the labor and delivery unit for induction of labor. What is a predictor of a successful induction?

A Bishop score of 10

Which statement is correct about toilet training? a. Bladder training is usually accomplished before bowel training. b. Wanting to please the parent helps motivate the child to use the toilet. c. Watching older siblings use the toilet confuses the child. d. Children must be forced to sit on the toilet when first learning.

ANS: B Voluntary control of the anal and urethral sphincters is achieved sometime after the child is walking. The child must be able to recognize the urge to let go and to hold on. The child must want to please the parent by holding on rather than pleasing self by letting go. Bowel training precedes bladder training. Watching older siblings provides role modeling and facilitates imitation for the toddler. The child should be introduced to the potty chair or toilet in a nonthreatening manner.

As pregnancy advances, the uterus presses on the abdominal vessels ( vena cava and aorta). What position is best for increasing perfucion according to the latest research? A. Left side lying position B. Knee chest position C. Side lying position D. Right side lying position

B. Recent research indicated that the knee chest position is best for increasing perfusion and that the sid lying position (either left or right side lying) is the second most disirable position to increase perfusion. Prior to this research, the left side lying position was usually encouraged

A nurse consults a mother and detects cord prolaspe. How should the examiner position the pregnant woman to relieve pressure on the cord? A. Side lying position B. Right side lying position C. High Fowlers position D. Knee chest position

D. If cord prolaspe is detected, the examiner should position the mother to relieve pressure on the cord, Knee chest position, or push the presenting part off the cord until Immediate cesarean delivery can be accomplished

A client who gave birth to a healthy 8 pound infant 3 hours ago is admitted to the postpartum unit. Which nursing plan is best in assisting this mother to bond with her newborn infant? A. Encourage the mother to provide total care for her infant. B. Provide privacy so the mother can develop a relationship with the infant. C. Encourage the father to provide most of the infant's care during hospitalization. D.Meet the mother's physical needs and demonstrate warmth toward the infant.

D.Meet the mother's physical needs and demonstrate warmth toward the infant.

Medical management of leukemia

Individualized; corticosteroids; chemotherapy

Which nursing diagnosis would be most appropriate for a child with idiopathic thrombocytopenic purpura? a) Risk for infection related to abnormal immune system b) Risk for altered urinary elimination related to kidney impairment c) Ineffective breathing pattern related to decreased white blood count d) Ineffective tissue perfusion related to poor platelet formation

Ineffective tissue perfusion related to poor platelet formation Explanation: Idiopathic thrombocytopenic purpura results in decreased platelets, so bleeding into tissue can occur

A nurse is teaching the parents of a child with sickle cell disease about factors that predispose the child to a sickle cell crisis. The nurse determines that the teaching was successful when the parents identify which of the following as a factor? a) Pallor b) Infection c) Fluid overload d) Respiratory distress

Infection Explanation: Factors that may precipitate a sickle cell crisis include: fever, infection, dehydration, hot or humid environment, cold air or water temperature, high altitude, or excessive physical activity. Respiratory distress and pallor are general signs and symptoms of a sickle cell crisis

What are risks for premature rupture of membranes?

Infection, multiple gestations, trauma, and incompetent cervix.

What is the adverse effect of Narcan in opioid dependent clients?

It can cause withdrawal.

What is the normal intrauterine fetal attitude?

It is the relationship of the fetal body parts to one another. The normal intrauterine attitude is flexion, in which the fetal back is rounded, the head is forward on the chest, and the arms and legs are folded in against the body.

A child with sickle cell disease is brought to the emergency department by his parents. He is in excruciating pain. A vaso-occlusive crisis is suspected and analgesia is prescribed. Which of the following would the nurse expect as least likely to be ordered? a) Nalbuphine b) Morphine c) Meperidine d) Hydromorphone

Meperidine Explanation: Meperidine is contraindicated for ongoing pain management in a child with vaso-occlusive crisis because it increases the risk for seizures. Analgesics such as morphine, nalbuphine, or hydromorphone are commonly used

Diagnostic tests for Sickle Cell Anemia

Sickle-turbidity test, hemoglobin electrophoresis

Which nursing intervention is most helpful in relieving postpartum uterine contractions or "afterpains?" a. Lying prone with a pillow on the abdomen b. Using a breast pump c. Massaging the abdomen d. Giving oxytocic medications

a. Lying prone with a pillow on the abdomen Lying prone (A) keeps the fundus contracted and is especially useful with multiparas, who commonly experience afterpains due to lack of uterine tone.

What is the maximum amount of time that a nurse should suction an artificial airway? a. 1 second b. 5 seconds c. 30 seconds d. 1 minute

b. 5 seconds The nurse should limit suctioning to no more than 5 seconds. p. 964

23. While taking a diet history, the nurse might be told that the expectant mother has cravings for ice chips, cornstarch, and baking soda. This represents a nutritional problem known as: a. Preeclampsia. b. Pyrosis. c. Pica. d. Purging.

c. Pica.

8. A pregnant woman's diet history indicates that she likes the following list of foods. The nurse would encourage this woman to consume more of which food to increase her calcium intake? a. Fresh apricots b. Canned clams c. Spaghetti with meat sauce d. Canned sardines

d. Canned sardines

29. Which pregnant woman should restrict her weight gain during pregnancy? a. Woman pregnant with twins b. Woman in early adolescence c. Woman shorter than 62 inches or 157 cm d. Woman who was 20 pounds overweight before pregnancy

d. Woman who was 20 pounds overweight before pregnancy

The mother of a preterm infant tells her nurse that she would like to visit her newborn, who is in the neonatal intensive care unit (NICU). Which of the following would be the most appropriate response by the nurse? a) "I'm sorry. You may not visit your son until he has been released from the NICU." b) "Certainly. You may only observe the child from a distance, however, as his immune system is still not developed adequately." c) "Certainly. You will need to wash your hands and gown before you can hold him, however." d) "I'm sorry. You may not visit the NICU, but we can arrange to have your son brought to your room so that you can hold him."

"Certainly. You will need to wash your hands and gown before you can hold him, however." Correct Explanation: Be certain the parents of a high-risk newborn are kept informed of what is happening with their child. They should be able to visit the special nursing unit to which the newborn is admitted as soon and as often as they choose, and, after washing and gowning, hold and touch their newborn, both actions which help make the child's birth more real to them.

A nurse provides instructions to parents regarding the methods that will decrease the risk of recurrent otitis media in infants. Which of the following should the nurse include in the instructions?

*1. "Feed the infant in an upright position."* 2. "Maintain bottle-feeding as long as possible." 3. "Discontinue breast-feeding as soon as possible." 4. "Allow the infant to have a bottle during nap time." *Rationale:* To decrease the risk of recurrent otitis media, parents should be encouraged to breast-feed during infancy, discontinue bottle-feeding as soon as possible, feed the infant in an upright position, and avoid giving the infant a bottle in bed. Parents should be told not to smoke in the child's presence because passive smoking increases the incidence of otitis media.

A nurse has provided discharge instructions to the mother of an 18-month-old child following surgical repair of hypospadias. Which statement by the mother indicates a need for further instruction?

*1. "I should carry my child by straddling the child on my hip."* 2. "I should use double diapers to hold the surgery site in place." 3. "I should avoid toilet training right now." 4. "I should encourage fluid intake." *Rationale:* Parent teaching following hypospadias repair includes restricting the child from activities that put pressure on the surgical site. Straddling the child on the hip will cause pressure on the surgical site. The parents should be instructed to use double diapers to hold the stent in place and should be instructed how to hold the child during the postoperative period. Fluids should be encouraged to maintain hydration. Toilet training should not be an issue during this stressful period.

A nurse is evaluating the parent's understanding of discharge care regarding the functioning of the infant's ventricular peritoneal shunt. Which statement by a parent indicates an understanding of the shunt complications?

*1. "If my baby has a high-pitched cry, I should call the doctor."* 2. "I should position my baby on the side with the shunt when sleeping." 3. "My baby will pass urine more often now that the shunt is in place." 4. "I should call my doctor if my baby refuses purees." *Rationale:* If the shunt is broken or malfunctioning, the fluid from the ventricle part of the brain will not be diverted to the peritoneal cavity. The cerebrospinal fluid will build up in the cranial area. The result is intracranial pressure, which then causes a high-pitched cry in the infant. The baby should not have pressure when on the shunt side. Skin breakdown and possible compression to the apparatus could result. This type of shunt affects the gastrointestinal system, not the genitourinary system. Option 4 is only a concern if the baby becomes malnourished or dehydrated, which could then raise the body temperature. Otherwise, refusal to eat purees has no direct relationship to the shunt functioning.

A nurse is assisting in collecting data on a child with seizures. The nurse is interviewing the child's parents to establish their adjustment to caring for their child with a chronic illness. Which statement by a parent would indicate a need for further teaching?

*1. "Our child sleeps in our bedroom at night."* 2. "We worry about injuries when our child has a seizure." 3. "Our child is involved in a swim program with neighbors and friends." 4. "Our babysitter just completed cardiopulmonary resuscitation (CPR) training." *Rationale:* Parents are especially concerned about seizures that might go undetected at night. The nurse should suggest a baby monitor. Reassurance by the nurse should ensure parental confidence and decrease parental overprotection. Option 2 is a common concern. Options 3 and 4 demonstrate the parents' ability to choose respite care and activities appropriately. The parents need to be reminded that, as the child grows, they cannot always observe their child, but their knowledge of seizure activity and care are appropriate to minimize complications.

A 13-year-old child is diagnosed with osteogenic sarcoma of the femur. Following a course of chemotherapy, it is decided that leg amputation is necessary. Following the amputation, the child becomes very frightened because of aching and cramping felt in the missing limb. Which statement made by the nurse will best assist in alleviating the child's fear?

*1. "This aching and cramping is normal and temporary and will subside."* 2. "The pain medication that I give you will take these feelings away." 3. "This always occurs after the surgery and we will teach you ways to deal with it." 4. "This pain is not real pain and relaxation exercises will help it go away." *Rationale:* Following amputation, phantom limb pain is a temporary condition that some children may experience. This sensation of aching or cramping in the missing limb is most distressing to the child. The child needs to be reassured that the condition is normal and only temporary.

A nursing student caring for a 6-month-old infant is asked to collect a urine specimen from the infant. The student collects the specimen by:

*1. Attaching a urinary collection device to the infant's perineum for collection* 2. Obtaining the specimen from the diaper by squeezing the diaper after the infant voids 3. Catheterizing the infant using the smallest available French Foley catheter 4. Noting the time of the next expected voiding and preparing to collect the specimen into a cup when the infant voids *Rationale:* Although many methods have been used to collect urine from an infant, the most reliable method is the urine collection device. This device is a plastic bag that has an opening that is lined with adhesive so that it may be attached to the perineum. Urine for certain tests, such as specific gravity, may be obtained from a diaper by collection of the urine with a syringe. Urinary catheterization is not to be done unless specifically prescribed because of the risk of infection. It is not reasonable to try to identify the time of the next voiding to attempt to collect the specimen.

A nurse is assigned to care for a child admitted to the hospital with a diagnosis of suspected bacterial endocarditis. The nurse prepares the child for which of the following diagnostic tests that will confirm the diagnosis?

*1. Blood cultures* 2. Chest x-ray 3. Echocardiogram 4. Transesophageal echocardiography *Rationale:* When endocarditis is suspected, a definitive diagnosis is achieved through blood cultures. A negative blood culture does not rule out the existence of endocarditis; it just indicates a lesser likelihood of its existence. A chest x-ray, echocardiogram, and transesophageal echocardiography are performed to aid in the diagnosis of endocarditis.

A nurse has reviewed the health care provider's prescriptions for a child suspected of a diagnosis of neuroblastoma and is preparing to implement diagnostic procedures that will confirm the diagnosis. The nurse prepares to:

*1. Collect a 24-hour urine sample.* 2. Perform a neurological assessment. 3. Send the child to the radiology department for a chest x-ray. 4. Assist with a bone marrow aspiration. *Rationale:* Neuroblastoma is a solid tumor found only in children. It arises from neural crest cells that develop into the sympathetic nervous system and the adrenal medulla. Typically the tumor infringes on adjacent normal tissue and organs. Neuroblastoma cells may excrete catecholamines and their metabolites. Urine samples will indicate elevated vanillylmandelic acid levels. A bone marrow aspiration will assist in determining marrow involvement. A neurological examination and a chest x-ray may be performed but will not confirm the diagnosis.

A nurse is monitoring a child with a head injury. On data collection, the nurse notes an abnormal flexion of the upper extremities and an extension of the lower extremities. The nurse documents that the child is experiencing:

*1. Decorticate posturing* 2. Decerebrate posturing 3. Flexion of the arms and legs 4. An expected position post-head injury *Rationale:* Decorticate posturing is an abnormal flexion of the upper extremities and an extension of the lower extremities with possible plantar flexion of the feet. Decerebrate posturing is an abnormal extension of the upper extremities with internal rotation of the upper arms and wrists, and an extension of the lower extremities with some internal rotation.

A child with sickle cell anemia is being discharged after treatment for a crisis. Which instructions should the nurse provide to prevent another crisis from occurring? *Select all that apply.*

*1. Drink plenty of fluids.* 2. Avoid foods high in folic acid. 3. Use cold packs to relieve joint pain. 4. Restrict all activity to quiet board games. *5. Wash hands before meals and after playing.* *6. Report a sore throat immediately.* *Rationale:* Sickle cell crisis can be precipitated by cold, dehydration, stress, or infection. Increasing the amount of fluids will reduce the viscosity of blood, thus preventing vascular occlusion. A conscious effort to wash hands can improve the child's health by preventing infection. A sore throat is a sign of an infection and must be reported. It is important to avoid cold temperatures of any kind because this can cause vaso-occlusion. Folic acid avoidance is not necessary. Children need to be encouraged to set their own limits in play.

A female adolescent with type 1 diabetes mellitus will become a member of the school's football cheerleader team. The adolescent excitedly reports to the school nurse to obtain information regarding adjustments needed in the treatment plan for the diabetes. The school nurse would instruct the adolescent to:

*1. Eat six graham crackers or drink a cup of orange juice before practice or game time.* 2. Eat half the amount of food normally eaten at lunchtime. 3. Take the prescribed insulin one half hour before practice or game time rather than in the morning. 4. Take two times the amount of prescribed insulin on practice and game days. *Rationale:* An extra snack of 15 to 30 g of carbohydrate eaten before activities such as cheerleader practice will prevent hypoglycemia. Six graham crackers or a cup of orange juice will provide 15 to 30 g of carbohydrate. The adolescent should not be instructed to adjust the amount or time of insulin administration. Meal amounts should not be decreased.

A child with a brain tumor returns from the recovery room following "debulking" of the tumor. The nurse assigned to care for the child monitors the child for brainstem involvement. Which of the following signs would indicate that brainstem involvement occurred during the surgical procedure?

*1. Elevated temperature* 2. Orthostatic hypotension 3. Inability to swallow 4. Altered hearing ability *Rationale:* Vital signs and neurological status are checked frequently. Special attention is paid to the child's temperature, which may be elevated because of hypothalamus or brainstem involvement during surgery. A cooling blanket should be in place on the bed or be readily available if the child becomes hyperthermic. Options 3 and 4 are related to functional deficits following surgery. An elevated blood pressure and a widened pulse pressure may be associated with increasing intracranial pressure.

A diagnostic workup is performed on a 1-year-old child suspected of a diagnosis of neuroblastoma. Which finding specifically associated with this type of tumor should the nurse expect to find documented in the child's record?

*1. Elevated vanillylmandelic acid (VMA) levels in the urine* 2. The presence of blast cells in the bone marrow 3. Projectile vomiting occurring often in the morning 4. Positive Babinski's sign *Rationale:* Neuroblastoma is a solid tumor found only in children. It arises from neural crest cells that develop into the sympathetic nervous system and the adrenal medulla. Typically the tumor infringes on adjacent normal tissue and organs. Neuroblastoma cells may excrete catecholamines and their metabolites. Urine samples will indicate elevated VMA levels. The presence of blast cells in the bone marrow occurs in leukemia. Projectile vomiting occurring most often in the morning and a positive Babinski's sign are clinical manifestations of a brain tumor.

A nurse is assisting in developing a plan of care for a diagnosed with acute glomerulonephritis. The nurse includes which intervention in the plan of care?

*1. Encourage limited activity and provide safety measures.* 2. Force intake of oral fluids to prevent hypovolemic shock. 3. Catheterize the child to strictly monitor intake and output. 4. Encourage classmates to visit and to keep the child informed of school events. *Rationale:* Activity is limited and most children, because of fatigue, voluntarily restrict their activities during the active phase of the disease. Catheterization may cause a risk of infection. Fluids should not be forced. Visitors should be limited to allow for adequate rest.

A mother of a 3-year-old child tells the nurse that the child has been continuously scratching the skin and has developed a rash. On data collection, which finding indicates that the child may have scabies?

*1. Fine, grayish-red lines* 2. Purple-colored lesions 3. Thick, honey-colored crusts 4. Clusters of fluid-filled vesicles *Rationale:* Scabies appears as burrows or fine, grayish-red lines. They may be difficult to see if they are obscured by excoriation and inflammation. Purple-colored lesions may be indicative of various disorders, including systemic conditions. Thick, honey-colored crusts are characteristic of impetigo. Clusters of fluid-filled vesicles are seen in clients with herpesvirus.

A nurse is checking the status of jaundice in a child with hepatitis. The nurse checks which of the following that will provide the best data regarding the presence of jaundice?

*1. Nailbeds* 2. Skin in the abdominal area 3. Skin in the sacral area 4. Membranes in the ear canal *Rationale:* Jaundice, if present, is best checked in the sclera, nailbeds, and mucous membranes. Generalized jaundice will appear in the skin throughout the body. Option 4 is not an appropriate assessment area for the presence of jaundice.

A nurse is reviewing the record of a child admitted to the hospital with nephrotic syndrome. Which finding would the nurse expect to note documented in the record?

*1. Proteinuria* 2. Weight loss 3. Increased appetite 4. Hyperalbuminemia *Rationale:* The term "nephrotic syndrome" refers to a kidney disorder characterized by proteinuria, hypoalbuminemia, and edema. The child experiences fatigue, anorexia, increased weight, abdominal pain, and a normal blood pressure.

The nurse is monitoring a neonate 1 hour after spontaneous vaginal delivery. Which of the following are expected findings? Select all that apply.

- Capillary glucose of 60 mg/dL (3.3 mmol/L) - Respirations of 56 breaths per minute - White papules on bridge of the nose

A baby's blood type is B negative. The baby is at risk for hemolytic jaundice if the mother has which of the following blood types? 1. Type O negative 2. Type A negative 3. Type B positive 4. Type AB positive

1. ABO incompatibility can arise when the mother is type O and the baby is either type A or B

During cardiopulmonary resuscitation, which of the following actions should be performed? 1. Provide assested ventilation at 40 to 60 breaths per minute 2. Begin chest compressions when heart rate is 0 to 20 bpm 3. Compress the chest using the three finger technique 4 Administer compressions and breaths in a 5:1 ratio

1. Assisted ventilations should be administered at a rate of 40 to 60 per minute

A baby, admitted to the nursery, was diagnosed with galactosemia from an anmiocentesis. Which of the following actions must the nurse take? 1. Feed the baby a specialty formula 2. Monitor the baby for central cyanosis 3. Do hemoccult testing on every stool 4. Monitor the baby for signs of abdominal pain

1. Galactosemia is one of the few diseases that is a contraindication for the intake of breast milk or any milk based formula

A client who is 18 weeks gestation has been diagnosed with hydatiform mole (gestational trophoblastic disease). In addition to vaginal loss, which of the following signs/symtoms would the nurse expect to see? 1. Hyperemesis and hypertension 2. Diarrhea nd hyperthermia 3. Polycythemia 4. Polydipsia

1. Hyperemesis and hypertension are often seen in clients with hydatiform mole

An adolescent is seen in the health care clinic with complaints of chronic fatigue. On physical examination, the nurse notes that the adolescent has swollen lymph nodes. A laboratory test is performed, and the results indicate the presence of Epstein-Barr virus (mononucleosis). The nurse calls the mother of the adolescent to inform the mother of the test results and provides instructions regarding the care of the adolescent. Which statement by the mother indicates an understanding of the care measures?

1. "I need to keep my child on bedrest for 3 weeks." 2. "I will call the health care provider if my child is still feeling tired in 1 week." *3. "I need to call the health care provider if my child complains of abdominal pain or left shoulder pain."* 4. "I need to isolate my child so that the respiratory infection is not spread to others." *Rationale:* The mother needs to be instructed to notify the health care provider if abdominal pain, especially in the left upper quadrant, or left shoulder pain occurs because this may indicate splenic rupture. Children with enlarged spleens are also instructed to avoid contact sports until splenomegaly resolves. Bedrest is not necessary, and children usually self-limit their activity. No isolation precautions are required, although transmission can occur via saliva, close intimate contact, or contact with infected blood. The child may still feel tired in 1 week as a result of the virus.

A nurse is caring for an 18-month-old child who has been vomiting. The appropriate position in which to place the child during naps and sleep time is:

1. A supine position *2. A side-lying position* 3. Prone, with the head elevated 4. Prone, with the face turned to the side *Rationale:* The vomiting child should be placed in an upright or side-lying position to prevent aspiration. Options 1, 3, and 4 will place the child at risk for aspiration if vomiting occurs.

A nurse is instructing the mother of a child with cystic fibrosis (CF) about the appropriate dietary measures. Which of the following meals best illustrates the most appropriate diet for a client with cystic fibrosis?

1. A veggie salad and a caramel apple 2. A strawberry jelly sandwich and pretzels 3. A plate of nachos and cheese and a cupcake *4. A piece of fried chicken and a loaded baked potato* *Rationale:* Children with CF are managed with a high-calorie, high-protein diet. Pancreatic enzyme replacement therapy is undertaken, and fat-soluble vitamin supplements are administered. Fats are not restricted unless steatorrhea cannot be controlled by increased levels of pancreatic enzymes. A piece of fried chicken and a loaded baked potato provides a high-calorie and high-protein meal that includes fat.

A nurse is preparing for the administration of ribavirin (Virazole) to a child with respiratory syncytial virus. Which of the following supplies will the nurse obtain for the administration of this medication?

1. An intravenous (IV) pole 2. An intramuscular (IM) syringe *3. A pair of goggles* 4. A protective isolation gown *Rationale:* Some caregivers experience headaches, burning nasal passages and eyes, and crystallization of soft contact lenses as a result of administration of ribavirin. Specific to this medication is the use of goggles. A mask may be worn. Handwashing is to be performed before and after any child contact. A gown is not necessary. The medication is administered via hood, face mask, or oxygen tent, not by the IM or IV route.

A nurse is assessing a pediatric client with a diagnosis of retinoblastoma. The nurse assesses for which most common clinical finding for a child with this diagnosis?

1. Blindness 2. Strabismus *3. Cat's-eye reflex* 4. Red, painful eye *Rationale:* Clinical manifestations of retinoblastoma include cat's-eye reflex (most common sign); strabismus (second most common sign); red, painful eye; and blindness (late signs). Cat's-eye reflex is commonly observed by the parent and is described as a whitish "glow" in the pupil. This represents visualization of the tumor as the light momentarily falls on the mass, and is the most common sign.

The primary goal to be included in the plan of care for a child who has cerebral palsy is to:

1. Eliminate the cause of the disease. 2. Improve muscle control and coordination. 3. Prevent the occurrence of emotional disturbances. *4. Maximize the child's assets and minimize the limitations.* *Rationale:* The goal of managing the child with cerebral palsy is early recognition and intervention to maximize the child's abilities. The cause of the disease cannot be eliminated. It is best to minimize emotional disturbances, if possible, but not to prevent them because it is healthy for the child to express emotions. Improvement of muscle control and coordination is a component of the plan, but the primary goal is to maximize the child's assets and minimize the limitations caused by the disease.

A child with croup is being discharged from the hospital. The nurse provides home care instructions to the mother and advises the mother to bring the child to the emergency department if the child:

1. Is irritable 2. Appears tired *3. Develops stridor* 4. Takes fluids poorly *Rationale:* The mother should be instructed that if the child develops stridor at rest, cyanosis, severe agitation or fatigue, moderate to severe retractions, or is unable to take oral fluids, to bring the child to the emergency department.

A nurse is observing a student preparing to suction a pediatric client through a tracheostomy. The nurse intervenes if the student verbalizes to:

1. Limit insertion and suctioning time to 5 seconds. 2. Reoxygenate the child between suction catheter passes. *3. Apply continuous suction when inserting the catheter.* 4. Use a twisting motion on the catheter when withdrawing the catheter. *Rationale:* The nurse would not use continuous suction on the catheter during insertion; suction is applied only when withdrawing the catheter. Options 1, 2, and 4 represent correct interventions regarding this procedure.

A nurse is reviewing the laboratory results of an infant suspected of having hypertrophic pyloric stenosis. Which of the following would the nurse expect to note in this infant?

1. Metabolic acidosis *2. Metabolic alkalosis* 3. Respiratory acidosis 4. Respiratory alkalosis *Rationale:* Laboratory findings in an infant with hypertrophic pyloric stenosis include metabolic alkalosis as a result of the vomiting (depletes acid) that occurs in this disorder. Additional findings include decreased serum potassium and sodium levels, increased pH and bicarbonate, and decreased chloride level.

A nursing student is asked to discuss the pathophysiology related to childhood leukemia during a clinical conference and reviews the planned presentation with the nursing instructor. The nursing instructor advises the student to review the disorder before the clinical conference if the student states that which of the following is associated with this type of cancer?

1. Normal bone marrow is replaced by blast cells. 2. Red blood cells (RBCs) and platelet production become affected. 3. The reticuloendothelial system is affected. *4. Reed-Sternberg cells are found on biopsy.* *Rationale:* In leukemia, normal bone marrow is replaced by malignant blast cells. As the blast cells take over the bone marrow, eventually RBC and platelet production is affected and the child becomes anemic and thrombocytopenic. The reticuloendothelial system is affected, thus disturbing the body's defense system and rendering these children unable to fight infections normally. The Reed-Sternberg cell is found in Hodgkin's disease.

A nurse checks the vital signs of an infant with a respiratory infection and notes that the respiratory rate is 50 breaths per minute. Which action is appropriate?

1. Notify the registered nurse. 2. Administer oxygen. 3. Recheck the respiratory rate in 15 minutes. *4. Document the findings.* *Rationale:* The normal respiratory rate in an infant is 30 to 60 breaths per minute. The nurse would document the findings.

A nurse provides instructions to the mother of a child with impetigo regarding the application of antibiotic ointment. The mother asks the nurse when the child can return to school. Which response by the nurse is accurate?

1. Ten days after using the antibiotic ointment 2. One week after using the antibiotic ointment *3. Forty-eight hours after using the antibiotic ointment* 4. Twenty-four hours after using the antibiotic ointment *Rationale:* The child should not attend school for 24 to 48 hours after the initiation of systemic antibiotics or for 48 hours after the use of the antibiotic ointment. The school should be notified of the diagnosis. Therefore options 1, 2, and 4 are incorrect.

A woman who states she smokes 2 pack of cigarettes each day is admitted to the labor and delivery suite in labor. The nurse should monitor this labor for which of the following? 1. Delayed placental seperation 2. Late decelerations 3. Shoulder dystocia 4. Precipitous fetal descent

2. The nurse should carefully monitor the labor for late decelerations

When does the uterus return to the pelvic cavity after birth? 1. 2 weeks postpartum 2. 7-9 days postpartum 3. 6 weeks postpartum 4. When lochia changes to alba

2. 7-9 days postpartum Rationale: The uterus involutes and returns to the pelvic cavity after 7-9 days. Failure of the uterus to return to the pelvic cavity is subinvolution.

A jaundice neonate must have a heel stick to assess bilirubin levels. Which of the following actions should the nurse make during the procedure? 1. Cover the foot with an iced wrap for one minute prior to the procedure 2. Avoid puncturing the lateral heel to prevent damageing sensitive structures 3. Blot the site with a dry gauze after rubbing it with an alcohol swab 4. Firmly grasp the calf of the baby during the procedure to prevent injury

3. Alcohol can irritate the punctured skin and cause hemolysis

A neonate is in the warming crib for poor thermoregulation. Which of the following sites is appropriate for the placement of the skin thermal sensor? 1. Xiphoid process 2. Forehead 3. Abdominal Wall 4. Great toe

3. The abdominal wall is the appropriate placement for the skin thermal sensor

At 32 weeks' gestation a 15-year-old primigravid client who is 5'2", has gained 20 lbs, with a 1 lb weight gain in the last 2 weeks. Urinalysis reveals negative glucose and a trace of protein. The nurse should advise the client that which of the following factors increases her risk for preeclampsia? 1. Total weight gain 2. Short stature 3. Adolescent age group 4. Proteinuria

3. Adolescent age group Rationale: Client's with increased risk for preeclampsia include primigravid clients younger than 20 years or older than 40 years, clients with 5 or more pregnancies, women of color, women with multifetal pregnancies, women with diabetes or heart issues. A total weight gain of 20 lbs in the at 32 weeks gestation with a 1 lb weight gain in the last 2 weeks is within normal limits. Trace amounts of protein in the urine is common during pregnancy but amounts of +1 or more may be pregnancy induced hypertension.

A nurse is instructing a patient on signs and symptoms of labor. Teaching was effective if the patient says which? 1. I will call my physician when I feel increased pelvic pressure. 2. I will call my physician when I pass my mucous plug. 3. I will call my physician when my contractions occur every 5 minutes for an hour. 4. I will call my physician when I have bloody show.

3. I will call my physician when my contractions occur every 5 minutes for 1 hour. Rationale: Contractions that occur regularly for an hour indicate the onset of labor. True labor causes contractions that are regular with decreasing intervals between contractions, contractions increase in intensity and duration, and the cervix dilates and effaces. Contractions every 3 to 4 minutes are a sign of active labor. Blood show occurs 24-48 hours before the onset of labor. Lightening occurs when the fetus settles into the pelvic inlet and usually occurs 2 weeks before labor.

The nurse is discussing possible causes of a 4-year-old child's cognitive impairment with his mother. Which causes would the nurse be correct in including in the discussion? 1.) Metabolic disorders, perinatal anoxia, hyperthyroidism 2.) Perinatal infection, Metabolic disorders, and post maturity. 3.) Lead poisoning, prematurity, perinatal anoxia 4.) Iron supplementation, toxoplasmosis, maternal drug use

3.) Lead poisoning, prematurity, perinatal anoxia Pg. 1056 -Foundations of Nursing

A baby has been admitted to the neonatal intensive care unit with a diagnosis of post maturity. The nurse expects to find which of the following during the initial newborn assessment? 1. Abundant lanugo 2. Flat breast tissue 3. Prominent clitoris 4. Wrinkled skin

4. The post term baby does have dry, wrinkled , and often dequamating skin. The baby's dehydration is secondary to a placenta that progressively deteriorates after 40 weeks gestation.

A woman is seeking counseling regarding tubal ligation. Which of the following should the nurse include in her discussion? 1. The woman wil no longer mensrtuate 2. The surgery shuld be done when the woman is ovulating 3. The surgery is easily reversible 4. The woman will be under anesthesia during the procedure

4. This response is correct. BTL surgery, usually performed laparoscopically, is done under general anesthesia

A nurse is caring for patients on the postpartum unit. Which patient is highest risk for postpartum hemorrhage? 1. Patient who received fundal massage 2. Patient with engorged breasts 3. Patient who gives oxytocin 4. Patient with a distended bladder

4. Patient with a distended bladder Rationale: A distended bladder will displace uterus to left or right and may prevent the uterus from fully contracting. An episiotomy can cause discomfort but is not a risk factor for hemorrhage. Fundal massage helps reduce postpartum bleeding.

Which stage of development is most unstable and challenging regarding development of personal identity? A) Adolescence B) Toddler hood C) Childhood D) Infancy

A) Adolescence

A patient suspected of abruptio placentae or placenta previa should be monitorized for bleeding at IV sites and gums due to increased risk of DIC. What isn't DIC related to? A. cervical carcinoma B. fetal demise C. infection/sepsis D. pregnancy-induced hypertension

A. DIC is related to fetal demise, infection/sepis, pregnancy-induced hypertension ( Preeclampsia) and abruptio placentae. Cervical carcinoma is related to podophyllin

A woman who thinks she could be pregnant calls her neighbor, a nurse, to ask when she could use a home pregnancy test to diagnose pregnancy. Which response is best? A. home pregnancy test can be used right after your first missed period. B.These tests are most accurate after you have missed your second period. C. Home pregnancy tests often give false positives and should not be trusted. D . The test can provide accurate information when used right after ovulation.

A. A home pregnancy test can be used right after your first missed period.

The nurse is performing a gestational age assessment on a full-term newborn during the first hour of transition using the Ballard (Dubowitz) scale. Based on this assessment, the nurse determines that the neonate has a maturity rating of 40-weeks. What findings should the nurse identify to determine if the neonate is small for gestational age (SGA)? (Select all that apply.)

A. Admission weight of 4 pounds, 15 ounces (2244 grams). B. Head to heel length of 17 inches (42.5 cm). C. Frontal occipital circumference of 12.5 inches (31.25 cm).

The mother of a toddler asks a nurse when it is safe to place the car safety seat in a face-forward position. The best nursing response is which of the following? A) When the toddler weighs 20 lbs B) The seat should not be placed in a face-forward position unless there are safety locks in the car C) The seat should never be place in a face-forward position because the risk of the child unbuckling the harness D) When the weight of the toddler is greater than 40 lbs

A. The transition point for switching to the forward facing position is defined by the manufacturer of the convertible car safety seat but is generally at a body weight of 9 kg or 20 lb and 1 year of age. Convertible car safety seats are used until the child weighs at least 40 lb. Options b, c, and d are incorrect

The nurse is assessing a 3 day old infant with a cephalohematoma in the newborn nursery. Which assessment finding should the nurse report to the healthcare provider? A. yellowish tinge to the skin B. babinski reflex present bilaterally C. pink papular rash on the face D. moro reflex noted after a loud noise

A. yellowish tinge to the skin

The nurse observes a 10-month-old infant using her index finger and thumb to pick up pieces of cereal. This behavior is evidence that the infant has developed: a. the pincer grasp. b. a grasp reflex. c. prehension ability. d. the parachute reflex.

ANS: A By 1 year, the pincer-grasp coordination of index finger and thumb is well established.

Peer relationships become more important during adolescence because: a. Adolescents dislike their parents. b. Adolescents no longer need parental control. c. They provide adolescents with a feeling of belonging. d. They promote a sense of individuality in adolescents.

ANS: C The peer group serves as a strong support to teenagers, providing them with a sense of belonging and strength and power. During adolescence the parent/child relationship changes from one of protection-dependency to one of mutual affection and quality. Parents continue to play an important role in the personal and health-related decisions. The peer group forms the transitional world between dependence and autonomy.

The most common cause of death in the adolescent age-group involves: a. Drownings. b. Firearms. c. Drug overdoses. d. Motor vehicles.

ANS: D 36% of all adolescent deaths in the United States are the result of motor vehicle accidents. Drownings, firearms, and drug overdoses are major concerns in adolescence but do not cause the majority of deaths.

How should the body be delivered after nunchal cord has been excluded?

Apply gentle traction on the anterior shoulder, advising the client not to push.

A nurse is preparing a teaching plan for a child with hemophilia and his parents. Which of the following would the nurse be least likely to include to manage a bleeding episode? a) Apply direct pressure to the area. b) Administer factor VIII replacement. c) Apply heat to the site of bleeding. d) Elevate the injured area such as a leg or arm.

Apply heat to the site of bleeding. Explanation: Ice or cold compresses, not heat, would be applied to the site of bleeding. Direct pressure is applied until the bleeding stops. The injured part is elevated unless elevating would contribute to further injury. Factor VIII replacement is given to replace the missing clotting factor.

The nurse preceptor should intervene if the graduate practical nurse performs which action when caring for a jaundiced newborn being treated with phototherapy?

Applying a shirt while the newborn is exposed to phototherapy

A primigravid client in early labor is admitted and reports intense back pain with contractions. The fetal position is determined to be right occiput posterior. Which action by the nurse would be most helpful for alleviating the client's back pain during early labor?

Applying counterpressure to the client's sacrum during contractions

Which of the following would lead the nurse to suspect that a large-for-gestational-age newborn has experienced birth trauma? a) Temperature instability b) Seizures c) Feeble sucking d) Asymmetrical movement

Asymmetrical movement Correct Explanation: A birth injury is typically characterized by asymmetrical movement. Temperature instability, seizures, and feeble sucking suggest hypoglycemia.

When examining a client after delivery, the nurse finds the fundus soft, boggy, and displaced above and to the right of the umbilicus. What action should the nurse take first in this case? A. Have the client empy her bladder B. Perform fundal massage C. Administer narcotic analgesics D. Administer codeine and meperidine

B. First the nurse should perform fundal massage; then have the client empt her bladder

Approximately 20 prior to nerve block anesthesia, the client should be hydrated with 500-1000 cc of lactated ringers IV. What should the nurse do if hypotension offurs? A. Administer Stadol B. Administer O2 at 10 L/min by face mask C. Administer CO2 at 10 L/min by face mask D. Administer Nubain

B. If hypotension occurs- turn client to her side, administer O2 at 10 L/min by face mask, and increase IV rate

A nurse is evaluating the developmental level of a 2-year-old. Which of the following does the nurse expect to observe in this child? A) Uses a fork to eat B) Uses a cup to drink C) Uses a knife for cutting food D) Pours own milk into a cup

B. By age 2 years, the child can use a cup and can use a spoon correctly but with some spilling. By ages 3 to 4, the child begins to use a fork. By the end of the preschool period, the child should be able to pour milk into a cup and begin to use a knife for cutting.

client who is attending antepartum classes asks the nurse why her healthcare provider has prescribed iron tablets. The nurse's response is based on what knowledge? A. Supplementary iron is more efficiently utilized during pregnancy. B. It is difficult to consume 18 mg of additional iron by diet alone. C. Iron absorption is decreased in the GI tract during pregnancy. D. Iron is needed to prevent megaloblastic anemia in the last trimester.

B. It is difficult to consume 18 mg of additional iron by diet alone.

In addition to the child's history, symptoms, and blood studies, which of the following helps to confirm the diagnosis of leukemia? a) Genetic studies b) Chest x-rays c) Bone marrow aspiration d) Modified Jones criteria

Bone marrow aspiration Explanation: In addition to the his tory, symptoms, and laboratory blood studies, a bone mar row aspiration must be done to confirm the diagnosis of leu kemia. Genetic studies are done for hereditary diseases such as sickle cell anemia and hemophilia. The modified Jones criteria are used as a guide to note the manifestations of rheumatic fever, and chest x-rays help in diagnosing congestive heart failure.

Supplemental iron is not givento clients with sickle cell anemia because the anemia is not caused by iron deficiency. What aspect is very important in treatment of sickle cell disease because it promotes hemodilution and circulation of red cells through the blood vessels? A. HgbAS B. HGBS C. Hydration D. Hydrotherapy

C. Hydration is very important in treatment of sickle cell disease because it promotes hemodilution and curculation of red cells through blood vessels

A pregnant client has a temperature over 100.4 F, Dysuria and fluid discharge from vagina. What could these signs most probable indicate? A. Preeclampsia B. Eclampsia C. Infection D. Change in fetal movement

C. Temperature ofve 100.4 F, Dysuria, and fluid discharge from vagina are signs of infection.

In autosomal recessive disease, both parents must be hererozygous, or carriers of the recessive trait, for the disease to be expressed in their offspring. If both parents are heterozygous, what is the chance the baby to have the disease as well? A. 1:2 B. 1:3 C. 1:4 D. 1:1

C. With each pregnancy, there is a 1:4 chance of the infant having the disease.

Which minerals and vitamins usually are recommended to supplement a pregnant woman's diet? A. Fat-soluble vitamins A and D B. Water-soluble vitamins C and B6 C. Iron and folate D. Calcium and zinc

C. Iron and folate Fat-soluble vitamins should be supplemented as a medical prescription, as vitamin D might be for lactose-intolerant women. Water-soluble vitamin C sometimes is consumed in excess naturally; vitamin B6 is prescribed only if the woman has a very poor diet. Iron generally should be supplemented, and folic acid supplements often are needed because folate is so important. Zinc sometimes is supplemented. Most women get enough calcium.

A 38-week primigravida who works as a secretary and sits at a computer 8 hours each day tells the nurse that her feet have begun to swell. Which instruction would be most effective in preventing pooling of blood in the lower extremities? A.Wear support stockings. B. Reduce salt in her diet. C. Move about every hour. D. Avoid constrictive clothing.

C. Move about every hour.

Diagnostic tests for Leukemia

CBC, bone marrow aspirate, peripheral blood smear. Performing a lumbar puncture is an option to determine whether the CNS has been infiltrated

Diagnostic tests for iron-deficiency anemia

CBC, reticulocyte count, serum ferritin, serum iron, total iron binding capacity

One hour after giving birth to an 8-␣␣␣␣␣␣␣␣␣␣␣␣␣␣␣␣␣␣␣␣␣␣␣␣␣␣␣␣␣␣␣␣␣␣␣␣␣ has increased from small to large and her fundus is boggy despite massage. The client's pulse is 84 beats/minute and blood pressure is 156/96. The healthcare provider prescribes Methergine 0.2 mg IM × 1. What action should the nurse take immediately?

Call the healthcare provider to question the prescription. Methergine is contraindicated for clients with elevated blood pressure, so the nurse should contact the healthcare provider and question the prescription

What are the assessments for stage 2 of labor?

Cervical dilation is complete. Progress of labor is measured by descent of the fetal head through the birth canal (fetal station). Uterine contractions occur every 2 to 3 minutes, lasting 60-75 seconds, and strong. Mother feels urge to bear down.

A pregnant client has chosen labor induction to have a vaginal birth after cesarean. During labor, which is the most concerning assessment finding?

Change in uterine shape and maternal tachycardia

A 4-week-old premature infant has been receiving epoetin alfa (Epogen) for the last three weeks. Which assessment finding indicates to the nurse that the drug is effective?

Changes in apical heart rate from the 180s to the 140s.

A 4+ reflex in a client with pregnancy-induced hypertension (A) indicates hyperreflexia, which is an indication of an impending seizure.

Check the infant's oxygen saturation rate.

The nurse is reviewing the chart of a pregnant client at 10 weeks gestation during the first prenatal appointment. Which finding indicates that client teaching is needed?

Client has a pet dog and cat at home

Monitoring for fetal position is important because the mother cannot tell you she has back pain, which is the cardinal sign of persistant posterior fetal position. Why do the regional blocks, especially epidural and caudal, often result in assisted delivery? A. inability to push effectively in 3rd stage B. inability to push effectively in 4th stage C. inability to push effectively in 1st stage D. inability to push effectively in 2nd stage

D. Regional blocks, especially epidural and caudal, often result in assisted delivery due to inability to push effectively in the 2nd stage.

A pregnant woman comes to the prenatal clinic for an initial visit. In reviewing her childbearing hx, the client indicates that she has delivered premature twins, one full term baby and has had no abortions. Which GTPAL should the nurse document in this client's record? A. 3-1-2-0-3 B. 4-1-2-0-3 C. 2-1-2-1-2 D. 3-1-1-0-3

D. 3-1-1-0-3

A woman who had a miscarriage 6 months ago becomes pregnant. Which instruction is most important for the nurse to provide this client? A.Elevate lower legs while resting. b.Increase caloric intake by 200 to 300 calories per day. c.Increase water intake to 8 full glasses per day. d.Take prescribed multivitamin and mineral supplements.

D. Take prescribed multivitamin and mineral supplements.

Just after delivery, a new mother tells the nurse, "I was unsuccessful breastfeeding my first child, but I would like to try with this baby." Which intervention is best for the nurse to implement first? A.Assess the husband's feelings about his wife's decision to breastfeed their baby. B.Ask the client to describe why she was unsuccessful with breastfeeding her last child. C.Encourage the client to develop a positive attitude about breastfeeding to help ensure success. D.Provide assistance to the mother to begin breastfeeding as soon as possible after delivery.

D.Provide assistance to the mother to begin breastfeeding as soon as possible after delivery.

A 33-week-gestation infant has just been born. The child's heartbeat is not audible. Which of the following is the priority nursing intervention? a) Depression of the sternum with both thumbs 1 to 2 cm at a rate of 100 times per minute b) Transfer to a transitional or high-risk nursery for continuous cardiac surveillance c) Administration of IV epinephrine, as prescribed d) Palpation for a femoral pulse

Depression of the sternum with both thumbs 1 to 2 cm at a rate of 100 times per minute Explanation: If an infant has no audible heartbeat, or if the cardiac rate is below 60 beats per minute, closed-chest massage should be started. Hold the infant with fingers encircling the chest and wrapped around the back and depress the sternum with both your thumbs, on the lower third of the sternum approximately one third of its depth (1 or 2 cm) at a rate of 100 times per minute. If the pressure and the rate of massage are adequate, it should be possible, in addition, to palpate a femoral pulse. If heart sounds are not resumed above 60 beats per minute after 30 seconds of combined positive-pressure ventilation and cardiac compressions, intravenous epinephrine may be prescribed. Following cardio-resuscitation, newborns need to be transferred to a transitional or high-risk nursery for continuous cardiac surveillance to be certain cardiac function is maintained.

A nurse caring for an 8-year-old patient with a bleeding disorder documents the following nursing diagnosis: ineffective tissue perfusion related to intravascular thrombosis and hemorrhage. This diagnosis is most appropriate for a patient with: a) Hemophilia b) von Willebrand disease c) Iron deficiency anemia d) Disseminated intravascular coagulation

Disseminated intravascular coagulation Explanation: Disseminated intravascular coagulation (DIC) is an acquired coagulopathy that, paradoxically, is characterized by both thrombosis and hemorrhage. The outcome for this patient is: The child will maintain adequate tissue perfusion of all body systems affected by DIC and regain adequate laboratory values for hemostasis.

A 24-hour-old newborn has a pink papular rash with vesicles superimposed on the thorax, back, and abdomen. What action should the nurse implement?

Document the finding in the infant's record.

A client who has an autosomal dominant inherited disorder is exploring family planning options and the risk of transmission of the disorder to an infant. The nurse's response should be based on what information?

Each pregnancy carries a 50% chance of inheriting the disorder.

A full-term newborn of a mother with gestational diabetes is slightly jittery with a blood glucose level of 45 mg/dL. The practical nurse (PN) is assisting the registered nurse to implement the appropriate intervention. Which action does the PN anticipate first?

Feed the newborn

The nurse is caring for a child with leukemia. Which of the following nursing interventions would be the highest priority for this child? a) Encouraging the child to share feelings b) Grouping nursing care c) Following guidelines for protective isolation d) Providing age appropriate activities

Following guidelines for protective isolation Explanation: The child with leukemia is susceptible to infection, especially during chemotherapy. Infections such as meningitis, septicemia, and pneumonia are the most common causes of death. To protect the child from infectious organisms, follow standard guidelines for protective isolation. Grouping nursing care to provide rest is important, but not the highest priority. Encouraging the child to share feelings and providing age appropriate activities are important, but psychological issues are a lower priority than physical.

After teaching a group of students about the effects of prematurity on various body systems, the instructor determines that the class was successful when the students identify which of the following as an effect of prematurity? a) Fragile cerebral blood vessels b) Enhanced ability to digest proteins c) Enlarged respiratory passages d) Rapid glomerular filtration rate

Fragile cerebral blood vessels Explanation: Preterm newborns have fragile blood vessels in the brain, and fluctuations in blood pressure can predispose these vessels to rupture, causing intracranial hemorrhage. The preterm newborn typically has smaller respiratory passages, leading to an increased risk for obstruction. Preterm newborns have a limited ability to digest proteins. The preterm newborn's renal system is immature, which reduces his or her ability to concentrate urine and slow the glomerular filtration rate.

Sickle Cell Anemia

Genetic disorder characterized by abnormal forms of hemoglobin with the erythrocyte and sickle cells. As sickled cells clump, circulation slows, resulting in obstructions with severe tissue hypoxia and necrosis

The nurse begins intermittent oral feedings for a small-for-gestational-age newborn to prevent which of the following? a) Asphyxia b) Meconium aspiration c) Hypoglycemia d) Polycythemia

Hypoglycemia Correct Explanation: Intermittent oral feedings are initiated to prevent hypoglycemia as the newborn now must assume control of glucose homeostasis. Hydration and frequent monitoring of hematocrit are important to prevent polycythemia. Resuscitation and suctioning are used to manage meconium aspiration. Immediate resuscitation is used to manage asphyxia.

In an infant who has hypothermia, what would be an appropriate nursing diagnosis? a) Ineffective parental attachment b) Impaired tissue perfusion c) Alteration in nutrition d) Impaired skin integrity

Impaired tissue perfusion Correct Explanation: Impaired tissue perfusion would be appropriate and may be related to cardiopulmonary, cerebral, gastrointestinal, peripheral, or renal issues.

The nurse in a newborn nursery is observing for developmentally appropriate care. Which of the following is an example of self-regulation? a) Infant has hand in mouth b) Infant is kicking feet c) Infant is crying d) Infant is quiet

Infant has hand in mouth Correct Explanation: Self-regulation is a form of self-soothing for an infant such as sucking on hands or putting hand to mouth type of movements.

A full term infant is transferred to the nursery from labor and delivery. Which information is most important for the nurse to receive when planning immediate care for the newborn?

Infant's condition at birth and treatment received. Immediate care is most dependent on the infant's current status (i.e., Apgar scores at 1 and 5 minutes) and any treatment or resuscitation that was indicated. The transitional care nurse needs the information listed in the choices

Immediately after birth a newborn infant is suctioned, dried, and placed under a radiant warmer. The infant has spontaneous respirations and the nurse assesses an apical heart rate of 80 beats/minute and respirations of 20 breaths/minute. What action should the nurse perform next?

Initiate positive pressure ventilation. The nurse should immediately begin positive pressure ventilation (A) because this infant's vital signs are not within the normal range, and oxygen deprivation leads to cardiac depression in infants. (The normal newborn pulse is 100 to 160 beats/minute and respirations are 40 to 60 breaths/minute.) Waiting until the infant is 1 minute old to intervene may worsen the infant's condition. According to neonatal resuscitation guidelines, CPR is not begun until the heart rate is 60 or below or between 60 and 80 and not increasing after 20 to 30 seconds of PPV. (D) can be checked after treating the respiratory rate.

What type of bleeding accounts for more hemophilic deaths than any other bleeding?

Intracranial hemorrhage (either spontaneous or secondary to head trauma) is life threatening and accounts for more hemophilic deaths than any other bleeding

A woman delivers her baby immediately on arrival at the emergency department and 5 minutes later delivers the placenta. The nurse's assessment is that the woman's uterus is boggy and midline. What action should the nurse take first?

Massage the fundus

A 35-year-old client has just given birth to a healthy newborn during her 43rd week of gestation. Which of the following should the nurse expect when assessing the condition of the newborn? a) Meconium aspiration in utero or at birth b) Yellow appearance of the newborn's skin c) Tremors, irritability, and high-pitched cry d) Seizures, respiratory distress, cyanosis, and shrill cry

Meconium aspiration in utero or at birth Correct Explanation: Infants born after 42 weeks of pregnancy are postterm. These infants are at a higher risk of swallowing or aspirating meconium in utero or after birth. As soon as the infant is born, the nurse usually suctions out the secretions and fluids in the newborn's mouth and throat before the first breath to avoid aspiration of meconium and amniotic fluid into the lungs. Seizures, respiratory distress, cyanosis, and shrill cry are signs and symptoms of infants with intracranial hemorrhage. Intracranial hemorrhage can be a dangerous birth injury that is primarily a problem for preterm newborns, not postterm neonates. Yellow appearance of the newborn's skin is usually seen in infants with jaundice. Tremors, irritability, high-pitched or weak cry, and eye rolling are seen in infants with hypoglycemia.

A client who gave birth to a healthy 8 pound infant 3 hours ago is admitted to the postpartum unit. Which nursing plan is best in assisting this mother to bond with her newborn infant?

Meet the mother's physical needs and demonstrate warmth toward the infant. It is most important to meet the mother's requirement for attention to her needs so that she can begin infant care-taking (D). Nurse theorist Reva Rubin describes the initial postpartal period as the "taking-in phase," which is characterized by maternal reliance on others to satisfy the needs for comfort, rest, nourishment, and closeness to families and the newborn

The nurse assesses a client admitted to the labor and delivery unit and obtains the following data: dark red vaginal bleeding, uterus slightly tense between contractions, BP 110/68, FHR 110 beats/minute, cervix 1 cm dilated and uneffaced. Based on these assessment findings, what intervention should the nurse implement?

Monitor bleeding from IV sites Monitoring bleeding from peripheral sites (C) is the priority intervention. This client is presenting with signs of placental abruption. Disseminated intravascular coagulation (DIC) is a complication of placental abruptio, characterized by abnormal bleeding. Invasive vaginal procedures (A and B) or (D) can increase the abruption and bleeding, so these interventions are contraindicated.

Which of the following data is indicative of hypothermia of the preterm infant?

Nasal flaring Correct Explanation: Nasal flaring is a sign of respiratory distress. Infants with hypothermia show signs of respiratory distress (cyanosis, increased respirations, low oxygen saturation, nasal flaring, and grunting). The other choices are normal findings.

The nurse is caring for a child with DIC. The nurse notices signs of neurological deficit. The appropriate nursing action is to: a) Continue to monitor neurological signs b) Notify the physician c) Inspect for signs of bleeding d) Evaluate respiratory status

Notify the physician Explanation: If neurological deficits are assessed, immediate reporting of the findings is necessary to begin treatment to prevent permanent damage.

Vasoocclusive Crisis

Obstruction of the small vessels of the hands and feet, resulting in edema, impaired ROM, and pain

A client at 30-weeks gestation, complaining of pressure over the pubic area, is admitted for observation. She is contracting irregularly and demonstrates underlying uterine irritability. Vaginal examination reveals that her cervix is closed, thick, and high. Based on these data, which intervention should the nurse implement first?

Obtain a specimen for urine analysis Obtaining a urine analysis (C) should be done first because preterm clients with uterine irritability and contractions are often suffering from a urinary tract infection, and this should be ruled out first.

What is the first action the nurse takes in surfactant administration? a) Hold feedings. b) Call pharmacy for medication. c) Obtain and document baseline vital signs. d) Change the infant's diaper.

Obtain and document baseline vital signs. Correct Explanation: Obtaining a baseline set of vital signs is the first step in surfactant administration. The nurse will need a baseline in case there is any reaction to the medication administration. The other choices are not the first thing done before instilling this medication.

After feeding a newborn, how should the nurse position the infant in the crib?

On the right side.

Medical management of Vasoocclusive crisis

Palliative (analgesics, hydration, oxygen)

Which of the following places newborns at risk for ongoing health problems? a) Average weight b) Term birth c) Vaginal delivery d) Perinatal asphyxia

Perinatal asphyxia Correct Explanation: Several disorders can place newborns at risk for ongoing health problems such as prematurity, low birth weight, congenital abnormalities, perinatal asphyxia, and birth trauma. These conditions need further nursing assessment and care for optimal growth and healing. The other choices do not place a risk on the infant.

Monitoring bleeding from peripheral sites (C) is the priority intervention. This client is presenting with signs of placental abruption. Disseminated intravascular coagulation (DIC) is a complication of placental abruptio, characterized by abnormal bleeding. Invasive vaginal procedures (A and B) or (D) can increase the abruption and bleeding, so these interventions are contraindicated.

Places the infant prone in the bassinet.

The small-for-gestation neonate is at increased risk for which complication during the transitional period? a) Hyperthermia due to decreased glycogen stores b) Polycythemia probably due to chronic fetal hypoxia c) Hyperglycemia due to decreased glycogen stores d) Anemia probably due to chronic fetal hypoxia

Polycythemia probably due to chronic fetal hypoxia Correct Explanation: The small-for-gestation neonate is at risk for developing polycythemia during the transitional period in an attempt to decrease hypoxia. This neonate is also at increased risk for developing hypoglycemia and hypothermia due to decreased glycogen stores

S/Sx of Ductus arteriosus

Poor eating, poor growth patterns, fatigue, widened pulse pressure, bounding pulses, machine-like murmur audible at the upper left sternal border, HF

An 18-year-old client has given birth in the 28th week of gestation, and her newborn is showing signs of respiratory distress syndrome (RDS). Which is true for a newborn with RDS? a) Respiratory symptoms of RDS typically improve within a short period of time. b) RDS is caused by a lack of alveolar surfactant. c) Glucocorticosteroid is given to the newborn following birth. d) RDS is characterized by heart rates below 50 beats per minute.

RDS is caused by a lack of alveolar surfactant. Correct Explanation: Respiratory distress syndrome (RDS) is a serious breathing disorder caused by a lack of alveolar surfactant. Betamethasone, a glucocorticosteroid, is often given to the mother 12 to 24 hours before a preterm birth to help reduce the severity of RDS, not to the newborn following birth. Respiratory symptoms in the newborn with RDS typically worsen within a short period of time after birth, not improve. Diagnosis of RDS is made based on a chest x-ray and the clinical symptoms of increasing respiratory distress, crackles, generalized cyanosis, and heart rates exceeding 150 beats per minute (not below 50 beats per minute).

A 30-year-old multiparous woman who has a 3-year-old boy and an newborn girl ␣␣␣␣␣␣␣␣␣␣␣␣␣␣␣␣␣␣␣␣␣␣␣␣␣␣␣␣␣␣␣␣␣␣␣␣␣␣␣␣␣␣␣␣␣␣␣␣␣␣␣␣␣␣␣␣␣␣␣␣␣␣␣␣␣␣␣␣␣␣␣␣␣␣␣␣␣␣␣␣␣␣ manage both children when I get home." How should the nurse respond?

Regression in behaviors in the older child is a typical reaction so he needs attention at this time.

A preterm newborn receives oxygen therapy to treat respiratory distress syndrome (RDS). Which of the following should the nurse consider as a complication of oxygen administration at a high concentration? a) Retinopathy of prematurity b) Diminished erythropoiesis c) Bronchopulmonary dysplasia d) Necrotizing enterocolitis

Retinopathy of prematurity Correct Explanation: Retinopathy of prematurity can occur as a complication associated with the use of high concentrations of oxygen. High concentrations of oxygen can damage the fragile retinal blood vessels of the preterm infants and cause retinopathy. Bronchopulmonary dysplasia, diminished erythropoiesis, and necrotizing enterocolitis are not complications associated with a high concentration of oxygen. Bronchopulmonary dysplasia is a chronic lung disease that results from the effect of long-term mechanical ventilation. Diminished erythropoiesis in the preterm newborn is due to immaturity of the hemopoietic system. Necrotizing enterocolitis is associated with ischemia of the bowel, leading to necrosis and perforation.

What intervention is indicated for a pregnant client with late decelerations?

She should lie on her left side, oxygen should be given, and the physician should be notified immediately.

A nurse is caring for a baby girl born at 34 weeks' gestation. Which of the following features should the nurse identify as those of a preterm newborn? a) Scant coating of vernix b) Closely approximated labia c) Paper-thin eyelids d) Shiny heels and palms

Shiny heels and palms Correct Explanation: A preterm newborn has shiny heels and palms with few creases. The eyelids of the preterm newborn are edematous, and not paper-thin. The external genitalia in the preterm baby girl appear large with widely spaced labia, and not closely approximated. Vernix is scant in post-term newborns and is excessive in premature infants.

A couple, concerned because the woman has not been able to conceive, is referred to a healthcare provider for a fertility workup and a hysterosalpingography is scheduled. Which postprocedure complaint indicates that the fallopian tubes are patent?

Shoulder pain If the tubes are patent (open), pain is referred to the shoulder (C) from a subdiaphragmatic collection of peritoneal dye/gas.

Nursing Interventions for Hemophilia patients

Teaching child and family about avoiding injury and control bleeding, and safety measures to take in regards to toys, throw rugs and stairs and to avoid contact sports

What is the normal postpartum temperature?

Temperature may increase to 100.4 because of inflammation associated with delivery. Higher temperature elevations may be caused by infection.

What is the rooting reflex?

Touching the newborn's lip, cheek, or corner of the mouth with a nipple causes the newborn to turn the head toward the nipple and open mouth. The newborn takes hold of the nipple and sucks. The rooting reflex usually disappears 3 to 4 months.

Treatment for Aplastic Anemia

Tx aims at restoring the function of the marrow through immunosuppressive Tx or at replacing the defective marrow through bone marrow transplantation

A nurse is providing care for a child with disseminated intravascular coagulation (DIC). Which of the following would alert the nurse to possible neurologic compromise? a) Widely fluctuating blood pressure b) Petechiae c) Equal pupillary response d) Hematuria

Widely fluctuating blood pressure Explanation: A key aspect of the nurse's role is to assess the child for signs and symptoms of impaired tissue perfusion in the various body systems that may be affected by DIC. Unstable or abnormal blood pressure such as wide fluctuations in blood pressure or unequal pupil size may suggest neurologic compromise. Hematuria would suggest renal compromise. Petechiae would be indicative of bleeding into the skin

A new mother asks the nurse, "How do I know that my daughter is getting enough breast milk?" Which explanation should the nurse provide?

Your milk is sufficient if the baby is voiding pale straw-colored urine 6 to 10 times a day.

A full-term infant is admitted to the newborn nursery and, after careful assessment, the nurse suspects that the infant may have an esophageal atresia. Which symptoms is this newborn likely to have exhibited? a. Choking, coughing, and cyanosis. b. Projectile vomiting and cyanosis. c. Apneic spells and grunting. d. organomegaly.

a. Choking, coughing, and cyanosis.

What was founded by Lillian Wald? a. National Commission on Children b. Henry Street Settlement c. White House Conference d. U.S. Children's Bureau

b. Henry Street Settlement Lillian Wald, regarded as the founder of public health, founded Henry Street Settlement, which provided nursing services and social assistance. p. 932

The nurse caring for a laboring client encourages her to void at least q2h, and records each time the client empties her bladder. What is the primary reason for implementing this nursing intervention?

chlamydia

30. The major source of nutrients in the diet of a pregnant woman should be composed of: a. Simple sugars b. Fats c. Fiber d. Complex carbohydrates

d. Complex carbohydrates

A nurse reinforces instructions to the mother of a child who has been hospitalized with croup. Which of the following statements, if made by the mother, would indicate the need for further instruction?

*1. "I will give my child cough syrup if a cough develops."* 2. "During an attack, I will take my child to a cool location." 3. "I will give acetaminophen (Tylenol) if my child develops a fever." 4. "I will be sure that my child drinks at least three to four glasses of fluids every day." *Rationale:* Cough syrups and cold medicines are not to be given, because they may dry and thicken secretions. During a croup attack, the child can be taken to a cool basement or garage. Acetaminophen is used if a fever develops. Adequate hydration of 500 to 1000 mL of fluids daily is important for thinning secretions.

A nurse reinforces instructions to the mother of a child with croup about the measures to take if an acute spasmodic episode occurs. Which statement by the mother indicates the need for further instruction?

*1. "I will place a steam vaporizer in my child's room."* 2. "I will take my child out into the cool, humid night air." 3. "I will place a cool-mist humidifier in my child's room." 4. "I will place my child in a closed bathroom and allow my child to inhale steam from the running water." *Rationale:* Steam from warm running water in a closed bathroom and cool mist from a bedside humidifier are effective for reducing mucosal edema. Cool-mist humidifiers are recommended as compared with steam vaporizers, which present a danger of scalding burns. Taking the child out into the cool, humid night air may also relieve mucosal swelling. Remember, however, that a cold mist may precipitate bronchospasm.

A 4-year-old child sustains a fall at home and is brought to the emergency department by the mother. Following x-ray examination, it has been determined that the child has a fractured arm, and a plaster cast is applied. The nurse provides instructions to the mother regarding cast care for the child. Which statement by the mother indicates a need for further instruction?

*1. "The cast will feel warm when it is dried."* 2. "If the cast becomes wet, a fan may be used to dry the cast." 3. "I need to call the health care provider if any blood or drainage appears on the cast." 4. "I can apply ice to the casted area to prevent swelling." *Rationale:* Once the cast dries, the cast will sound hollow and will be cool (not warm) to touch. A fan can be directed toward the cast to facilitate drying. The mother must be instructed to call the health care provider if any blood or drainage appears on the cast. Ice can be applied to the casted area to prevent swelling.

A nurse is checking the capillary refill of a child with a cast applied to the left arm. The nurse compresses the nail bed of a finger and it returns to its original color in 2 seconds. Which action should be taken by the nurse?

*1. Document the findings.* 2. Notify the registered nurse (RN). 3. Prepare the child for bivalving the cast. 4. Elevate the extremity and recheck the capillary refill immediately. *Rationale:* When checking capillary refill, the nurse would expect to note that a compressed nail bed will return to its original color in less than 3 seconds. Options 2, 3, and 4 are unnecessary actions.

A nurse is assigned to care for a child after a myringotomy with the insertion of tympanostomy tubes. The nurse notes a small amount of reddish drainage from the child's ear after the surgery. On the basis of this finding, the nurse takes which action?

*1. Documents the findings* 2. Notifies the registered nurse immediately 3. Changes the ear tubes so that they do not become blocked 4. Checks the ear drainage for the presence of cerebrospinal fluid *Rationale:* After a myringotomy with the insertion of tympanostomy tubes, the child is monitored for ear drainage. A small amount of reddish drainage is normal during the first few days after surgery. However, any heavy bleeding or bleeding that occurs after 3 days should be reported. The nurse would document the findings. Options 2, 3, and 4 are not necessary.

A school-age child with type 1 diabetes mellitus has soccer practice three afternoons a week. The nurse reinforces instructions regarding how to prevent hypoglycemia during practice. The nurse tells the child to:

*1. Drink a half a cup of orange juice before soccer practice.* 2. Eat twice the amount that is normally eaten at lunchtime. 3. Take half of the amount of prescribed insulin on practice days. 4. Take the prescribed insulin at noontime rather than in the morning. *Rationale:* An extra snack of 10 g to 15 g of carbohydrates eaten before activities and for every 30 to 45 minutes of activity will prevent hypoglycemia. A half cup of orange juice will provide the needed carbohydrates. The child or parents should not be instructed to adjust the amount or time of insulin administration, and meal amounts should not be doubled.

A 1-year-old child with hypospadias is scheduled for surgery to correct this condition. A nurse is asked to assist in preparing a plan of care for this child and makes suggestions, knowing that this surgery is taking place at a time when:

*1. Fears of separation and mutilation are present* 2. Sibling rivalry will cause regression to occur 3. Embarrassment of voiding irregularities is common 4. Concern over size and function of the penis is present *Rationale:* At the age of 1 year, a child's fears of separation and mutilation are present because the child is facing the developmental task of trusting others. As the child gets older, fears about virility and reproductive ability may surface. The question does not provide enough data to determine that siblings exist. Options 3 and 4 may be issues if the child were older.

A nurse is caring for a 3-year-old child with suspected bacterial meningitis. Which signs and symptoms would the nurse expect to find during the initial data collection? *Select all that apply.*

*1. Fever* 2. Cough *3. Irritability* 4. Hypothermia *5. Nuchal rigidity* 6. Closed anterior fontanel *Rationale:* The initial signs and symptoms of bacterial meningitis include fever, nuchal rigidity, and irritability. The anterior fontanel closes by 12 to 18 months of age. Cough usually is not associated with bacterial meningitis.

A 4-year-old child with acute lymphocytic leukemia has been admitted to the hospital in relapse. The priority concern is infection due to immunosuppression. Which of the following interventions would the nurse include in the plan of care?

*1. Perform oral hygiene four times a day.* 2. Monitor vital signs once a shift. 3. Inspect the child's mouth daily for mouth ulcers. 4. Administer acetaminophen (Tylenol) suppositories for increased temperature. *Rationale:* The child who is immunosuppressed is at risk for infection, and interventions must be performed frequently to prevent infection. The nursing interventions as stated in options 2 and 3 are incorrect because of the time frames. Suppositories are never administered when a child is immunocompromised because of the risk of perineal fissures.

A nurse is caring for a hospitalized child with a diagnosis of rubella (German measles). The nurse reviews the health care provider's progress notes and reads that the child has developed Forchheimer sign. Based on this documentation, which of the following should the nurse expect to note in the child?

*1. Petechiae spots located on the palate* 2. Small blue-white spots noted on the buccal mucosa 3. A fiery red edematous rash on the cheeks 4. Swelling of the parotid gland *Rationale:* Forchheimer sign refers to petechiae spots, which are reddish and pinpoint and located on the soft palate. Small blue-white spots noted on the buccal mucosa are known as Koplik's spots seen in rubeola. A fiery red edematous rash on the cheeks, also called "slapped cheeks" is seen in erythema infectiosum. Swelling of the parotid gland is seen in mumps.

A child with a diagnosis of pertussis (whooping cough) is being admitted to the pediatric unit. As soon as the child arrives in the unit, the nurse would first:

*1. Place the child on a pulse oximeter.* 2. Weigh the child. 3. Take the child's temperature. 4. Ask the parents about the child. *Rationale:* To adequately determine whether the child is getting enough oxygen, the child is placed on a pulse oximeter. The pulse oximeter will then provide ongoing information on the child's oxygen level. The child is also immediately placed on a cardiorespiratory monitor to provide early identification of periods of apnea and bradycardia. The nurse would then gather data including taking the child's temperature and weight and asking the parents about the child.

An 8-year-old boy is being treated with percussion treatments for cystic fibrosis. How would the nurse determine whether the treatment is effective?

*1. The child has a productive cough of thick sputum.* 2. The child no longer has a fever. 3. The child's skin is no longer high in sodium. 4. The child's bowel movements are firmer. *Rationale:* Percussion treatments are intended to produce sputum. Thick sputum is characteristic of cystic fibrosis. Being afebrile is not necessarily reflective of effectiveness of percussion treatments. Although a high sodium content in the skin is a sign associated with cystic fibrosis, percussion treatments will not help this characteristic. The percussion treatments will not help bowel movements.

A child with a tracheal obstruction is brought to the emergency department by emergency medical services. The child aspirated a grape, and the foreign body was removed by direct laryngoscopy. Following the procedure, the nurse plans to inform the mother of the child that:

*1. The child will need to be hospitalized for observation.* 2. The child may go home with a prescription for antibiotics. 3. The child will need to return to the hospital for a chest x-ray in 1 week. 4. The child will require a bronchoscopy for follow-up evaluation in 1 month. *Rationale:* Removal of foreign bodies from the respiratory tract may need to be performed by direct laryngoscopy or bronchoscopy. After the procedure the child should remain hospitalized for observation for laryngeal edema and respiratory distress. Cool mist is provided, and antibiotic therapy is prescribed if appropriate. Options 2, 3, and 4 are incorrect.

A nurse is developing a plan of care for a child who is at risk for seizures. Which interventions apply if the child has a seizure? *Select all that apply.*

*1. Time the seizure.* 2. Restrain the child. *3. Stay with the child.* 4. Place the child in a prone position. *5. Move furniture away from the child.* 6. Insert a padded tongue blade into the child's mouth. *Rationale:* During a seizure, the child is placed on his or her side in a lateral position. This type of positioning will prevent aspiration, because saliva will drain out of the corner of the child's mouth. The child is not restrained, because this could cause injury. The nurse would loosen clothing around the child's neck and ensure a patent airway. Nothing is placed into the child's mouth during a seizure, because this action may cause injury to the child's mouth, gums, or teeth. The nurse would stay with the child to reduce the risk of injury and allow for the observation and timing of the seizure.

An infant with congestive heart failure (CHF) is receiving diuretic therapy, and the nurse is closely monitoring the intake and output (I&O). Which is the best method for the nurse to use to monitor the urine output?

*1. Weighing the diapers* 2. Inserting a Foley catheter 3. Comparing intake with output 4. Measuring the amount of water added to formula *Rationale:* The best method to monitor urine output in an infant on diuretic therapy is to weigh the diapers. The weight of dry diapers is subtracted from the weight of wet diapers to determine the amount of urine excreted: 1 g is equivalent to 1 mL of urine. Comparing intake with output would not provide an accurate measure of urine output. Measuring the amount of water added to formula is unrelated to the amount of output. Although Foley catheter drainage is most accurate in determining output, it is not the best method and places the infant at risk for infection.

A client comes to the clinic indicating that a home pregnancy test was positive. The client's last menstrual period was September 7. Today is December 7. Which are true statements for this client? Select all that apply.

- According to Naegele's rule, the expected date of delivery is June 14 - Detection of the fetal heart rate via Doppler is possible - Urinary frequency is a common symptom

Exhibit A practical nurse is evaluating the external fetal monitoring strip of a laboring primigravida who is at 36 weeks gestation. Which nursing interventions should the practical nurse anticipate? Click on the exhibit button for additional information. Select all that apply.

- Administer supplemental oxygen by mask - Increase the IV fluid rate - Stop the client's oxytocin infusion

The nurse is caring for a full-term newborn following vaginal delivery. Which nursing interventions should be implemented? Select all that apply.

- Always wear gloves when handling the newborn before bathing - Cover the newborn to maintain a body temperature of 97.5-99 F (36.4-37.2 C) - Give a single dose of vitamin K intramuscularly - Suction the pharynx first, then the nasal passages

A client indicates to the nurse a desire to become pregnant. The client drinks 1-2 glasses of wine on weekends. BMI is 32 kg/m2. Which teachings should the nurse reinforce as part of proper preconception health care for this client? Select all that apply.

- Avoid eating undercooked hamburgers - Receive a rubella vaccine at least 3 months before attempting pregnancy - Take 0.4 mg folic acid supplementation daily

A pregnant client at 35 weeks gestation has brisk, painless vaginal bleeding. The health care provider suspects placenta previa. The nurse should prepare for which procedures? Select all that apply.

- Blood draw for hemoglobin - Electronic fetal monitoring - Pelvic ultrasound

A practical nurse (PN) is assisting the registered nurse in caring for a client on oxytocin to induce labor. Which assessments does the PN anticipate during the infusion? Select all that apply

- Blood pressure - Fetal heart rate tracings - Intake and output - Uterine contraction pattern - Vaginal examination

A client who reports sudden-onset severe right lower abdominal pain and dizziness is being evaluated for suspected ectopic pregnancy. Which assessment findings should the nurse anticipate? Select all that apply.

- Blood pressure 82/64 mm Hg - Pulse 120/min - Shoulder pain

The nurse is reinforcing information about the prevention of sudden infant death syndrome to a client with a newborn. Which recommendations should the nurse include? Select all that apply.

- Breastfeeding - Pacifier use at bedtime - Smoking cessation by parents

The nurse is reinforcing teaching to the mother of a newborn about gastroesophageal reflux. What does the nurse suggest to help prevent reflux? Select all that apply.

- Burp infant during and after feeds - Offer infant smaller but more frequent feeds - Place infant on tummy after feeding

An infant is born with a cleft palate. Which actions will promote oral intake until the defect can be repaired? Select all that apply.

- Burping the infant often - Feeding in an upright position - Using a specialty bottle or nipple

A 36-year-old multigravida is admitted with a diagnosis of severe preeclampsia. IV magnesium sulfate is prescribed. Which nursing measures does the practical nurse anticipate in this client's plan of care? Select all that apply

- Check deep tendon reflex frequently - Ensure calcium gluconate is available - Have suction equipment ready to use - Monitor for right upper quadrant pain

A pregnant client in the third trimester completes an intake form for a clinic visit. Which signs and symptoms are priority problems for the nurse to evaluate? Select all that apply.

- Frequent urination with dysuria and nocturia - Headache and blurred vision - Nonmalodorous, copious, clear vaginal discharge

A client at 21 weeks gestation has intense heartburn (pyrosis). What should the nurse recommend? Select all that apply.

- High-protein, low-fat diet - Six small meals a day

A nurse is reinforcing discharge instructions to the mother of a 2-year-old child who has had an orchiopexy to correct cryptorchidism. Which of the following statements, if made by the mother of the child, indicates that further teaching is necessary?

1. "I'll check his temperature." 2. "I'll give him medication so he'll be comfortable." *3. "I'll let him decide when to return to his play activities."* 4. "I'll check his voiding to be sure there are no problems." *Rationale:* All vigorous activities should be restricted for 2 weeks after surgery to promote healing and prevent injury. This will prevent dislodging of the suture, which is internal. Normally 2-year-old children will want to be very active. Therefore, allowing the child to decide when to return to his play activities may prevent healing and cause injury. The parents should be taught to monitor the child's temperature, provide analgesics, as needed, and monitor the urine output.

A nurse provides instructions to the mother of a child diagnosed with pediculosis (head lice). Permethrin (Nix) has been prescribed. Which statement by the mother regarding the use of the medication indicates a need for further instructions?

1. "After rinsing out the medication, I need to avoid washing my child's hair for 24 hours." 2. "I need to shampoo my child's hair, apply the medication, leave it on for 10 minutes, and then rinse it out." *3. "I need to shampoo my child's hair, apply the medication, and leave the medication on for 24 hours."* 4. "I need to purchase the medication from the pharmacy." *Rationale:* Permethrin is an over-the-counter antilice product that kills both lice and eggs with one application and has residual activity for 10 days. It is applied to the hair after shampooing and left for 10 minutes before rinsing out. The hair should not be shampooed for 24 hours after the rinsing treatment.

The mother of a child with juvenile idiopathic arthritis calls the nurse because the child is experiencing a painful exacerbation of the disease. The mother asks the nurse if the child should perform range-of-motion (ROM) exercises at this time. The nurse makes which response to the mother?

1. "Avoid all exercise during painful periods." 2. "The ROM exercises must be performed every day." *3. "Have the child perform simple isometric exercises during this time."* 4. "Administer additional pain medication before performing the ROM exercises." *Rationale:* During painful episodes, hot or cold packs, splinting, and positioning the affected joint in a neutral position help to reduce the pain. Although resting the extremity is appropriate, it is important to begin simple isometric or tensing exercises as soon as the child is able. These exercises do not involve joint movement.

A pediatric nursing instructor asks a nursing student to describe the cause of the clinical manifestations that occur in sickle cell disease. Which is the correct response by the nursing student?

1. "Bone marrow depression occurs because of the development of sickled cells." 2. "Sickled cells increase the blood flow through the body and cause a great deal of pain." 3. "The sickled cells mix with the unsickled cells and cause the immune system to become depressed." *4. "Sickled cells are unable to flow easily through the microvasculature, and their clumping obstructs blood flow."* *Rationale:* All the clinical manifestations of sickle cell disease are a result of the sickled cells being unable to flow easily through the microvasculature, and their clumping obstructs blood flow. With reoxygenation, most of the sickled red blood cells resume their normal shape. Options 1, 2, and 3 are inaccurate.

A nurse is providing home care instructions to the mother of a child diagnosed with pneumonia. Which statement by the mother indicates the need for further instructions?

1. "I can administer acetaminophen [Tylenol] for a fever." *2. "I can use a warm mist humidifier to keep the secretions loose."* 3. "I should administer the antibiotics until the prescribed amount is completed." 4. "I can give my child warm liquids to loosen secretions." *Rationale:* A cool mist humidifier rather than a warm mist should be used for the child with pneumonia. In addition, vaporizers that produce steam pose a danger of burns. Options 1, 3, and 4 are appropriate home care instructions regarding care of the child with pneumonia.

A nurse provides home care instructions to a client with sickle cell anemia. Which statement by the client indicates a need for further instruction?

1. "I'm going to take a painting class." 2. "I've learned to knit and sew my own clothes." *3. "When I'm feeling better, I'm returning to the soccer team."* 4. "I'm using a schedule to maintain my increased fluid intake." *Rationale:* Clients with sickle cell anemia are advised to avoid strenuous activities. Quiet activities as tolerated are recommended when the client is feeling well. Increasing fluid intake is encouraged to assist in preventing sickle cell crisis.

A nurse is providing dietary instructions to the mother of a child with celiac disease. Which statement by the mother indicates a need for further instructions?

1. "I can give my child rice." *2. "I am so pleased that I won't have to eliminate oatmeal from my child's diet."* 3. "My child loves corn. I will be sure to include corn in the diet." 4. "I will be sure to give my child vitamin supplements every day." *Rationale:* Dietary management is the mainstay of treatment for the child with celiac disease. All wheat, rye, barley, and oats should be eliminated from the diet and replaced with corn and rice. Vitamin supplements, especially fat-soluble vitamins and folate, may be needed in the early period of treatment to correct deficiencies.

When instructing the caregiver of a child about cast care, the nurse anticipates the need for further teaching when the caregiver states:

1. "I will encourage my child to avoid standing for too long." 2. "I will instruct my child to not put anything inside the cast." *3. "I will allow my child to put cotton balls inside the cast to relieve pressure."* 4. "I will encourage my child to keep the injured extremity elevated while resting." *Rationale:* Cast care includes keeping the casted extremity elevated on pillows or similar support for the first day, or as directed by the health professional; elevating a lower limb when sitting and avoid standing for too long; encouraging frequent rest for a few days; keeping the injured extremity elevated while resting; and not allowing the child to put anything inside the cast.

A nurse provides discharge instructions to the mother of a child following a myringotomy with insertion of tympanostomy tubes. Which statement by the mother indicates a need for further instructions?

1. "I will not allow my child to swim in lake water." 2. "I will not allow my child to swim in deep water." 3. "I will put earplugs in my child's ears during bathing." *4. "I need to be sure my child uses soft tissues to blow his nose."* *Rationale:* Parents need to be instructed that the child should not blow the nose for 7 to 10 days. Bath and lake water are potential sources of bacterial contamination. Diving and swimming deeply under water are prohibited. The child's ears need to be kept dry. Options 1, 2, and 3 are appropriate statements.

A nurse provides home care instructions to the parents of a child with congestive heart failure regarding the procedure for the administration of digoxin (Lanoxin). Which statement, if made by a parent, indicates the need for further instruction?

1. "I will not mix the medication with food." 2. "If more than one dose is missed, I will call the health care provider." 3. "I will take my child's pulse before administering the medication." *4. "If my child vomits after medication administration, I will repeat the dose."* *Rationale:* The parents need to be instructed that, if the child vomits after the digoxin is administered, they are not to repeat the dose. Options 1, 2, and 3 are accurate instructions regarding the administration of this medication. Additionally, the parents should be instructed that if a dose is missed and it is not noticed until 4 hours later, the dose should not be administered.

A nurse reinforces home-care instructions to the parents of a 3-year-old child who has been hospitalized with hemophilia. Which statement by a parent indicates the need for further instructions?

1. "I will supervise my child closely." 2. "I will pad the corners of the furniture." 3. "I will remove household items that can easily fall over." *4. "I will avoid immunizations and dental hygiene treatments for my child."* *Rationale:* The nurse needs to stress the importance of immunizations, dental hygiene, and routine well-child care. Options 1, 2, and 3 are appropriate statements. The parents are also provided instructions regarding measures to take in the event of blunt trauma (especially trauma that involves the joints), and they are instructed to apply prolonged pressure to superficial wounds until the bleeding has stopped.

A nurse has reinforced prior teaching of a school-age child who was given a brace to wear for the treatment of scoliosis. The nurse interprets that the client has not fully understood the information presented if the child makes which statement?

1. "I will wear my brace under my clothes." 2. "I will do back exercises at least five times a week." 3. "I will wear my brace whenever I am not sleeping." *4. "This brace will correct my curve."* *Rationale:* Bracing can halt the progression of most curvatures, although it is not curative for scoliosis. The statements in options 1, 2, and 3 represent correct understanding on the part of the child.

A 6-year-old child has just been diagnosed with localized Hodgkin's disease, and chemotherapy is planned to begin immediately. The mother of the child asks the nurse about radiation therapy because it was not prescribed as a part of treatment. The appropriate response to the mother is:

1. "I'm not sure. I'll discuss it with the health care provider." *2. "The child is too young to have radiation therapy."* 3. "It's very costly, and chemotherapy works just as well." 4. "The health care provider would prefer that you discuss treatment options with the oncologist." *Rationale:* Radiotherapy is usually delayed until a child is 8 years of age whenever possible to prevent retardation of bone growth and soft tissue development. Low doses of radiation may also be recommended. Options 1 and 4 are nontherapeutic and place the mother's inquiry on hold. Option 3 is a blunt and uncaring response.

A child is diagnosed with lactose intolerance. The child's mother asks the nurse about the disease. The appropriate nursing response is which of the following?

1. "It is the inability to fully digest the protein part of wheat, barley, rye, and oats." *2. "It is the inability to tolerate sugar found in dairy products."* 3. "It results from the absence of ganglion cells in the rectum." 4. "It results from increased bowel motility that leads to spasm and pain." *Rationale:* Lactose intolerance is the inability to tolerate lactose, the sugar found in dairy products. It results from absence or deficiency of lactase, an enzyme found in the secretions of the small intestine required for the digestion of lactose. Option 1 describes celiac disease. Option 3 describes Hirschsprung's disease. Option 4 describes irritable bowel syndrome.

A nurse determines that an adolescent client with diabetes mellitus needs further information about glycosylated hemoglobin levels and their purpose if the client made which statement when told that a level will be drawn?

1. "Most of my recent blood glucose levels were close to 170 mg/dL, so this result will probably be a little high." 2. "Last time this test was taken the result was 13. I hope it will be lower this time." *3. "I already had a complete blood cell [CBC] count drawn an hour ago, so this test is not necessary."* 4. "I have followed my diet these past 3 months, so hopefully the test result will be OK." *Rationale:* Glycosylated hemoglobin reflects the average blood glucose levels during the previous 3 to 4 months. It assesses glucose control in the client with diabetes mellitus. Glucose molecules attach to the hemoglobin A molecules found in red blood cells (RBCs) and remain there for the lifetime of the RBCs, approximately 120 days.

A nursing instructor asks a nursing student about phenylketonuria (PKU). Which statement, if made by the student, indicates an understanding of this disorder?

1. "PKU is an autosomal-dominant disorder." 2. "PKU primarily affects the gastrointestinal system." 3. "Treatment of PKU includes the dietary restriction of tyramine." *4. "All 50 states require routine screening of all newborns for PKU."* *Rationale:* PKU is an autosomal-recessive disorder. Treatment includes the dietary restriction of phenylalanine intake (not tyramine intake). PKU is a genetic disorder that results in central nervous system (CNS) damage from toxic levels of phenylalanine in the blood.

The mother of a child who had a myringotomy with insertion of tympanostomy tubes calls the nurse and tells the nurse that the "tubes" fell out. The nurse makes which response to the mother?

1. "Replace the tubes immediately so that the created opening does not close." 2. "This is an emergency and requires immediate intervention. Bring the child to the emergency department." *3. "This is not an emergency. I will speak to the health care provider and call you right back."* 4. "Soak the tubes in alcohol for 1 hour before replacing them in the child's ears." *Rationale:* The size and appearance of the tympanostomy tubes should be described to the parents following surgery. They should be reassured that if the tubes fall out, it is not an emergency but that the health care provider should be notified. Options 1, 2, and 4 are incorrect.

Several children have contracted measles (rubeola) in a local school, and the nurse provides information to the mothers of the children about this communicable disease. Which statement by a mother indicates a need for further information?

1. "Respiratory symptoms such as a profuse runny nose, cough, and fever occur before the development of a rash." 2. "Small blue-white spots with a red base may appear in the mouth." 3. "The rash usually begins behind the ears at the hairline." *4. "The infectious period ranges from 10 days before symptoms start to 15 days after the rash appears."* *Rationale:* The infectious period for rubeola ranges from 1 to 2 days before the onset of symptoms to 4 days after the rash appears. Options 1, 2, and 3 are accurate descriptions of rubeola. Option 4 describes the infectious period for rubella (German measles).

Several children have contracted rubeola (measles) in a local school and the school nurse conducts a teaching session for the parents of the school-children. Which statement, if made by a mother, indicates a need for further teaching regarding this communicable disease?

1. "Small blue-white spots with a red base may appear in the mouth." 2. "The rash usually begins centrally and spreads downward to the limbs." *3. "The communicable period ranges from 10 days before the onset of symptoms to 15 days after the rash appears."* 4. "Respiratory symptoms such as a very runny nose, cough, and fever occur before the development of a rash." *Rationale:* The communicable period for rubeola ranges from 4 days before to 5 days after the rash appears, mainly during the prodromal (catarrhal) stage. Options 1, 2, and 4 are accurate descriptions of rubeola. The small blue-white spots found in this communicable disease are called Koplik spots. Option 3 describes the incubation period for rubella, not rubeola.

A nursing student is caring for a child with increased intracranial pressure. On review of the chart, the student nurse notes that a transtentorial herniation has occurred. A nursing instructor asks the student about this type of herniation. Which statement by the student indicates a need for further research about this condition?

1. "The brain herniates downward and around the tentorium cerebelli." 2. "The herniation can be either unilateral or bilateral in nature." *3. "It involves only the anterior portions of the client's brain."* 4. "It can cause death if large amounts of tissue are involved." *Rationale:* Transtentorial herniation occurs when part of the brain herniates downward and around the tentorium cerebelli. It can be unilateral or bilateral and may involve anterior or posterior portions of the brain. If a large amount of tissue is involved, it can cause death because vital brain structures are compressed and become unable to perform their functions.

A 4-year-old child sustains a fall at home and is brought to the emergency department by the mother. After an x-ray, it is determined that the child has a fractured arm, and a plaster cast is applied. The nurse provides instructions to the mother regarding cast care for the child. Which statement by the mother indicates the need for further instructions?

1. "The cast may feel warm as it dries." *2. "I can use lotion or powder around the cast edges to relieve itching."* 3. "A small amount of white shoe polish can touch up a soiled white cast." 4. "If the cast becomes wet, a blow-dryer set on the cool setting may be used to dry it." *Rationale:* The mother needs to be instructed to not use lotion or powders on the skin around the cast edges or inside the cast, because they can become sticky or caked and cause skin irritation. Options 1, 3, and 4 are appropriate instructions.

A child is brought to the emergency department, and a fracture of the left lower arm is suspected. The mother states that the child was rollerblading and attempted to break a fall with an outstretched arm. The child receives diagnostic x-rays, from which it has been determined that a fracture is present. A plaster of Paris cast is applied to the arm, and the nurse provides instructions to the mother regarding cast care at home. Which of the following instructions would the nurse provide to the mother?

1. "The cast should be dry in about 6 hours." 2. "The cast is water resistant, so the child is able to take a bath or a shower." 3. "The cast will not mold to the body and should heal the fracture in no time at all." *4. "The cast needs to be kept dry because, when wet, it will begin to disintegrate."* *Rationale:* Plaster of Paris is a heavier material than that used in a synthetic cast. It molds easily to the extremity and is less expensive than a synthetic cast. It takes about 24 hours to dry, and drying time could be longer depending on the size of the cast. Plaster of Paris is not water resistant, and when wet, it will begin to disintegrate.

A nursing student is asked to discuss juvenile idiopathic arthritis (JIA) at a clinical conference scheduled for the end of the clinical day. Which statement by the nursing student indicates the need to further research this disorder?

1. "The cause of this disease is unknown." 2. "JIA most often occurs by age of 10 years." *3. "This disease is twice as likely to occur in boys rather than girls."* 4. "Clinical manifestations include morning stiffness and painful, stiff, swollen joints." *Rationale:* JIA is twice as likely to occur in girls as in boys. The cause of JIA is unknown. JIA has two peak ages of onset: between 1 and 3 years of age and between 8 and 10 years of age. This autoimmune inflammatory disease causes painful inflammation of joints.

A hospitalized 2-year-old child with croup is receiving corticosteroid therapy. The mother asks the nurse why the health care provider did not prescribe antibiotics. The nurse makes which response to the mother?

1. "The child may be allergic to antibiotics." 2. "The child is too young to receive antibiotics." *3. "Antibiotics are not indicated unless a bacterial infection is present."* 4. "The child still has the maternal antibodies from birth and does not need antibiotics." *Rationale:* Antibiotics are not indicated in the treatment of croup unless a bacterial infection is present. Options 1, 2, and 4 are incorrect. In addition, the question does not include any supporting data to indicate that the child may be allergic to antibiotics.

A child is scheduled for a tonsillectomy in the day-stay surgical unit. On the day following surgery, the mother calls the surgical unit and expresses concern because the child has a very bad mouth odor. The nurse makes which response to the mother?

1. "The child probably has an infection." 2. "You need to contact the health care provider immediately." *3. "Bad mouth odor is normal and may be relieved by drinking more liquids."* 4. "Have the child gargle with mouthwash." *Rationale:* Bad mouth odor is normal following tonsillectomy and may be relieved by drinking more liquids. Options 1, 2, and 4 are incorrect. Additionally, mouthwash gargles will irritate the throat.

A nursing instructor asks a student nurse to describe osteogenic sarcoma. Which statement by the student indicates the need to further research the disease?

1. "The femur is the most common site of this sarcoma." *2. "The child does not experience pain at the primary tumor site."* 3. "If a weight-bearing limb is affected, then limping is a clinical manifestation." 4. "The symptoms of the disease during the early stage are almost always attributed to normal growing pains." *Rationale:* Osteogenic sarcoma is the most common bone tumor in children. A clinical manifestation of osteogenic sarcoma is progressive, insidious, intermittent pain at the tumor site. By the time these children receive medical attention, they may be in considerable pain from the tumor. Options 1, 3, and 4 are accurate regarding osteogenic sarcoma.

A nurse provides instructions regarding the use of permethrin 1% (Nix) to the parents of a child who has been diagnosed with pediculosis capitis (head lice). Which statement by a parent indicates the need for further instruction?

1. "The hair should not be shampooed for 24 hours after treatment." 2. "The medication can be obtained over the counter in a local pharmacy." *3. "The medication is applied to the hair after shampooing and left on for 24 hours."* 4. "The medication is applied to the hair after shampooing, left on for 5 to 10 minutes, and then rinsed out." *Rationale:* Permethrin 1% is an over-the-counter anti-lice product that kills lice and eggs with one application and that has residual activity for 10 days. It is applied to dried hair after shampooing and left for 5 to 10 minutes before it is rinsed (not shampooed) out. The hair should not be shampooed for 24 hours after the treatment.

A nurse is providing instructions to a mother of a child with cystic fibrosis regarding the immunization schedule for the child. Which statement should the nurse make to the mother?

1. "The immunization schedule will need to be altered." 2. "The child should not receive any hepatitis vaccines." 3. "The child will receive all of the immunizations except for the polio series." *4. "The child will receive the recommended basic series of immunizations along with a yearly influenza vaccination."* *Rationale:* It is essential that children with cystic fibrosis be adequately protected from communicable diseases by immunization. It is recommended that in addition to the basic series of immunizations, children with cystic fibrosis also should receive yearly influenza vaccines.

A nurse is teaching cardiopulmonary resuscitation to a group of nursing students. The nurse asks a student to describe the reason why blind finger sweeps are avoided in infants. The nurse determines that the student understands the reason if the student makes which statement?

1. "The object may have been swallowed." 2. "The infant may bite down on the finger" 3. "The mouth is too small to see the object." *4. "The object may be forced back further into the throat."* *Rationale:* Blind finger sweeps are not recommended for infants and children because of the risk of forcing the object further down into the airway. Options 1, 2, and 3 are not related directly to the subject of the question.

A nurse provides information to the mother of a 2-week-old infant who was diagnosed with clubfoot at the time of birth. Which statement by the mother indicates the need for further instruction regarding this disorder?

1. "Treatment needs to be started as soon as possible." 2. "I realize my child will require follow-up care until full grown." *3. "I need to bring my child back to the clinic in 1 month for a new cast."* 4. "I need to come to the clinic every week with my child for the casting." *Rationale:* The treatment for clubfoot is started as soon as possible after birth. Serial manipulation and casting are performed at least weekly. If sufficient correction is not achieved within 3 to 6 months, surgery is usually indicated. Because clubfoot can recur, all children with the condition require long-term interval follow-up until they reach skeletal maturity to ensure an optimal outcome.

A nurse reinforces home care instructions to the parents of a child hospitalized with pertussis. The child is in the convalescent stage and is being prepared for discharge. Which statement by the parents indicates a need for further instructions?

1. "We need to encourage adequate fluid intake." 2. "Coughing spells may be triggered by dust or smoke." *3. "We need to maintain respiratory precautions and a quiet environment for at least 2 weeks."* 4. "Good hand-washing techniques need to be instituted to prevent spreading the disease to others." *Rationale:* Pertussis is transmitted by direct contact or respiratory droplets from coughing. The communicable period occurs primarily during the catarrhal stage. Respiratory precautions are not required during the convalescent phase. Options 1, 2, and 4 are components of home care instructions.

A nurse is reinforcing instructions to the parents of a child with a hernia regarding measures that will promote reducing the hernia. The nurse determines that the parents understand these measures if they state which of the following?

1. "We will be sure to give our child a Fleet enema every day to prevent constipation." 2. "We will make sure that our child participates in physical activity every day." *3. "We will provide comfort measures to reduce any crying periods by our child."* 4. "We will encourage our child to cough every few hours on a daily basis." *Rationale:* A warm bath and comfort measures to reduce crying periods are all simple measures to promote reducing a hernia. Coughing and crying increase the strain on the hernia. Likewise, physical activities and enemas of any type would increase the strain on the hernia.

A patient in labor is 8 cm dilated. The fetus is vertex and at the ischial spines. Which is the station of the fetus? 1. 0 station 2. +1 station 3. -1 station 4. +2 station

1. 0 station Rationale: Station refers to the level of the presenting part in relation to the pelvic inlet and the ischial spines. 0 station indicates that the presenting part is at the level of the ischial spines. Other stations are defined by their distance in centimeters above or below the ischial spines up to 5+ at the introitus. Negative is above the spines. Positive is below the spines.

A mother of a 5-year-old child brings the child to the emergency department and tells the nurse that the child fell. A fracture is suspected and an x-ray is taken. The results indicate that the child has a comminuted fracture of the right humerus. The mother asks the nurse to describe this type of fracture, and the nurse draws a picture for the mother. Which picture identifies this type of a fracture? Refer to figure.

1. 1 *2. 2* 3. 3 4. 4 *Rationale:* When small fragments of bone are broken from the fracture shaft and lie in the surrounding tissues, the fracture is called comminuted. An open or compound fracture (option 1) is a fracture with an open wound from which the bone is or has protruded. In an oblique fracture (option 3), a diagonal line across the bone is noted. In a greenstick fracture (option 4), the bone is partially bent and partially broken.

An infant returns to the nursing unit after the surgical repair of a cleft lip located on the right side of the lip. The best position in which to place this infant at this time is:

1. A flat position 2. A prone position *3. On his or her left side* 4. On his or her right side *Rationale:* After the repair of a cleft lip, the infant should be positioned on the side opposite to the repair to prevent contact of the suture lines with the bed linens. In this case, it is best to place the infant on his or her left side. Additionally, the flat or prone position can result in aspiration if the infant vomits.

A preliminary diagnosis is made for a child with acute lymphoblastic leukemia (ALL). In reviewing the complete blood cell count (CBC) of the child, the nurse would expect to find:

1. A hematocrit (Hct) count of 36 cells in 1 mL of peripheral blood 2. A hemoglobin (Hgb) count of 12 cells in 1 mL of peripheral blood *3. An erythrocyte (red blood cell [RBC]) count of 2 cells in 1 mL of peripheral blood* 4. A white blood cell (WBC) count of 15,000 cells in 1 mL of peripheral blood *Rationale:* ALL is diagnosed based on the history and the signs and symptoms a child presents during a health care provider's visit. The diagnosis also can be made during a routine physical exam or unrelated injury in which the child is brought to a medical clinic. A CBC is done initially to assist with a diagnosis. The CBC would indicate leukopenia (lower than normal WBCs), anemia, and thrombocytopenia. The values in options 1 and 2 are normal results for this age group and do not indicate anemia. Option 4 does not indicate leukopenia. Option 3 is the only laboratory value that is a lower result than a normal RBC count for this age group and is an indicator of anemia or possible leukemia.

The parents of a newborn have been told that their child was born with bladder exstrophy, and the parents ask the nurse about this condition. The nurse bases the response on knowledge that this condition is:

1. A hereditary disorder that occurs in every other generation 2. Caused by the use of medications taken by the mother during pregnancy 3. A condition in which the urinary bladder is abnormally located in the pelvic cavity *4. An extrusion of the urinary bladder to the outside of the body through a defect in the lower abdominal wall* *Rationale:* Bladder exstrophy is a congenital anomaly that is characterized by the extrusion of the urinary bladder to the outside of the body through a defect in the lower abdominal wall. The cause is unknown, and there is a higher incidence among males. Options 1, 2, and 3 are not characteristics of this disorder.

A 4-year-old child is admitted to the hospital with suspected acute lymphocytic leukemia (ALL). The nurse understands that which diagnostic study will confirm this diagnosis?

1. A platelet count 2. A lumbar puncture *3. Bone marrow biopsy* 4. White blood cell (WBC) count *Rationale:* The confirmatory test for leukemia is microscopic examination of bone marrow obtained by bone marrow aspirate and biopsy. The WBC count may be high or low in leukemia. A lumbar puncture may be done to look for blast cells in the spinal fluid that are indicative of central nervous system disease. An altered platelet count occurs as a result of chemotherapy.

A nurse assigned to care for a child with mumps is monitoring the child for the signs and symptoms associated with the common complication of mumps. The nurse monitors for which of the following that is indicative of this common complication?

1. A red, swollen testicle *2. Nuchal rigidity* 3. Pain 4. Deafness *Rationale:* The most common complication of mumps is aseptic meningitis, with the virus being identified in the cerebrospinal fluid. Common signs include nuchal rigidity, lethargy, and vomiting. A red, swollen testicle may be indicative of orchitis. Although this complication appears to cause most concern among parents, it is not the most common complication. Although mumps is one of the leading causes of unilateral nerve deafness, it does not occur frequently. Muscular pain, parotid pain, or testicular pain may occur, but pain does not indicate a sign of a common complication.

A nurse is assigned to care for an infant with a diagnosis of tricuspid atresia. The nurse plans care, knowing that in this disorder:

1. A single vessel overrides both ventricles. 2. Frequent episodes of hypercyanotic spells occur. *3. There is no communication from the right atrium to the right ventricle.* 4. There is no communication from the systemic and pulmonary circulations. *Rationale:* In tricuspid atresia, there is no communication from the right atrium to the right ventricle. Option 1 describes truncus arteriosus. Option 4 describes transposition of the great arteries. Frequent episodes of hypercyanotic spells occur in tetralogy of Fallot.

A nurse is assisting in preparing to care for a child with a brain tumor who will be returning from the recovery room following debulking of the tumor. Which of the following items will the nurse place at the bedside in preparation for the child's return from surgery?

1. A suction machine *2. A cooling blanket* 3. Protective isolation equipment 4. Skeletal traction equipment *Rationale:* Special attention is paid to the child's temperature postoperatively, which may be elevated because of hypothalamus or brainstem involvement during surgery. A cooling blanket should either be in place on the bed or readily available if the child becomes hyperthermic. Suctioning is avoided because it can cause increased intracranial pressure. Protective isolation is unnecessary, and there is no need for skeletal traction equipment.

A nursing instructor is observing a nursing student caring for an infant with a diagnosis of bladder exstrophy. The nursing student provides appropriate care to the infant by:

1. Covering the bladder with a dry sterile dressing 2. Covering the bladder with a wet-to-dry dressing 3. Applying sterile water soaks to the bladder mucosa *4. Covering the bladder with a nonadhering plastic wrap* *Rationale:* Care should be taken to protect the exposed bladder tissue from drying while allowing drainage of urine. This is best accomplished by covering the bladder with a nonadhering plastic wrap. The use of wet-to-dry dressings should be avoided because this type of dressing adheres to the mucosa and may damage the delicate tissue when removed. Sterile dressings and dressings soaked in solutions can also dry out and damage the mucosa when removed.

A nurse assists with admitting a child with a diagnosis of acute-stage Kawasaki disease. When obtaining the child's medical history, which clinical manifestation is likely to be reported?

1. Cracked lips 2. A normal appearance *3. Conjunctival hyperemia* 4. Desquamation of the skin *Rationale:* During the acute stage of Kawasaki disease, the child presents with fever, conjunctival hyperemia, a red throat, swollen hands, a rash, and enlargement of the cervical lymph nodes. During the subacute stage, cracking lips and fissures, desquamation of the skin on the tips of the fingers and toes, joint pain, cardiac manifestations, and thrombocytosis occur. During the convalescent stage, the child appears normal, but signs of inflammation may be present.

A nurse is reviewing the laboratory results of a child scheduled for tonsillectomy. Which laboratory value would be significant to review?

1. Creatinine 2. Urinalysis *3. Platelet count* 4. Blood urea nitrogen (BUN) *Rationale:* Before the surgical procedure, the child is assessed for signs of active infection and for redness and exudate of the throat. Because the tonsillar area is so vascular, postoperative bleeding is a concern. The prothrombin (PT), partial thromboplastin time (PTT), platelet count, hemoglobin and hematocrit (H&H), white blood cell (WBC) count, and urinalysis are performed preoperatively. The platelet count result would identify a potential for bleeding. The BUN and creatinine would not determine the potential for bleeding but rather evaluate renal function.

A nurse is assigned to care for an infant with cryptorchidism. The nurse anticipates that diagnostic studies will be prescribed to evaluate:

1. DNA synthesis 2. Babinski reflex *3. Kidney function* 4. Chromosomal analysis *Rationale:* Cryptorchidism may be the result of hormone deficiency, intrinsic abnormality of a testis, or a structural problem. Diagnostic tests would assess kidney function, because the kidneys and testes arise from the same germ tissue. Babinski's reflex tests neurological function and is unrelated to this diagnosis. DNA synthesis and a chromosomal analysis are also unrelated to this diagnosis.

A nurse collecting data on a child suspects physical abuse. The nurse understands that which of the following is a primary and legal nursing responsibility?

1. Document the child's physical assessment findings accurately and thoroughly. *2. Report the case in which the abuse is suspected.* 3. Refer the family to the appropriate support groups. 4. Assist the family in identifying resources and support systems. *Rationale:* The primary legal nursing responsibility when child abuse is suspected is to report the case. All 50 states require health care professionals to report all cases of suspected abuse. Although documentation of findings, assisting the family, and referring the family to appropriate resources and support groups are important, the primary legal responsibility is to report the case.

A child is seen in the clinic, and the primary health care provider documents a diagnosis of primary nocturnal enuresis. The mother asks the nurse about the diagnosis. The nurse bases the response on the fact that primary nocturnal enuresis:

1. Does not respond to treatment 2. Is caused by a psychiatric problem 3. Requires surgical intervention to improve the problem *4. Is common and most children will outgrow bed-wetting without therapeutic intervention* *Rationale:* Primary nocturnal enuresis is bedwetting and is described as occurring in a child that has never been dry at night for extended periods. It is common in children, most of whom will outgrow bedwetting without therapeutic intervention. The child is not able to sense a full bladder and does not awaken to void. The child may have delayed maturation of the central nervous system (CNS). It is not caused by a psychiatric problem. Behavioral conditioning with use of alarms has been used for treatment in the older child with nocturnal enuresis. A device that contains a moisture-sensitive alarm is worn on the child's pajamas. As the child starts to void, the alarm goes off, awakening the child. The alarm system may need to be used consistently over 15 weeks for resolution.

A nurse is monitoring for fluid volume deficit in an infant who is vomiting and having diarrhea. The nurse weighs the infant's diaper after each voiding and stool and carefully calculates fluid volume, knowing that:

1. Each gram of diaper weight is equivalent to 0.5 mL of urine. *2. Each gram of diaper weight is equivalent to 1 mL of urine.* 3. Each gram of diaper weight is equivalent to 2 mL of urine. 4. Each gram of diaper weight is equivalent to 2.5 mL of urine. *Rationale:* When monitoring for fluid volume deficit, the nurse should weigh the infant's diaper after each voiding and stool. Each gram of diaper weight is equivalent to 1 mL of urine. Therefore options 1, 3, and 4 are incorrect.

A nurse is performing a neurovascular check on a child with a cast applied to the lower leg. The child complains of tingling in the toes distal to the fracture site. Which action should be taken by the nurse?

1. Elevate the extremity. 2. Document the findings. *3. Notify the health care provider (HCP).* 4. Ambulate the child with crutches. *Rationale:* Reduced sensation to touch or complaints of numbness or tingling at a site distal to the fracture may indicate poor tissue perfusion. This finding should be reported to the registered nurse or HCP. Options 1, 2, and 4 are inappropriate and would delay the required and immediate interventions.

A nurse is assisting in preparing a plan of care for a child who will be returning from surgery following the application of a hip spica cast. Which of the following would be the priority in the plan of care for this child on return from the procedure?

1. Elevate the head of the bed. 2. Turn the child onto the right side. *3. Check circulation in the feet.* 4. Abduct the hips using pillows. *Rationale:* During the first few hours after a cast is applied, the primary concern is swelling that may cause the cast to produce a tourniquet-like effect and restrict circulation. Therefore circulatory assessment is a priority. Elevating the head of the bed of a child in a hip spica would cause discomfort. Using pillows to abduct the hips is not necessary because a hip spica immobilizes the hip and the knee. Turning the child side to side at least every 2 hours is important because it allows the body cast to dry evenly and prevents complications related to immobility; however, it is not as important as checking circulation.

A nurse is administering medications to a 6-year-old child with nephrotic syndrome. To reduce proteinuria, the nurse should expect that which medication would be prescribed?

1. Enalapril (Vasotec) *2. Furosemide (Lasix)* 3. Prednisone 4. Cyclophosphamide *Rationale:* The child is usually placed on diuretic therapy with furosemide (Lasix) until protein loss is controlled. Enalapril is most commonly used to control hypertension. Corticosteroids, such as prednisone, may be prescribed to decrease inflammation. Corticosteroids also suppress the autoimmune response and stimulate vascular reabsorption of edema. Cyclophosphamide is an alkylating agent and may be used in maintaining remission.

A nurse who is caring for a child with aplastic anemia reviews the laboratory results and notes a white blood cell (WBC) count of 6000 cells/ mm3 and a platelet count of 27,000 cells/mm3. Which nursing intervention should be incorporated into the plan of care?

1. Encourage naps. 2. Encourage a diet high in iron. *3. Encourage quiet play activities.* 4. Maintain strict isolation precautions. *Rationale:* Precautionary measures to prevent bleeding should be taken when a child has a low platelet count. These include no injections, no rectal temperatures, the use of a soft toothbrush, and abstinence from contact sports or activities that could cause an injury. Strict isolation would be required if the WBC count was low. Naps and a diet high in iron are unrelated to the risk of bleeding.

A child with rubeola (measles) is being admitted to the hospital. When preparing for the admission of the child, which precautions should be implemented?

1. Enteric 2. Contact 3. Protective *4. Respiratory* *Rationale:* Rubeola is transmitted via airborne particles or direct contact with infectious droplets. Respiratory precautions are required, and a mask is worn by those who come in contact with the child. Gowns and gloves are not indicated. Articles that are contaminated should be bagged and labeled. Options 1, 2, and 3 are not indicated for rubeola.

A nurse is reviewing the health record of an infant with a diagnosis of gastroesophageal reflux. Which clinical manifestation of this disorder would the nurse expect to note documented in the record?

1. Excessive oral secretions 2. Bowel sounds heard over the chest *3. Hiccupping and spitting up after a meal* 4. Coughing, wheezing, and short periods of apnea *Rationale:* Clinical manifestations of all types of gastroesophageal reflux include vomiting (spitting up) after a meal, hiccupping, and recurrent otitis media related to pooled secretions in the nasopharynx during sleep. Option 1 is a clinical manifestation of esophageal atresia and tracheoesophageal fistula. Option 2 is a clinical manifestation of congenital diaphragmatic hernia. Option 4 is a clinical manifestation of hiatal hernia.

A nurse is assisting in developing a plan of care for a child admitted with a diagnosis of Kawasaki disease. In developing the initial plan of care, the nurse suggests that the child should be monitored for signs of:

1. Failure to thrive 2. Bleeding *3. Congestive heart failure (CHF)* 4. Decreased tolerance to stimulation *Rationale:* Nursing care for Kawasaki disease initially centers around observing for signs of CHF. The nurse monitors for increased respiratory rate, increased heart rate, dyspnea, lung congestion, and abdominal distention. Options 1, 2, and 4 are not findings directly associated with this disorder.

A nurse reinforces instructions to the mother about dietary measures for a 5-year-old child with lactose intolerance. The nurse tells the mother that which of the following supplements will be required as a result of the need to avoid lactose in the diet?

1. Fats 2. Zinc *3. Calcium* 4. Thiamine *Rationale:* Lactose intolerance is the inability to tolerate lactose, which is the sugar that is found in dairy products. Removing milk from the diet can provide relief from symptoms. Additional dietary changes may be required to provide adequate sources of calcium and, if the child is an infant, protein and calories.

A child with a diagnosis of a hernia has been scheduled for a surgical repair in 2 weeks. The nurse reinforces instructions to the parents about the signs of possible hernial strangulation. The nurse tells the parents that which of the following signs would require health care provider (HCP) notification by the parents?

1. Fever 2. Diarrhea *3. Vomiting* 4. Constipation *Rationale:* The parents of a child with a hernia need to be instructed about the signs of strangulation. These signs include vomiting, pain, and an irreducible mass. The parents should be instructed to contact the HCP immediately if strangulation is suspected. Fever, diarrhea, and constipation are not associated with strangulation of a hernia.

A nurse is reviewing the health record of a 14-year-old child who is suspected of having Hodgkin's disease. Which of the following is the primary characteristic of this disease?

1. Fever and malaise 2. Anorexia and weight loss 3. Painful, enlarged inguinal lymph nodes *4. Painless, firm, and movable lymph nodes in the cervical area* *Rationale:* Clinical manifestations specifically associated with Hodgkin's disease include painless, firm, and movable adenopathy in the cervical and supraclavicular areas. Hepatosplenomegaly is also noted. Although anorexia, weight loss, fever, and malaise are associated with Hodgkin's disease, these manifestations are not the primary characteristics and are seen with many disorders.

A mother of a 6-year-old-child calls a nurse who lives in the neighborhood and tells the nurse that her child accidentally rubbed waterproof sunscreen in his eyes. The nurse should tell the mother to immediately:

1. Flush the child's eyes for 15 minutes with water. 2. Have the child wipe the eyes with a wet towel. 3. Tell the child to blink continuously to get the sunscreen out of the eye. *4. Call the poison control center.* *Rationale:* Waterproof sunscreen should never be placed near the eyes. Waterproof sunscreen causes severe pain and a chemical burn that can damage the child's vision. Flushing the eyes with water does not stop the burning. The mother should be instructed to call the poison control center and to take the child to the emergency department. Special chemicals will be needed to flush the sunscreen out of the eyes and preserve vision. Wiping the eyes will increase the pain and burning. Blinking will not alleviate the pain or remove the sunscreen from the eyes.

A nurse is assigned to care for a 2-year-old child who has been admitted to the hospital for surgical correction of cryptorchidism. The highest priority in the postoperative plan of care for this child is to:

1. Force oral fluids. *2. Prevent tension on the suture.* 3. Test the urine for glucose. 4. Encourage coughing. *Rationale:* When a child returns from surgery, the testicle is held in position by an internal suture that passes through the testes and scrotum and is attached to the thigh. It is important not to dislodge this suture. Depending on the type of anesthesia used, option 4 may be appropriate but is not the priority. Although adequate hydration is important to maintain, fluids should not be forced. Testing urine for glucose is not related to this type of surgery.

A nurse is reviewing the health record of a child with a diagnosis of celiac disease. Which clinical manifestation should the nurse expect to note documented in the health record?

1. Frothy diarrhea *2. Profuse watery diarrhea and vomiting* 3. Foul-smelling ribbon stools 4. Diffuse abdominal pain unrelated to meals or activity *Rationale:* Celiac disease causes profuse watery diarrhea and vomiting. Option 1 is a clinical manifestation of lactose intolerance. Option 3 is a clinical manifestation of Hirschsprung's disease. Option 4 is a clinical manifestation of irritable bowel syndrome.

A mother of a child who underwent a myringotomy with insertion of tympanostomy tubes calls the nurse and reports that the child is complaining of discomfort. The nurse tells the mother to:

1. Give the child children's aspirin for the discomfort. 2. Be sure that the child is resuming normal activities. *3. Give the child acetaminophen (Tylenol) for the discomfort.* 4. Speak to the health care provider because the child should not be having any discomfort. *Rationale:* Following myringotomy with insertion of tympanostomy tubes, the child may experience some discomfort. Acetaminophen can be given to relieve the discomfort. Aspirin should not be administered to the child. The child should rest if discomfort is present.

A nurse collects a urine specimen preoperatively from a child with epispadias who is scheduled for surgical repair. The nurse reviews the child's record for the laboratory results of the urine test and would most likely expect to note which of the following?

1. Hematuria *2. Bacteriuria* 3. Glucosuria 4. Proteinuria *Rationale:* Epispadias is a congenital defect that involves the abnormal placement of the urethral orifice of the penis. In clients with this condition, the urethral opening is located anywhere on the dorsum of the penis. This anatomical characteristic leads to the easy access of bacterial entry into the urine. Options 1, 3, and 4 are not characteristically noted with this condition.

A mother is 11 weeks pregnant and presents for her first prenatal care visit. Which test is the most appropriate? 1. Hemoglobin A1c 2. Triple screen 3. Coombs Test 4. Amniocentesis

1. Hemoglobin A1c Hemoglobin A1C test screens for gestational diabetes. This test is recommended for all pregnant women at the first prenatal test. Triple screen is a blood study done between 15 & 20 weeks gestation that detects chromosomal abnormalities. The indirect Coombs test screens maternal blood for anti-RBC antibodies. Amniocentesis is needle aspiration of fluid from the sac to detect fetal abnormalities.

A nursing instructor asks a nursing student about the cause of hemophilia. The student correctly responds by telling the instructor that:

1. Hemophilia is a Y-linked hereditary disorder. 2. A splenectomy resolves the bleeding disorders. *3. Hemophilia A results from deficiency of factor VIII.* 4. A bone marrow transplant is the treatment of choice. *Rationale:* The term "hemophilia" refers to a group of bleeding disorders. The identification of the specific factor deficiencies allows for definitive treatment with replacement agents. Hemophilia A results from a deficiency of factor VIII. Hemophilia B (Christmas disease) is a deficiency of factor IX. Hemophilia is inherited in a recessive manner via a genetic defect on the X chromosome, not the Y chromosome. Neither a bone marrow transplant nor splenectomy is used to treat this disorder.

A nurse assists in preparing a plan of care for the infant with bladder exstrophy. The nurse identifies which of the following immediate problems as the priority for the infant?

1. Infection 2. Elimination *3. Skin disruption* 4. Lack of parental understanding *Rationale:* In bladder exstrophy, the bladder is exposed and external to the body. The highest priority is skin disruption related to the exposed bladder mucosa. Although the infant needs to be monitored for elimination patterns and kidney function, this is not the priority concern for this condition. Lack of parenteral understanding related to the diagnosis and treatment of the condition will need to be addressed, but again is not the priority. Although infection related to the anatomically located defect can be a problem, it is not the immediate one.

A male child who had surgery to correct hypospadias is seen in a health care provider's office for a well-baby check-up. The nurse provides instructions to the mother, knowing that which long-term complication is associated with hypospadias?

1. Infertility *2. Renal anomalies* 3. Erectile dysfunction 4. Decreased urinary output *Rationale:* The nurse should ask the child's parents about the child's kidney function because hypospadias may be associated with renal anomalies. The incorrect options are not associated with a long-term effect of hypospadias.

A nurse is caring for a hospitalized infant with bronchiolitis. Diagnostic tests have confirmed respiratory syncytial virus (RSV). On the basis of this finding, which of the following would be the appropriate nursing action?

1. Initiate strict enteric precautions. 2. Wear a mask when caring for the child. *3. Plan to move the infant to a room with another child with RSV.* 4. Leave the infant in the present room, because RSV is not contagious. *Rationale:* RSV is a highly communicable disorder, but it is not transmitted via the airborne route. It is usually transferred by the hands, and meticulous handwashing is necessary to decrease the spread of organisms. The infant with RSV is isolated in a single room or placed in a room with another child with RSV. Enteric precautions are not necessary; however, the nurse should wear a gown when the soiling of clothing may occur.

A nurse who is working in the emergency department is caring for a child who has been diagnosed with epiglottitis. Indications that the child may be experiencing airway obstruction include which of the following?

1. Nasal flaring and bradycardia *2. The child thrusts the chin forward and opens the mouth* 3. A low-grade fever and complaints of a sore throat 4. The child leans backward, supporting himself or herself with the hands and arms *Rationale:* Clinical manifestations that are suggestive of airway obstruction include tripod positioning (leaning forward supported by the hands and arms with the chin thrust out and the mouth open), nasal flaring, tachycardia, a high fever, and a sore throat.

A mother arrives at the emergency department with her 5-year-old child and states that the child fell off a bunk bed. A head injury is suspected, and a nurse is monitoring the child continuously for signs of increased intracranial pressure (ICP). Which of the following is a late sign of increased ICP in this child?

1. Nausea *2. Bradycardia* 3. Bulging fontanel 4. Dilated scalp veins *Rationale:* Late signs of increased ICP include a significant decrease in the level of consciousness, bradycardia, and fixed and dilated pupils. Nausea is an early sign of increased ICP. A bulging fontanel and dilated scalp veins are early signs of increased ICP and would be noted in an infant rather than in a 5-year-old child.

A nurse is reinforcing information to parents regarding the signs of meningitis. The nurse informs the parents that the primary signs of meningitis include:

1. Nausea and delirium 2. Anorexia and back pain 3. Night blindness and confusion *4. Severe headache and neck stiffness* *Rationale:* The primary signs of meningitis include severe headache, fever, stiff neck, and a change in the level of consciousness. Although nausea, confusion, delirium, and back pain may occur in meningitis, these are not the classic signs. Night blindness is not related to meningitis.

A 3-year-old child is seen in the health care clinic, and a diagnosis of encopresis is made. The nurse reviews the record, expecting to note which of the following that is a sign of this disorder?

1. Nausea and vomiting 2. Diarrhea *3. Evidence of soiled clothing* 4. Malaise anorexia *Rationale:* Encopresis is defined as fecal incontinence and is a major concern if the child is constipated. Signs include evidence of soiling clothing, scratching, or rubbing the anal area because of irritation, fecal odor without apparent awareness by the child, and social withdrawal.

Which of the following represents a primary characteristic of autism?

1. Normal social play 2. Consistent imitation of others' actions *3. Lack of social interaction and awareness* 4. Normal verbal and nonverbal communication *Rationale:* Autism is a severe form of an autism spectrum disorder. A primary characteristic is a lack of social interaction and awareness. Social behaviors in autism include a lack of or an abnormal imitation of others' actions and a lack of or abnormal social play. Additional characteristics include a lack of or impaired verbal communication and markedly abnormal nonverbal communication.

A nurse is assigned to assist in caring for a newborn with a colostomy that was created during surgical intervention for imperforate anus. When the newborn returns from surgery, the nurse checks the stoma and notes that it is red and edematous. Which of the following is the appropriate nursing intervention?

1. Notify the registered nurse immediately. *2. Document the findings.* 3. Apply ice immediately. 4. Elevate the buttocks. *Rationale:* A fresh colostomy stoma will be red and edematous but this will decrease with time. The colostomy site will then be pink without evidence of abnormal drainage, swelling, or skin breakdown. The nurse would document these findings because this is a normal expectation. Options 1, 3, and 4 are inappropriate interventions.

A nurse is assigned to care for a child with a spica cast. Which action should be avoided when caring for the child?

1. Observing for nonverbal signs of pain *2. Using pillows to elevate the head and shoulders* 3. Checking neurovascular status of the extremities 4. Placing the child on a stretcher and bringing the child to the playroom *Rationale:* Pillows should not be used to elevate the head or shoulders of a child in a body cast because the pillows will thrust the child's chest against the cast and cause discomfort and respiratory difficulty. Neurovascular checks are a critical component of care to ensure that the cast is not causing circulatory compromise. The nurse should observe for nonverbal signs of pain and ask the older child if pain is experienced. A ride on a stretcher to the playroom or around the hospital provides changes of position and scenery.

A nurse determines that a child with type 1 diabetes mellitus is having a hypoglycemic reaction. The nurse should give the child which of the following to treat the reaction?

1. One sugar cube 2. 1 teaspoon of sugar 3. ½ cup of diet cola *4. ½ cup of fruit juice* *Rationale:* Hypoglycemia is immediately treated with 10 to 15 g of carbohydrate. Glucose tablets or glucose gel may be administered. Other items used to treat hypoglycemia include ½ cup of fruit juice, ½ cup of regular (nondiet) soft drink, 8 ounces of skim milk, 6 to 10 hard candies, 4 cubes of sugar or 4 teaspoons of sugar, 6 saltines, 3 graham crackers, or 1 tablespoon of honey or syrup. The items in options 1, 2, and 3 would not adequately treat hypoglycemia.

A mother arrives at the clinic with her child. The mother tells the nurse that the child has had a fever and a cough for the past 2 days, and this morning the child began to wheeze. Viral pneumonia is diagnosed. Which of the following would the nurse anticipate to be a component of the treatment plan?

1. Oral antibiotics *2. Supportive treatment* 3. Hospitalization and antibiotics 4. Intravenous (IV) fluid administration *Rationale:* With viral pneumonia, treatment is supportive. More severely ill children may be hospitalized and given oxygen, chest physiotherapy, and IV fluids. Antibiotics are not given. Bacterial pneumonia, however, is treated with antibiotic therapy.

A nurse is assigned to care for a child who is scheduled for an appendectomy. Which prescriptions does the nurse anticipate will be prescribed? *Select all that apply.*

1. Administer a Fleet enema. *2. Initiate an intravenous line.* *3. Maintain nothing-by-mouth status.* *4. Administer intravenous antibiotics.* *5. Administer preoperative medications.* 6. Place a heating pad on the abdomen to decrease pain. *Rationale:* During the preoperative period, enemas or laxatives should not be administered. In addition, heat should not be applied to the abdomen. Any of these interventions can cause the rupture of the appendix and resultant peritonitis. Intravenous fluids would be started, and the child should receive nothing by mouth while awaiting surgery. Antibiotics are usually administered because of the risk of perforation. Preoperative medications are administered as prescribed.

A mother of a 9-year-old child calls the emergency department and tells the nurse that her child received a minor burn on the hand after accidentally touching a grill during a family cookout. The mother asks the nurse for advice on how to treat the burn. The nurse tells the mother to immediately:

1. Apply a tepid compress to the child's hand. 2. Apply an ice pack to the child's hand. *3. Place the child's hand under cool running water.* 4. Apply a sterile bandage tightly over the burn area to prevent swelling. *Rationale:* Most minor burns can be handled at home by the parents. For minor burns, exposure to cool running water is the best treatment. This stops the burning process and helps alleviate pain. Ice is contraindicated because it may add more damage to already injured skin. Option 4 is an incorrect measure. In addition, the mother may not have a sterile dressing available.

A corticosteroid cream is prescribed by a health care provider for a child with atopic dermatitis (eczema). The nurse teaches the mother how to apply the cream. Which instruction is appropriate?

1. Apply the cream over the entire body. 2. Apply a thick layer of cream to affected areas only. 3. Avoid cleansing the area before applying the cream. *4. Apply a thin layer of cream, and rub it into the area thoroughly.* *Rationale:* Corticosteroid cream should be applied sparingly and rubbed into the area thoroughly. The affected area should be cleansed gently before application. The cream should not be applied over extensive areas. Systemic absorption is more likely to occur with extensive application.

Permethrin 5% (Elimite) is prescribed for a 4-year-old child with a diagnosis of scabies. The nurse instructs the mother regarding the use of this treatment. Which instruction is appropriate?

1. Apply the lotion and leave it on for 4 hours. 2. Apply the lotion to the hair, the face, and the entire body. 3. The child should wear no clothing while the lotion is in place. *4. Apply the lotion to cool, dry skin at least half an hour after bathing.* *Rationale:* Permethrin is applied from the neck downward, with care taken to ensure that the soles of the feet, the areas behind the ears, and the areas under the toenails and fingernails are covered. The lotion should be kept on for 8 to 14 hours, and then the child should be given a bath. The lotion should be applied at least 30 minutes after bathing, and it should be applied only to cool, dry skin. The child should be clothed during treatment.

Acetylsalicylic acid (aspirin) is prescribed for a child with rheumatic fever (RF). The nurse would question this prescription if the child had documented evidence of which condition?

1. Arthralgia 2. Joint pain 3. Facial edema *4. A viral infection* *Rationale:* Anti-inflammatory agents, including aspirin, may be prescribed by the health care provider for the child with RF. Aspirin should not be given to a child who has chickenpox or other viral infections such as influenza because of the risk of Reye's syndrome. Options 1 and 2 are clinical manifestations of RF. Facial edema may be associated with the development of a cardiac complication.

A nursing student is assigned to care for an infant with a diagnosis of congestive heart failure (CHF). The student develops a plan of care for the child that is focused on monitoring for fluid overload. The student plans to best assess the urine output of the infant by:

1. Asking the health care provider for permission to insert a Foley catheter 2. Monitoring the intake closely 3. Comparing the intake with the output *4. Weighing the diapers* *Rationale:* The best method to assess urine output in an infant is to weigh the diapers. Comparing intake with output would not provide an accurate measure of urine output. Measuring the intake is not directly related to the subject of the question. Although Foley catheter drainage is most accurate in determining output, it is not the most appropriate method in an infant. In addition, insertion of a Foley catheter places the infant at risk for infection.

A nurse is reviewing the record of a child scheduled for a health care provider's visit. Before data collection, the nurse notes documentation that the child has enuresis. Based on this diagnosis, the nurse plans to focus on which of the following when collecting data?

1. Bowel function *2. Bladder function* 3. Motor development 4. Nutritional status and weight gain *Rationale:* Enuresis refers to a condition in which the child is unable to control bladder function, although he or she has reached an age at which control of voiding is expected. Nocturnal enuresis, or bed-wetting, is common in children.

A nurse is collecting data on a 12-month-old child with iron deficiency anemia. Which of the following findings would the nurse expect to note in this child?

1. Bradycardia *2. Tachycardia* 3. Hyperactivity 4. A reddened appearance to the cheeks *Rationale:* Clinical manifestations of iron deficiency anemia will vary with the degree of anemia but usually include extreme pallor with porcelain-like skin, tachycardia, lethargy, and irritability.

A nurse is monitoring the daily weight of an infant with congestive heart failure (CHF). Which of the following alerts the nurse to suspect fluid accumulation and thus to the need to notify the registered nurse?

1. Bradypnea 2. Diaphoresis 3. Decreased blood pressure (BP) *4. A weight gain of 1 lb in 1 day* *Rationale:* A weight gain of 0.5 kg (1 lb) in 1 day is a result of the accumulation of fluid. The nurse should monitor the urine output, monitor for evidence of facial or peripheral edema, check the lung sounds, and report the weight gain. Tachypnea and an increased BP would occur with fluid accumulation. Diaphoresis is a sign of CHF, but it is not specific to fluid accumulation, and it usually occurs with exertional activities.

A nursing instructor asks a nursing student about the use of bacillus Calmette-Guerin vaccine (BCG). The nursing student responds correctly, knowing that the BCG vaccine is used for:

1. Children with a positive Mantoux test 2. All children to prevent tuberculosis (TB) 3. Children with both a positive Mantoux test and positive chest x-ray *4. Asymptomatic human immunodeficiency virus (HIV)-infected children who are at increased risk for developing TB* *Rationale:* The BCG vaccine is used mainly for children with a negative chest x-ray and skin test results who have had repeated exposures to TB and for asymptomatic HIV-infected children who are at increased risk for developing TB.

A sweat test is performed on a child with a suspected diagnosis of cystic fibrosis (CF). Which test result is suggestive of cystic fibrosis and will require further assessment and investigation?

1. Chloride level of 5 mEq/L 2. Chloride level of 10 mEq/L 3. Chloride level of 20 mEq/L *4. Chloride level of 40 mEq/L* *Rationale:* In a sweat test, sweating is stimulated on the child's forearm with pilocarpine, the sample is collected on absorbent material, and the amount of sodium and chloride is measured. A sample of at least 50 mg of sweat is required for accurate results. A chloride level greater than 60 mEq/L is considered to be a positive test result. A chloride level of 40 mEq/L is suggestive of CF and requires a repeat test. Options 1, 2, and 3 do not identify results that are positive for CF.

A nurse is caring for a 9-year-old child with leukemia who is hospitalized for the administration of chemotherapy. The nurse monitors the child for central nervous system (CNS) involvement by checking which of the following?

1. Color, motion, and sensation of the extremities 2. Pupillary reaction *3. Level of consciousness (LOC)* 4. The presence of petechiae in the sclera *Rationale:* The CNS is monitored because of the risk of infiltration of blast cells into the CNS. The child's level of consciousness is assessed, and the child is monitored for signs of irritability, vomiting, and lethargy. Color, motion, and sensation of the extremities are neurovascular assessments. Changes in pupillary reaction are most often noted in conditions related to increased intracranial pressure. The presence of petechiae in the sclera is an objective sign that may occur in leukemia.

A nurse is caring for a child diagnosed with Down syndrome. In describing the disorder to the parents, the nurse bases the explanation on the fact that Down syndrome is a:

1. Condition characterized by above-average intellectual functioning with deficits in adaptive behavior 2. Condition characterized by average intellectual functioning and the absence of deficits in adaptive behavior 3. Condition characterized by subaverage intellectual functioning with the absence of deficits in adaptive behavior *4. Congenital condition that results in moderate to severe retardation and has been linked to an extra chromosome 21 (group G)* *Rationale:* Down syndrome is a form of mental retardation. It is a congenital condition that results in moderate to severe mental retardation. The syndrome has been linked to an extra group G chromosome, chromosome 21 (trisomy 21). Options 1, 2, and 3 are incorrect descriptions.

A nurse is initiating seizure precautions for a child being admitted to the nursing unit. Which of the following items should the nurse place at the bedside?

1. Oxygen and a tongue depressor *2. A suction apparatus and oxygen* 3. An airway and a tracheotomy set 4. An emergency cart and an oxygen mask *Rationale:* Seizures cause a tightening of all body muscles that is followed by tremors. An obstructed airway and increased oral secretions are the major complications during and after the seizure. Suctioning and oxygen are helpful to prevent choking and cyanosis. Option 1 is incorrect; a tongue depressor is not needed and nothing is placed into the client's mouth during a seizure because of the risk for injury. Option 3 is incorrect, because inserting a tracheostomy is not done. Option 4 is incorrect, because an emergency cart would not be left at the bedside; however, it would be available in the treatment room or on the nursing unit.

A nursing instructor asks the nursing student to plan and conduct a clinical conference on phenylketonuria (PKU). The student researches the topic and plans to include which of the following in the conference?

1. PKU is an autosomal dominant disorder. *2. PKU results in central nervous system (CNS) damage.* 3. Some state laws require routine screening of all newborn infants for PKU. 4. Treatment includes dietary restriction of sodium. *Rationale:* PKU is an autosomal recessive disorder. Treatment includes dietary restriction of phenylalanine intake (not sodium). PKU is a genetic disorder that results in CNS damage from toxic levels of phenylalanine in the blood. All 50 states require routine screening of all newborn infants for PKU.

A newborn is transferred to the neonatal intensive care unit with an admitting diagnosis of esophageal atresia accompanied by a distal tracheoesophageal fistula (TEF). When assisting to care for the newborn, the priority concern would be:

1. Pain 2. Infection *3. Aspiration* 4. The parents' concerns *Rationale:* Because TEF manifests itself with regurgitation and coughing, the concern that has the highest priority is aspiration. Although the other problems are an important part of care, the one with the highest concern relates to airway.

A nurse is assisting in planning discharge instructions to the mother of a child following orchiopexy, which was performed on an outpatient basis. Which of the following is the priority in the plan of care?

1. Pain control measures 2. Measurement of intake *3. Wound care* 4. Cold and heat applications *Rationale:* The most common complications associated with orchiopexy are bleeding and infection. Discharge instruction should include demonstration of proper wound cleansing and dressing and teaching parents to identify signs of infection such as redness, warmth, swelling, or discharge. Testicles will be held in a position to prevent movement, and great care should be taken to prevent contamination of the suture line. Analgesics may be prescribed but are not the priority, considering the options presented. Option 2 is not necessary. Option 4 is not a prescribed treatment measure.

A child is to be admitted to the orthopedic unit following a Harrington rod insertion for the treatment of scoliosis. The nurse is assisting in preparing a plan of care for the child. The nurse plans to monitor which priority item in the immediate postoperative period?

1. Pain level *2. Capillary refill, sensation, and motion in all extremities* 3. Ability to turn using the logroll technique 4. Ability to flex and extend the lower extremities *Rationale:* When the spinal column is manipulated during surgery, altered neurovascular status is a possible complication; therefore neurovascular assessments including circulation, sensation, and motion should be done every 2 hours. Level of pain and ability to flex and extend the lower extremities are important postoperative assessments but not the priorities of the options provided. Logrolling would be performed by nurses.

A nurse caring for an infant with congenital heart disease is monitoring the infant closely for signs of congestive heart failure (CHF). The nurse looks for which early sign of CHF?

1. Pallor 2. Cough *3. Tachycardia* 4. Slow and shallow breathing *Rationale:* The early signs of CHF include tachycardia, tachypnea, profuse scalp sweating, fatigue, irritability, sudden weight gain, and respiratory distress. A cough may occur with CHF as a result of mucosal swelling and irritation, but it is not an early sign. Pallor may be noted in the infant with CHF, but it is also not an early sign.

A nurse reviews the record of a child who was just seen by a health care provider (HCP). The HCP has documented a diagnosis of suspected aortic stenosis. Which clinical manifestation that is specifically found in children with this disorder should the nurse anticipate?

1. Pallor 2. Hyperactivity *3. Exercise intolerance* 4. Gastrointestinal disturbances *Rationale:* The child with aortic stenosis shows signs of exercise intolerance, chest pain, and dizziness when standing for long periods. Pallor may be noted, but it is not specific to this type of disorder alone. Options 2 and 4 are not related to this disorder.

A nurse is asked to prepare for the admission of a child to the pediatric unit with a diagnosis of Wilms' tumor. The nurse assists in developing a plan of care for the child and suggests including which of the following in the plan of care?

1. Palpate the abdomen for an increase in the size of the tumor every 8 hours. *2. Inspect the urine for the presence of hematuria at each voiding.* 3. Monitor the temperature for hypothermia. 4. Monitor the blood pressure for hypotension. *Rationale:* If Wilms' tumor is suspected, the tumor mass should not be palpated. Excessive manipulation can cause seeding of the tumor and cause spread of the cancerous cells. Fever (not hypothermia), hematuria, and hypertension (not hypotension) are clinical manifestations associated with Wilms' tumor.

A preschool child who was admitted to the hospital for a minor surgery develops a rash on the second day after hospitalization and is diagnosed with chickenpox (varicella). The nurse should take which of the following actions to provide safety for all children on the unit?

1. Place only the infected child in isolation. 2. Keep siblings from visiting the infected child. 3. Place the child and any other child who were exposed in isolation. *4. Place the infected child and any immunocompromised children in isolation.* *Rationale:* The period of communicability for chickenpox is 1 day before the eruption of vesicles to about 1 week when crusts are formed. The infected child should be isolated until vesicles have dried, and other high-risk children (immunocompromised) should be isolated from the infected client.

A health care provider prescribes "eye patching" for a child with strabismus of the right eye. The nurse instructs the mother regarding this procedure and tells the mother to:

1. Place the patch on both eyes. *2. Place the patch on the left eye.* 3. Place the patch on the right eye. 4. Alternate the patch from the right to left eye hourly. *Rationale:* Eye patching may be used in the treatment of strabismus to strengthen the weak eye. In this treatment, the "good" eye is patched. This encourages the child to use the weaker eye. It is most successful when done during the preschool years. The schedule for patching is individualized and is prescribed by the ophthalmologist.

An infant returns to the nursing unit following surgery for an esophageal atresia with tracheoesophageal fistula (TEF). The infant is receiving intravenous (IV) fluids, and a gastrostomy tube is in place. The nurse who is assisting in caring for the infant will ensure that the gastrostomy tube is:

1. Placed to gravity 2. Attached to low suction 3. Taped to the bed linens *4. Elevated* *Rationale:* In the immediate postoperative period, the gastrostomy tube is elevated, allowing gastric contents to pass to the small intestine and air to escape. This promotes comfort and decreases the risk of leakage at the anastomosis. Options 1, 2, and 3 are incorrect.

A child suspected of having sickle cell disease (SCD) is seen in a clinic, and laboratory studies are performed. Which laboratory value is likely to be increased in sickle cell disease?

1. Platelet count 2. Hematocrit level 3. Hemoglobin level *4. Reticulocyte count* *Rationale:* A diagnosis is established on the basis of a complete blood count, examination for sickled red blood cells (RBCs) in the peripheral smear, and hemoglobin electrophoresis. Laboratory studies will show decreased hemoglobin and hematocrit levels and a decreased platelet count, an increased reticulocyte count, and the presence of nucleated red blood cells. Increased reticulocyte counts occur in children with SCD because the life span of their sickled RBCs is shortened.

The mother of a child arrives at the clinic because the child has been experiencing scratchy, red, and swollen eyes. The nurse notes a discharge from the eyes and a culture is sent to the laboratory for analysis. Chlamydial conjunctivitis is diagnosed. Based on this diagnosis, which of the following would require further investigation?

1. Possible trauma *2. Possible sexual abuse* 3. The presence of an allergy 4. The presence of a respiratory infection *Rationale:* A diagnosis of chlamydial conjunctivitis in a non-sexually active child should signal the health care provider to assess the child for possible sexual abuse. Allergy, infection, and trauma can cause conjunctivitis but not chlamydial conjunctivitis.

A child with diabetes mellitus is brought to the emergency department by her mother, who states that her daughter has been complaining of abdominal pain and has a fruity odor on the breath. Diabetic ketoacidosis (DKA) is diagnosed. The nurse assisting to care for the child checks the intravenous (IV) and medication supply area for which of the following?

1. Potassium 2. NPH insulin 3. 5% dextrose IV infusion *4. 0.9% normal saline IV infusion* *Rationale:* Rehydration is the initial step in resolving DKA. Normal saline is the initial IV rehydration fluid. NPH insulin is never administered by the IV route. Dextrose solutions are added to the treatment when the blood glucose levels reach an acceptable level. IV potassium may be required depending on the potassium level, but would not be part of the initial treatment.

A nurse is assisting in admitting to the hospital a 4-month-old infant with a diagnosis of vomiting and dehydration. The nurse assists in developing a plan of care for the infant and suggests including in the plan to position the infant in a(n):

1. Prone position *2. Side-lying position* 3. Modified Trendelenburg's position 4. Infant car seat with the head of the seat in a flat position *Rationale:* The vomiting infant or child should be placed in an upright or side-lying position to prevent aspiration. The positions identified in options 1, 3, and 4 will increase the risk of aspiration if vomiting occurs.

A nurse assists the health care provider in performing a lumbar puncture on a 3-year-old child with leukemia suspected of central nervous system (CNS) disease. In which position will the nurse place the child during this procedure?

1. Prone with knees flexed to the abdomen and head bent with chin resting on the chest 2. Modified Sims' position *3. Lateral recumbent with the knees flexed to the abdomen and head bent with the chin resting on the chest* 4. Lithotomy position *Rationale:* A lateral recumbent with the knees flexed to the abdomen and the head bent with the chin resting on the chest is assumed for a lumbar puncture. This position separates the spinal processes and facilitates needle insertion into the subarachnoid space. Options 1, 2, and 4 are incorrect positions.

A nurse is assisting in preparing a plan of care for a 4-year-old child hospitalized with nephrotic syndrome. Which dietary intervention is most appropriate for this child?

1. Provide a high-salt diet. 2. Provide a high-protein diet. 3. Discourage visitors at mealtimes. *4. Encourage the child to eat in the playroom.* *Rationale:* Mealtimes should center on pleasurable socialization. The child should be encouraged to eat meals with other children on the unit. A diet that is normal in protein with a sodium restriction is normally prescribed for a child with nephrotic syndrome. Parents or other family members should be encouraged to be present at mealtimes with a hospitalized child.

A 3-year-old child has returned to his room following a tonsillectomy. Which assessment finding needs immediate notification of the registered nurse?

1. Pulse rate 90, respirations 24 per minute *2. Nasal flaring and rib retractions* 3. Drooling slightly blood-tinged saliva 4. Refusal to take sips of his favorite soda *Rationale:* Nasal flaring and rib retractions are signs of respiratory distress, a major concern following a tonsillectomy. These signs require immediate notification. The vital signs are normal for a 3-year-old child. Drooling slightly blood-tinged saliva and refusal to take sips of liquids are common after a tonsillectomy.

Which of the following is the most appropriate location for assessing the pulse of an infant who is less than 1 year old?

1. Radial 2. Carotid *3. Brachial* 4. Popliteal *Rationale:* To assess a pulse in an infant (i.e., a child <1 year old), the pulse is checked at the brachial artery. The infant's relatively short, fat neck makes palpation of the carotid artery difficult. The popliteal and radial pulses are also difficult to palpate in an infant.

During labor, the nurse determines that a full term client is demonstrating late decelerations. In which sequence should the nurse implement these nursing actions? (Arrange in order) a. Provide oxygen via face mack b. Reposition the client c. Increase IV fluid d. Call the healthcare provider

1. Reposition the Client 2. Provide oxygen via face mask 3. Increase IV fluid 4. Call the healthcare provider To stabilize the fetus, intrauterine resuscitation is the first priority, and to enhance the fetal blood supply, the laboring client should be repositioned (1) to displace the gravid uterus and improve fetal perfusion. Secondly, to optimize oxygenation of the circulatory blood volume, oxygen via face mask (2) should be applied to the mother. Next, the IV fluids should be increased (3) to expand the maternal circulating blood volume. Then, the primary healthcare provider should be notified (4) for additional interventions to resolve the fetal stress.

During labor, the nurse determine that a full term client is demonstrating late decelerations. In which sequence should the nurse implement these nursing actions.

1. Reposition the client 2. Provide O2 via face mask 3. Increase IV fluid 4. Call the healthcare provider

A nurse is caring for a child with osteosarcoma following amputation of the left lower limb. The child is continually complaining of aching and cramping in the missing limb. The initial nursing action is which of the following?

1. Request a referral for a psychiatric consultation. 2. Ask the health care provider for a prescription for a placebo. *3. Reassure the child that this is a temporary condition.* 4. Tell the child that the prosthesis will relieve this sensation. *Rationale:* Following amputation, phantom limb pain is a temporary condition that some children may experience. This sensation of burning, aching, or cramping in the missing limb is most distressing to the child. The child needs to be reassured that the condition is normal and only temporary. Options 1 and 2 are inappropriate. Although the sensation of phantom pain is temporary, the prosthesis will not necessarily relieve this sensation.

A child with a fractured femur is placed in Buck's skin traction and the nurse is planning care for the client. Which information about this type of traction is correct?

1. Requires frequent pin care 2. Places the child at risk for infection 3. Uses skeletal traction and weights to provide a counterforce *4. Is a type of skin traction that pulls the hip and leg into extension* *Rationale:* Buck's skin traction is a type of skin traction used in fractures of the femur and in hip and knee contractures. It pulls the hip and leg into extension. Countertraction is applied by the child's body. Options 1, 2, and 3 describe skeletal traction.

The nurse provides instructions regarding respiratory precautions to the mother of a child with mumps. The mother asks the nurse about the length of time required for the respiratory precautions. Which response by the nurse is accurate?

1. Respiratory isolation is not necessary. 2. Mumps is not transmitted by the respiratory system. *3. Respiratory precautions are indicated during the period of communicability.* 4. Respiratory precautions are indicated for 18 days after the onset of parotid swelling. *Rationale:* Mumps is transmitted via direct contact or droplets spread from an infected person and possibly by contact with urine. Respiratory precautions are indicated during the period of communicability. Options 1, 2, and 4 are incorrect.

A nursing student is asked to administer a tepid bath to a child with a fever. The student avoids which of the following when performing this procedure?

1. Squeezes water over the child's body, using a washcloth *2. Applies alcohol-soaked cloths over the child's body* 3. Uses a water toy to distract the child during the bath 4. Places lightweight pajamas on the child after the bath *Rationale:* Alcohol should never be used for bathing the child with a fever because it can cause rapid cooling, peripheral vasoconstriction, and chilling, thus elevating the temperature further. Washcloths can be used to squeeze water over the child's body. Towels are used to dry the child. Toys, especially water toys, can be used to provide distraction during the bath. Lightweight clothing should be placed on the child after the child is dried.

A nurse instructs the mother of a child with sickle cell disease regarding the precipitating factors related to pain crisis. Which of the following, if identified by the mother as a precipitating factor, indicates the need for further instructions?

1. Stress 2. Trauma 3. Infection *4. Fluid overload* *Rationale:* Pain crisis may be precipitated by infection, dehydration, hypoxia, trauma, or general stress. The mother of a child with sickle cell disease should encourage a fluid intake of 1.5 to 2 times the daily requirement to prevent dehydration.

A nurse is assisting with data collection from an infant who has been diagnosed with hydrocephalus. If the infant's level of consciousness diminishes, a priority intervention is:

1. Taking the apical pulse 2. Taking the blood pressure 3. Testing the urine for protein *4. Palpating the anterior fontanel* *Rationale:* A full or bulging anterior fontanel indicates an increase in cerebrospinal fluid collection in the cerebral ventricle. Apical pulse and blood pressure changes and proteinuria are not specifically associated with increasing cerebrospinal fluid in the brain tissue in an infant.

A nurse is preparing to perform a neurovascular check for tissue perfusion in the child with an arm cast. Which of the following is the priority when performing this procedure?

1. Taking the temperature 2. Taking the blood pressure 3. Checking the apical heart rate *4. Checking the peripheral pulse in the affected arm* *Rationale:* The neurovascular check for tissue perfusion is performed on the toes or fingers distal to an injury or cast and includes checking peripheral pulse, color, capillary refill time, warmth, motion, and sensation. Options 1, 2, and 3 may be components of care, but they are not the priority in this situation.

Following a cleft lip repair, the nurse provides instructions to the parents regarding cleaning of the lip repair site. Which of the following solutions would the nurse use in demonstrating this procedure to the parents?

1. Tap water *2. Sterile water* 3. Full-strength hydrogen peroxide 4. Half-strength hydrogen peroxide *Rationale:* The lip repair site is cleansed with sterile water using a cotton swab after feeding and as prescribed. The parents should be instructed to use a rolling motion from the suture line out. The parents should also demonstrate performance of the correct procedure to the nurse.

A nurse prepares to administer a pancreatic enzyme powder to the child with cystic fibrosis (CF). Which of the following food items will the nurse mix with the medication?

1. Tapioca *2. Applesauce* 3. Hot oatmeal 4. Mashed potatoes *Rationale:* Pancreatic enzyme powders are not to be mixed with hot foods or foods containing tapioca or other starches. Enzyme powder should be mixed with non-fat, non-protein foods such as applesauce. Pancreatic enzymes are inactivated by heat and are partially degraded by gastric acids.

A child with croup is placed in a cool-mist tent. The mother becomes concerned because the child is frightened, consistently crying, and tries to climb out of the tent. The appropriate nursing action would be to:

1. Tell the mother that the child must stay in the tent 2. Call the health care provider and obtain a prescription for a mild sedative 3. Place a toy in the tent to make the child feel more comfortable *4. Let the mother hold the child and direct a cool mist over the child's face* *Rationale:* Crying aggravates laryngospasm and increases hypoxia, which may cause airway obstruction. If the use of a tent or hood is causing distress, treatment may be more effective if the child is held by the parent and a cool mist is directed toward the child's face. A mild sedative would not be administered to the child. Options 1 and 3 will not alleviate the child's fear.

A nurse provides instructions to the parents of an infant with hip dysplasia regarding care of the Pavlik harness. Which instruction provided by the nurse is accurate?

1. The harness must be worn 8 hours a day. 2. The infant should never be moved when out of the harness. *3. The harness needs to be removed to check the skin and for bathing.* 4. The harness must be removed for diaper changes and for feeding. *Rationale:* The harness should be worn 23 hours a day and should be removed only to check the skin and for bathing. The hips and buttocks should be supported carefully when the infant is out of the harness. The harness does not need to be removed for diaper changes or feedings.

A nurse is reviewing the immunization schedule for a child with human immunodeficiency virus (HIV) infection with the mother. Which of the following will be a component of the instructions that the nurse provides to the mother?

1. The immunization schedule must be altered because of the HIV infection. *2. No live virus vaccines should be administered to the child.* 3. Immunizations will not be given to the child with HIV infection. 4. Immunizations will be given to the child with HIV infection but will not be initiated until the child is 3 years old. *Rationale:* The mother should be instructed that the child with HIV should keep immunizations up to date. No live virus vaccines should be administered because the child with HIV is immunocompromised. The immunization schedule would not be altered in any other way, and it is important for the mother to understand the immunization schedule clearly.

A child is admitted to the hospital, and a diagnosis of bacterial meningitis is suspected. A lumbar puncture is performed, and the results reveal cloudy cerebrospinal fluid (CSF) with high protein and low glucose levels. The nurse determines that these results are indicative of:

1. The need to repeat the test 2. Possible contamination of the specimen *3. Confirmation of the diagnosis* 4. A negative test *Rationale:* A diagnosis of meningitis is made by testing CSF obtained by lumbar puncture. In the case of bacterial meningitis, findings usually include increased pressure, cloudy CSF, and high protein and low glucose levels. Options 2 and 4 are incorrect. Option 1 is an unnecessary measure.

An infant is suspected to be human immunodeficiency virus (HIV) positive, and the nurse provides information to the parents about the care of their infant. Which of the following indicates to the nurse that the parents need further information about the care of their HIV-positive infant?

1. The parents state they will not allow anyone with a cold to hold and kiss the baby. 2. The parents are able to verbalize signs and symptoms of failure to thrive. 3. The parents ask about a prescription for an antiretroviral medication. *4. The parents plan to use rice cereal to help with watery stools when they occur.* *Rationale:* If an infant is having diarrhea, the parents need to seek medical attention because this could be the beginning of an opportunistic infection. Self-treatment is not encouraged. Asking for antiretroviral therapy, understanding signs and symptoms of failure to thrive, and being protective of an immunocompromised infant are evidence of understanding the needs of the infant.

A nurse employed in an emergency department is instructed to monitor a child diagnosed with epiglottitis. The nurse notes that the child is leaning forward with the chin thrust out. The nurse interprets this finding as indicating:

1. The presence of dehydration 2. The presence of pain 3. Extreme fatigue *4. An airway obstruction* *Rationale:* Clinical manifestations suggestive of airway obstruction include tripod positioning (leaning forward supported by arms, chin thrust out, mouth open), nasal flaring, tachycardia, a high fever, and sore throat. The data in the question do not relate to options 1, 2, or 3.

A child is diagnosed with intussusception. The nurse collects data on the child, knowing that which of the following is a characteristic of this disorder?

1. The presence of fecal incontinence 2. Incomplete development of the anus 3. The infrequent and difficult passage of dry stools *4. Invagination of a section of the intestine into the distal bowel* *Rationale:* Intussusception is an invagination of a section of the intestine into the distal bowel. It is the most common cause of bowel obstruction in children age 3 months to 6 years. Option 1 describes encopresis. Option 2 describes imperforate anus, and this disorder is diagnosed in the neonatal period. Option 3 describes constipation. Constipation can affect any child at any time, although it peaks at ages 2 to 3 years. Encopresis generally affects preschool and school-age children.

A nurse is monitoring a newborn with a suspected diagnosis of imperforate anus. The nurse understands that which of the following is unassociated with this disorder?

1. The presence of stool in the urine 2. Failure to pass a rectal thermometer 3. Failure to pass meconium in the first 24 hours after birth *4. The passage of currant jelly-like stools* *Rationale:* During the newborn assessment, imperforate anus should be easily identified visually. However, a rectal thermometer or tube may be necessary to determine patency if meconium is not passed in the first 24 hours after birth. The presence of stool in the urine or vagina should be reported immediately as an indication of abnormal anorectal development. Currant jelly-like stool is not a clinical manifestation of this disorder.

A 9-year-old child is diagnosed with chlamydial conjunctivitis. The nurse consults with the primary health care provider regarding necessary follow-up because this infection can be associated with:

1. The presence of systemic allergies 2. The cleanliness of the home environment 3. The presence of otitis media *4. Possible sexual abuse* *Rationale:* A diagnosis of chlamydial conjunctivitis in a child who is not sexually active should signal the health care provider to assess the child for possible sexual abuse. Allergy and infection can cause conjunctivitis, but the infecting organism would not be chlamydial. Although the infection can be transmitted, it is not directly associated with cleanliness in the home. Chlamydial conjunctivitis also may be suspected in a sexually active adolescent with chronic infection that is unresponsive to other treatment.

A nurse is providing information to the family of a child about a synthetic cast that has been applied to the child for the treatment of a clubfoot. Which information should the nurse provide to the mother?

1. The synthetic cast takes 24 hours to dry. 2. The synthetic cast is heavier than a plaster cast. 3. The synthetic cast is stronger than a plaster cast. *4. The synthetic cast allows for greater mobility than a plaster cast.* *Rationale:* Synthetic casts dry quickly (in less than 30 minutes) and are lighter than plaster casts. Synthetic casts allow for greater mobility than a plaster cast. However, synthetic casts are not as strong as plaster casts and are more expensive.

A nurse provides feeding instructions to a mother of an infant diagnosed with gastroesophageal reflux (GER). To assist in reducing the episodes of emesis, the nurse tells the mother to:

1. Thin the feedings by adding water to the formula. *2. Thicken the feedings by adding rice cereal to the formula.* 3. Provide less frequent, larger feedings. 4. Burp less frequently during feedings. *Rationale:* Small, more frequent feedings with frequent burping are often tried as the first line of treatment in GER. Feedings thickened with rice cereal may reduce episodes of emesis. Thickened feedings do not affect reflux time, however. If thickened formula is prescribed, 1 to 3 teaspoons of rice cereal per ounce of formula is most commonly used and may require cross-cutting the nipple. Options 1, 3, and 4 are incorrect.

A child is seen in the health care clinic and received an immunization of DPT (diphtheria, pertussis, tetanus vaccine). One hour later, the mother calls the clinic and tells the nurse that the injection site is painful and red. Which of the following instructions would the nurse provide to the mother?

1. To return to the health care clinic immediately 2. To call the health care provider 3. To apply warm compresses on the site *4. To apply cold compresses for 24 hours following the injection* *Rationale:* For painful or red injection sites, the nurse should instruct the mother to apply cold compresses for the first 24 hours, then to use warm or cold compresses as long as needed. Options 1, 2, and 3 are incorrect. It is not necessary for the mother to bring the child to the clinic immediately, and it is not necessary for the mother to contact the health care provider.

A mother of a child with cystic fibrosis asks the clinic nurse about the disease. The nurse tells the mother that it is:

1. Transmitted as an autosomal dominant trait *2. A chronic multisystem disorder affecting the exocrine glands* 3. A disease that causes the formation of multiple cysts in the lungs 4. A disease that causes dilation of the passageways of many organs *Rationale:* Cystic fibrosis is a chronic multisystem disorder affecting the exocrine glands. The mucus produced by these glands (particularly those of the bronchioles, small intestine, and the pancreatic and bile ducts) is abnormally thick, causing obstruction of the small passageways of these organs. It is transmitted as an autosomal recessive trait.

A 10-year-old child with asthma is treated for acute exacerbation. Which finding would indicate that the condition is worsening?

1. Warm, dry skin 2. Increased wheezing *3. Decreased wheezing* 4. A pulse rate of 90 beats per minute *Rationale:* Decreased wheezing in a child who is not improving clinically may be interpreted incorrectly as a positive sign, when in fact it may signal an inability to move air. A "silent chest" is an ominous sign during an asthma episode. With treatment, increased wheezing may actually signal that the child's condition is improving. Warm, dry skin indicates an improvement in the condition because the child is normally diaphoretic during exacerbation. The normal pulse rate in a 10-year-old is 70 to 110 beats per minute.

A nurse is reviewing the record of a child with a diagnosis of pyloric stenosis. Which data would the nurse expect to note as having been documented in the child's record?

1. Watery diarrhea *2. Projectile vomiting* 3. Increased urine output 4. Vomiting large amounts of bile *Rationale:* Clinical manifestations of pyloric stenosis include projectile, nonbilious vomiting; irritability; hunger and crying; constipation; and signs of dehydration, including a decrease in urine output.

A nurse is caring for a child with a diagnosis of intussusception. Which of the following symptoms would the nurse expect to note in this child?

1. Watery diarrhea 2. Ribbon-like stools 3. Profuse projectile vomiting *4. Blood and mucus in the stools* *Rationale:* The child with intussusception classically presents with severe abdominal pain that is crampy and intermittent and that causes the child to draw in his or her knees to the chest. Vomiting may be present, but it is not projectile. Bright red blood and mucus are passed through the rectum and commonly described as currant jelly-like stools. Ribbon-like stools are not a manifestation of this disorder.

A nurse prepares a teaching plan regarding the administration of eardrops for the parents of a 2-year-old child. Which of the following would be included in the plan?

1. Wear gloves when administering the eardrops. 2. Pull the ear up and back before instilling the eardrops. *3. Pull the earlobe down and back before instilling the ear drops.* 4. Hold the child in a sitting position when administering the ear drops. *Rationale:* When administering eardrops to a child who is less than 3 years old, the ear should be pulled down and back. For children who are more than 3 years old, the ear is pulled up and back. Gloves do not need to be worn by the parents, but handwashing needs to be performed before and after the procedure. The child should be in a side-lying position with the affected ear facing upward to facilitate the flow of medication down the ear canal by gravity.

The student nurse is preparing a community presentation on urinary tract infections. What would the student include as risk factors? (Select all that apply) 1.) A short urethra in young girls 2.) Frequent emptying of the bladder 3.) Increased fluid intake 4.) Ingestion of highly acidic juices 5.) Cleaning the perineal area from back to front

1.) A short urethra in young girls 5.) Cleaning the perineal area from back to front Pg. 1016 -Foundations of Nursing

How is Hirschsprung disease best described? 1.) Absence of parasympathetic ganglion cells in a segment of the colon. 2.) Passage of excessive amounts of meconium by the newborn. 3.) Results in excessive peristaltic movements within the GI tract. 4.) Results in frequent evacuation of solids, liquids, and gas

1.) Absence of parasympathetic ganglion cells in a segment of the colon. Pg. 1014 -Foundations of Nursing

The nurse is caring for a child with nephrotic syndrome. The nurse is correct when questioning the HCP's order of which class of medications? (Select all that apply) 1.) Antibiotics 2.) Diuretics 3.) Vitamins 4.) Corticosteroids 5.) Antifungals

1.) Antibiotics 3.) Vitamins 5.) Antifungals Pg. 1017 -Foundations of Nursing

The LPN/LVN is assisting in preparing a care plan for a patient who has received a diagnosis of a seizure disorder. What is an important nursing intervention that should be included in caring for a child who is experiencing a seizure. 1.) Describe and record the seizure activity observed 2.) Restrain the child when seizures occur to prevent bodily harm. 3.) Place a tongue blade between the teeth if they become clenched. 4.) Suction the child during a seizure to prevent aspiration.

1.) Describe and record the seizure activity observed Pg. 1036 -Foundations of Nursing

A 3-year-old patient has received a diagnosis of cognitive impairment. Which intervention is most important in dealing with the patient and her family? 1.) Encourage the family to enroll the child in an early intervention program. 2.) Discourage play with "normal" children to prevent feelings of inadequacy. 3.) Instruct the family not to discuss their feelings in front of the child. 4.) Educate the family that they should treat the child in a special manner because she is "slow."

1.) Encourage the family to enroll the child in an early intervention program. Pg. 1056 -Foundations of Nursing

A 7-year-old patient has a diagnosis of recurrent abdominal (RAP). What nursing interventions would be most appropriate to include in the plan of care? (Select all that apply) 1.) Encourage the parents to maintain a normal schedule for their child with regard to school. 2.) Support the parents in deemphasizing their child's complaints. 3.) Educate the parents to contact the health care provider if symptoms worsen. 4.) Help the parents choose appropriate exercise for the child. 5.) Discourage the parents from allowing the child to play with other children until symptoms subside.

1.) Encourage the parents to maintain a normal schedule for their child with regard to school. 2.) Support the parents in deemphasizing their child's complaints. 3.) Educate the parents to contact the health care provider if symptoms worsen. 4.) Help the parents choose appropriate exercise for the child. Pg. 1063 -Foundations of Nursing

A pediatric patient has received a diagnosis of Wilms tumor. Which statement regarding Wilms tumor is true? 1.) The tumor manifests as a firm, nontender, intra-abdominal mass. 2.) The tumor is sometimes difficult to distinguish from the spleen. 3.) The tumor usually crosses the midline 4.) If the surgeon successfully removes the tumor without a tear in its capsule, no chemotherapy is needed.

1.) The tumor manifests as a firm, nontender, intra-abdominal mass. Pg. 1019 -Foundations of Nursing

A laboring client weighing 187 lb is 5 cm dilated and having contractions every 2-3 minutes. The client rates the pain at 7 out of 10. Nalbuphine hydrochloride 10 mg/70 kg IV push × 1 is prescribed by the health care provider. Nalbuphine hydrochloride 10 mg/1 mL is available. How many milliliters does the nurse administer? Record your answer using one decimal place. Answer: (mL)

1.2

What symptoms should alert the nurse to the possibility of an ectopic pregnancy? 1. Nausea and vomiting 2. Abdominal pain, vaginal bleeding, and a positive pregnancy test 3. Amenorrhea and a negative pregnancy test 4. Copious discharge of clear mucus

2. Abdominal pain, vaginal bleeding, and a positive pregnancy test Rationale: Abdominal pain, vaginal bleeding, and a positive pregnancy test are signs of an ectopic pregnancy. Amenorrhea and a negative pregnancy test may indicate a metabolic disorder such as hypothyroidism.

A 5-year-old patient is admitted to the pediatric unit in sickle cell crisis. Which nursing intervention will be included in the plan of care? (Select all that apply) 1.) Strenuous exercise to the extremities to increase oxygen to the area. 2.) Administration of IV fluids to improve circulation and hydration. 3.) Administration of analgesics as ordered 4.) Administration of oxygen as ordered 5.) Applying ice packs to the affected areas

2.) Administration of IV fluids to improve circulation and hydration. 3.) Administration of analgesics as ordered 4.) Administration of oxygen as ordered 5.) Applying ice packs to the affected areas Pg. 984 -Foundations of Nursing

A 6-month-old patient is suspected of having cystic fibrosis. What diagnostic tests does the nurse expect to be ordered for this patient? (Select all that apply) 1.) Bronchoscopy, w/ pulmonary washings 2.) Chest X-ray study 3.) Upper GI series 4.) Sweat test 5.) Pulmonary Function Test

2.) Chest X-ray study 4.) Sweat test 5.) Pulmonary Function Test Pg. 1002 -Foundations of Nursing

The parents of a 12-year-old patient are concerned about the child's recent diagnosis of school avoidance. Which response by the nurse would be the most appropriate? 1.) Emphasize that the child is sick and needs to stay at home from school. 2.) Discuss the importance of the child's returning to school 3.) Ignore any somatic complaints the child may have 4.) Instruct them that this is a psychiatric disorder

2.) Discuss the importance of the child's returning to school Pg. 1059 -Foundations of Nursing

A newborn was admitted to the nursery with a complete bilateral cleft lip and palate. The HCP explained the plan of therapy and its expected good results; however, the mother refuses to see or hold her baby. What is the most therapeutic initial approach to the mother? 1.) Restate what the HCP has told her about plastic surgery 2.) Encourage her to express her feelings 3.) Emphasize then normal characteristics of her baby and the baby's need for mothering. 4.) Keep the baby and mother apart until the lip has been repaired.

2.) Encourage her to express her feelings Pg. 1006 -Foundations of Nursing

The nurse is answering questions from a parent whose child has acute glomerulonephritis. Which explanation of the disease is most accurate? 1.) It is a syndrome in which there is reabsorption of bicarbonate or in which excretion of hydrogen ions is impaired. 2.) It occurs after an antecedent streptococcal infection 3.) It is a disorder manifested by gross bacteria 4.) It is a disorder associated with a defect in the ability to concentrate urine.

2.) It occurs after an antecedent streptococcal infection Pg. 1018 -Foundations of Nursing

When caring for a patient who is 2.5 years old and has Wilms tumor, what assessment is most likely to reveal clinical manifestations of the disease? 1.) Auscultation of lung sounds 2.) Palpation of the abdomen 3.) Assessment of skin turgor 4.) Palpation of femoral and dorsalis pedis pulses

2.) Palpation of the abdomen Pg. 1019 -Foundations of Nursing

A 2-month-old infant receives a diagnosis of Down syndrome. Which clinical manifestation supports this diagnosis? 1.) Pointed nose 2.) Small, rounded skull with flat occipitut; simian crease 3.) Small tongue 4.) Downward-slanting eyes

2.) Small, rounded skull with a flat occiput; simian crease Pg. 1056 -Foundations of Nursing

A baby who is declared AGA (appropriate for gestational age) falls in what weight percentile? a) 95 b) 20 c) 5 d) 9

20 Explanation: AGA infants are infants that fall between the 10th and 90th percentile for weight.

A patient in labor has red amniotic fluid. What does this finding suggest? 1. Normal amniotic fluid 2. Increased bloody show 3. Abruptio placentae 4. Meconium

3. Abruptio placentae Rationale: Red amniotic fluid is a sign of abruptio placentae, which is a premature separation of the placenta from the uterine wall. Increased bloody show is normal and causes pink amniotic fluid.

A patient who is 32 weeks pregnant presents to the emergency department with painless, bright red bleeding. What should the nurse do first? 1. Assess BP 2. Perform a pelvic exam 3. Assess fetal heart rate 4. Order a hemoglobin and hematocrit.

3. Assess fetal heart rate Rationale: For patients with 3rd trimester vaginal bleeding, assess the fetal heart rate for decelerations or tachycardia. The nurse should not perform a pelvic exam because of the possibility of placenta previa, which presents as painless, bright red bleeding. Assess the BP after the fetal heart rate pattern. Hematocrit and hemoglobin are usually normal after acute bleeding for a few hours.

The nurse is counseling a woman who wants to become pregnant. The woman tells the nurse that she has a 36-day menstrual cycle and the first day of her last menstrual period was January 8. The nurse correctly calculates that the woman's next fertile period is

January 30-31

When caring for a neonate of a mother with diabetes, which physiologic finding is most indicative of a hypoglycemic episode? a) Serum glucose level of 60 mg/dl b) Jitteriness c) Hyperalert state d) Loud and forceful crying

Jitteriness Explanation: Hypoglycemia in a neonate is expressed as jitteriness, lethargy, diaphoresis, and a serum glucose level below 40 mg/dl. A hyperalert state in a neonate is more suggestive of neuralgic irritability and has no correlation to blood glucose levels. Weak crying is found in babies with hypoglycemia. A serum glucose level of 60 mg/dl is a normal level.

A nurse who has just performed a vaginal examination notes that the fetus is in the LOP position. Which of the following clinical assessments would the nurse expect to note at this time? 1. Complaints of severe back pain 2. Rapid descent and effacement 3. Irregular and hypotonic contractions 4. Rectal pressure with blood show

1. The nurse would expect the client to complain of severe back pain.

A baby exhibits weak rooting and sudking reflexes. Which of the following nursing diagnoses would be appropriate? 1. Risk for deficient fluid volume 2. Activity intolerance 3. Risk for aspiration 4. Feeding self care deficit

1. When a boby roots and sucks poorly, the baby is unable to trasnsfer milk effectively. Because milk intake is the baby's source of fluid, the baby is high risk for fluid volume deficit

Which of the following would lead the nurse to suspect cold stress syndrome in a newborn with a temperature of 96.5 F? 1. Blood glucose of 50 mg/dL 2. Acrocyanosis 3. Tachypnea 4. Oxygen saturation of 96%

3. Babies who have cold stress syndrome will develop respiratory distress. One symptom of the distress is tachypnea

What 3 problems develop as a result from the leukemic cells infiltrating and overcrowding normal cells in leukemia?

(1) Decrease in RBCs cause anemia, (2) neutropenia leads to infection, and (3) the decrease in platelets causes bleeding

Types of heart defects

(1) Increased pulmonary blood flow (2) Decreased pulmonary blood flow (3) Obstruction to systemic blood flow (4) Mixed blood flow

A nurse is providing instructions to the mother of a child with juvenile idiopathic arthritis regarding measures to take if a painful exacerbation of the disease occurs. Which statement by the mother indicates the need for further instruction?

*1. "The full range-of-motion (ROM) exercises must be performed every day, even during the exacerbations."* 2. "Hot or cold packs will assist in reducing discomfort." 3. "The painful joint should be splinted and positioned in a neutral position." 4. "I should have my child perform simple isometric exercises during exacerbations." *Rationale:* During painful episodes, hot or cold packs and splinting and positioning the affected joint in a neutral position help reduce the pain. Full ROM exercises will cause significant pain during exacerbation and should be avoided during this time. Although resting the extremity is appropriate, it is important to begin simple isometric or tensing exercises as soon as the child is able. These exercises do not involve joint movement.

A nurse is providing instructions to a child with cystic fibrosis regarding how to perform the "huff" maneuver. The child asks the nurse about the purpose of this type of breathing. The appropriate nursing response is which of the following?

*1. "This type of breathing is used to mobilize secretions so that they can be easily coughed out."* 2. "This type of breathing prolongs inspiration time." 3. "This type of breathing moves air out of the lower lungs." 4. "This type of breathing moves air through the lungs." *Rationale:* The "huff" maneuver (forced expiratory technique) is used to mobilize secretions. This technique reduces the likelihood of bronchial collapse. The child is taught to cough with an open glottis by taking a deep breath, then exhaling rapidly whispering the word, "huff." Options 2, 3, and 4 are not the purpose of this breathing technique.

Which statement should the nurse include when providing safety instructions to the parents of an infant with a diagnosis of hydrocephalus?

*1. "When picking up your infant, support the infant's neck and head with the open palm of your hand."* 2. "Feed your infant in a side-lying position." 3. "Place a helmet on your infant when in bed." 4. "Hyperextend your infant's head with a rolled blanket under the neck area." *Rationale:* Hydrocephalus is a condition characterized by an enlargement of the cranium caused by an abnormal accumulation of cerebrospinal fluid within the cerebral ventricular system. This characteristic causes the increase in the weight of the infant's head. The infant's head becomes top heavy. Supporting the infant's head and neck when picking it up prevents hyperextension of the neck area and the infant from falling backward. The infant should be fed with the head elevated for proper motility of food processing. A helmet could suffocate an unattended infant during rest and sleep times, and hyperextension of the infant's head can put pressure on the neck vertebrae, causing injury.

A nurse is monitoring the laboratory values of a child with leukemia who is receiving chemotherapy. The nurse prepares to implement bleeding precautions if the child becomes thrombocytopenic and the platelet count is less than:

*1. 20,000/mm3* 2. 100,000/mm3 3. 120,000/mm3 4. 150,000/mm3 *Rationale:* If a child is severely thrombocytopenic, with a platelet count of less than 20,000/mm3, precautions need to be taken because of the increased risk of bleeding. The precautions include limiting activity that could result in head injury, using soft toothbrushes or Toothettes, checking urine and stools for blood, and administering stool softeners to prevent straining with constipation. In addition, suppositories and rectal temperatures are avoided. The normal platelet count ranges from 150,000 to 400,000/mm3.

A nurse is caring for a child who is scheduled for an appendectomy. When the nurse reviews the health care provider's preoperative prescriptions, which of the following would be questioned?

*1. Administer a Fleet enema.* 2. Maintain nothing per mouth (NPO) status. 3. Maintain intravenous (IV) fluids as prescribed. 4. Administer preoperative medication on call to the operating room. *Rationale:* In the preoperative period, enemas or laxatives should not be administered. No heat should be applied to the abdomen because this may increase the chance of perforation secondary to vasodilation. IV fluids would be started and the child would be NPO. Prescribed preoperative medications most likely would be administered on call to the operating room.

A nurse obtains a health history from a mother of a 15-month-old child before administering a measles, mumps, and rubella (MMR) vaccine. Which of the following is essential information to obtain before the administration of this vaccine?

*1. Allergy to eggs* 2. A recent cold 3. The presence of diarrhea 4. Any recent ear infections *Rationale:* Before the administration of MMR vaccine, a thorough health history needs to be obtained. MMR is used with caution in a child with a history of allergy to gelatin or eggs because the live measles vaccine is produced by chick embryo cell culture. MMR also contains a small amount of the antibiotic neomycin. Options 2, 3, and 4 are not contraindications to administering this immunization.

An adolescent is seen in the emergency department following an athletic injury, and it is suspected that the child sprained an ankle. X-rays are taken, and a fracture has been ruled out. The nurse provides instructions to the adolescent regarding home care for treatment of the sprain and tells the adolescent which of the following?

*1. Apply ice to the injured area for a period of 30 minutes every 4 to 6 hours for the first 24 hours.* 2. Apply heat to the injured area every 4 hours for the first 48 hours, and then begin to apply ice. 3. Immobilize the extremity and maintain the extremity in a dependent position. 4. Elevate the extremity and maintain strict bedrest for a period of 7 days. *Rationale:* To treat a sprain, the injured area should be wrapped immediately to support the joint and control the swelling. Ice is applied to reduce the swelling and should be applied for no longer than 30 minutes every 4 to 6 hours for the first 24 to 48 hours. The joint should be immobilized and elevated, but strict bedrest for a period of 7 days is not required. A dependent position will cause swelling in the affected area.

A nurse teaches a child with cystic fibrosis how to perform the "huff" maneuver and tells the child to take a:

*1. Deep breath then exhale, rapidly whispering the word "huff"* 2. Shallow breath then exhale, rapidly whispering the word "huff" 3. Deep breath, hold it for 15 seconds, then exhale slowly, whispering the word "huff" 4. Shallow breath, hold it for 10 seconds, then exhale rapidly, whispering the word "huff" *Rationale:* The "huff" maneuver (forced expiratory technique) is used to mobilize secretions. This technique reduces the likelihood of bronchial collapse. The child is taught to cough with an open glottis by taking a deep breath, then exhaling rapidly, whispering the word "huff."

Choose the home care instructions that the nurse would provide to the mother of a child with acquired immunodeficiency syndrome (AIDS). *Select all that apply.*

*1. Frequent handwashing is important.* *2. The child should avoid exposure to other illnesses.* 3. The child's immunization schedule will need revision. 4. Kissing the child on the mouth will never transmit the virus. *5. Clean up body fluid spills with bleach solution (10:1 ratio of water to bleach).* 6. Fever, malaise, fatigue, weight loss, vomiting, and diarrhea are expected to occur and do not require special intervention. *Rationale:* AIDS is a disorder that is caused by the human immunodeficiency virus (HIV) and is characterized by a generalized dysfunction of the immune system. Both cellular and humoral immunity are compromised. The horizontal transmission of HIV occurs through intimate sexual contact or parenteral exposure to blood or body fluids that contain visible blood. Vertical (perinatal) transmission occurs when an HIV-infected pregnant woman passes the infection to her infant. Home care instructions include the following: frequent handwashing; monitoring for fever, malaise, fatigue, weight loss, vomiting, diarrhea, altered activity level, and oral lesions and notifying the health care provider if these occur; monitoring for signs and symptoms of opportunistic infections; administering antiretroviral medications, as prescribed; avoiding exposure to other illnesses; keeping immunizations up to date; avoiding kissing the child on the mouth; monitoring the weight and providing a high-calorie, high-protein diet; washing eating utensils in the dishwasher; and avoiding the sharing of eating utensils. Gloves are worn for care, especially when in contact with body fluids or changing diapers. Diapers are changed frequently and away from food areas, and soiled disposable diapers are folded inward, closed with their tabs, and disposed of in a tightly covered plastic-lined container. Any body fluid spills are cleaned with a bleach solution made up of a 10:1 ratio of water to bleach.

A nurse is admitting a child with a diagnosis of lactose intolerance. Which finding does the nurse expect to assess?

*1. Frothy stools* 2. Foul-smelling ribbon stools 3. Profuse, watery diarrhea and vomiting 4. Diffuse abdominal pain unrelated to meals or activity *Rationale:* Lactose intolerance causes frothy stools. Abdominal distention, crampy abdominal pain, and excessive flatus may also occur. Option 2 is a clinical manifestation of Hirschsprung's disease. Option 3 is a clinical manifestation of celiac disease. Option 4 is a clinical manifestation of irritable bowel syndrome.

A newborn infant is diagnosed with gastroesophageal reflux (GER). The mother of the infant asks the nurse to explain the diagnosis. The nurse plans to base the response on which description of this disorder?

*1. Gastric contents regurgitate back into the esophagus.* 2. The esophagus terminates before it reaches the stomach. 3. Abdominal contents herniate through an opening of the diaphragm 4. A portion of the stomach protrudes through the esophageal hiatus of the diaphragm. *Rationale:* GER is regurgitation of gastric contents back into the esophagus. Option 2 describes esophageal atresia. Option 3 describes a congenital diaphragmatic hernia. Option 4 describes a hiatal hernia.

A breast-feeding mother of an infant with lactose intolerance asks the nurse about dietary measures. Which of the following foods would the nurse instruct the mother to avoid?

*1. Hard cheeses* 2. Green, leafy vegetables 3. Dried beans 4. Egg yolk *Rationale:* Breast-feeding mothers of an infant with lactose intolerance need to be encouraged to limit dairy products. Cheese is a dairy product. Alternative calcium sources include egg yolk; green, leafy vegetables; dried beans; cauliflower; and molasses.

When checking a child's trochlear nerve function, the nurse would perform which data collection technique?

*1. Have the child look down and in.* 2. Have the child look toward the temporal side. 3. Have the child bite down hard and open the jaw. 4. Have the child show the teeth to note symmetry of expression. *Rationale:* Having the child look down and in will assess the function of the trochlear nerve. Option 2 is the technique for checking the abducens nerve. Option 3 is the technique for checking the trigeminal nerve. Option 4 is the data collection technique for checking the facial nerve.

A child has epistaxis. The nurse understands that an appropriate treatment for epistaxis is which of the following?

*1. Have the child sit up and lean forward.* 2. Have the child assume a supine position. 3. Have the child sit up and tilt the head backward. 4. Apply continuous pressure to the nose for at least 3 minutes. *Rationale:* Correct treatment for epistaxis (a nosebleed) involves having a client sit up and lean forward. Therefore options 2 and 3 are incorrect. Continuous pressure should be applied to the nose for at least 10 minutes.

A nurse is reviewing a health care provider's prescription for a child who was just admitted to the hospital with a diagnosis of Kawasaki disease. Which prescription should the nurse anticipate being part of the treatment plan?

*1. Immune globulin* 2. Heparin infusion 3. Morphine sulfate 4. Digoxin (Lanoxin) *Rationale:* Intravenous immune globulin (IVIG) is administered to the child with Kawasaki disease to decrease the incidence of coronary artery lesions and aneurysms and to decrease fever and inflammation. Options 2, 3, and 4 are not components of the treatment plan for this disease.

To ensure a safe environment for a child admitted to the hospital for a craniotomy to remove a brain tumor, the nurse should include which of the following in the plan of care?

*1. Initiating seizure precautions* 2. Using a wheelchair for out-of-bed activities 3. Assisting the child with ambulation at all times 4. Avoiding contact with other children on the nursing unit *Rationale:* Safety of the child is the nursing priority. Seizure precautions should be implemented for any child with a brain tumor, both preoperatively and postoperatively. A thorough neurological assessment should be performed on the child, and the child's safety should be assessed before allowing the child to get out of bed without help. Assessment of the child's gait should be assessed daily. However, options 2 and 3 are not required unless functional deficits exist. Isolating the child, option 4, is not necessary.

A mother of an infant diagnosed with Hirschsprung's disease asks the nurse about the disorder. The nurse plans to base the response on which of the following?

*1. It is a congenital aganglionosis or megacolon.* 2. It is a complete small intestinal obstruction. 3. It is a condition that causes the pyloric valve to remain open. 4. It is a severe inflammation of the gastrointestinal tract. *Rationale:* Hirschsprung's disease, also known as "congenital aganglionosis" or "megacolon," is the result of an absence of ganglion cells in the rectum and to varying degrees upward in the colon. Options 2, 3, and 4 are incorrect.

A nurse working with a 24 hour old neonate is the well baby nursery has made the following nursing diagnosis: Risk for altered growth. Which of the following assessments would warrant this diagnosis? 1. The baby has lost 8% of weight since birth 2. The baby has not urinated since birth 3. The baby weighed 3,000 grams at birth 4. The baby exhibited signs of torticollis

1. A baby who has lost 8% of his or her weight after only 24 hours of life is very high risk for altered growth

A nursing student is preparing a clinical conference, and the topic of the discussion is caring for the child with cystic fibrosis (CF). The student prepares a handout for the group and lists which of the following on the handout? *Select all that apply.*

*1. It is a disease that causes mucus formation to be abnormally thick.* *2. It is a chronic multisystem disorder affecting the exocrine glands.* *3. It is transmitted as an autosomal recessive trait.* 4. It is a disease that causes dilation of the passageways of all organs. 5. It is a disease that affects males only. 6. It is a disease that affects the lungs only. *Rationale:* CF is a chronic multisystem disorder affecting the exocrine glands. The mucus produced by these glands (particularly those of the bronchioles, small intestine, and pancreatic and bile ducts) is abnormally thick, causing obstruction of the small passageways of these organs. It is transmitted as an autosomal recessive trait and can affect both males and females.

A child seen in the clinic is found to have rubeola (measles), and the mother asks the nurse how to care for the child. The nurse tells the mother that she should:

*1. Keep the child in a room with dim lights.* 2. Give the child warm baths to help prevent itching. 3. Allow the child to play outdoors, because sunlight will help the rash. 4. Take the child's temperature every 4 hours and administer 1 baby aspirin for fever. *Rationale:* A nursing consideration in rubeola is eye care. The child usually has photophobia, so the nurse should suggest that the parent keep the child out of brightly lit areas. Children with viral infections are not to be given aspirin because of the risk of Reye's syndrome. Warm baths and the sun will aggravate itching. In addition, the child needs to rest.

A cooling blanket is prescribed for a child with a fever. The nurse prepares to use the cooling blanket and avoids which of the following?

*1. Keeping the child uncovered to assist in reducing the fever* 2. Placing the cooling blanket on the bed and covering it with a sheet 3. Keeping the child dry while on the cooling blanket to prevent the risk of frostbite 4. Checking the skin condition of the child before, during, and after the use of the cooling blanket *Rationale:* While on a cooling blanket, the child should be covered lightly to maintain privacy and reduce shivering. Options 2, 3, and 4 are important interventions to prevent shivering, frostbite, and skin breakdown.

A nurse is caring for a child who was burned in a house fire. The nurse assists in developing a plan of care for monitoring the child during the treatment for burn shock. The nurse identifies which of the following assessments as providing the most accurate guide to determine the adequacy of fluid resuscitation?

*1. Level of consciousness* 2. Amount of edema at the site of the burn injury 3. Heart rate 4. Lung sounds *Rationale:* The sensorium, or level of consciousness, is an important guide to the adequacy of fluid resuscitation. The burn injury itself does not affect the sensorium, so the child should be alert and oriented. Any alteration in sensorium should be evaluated further. A neurological assessment would determine the level of consciousness in the child. Options 2, 3, and 4, although important in the assessment of the child with a burn injury, would not provide an accurate assessment of the adequacy of fluid resuscitation.

A nurse is reviewing the laboratory results of a child with aplastic anemia and notes that the white blood cell (WBC) count is 2000 cells/mm3 and the platelet count is 150,000 cells/mm3. Which of the following nursing interventions will the nurse incorporate into the plan of care?

*1. Maintain strict isolation precautions.* 2. Encourage the child to use a soft toothbrush. 3. Avoid unnecessary injections. 4. Encourage quiet play activities. *Rationale:* The normal WBC ranges from 5000 to 10,000 cells/mm3 and the normal platelet count ranges from 150,000 to 400,000/mm3. Strict isolation procedures would be required if the WBC count were low to protect the child from infection. Precautionary measures to prevent bleeding should be taken when a child has a low platelet count. These include no injections, no rectal temperatures, use of a soft toothbrush, and abstinence from contact sports or activities that could cause an injury.

A nurse is collecting data from a child suspected of having juvenile idiopathic arthritis (JIA). Which findings would the nurse expect to note if JIA were present? *Select all that apply.*

*1. Malaise, fatigue, and lethargy* *2. Painful, stiff, and swollen joints* *3. Limited range of motion of the joints* 4. Stiffness that develops later in the day 5. Cool temperature of the skin over the affected joints *6. History of late afternoon temperature, with temperature spiking up to 105° F* *Rationale:* Clinical manifestations associated with JIA include intermittent joint pain that lasts longer than 6 weeks and painful, stiff, and swollen joints that are warm to the touch, with limited range of motion. The child will complain of morning stiffness and may protect the affected joint or refuse to walk. Systemic symptoms include malaise, fatigue and lethargy, anorexia, weight loss, and growth problems. A history of a late afternoon fever with temperature spiking up to 105° F will also be part of the clinical manifestations.

A nurse is monitoring a 7-year-old child who sustained a head injury in a motor vehicle accident for signs of increased intracranial pressure (ICP). The nurse assesses the child frequently for which early sign of increased ICP?

*1. Nausea* 2. Papilledema 3. Decerebrate posturing 4. Alterations in pupil size *Rationale:* Nausea is an early sign of increased ICP. Late signs of increased ICP include a significant decrease in level of consciousness, Cushing's triad (increased systolic blood pressure and widened pulse pressure, bradycardia, and irregular respirations), and fixed and dilated pupils. Other late signs include decreased motor response to command, decreased sensory response to painful stimuli, posturing, Cheyne-Stokes respirations, and papilledema.

A clinic nurse reads the results of a Mantoux test performed on a 5-year-old child. The results indicate an area of induration measuring 8 mm. The nurse should interpret these results as:

*1. Negative* 2. Positive 3. Inconclusive 4. Definitive and requiring a repeat test *Rationale:* Induration measuring 15 mm or greater is considered a positive result in a child 4 years or older who has no associated risk factors. Since this child's results show an area of induration measuring 8 mm, the finding is negative. Options 2, 3, and 4 are incorrect interpretations.

A baby with hemolytic jaundice is being treated with fluorescent phototherapy. To provide safe newborn care, which of the following actions should the nurse perform? 1. Cover the baby's eyes with eye pads 2. Turn the lights on for ten minutes every hour 3. Clothe the baby in a shirt and diaper only 4. Tightly swaddle the baby in a baby blanket

1. When phototherapy is administered, the baby's eyes must be protected from the light source

A nurse is caring for a child following surgical removal of a brain tumor. The nurse is monitoring the child and notes that the pulse rate has increased and the blood pressure has dropped significantly. Bloody drainage also is noted on the posterior dressing. The initial nursing action is to:

*1. Notify the registered nurse (RN).* 2. Document the findings. 3. Recheck the vital signs in 1 hour. 4. Place the child in Trendelenburg's position. *Rationale:* In the event of bleeding and suspected shock, the health care provider is notified immediately. The nurse would contact the RN, who would then contact the health care provider. The child is never placed in Trendelenburg's position because it increases intracranial pressure (ICP) and the risk of bleeding. Rechecking the vital signs in 1 hour will delay necessary treatment. The nurse would document the findings, but the initial action would be to notify the RN to avoid any delays in treating this life-threatening situation.

A licensed practical nurse (LPN) is bathing a neonate and notices small dark tufts of fine hair on the neonate's lower back. The LPN should take which best action?

*1. Notify the registered nurse of the finding.* 2. Assess for other associated anomalies and document carefully. 3. Tell the mother and father that this may indicate spina bifida. 4. Recognize that this is normal in the neonate and continue the bath. *Rationale:* The legal role of the LPN is to practice under the supervision of the registered nurse. In this instance, the tuft of hair may be indicative of a spinal anomaly, and the registered nurse should be notified of the finding. It is inappropriate to discuss abnormal findings with the parents because this is the responsibility of the health care provider, if an anomaly is suspected or diagnosed. The LPN should take the priority intervention of notifying the registered nurse before documenting in the chart.

A nurse is checking a child for dehydration and documents that the child is moderately dehydrated. Which of the following symptoms would be noted in determining this finding?

*1. Oliguria* 2. Pale skin color 3. Severely depressed fontanels 4. Slightly dry, mucous membranes *Rationale:* In moderate dehydration, the fontanels would be slightly sunken, the mucous membranes would be very dry, the skin color would be dusky, and oliguria would be present. Options 2 and 4 describe mild dehydration. In mild dehydration, urine output would be decreased, but oliguria would not be present. Option 3 describes severe dehydration.

A nurse is assisting in preparing a plan of care for a child who is being admitted to the pediatric unit with a diagnosis of seizures. Which of the following would be a component of the plan of care? *Select all that apply.*

*1. Pad the side rails of the bed with blankets.* *2. Maintain the bed in a low position.* 3. Restrain the child if a seizure occurs. *4. Place the child in a side-lying lateral position if a seizure occurs.* *5. Protect the child's head, body, and extremities if a seizure occurs.* 6. Place a padded tongue blade in the child's mouth if a seizure occurs. *Rationale:* Restraints are not to be applied to a child with a seizure because they could cause injury to the child. The side rails of the bed are padded with blankets, and the bed is maintained in a low position to provide safety if the child has a seizure. The child's head and the rest of the body are protected from injury if a seizure occurs. Positioning the child on his or her side will prevent aspiration as the saliva drains out of the child's mouth during the seizure. Neither a padded tongue blade nor any other object is placed in the child's mouth once a seizure has started.

A child is admitted to the hospital with sickle cell crisis. The nurse checks this child for which frequent symptom of the disorder?

*1. Pain* 2. Diarrhea 3. Bradycardia 4. Blurred vision *Rationale:* Sickling crisis often causes pain in the bones and joints, accompanied by joint swelling. Pain is a classic symptom of the disease and may require large doses of opioid analgesics when it is severe. The symptoms listed in the other options are not part of the clinical picture.

In planning care for a child with contact dermatitis, which concern is the highest priority for the child?

*1. Pain* 2. Skin breaks 3. Infection 4. Parental knowledge about care *Rationale:* In any skin disorder, the goal with children is to offer comfort interventions so that the child can rest. Once pain has decreased, the skin can be assessed for integrity and infection. Although important, teaching is not the priority in this situation.

A child is admitted to the hospital with a probable diagnosis of nephrotic syndrome. Which findings would the nurse expect to observe? *Select all that apply.*

*1. Pallor* *2. Edema* *3. Anorexia* *4. Proteinuria* 5. Weight loss 6. Decreased serum lipids *Rationale:* Nephrotic syndrome is a kidney disorder that is characterized by massive proteinuria, hypoalbuminemia, edema, elevated serum lipids, anorexia, and pallor. The urine volume is decreased, and the urine is dark and frothy in appearance. The child with this condition gains weight.

A 4-year-old child is hospitalized with a suspected diagnosis of Wilms' tumor. The nurse assists with developing a plan of care. The nurse questions which intervention that is written in the plan of care?

*1. Palpating the abdomen for a mass* 2. Checking the urine for the presence of hematuria 3. Monitoring the blood pressure for the presence of hypertension 4. Monitoring the temperature for the presence of a kidney infection *Rationale:* Wilms' tumor is an intra-abdominal and kidney tumor. If Wilms' tumor is suspected, the mass should not be palpated. Excessive manipulation can cause seeding of the tumor and thus cause the spread of the cancerous cells. Hematuria, hypertension, and fever are clinical manifestations that are associated with Wilms' tumor.

A child is diagnosed with scarlet fever. A nurse collects data regarding the child. Which of the following is a clinical manifestation of scarlet fever?

*1. Pastia's sign* 2. Abdominal pain and flaccid paralysis 3. Dense pseudoformation membrane in the throat 4. Foul-smelling and mucopurulent nasal drainage *Rationale:* Pastia's sign is a rash seen among children with scarlet fever that will blanch with pressure, except in areas of deep creases and in the folds of joints. The tongue is initially coated with a white furry covering with red projecting papillae (white strawberry tongue). By the fourth to fifth day, the white strawberry tongue sloughs off and leaves a red, swollen tongue (strawberry tongue). The pharynx is edematous and beefy red in color. Option 2 is associated with poliomyelitis. Options 3 and 4 are characteristics of diphtheria.

Which statement made by a 12-year-old patient with type 1 diabetes indicates a need for more teaching? 1.) "My pancreas is sick and needs insulin until it gets better." 2.) "I will need to take my insulin everyday." 3.) "I need to keep a piece of candy in my pocket in case I start to feel shaky." 4.) "My mom has to give me insulin shots twice a day."

1.) "My pancreas is sick and needs insulin until it gets better." Pg. 1022 -Foundations of Nursing

A nurse is preparing for the admission of an infant with a diagnosis of bronchiolitis caused by the respiratory syncytial virus (RSV). Choose the interventions that would be included in the plan of care. *Select all that apply.*

*1. Place the infant in a private room.* *2. Place the infant in a room near the nurses' station.* 3. Ensure that the infant's head is in a flexed position. 4. Wear a mask at all times when in contact with the infant. 5. Place the child in a tent that delivers warm, humidified air. 6. Position the infant side-lying, with the head lower than the chest. *Rationale:* The infant with RSV should be isolated in a private room or in a room with another child with RSV. The infant should be placed in a room near the nurses' station for close observation. The infant should be positioned with the head and chest at a 30- to 40-degree angle and the neck slightly extended to maintain an open airway and to decrease pressure on the diaphragm. Cool, humidified oxygen is delivered to relieve dyspnea, hypoxemia, and insensible water loss from tachypnea. Contact precautions (wearing gloves and a gown) reduce the nosocomial transmission of RSV.

A nurse reviews the results of a Mantoux test performed on a 3-year-old child. The results indicate an area of induration that measures 10 mm. The nurse would interpret these results as:

*1. Positive* 2. Negative 3. Inconclusive 4. Definitive, requiring a repeat test *Rationale:* An induration that measures 10 mm or more is considered to be a positive result for children who are younger than 4 years old and for those with chronic illness or with a high risk for environmental exposure to tuberculosis. A reaction of 5 mm or more is considered to be a positive result for those in the highest-risk groups. Repeat tests are not done, especially when a positive reaction occurs.

A nurse is assigned to care for a child with hypertrophic pyloric stenosis who is scheduled for pyloromyotomy. Which of the following positions would the nurse place the child in during the preoperative period?

*1. Prone with the head of the bed elevated* 2. Prone with the head of the bed lowered to promote drainage 3. Supine with the head of the bed at a 30-degree angle 4. Supine with the head of the bed at a 45-degree angle *Rationale:* In the preoperative period, the infant is positioned prone with the head of the bed elevated to reduce the risk of aspiration. Options 2, 3, and 4 are inappropriate positions to prevent this risk.

Choose the interventions that a nurse would include when writing a care plan for a child with hepatitis? *Select all that apply.*

*1. Providing a low-fat, well-balanced diet* 2. Notifying the health care provider if jaundice is present *3. Teaching the child effective hand washing techniques* 4. Scheduling play time in the playroom with other children *5. Instructing the parents about the risks associated with taking medications* 6. Arranging for indefinite home schooling because the child will not be able to return to school *Rationale:* Because hepatitis can be viral, standard precautions should be instituted in the hospital. The child should be discouraged from sharing toys, so playtime in the playroom with other children is not part of the plan of care. The child will be allowed to return to school 1 week after the onset of jaundice, so indefinite home schooling would not need to be arranged. Jaundice is an expected finding with hepatitis and would not warrant notification of the health care provider. Provision of a low-fat, well-balanced diet is recommended. Parents are cautioned about administering any medication to the child because normal doses of many medications may become dangerous because of the liver's inability to detoxify and excrete them. Handwashing is the single most effective measure in control of hepatitis in any setting, and effective handwashing can prevent the compromised child from picking up an opportunistic type of infection.

A nurse is monitoring for signs of dehydration in a 1-year-old child who has been hospitalized for diarrhea and prepares to take the child's temperature. Which method of temperature measurement should be avoided?

*1. Rectal* 2. Axillary 3. Electronic 4. Tympanic *Rationale:* Rectal temperature measurements should be avoided if diarrhea is present. The use of a rectal thermometer can stimulate peristalsis and cause more diarrhea. Axillary or tympanic measurements of temperature would be acceptable. Most measurements are performed via electronic devices.

A nurse is monitoring a child following a tonsillectomy. Which finding may indicate that the child is bleeding?

*1. Restlessness* 2. A decreased pulse rate 3. Complaints of discomfort 4. An elevation in blood pressure (BP) *Rationale:* Frequent swallowing, restlessness, a fast and thready pulse, and vomiting bright red blood are signs of bleeding. An elevated BP is not an indication of bleeding. Complaint of discomfort is an expected finding following a tonsillectomy.

A nurse is assisting in developing a plan of care for a child who will be returning from the operating room following a tonsillectomy. The nurse plans to place the child in which position on return from the operating room?

*1. Side-lying* 2. Trendelenburg's and on the right side 3. Supine 4. High Fowler's and on the left side *Rationale:* The child should be placed in a prone or side-lying position following tonsillectomy to facilitate drainage. Options 2, 3, and 4 will not facilitate drainage.

A nurse reviews the plan of care for a child with Reye's syndrome. The nurse prioritizes the nursing interventions included in the plan and prepares to monitor for:

*1. Signs of increased intracranial pressure* 2. The presence of protein in the urine 3. Signs of a bacterial infection 4. Signs of hyperglycemia *Rationale:* Intracranial pressure and encephalopathy are major complications of Reye's syndrome. Protein is not present in the urine. Reye's syndrome is related to a history of viral infections, and hypoglycemia is a symptom of this disease.

A licensed practical nurse is providing care for a child with hydrocephalus who has had a ventriculoperitoneal shunt revision. Which data collection finding should be reported to the registered nurse immediately?

*1. Temperature 100.9° F* 2. Pulse 78 beats per minute 3. Blood pressure 110/70 mm Hg 4. Respirations 22 breaths per minute *Rationale:* Fever may be an indication of an infection of the shunt, which is the primary concern in the postoperative period, related to a shunt insertion. All of the other vital signs are normal findings for this child.

A practical nurse is collaborating with the registered nurse to form a care plan for a client diagnosed with placenta previa at 33 weeks gestation. What does the nurse anticipate being included in the plan of care? Select all that apply.

- Nonstress test 1 or 2 times a week - Prepare for cesarean birth at any time - Type and screen blood Explain: In placenta previa, the placenta is implanted over or very near the cervix. As a result, placental blood vessels may be disrupted during dilation and effacement. Because of the increased risk of hemorrhage, the client should have a type and screen on file at the selected hospital. A nonstress test or biophysical profile should be performed once or twice a week to ensure fetal well-being. With asymptomatic clients, a cesarean birth is planned after 36 weeks gestation and prior to the onset of labor to prevent blood loss of mother and fetus. However, if the client is bleeding profusely or constantly or goes into active labor, a cesarean birth is typically performed immediately. (Option 1) The recommended activity for a client at less than 36 weeks gestation with diagnosed placenta previa is bed rest with bathroom privileges. A stable client may be released to continue bed rest at home, but the client must be closely monitored and return to the hospital immediately if bleeding occurs.

A child abruptly develops miniature petechiae over his legs, along with epistaxis and bleeding into the joints. Laboratory results reveal a platelet count of 20,000/mm3. The child is eventually diagnosed with idiopathic thrombocytopenic purpura. The mother of the child is distraught and asks the nurse what the course of this disorder typically is. Which of the following should the nurse mention? a) 1 to 3 months b) Chronic condition c) 4 to 6 weeks d) Terminal condition

1 to 3 months In most children, ITP runs a limited, 1- to 3-month course. A few children develop chronic ITP

The caregiver of a child with sickle cell disease asks the nurse how much fluid her child should have each day after the child goes home. In response to the caregiver's question, the nurse would explain that for the child with sickle cell disease, it is best that the child have: a) 1,000 to 1,200 mL of fluid per day b) 2,500 to 3,200 mL of fluid per day c) 1,500 to 2,000 mL of fluid per day d) 300 to 800 mL of fluid per day

1,500 to 2,000 mL of fluid per day Explanation: Prevention of crises is the goal between episodes. Adequate hydration is vital; fluid intake of 1,500 to 2,000 mL daily is desirable for a child weighing 20 kg and should be increased to 3,000 mL during the crisis.

The triage nurse in an obstetric clinic received the following forur messages during the lunch hour. Which of the woman should the nurse telephone first? 1. " My section incision from last week is leaking a whitish yellow discharge and I have a fever. What should I do?" 2. " I am 39 weeks pregnant with my first baby. I am having contractions about every 10 minutes." 3. " My boyfriend and I had intercourse this morning and our condom broke. What should we do?" 4. " I started my period yesterday. I need some medicine for these terrible menstrual cramps."

1. The nurse should call the postaperative cesarean client back first. It sounds, from her description, that she has a wound infection

Four babies are in the newborn nursery. The nurse pages the neonatologist to see the baby who exhibits which of the following? 1. Intracostal retraction 2. Erythema toxicum 3. Pseudostrabismus 4. Vernix caseosa

1. Intracostal retractions are symptomatic of respiratory distress syndrome

Which of the following laboratory findings would the nurse expect to see in a baby diagnosed with erythroblastosis fetalis? 1. Hemacrit 24% 2. Leukocyte count 45,000 cells/mm 3. Sodium 125 mEq/L 4. Potassium 5.5 mEq/L

1. The baby with erythroblastosis fetalis would exhibit signs of severe anemia, which a hematocrit of 24% reflects

A nurse is providing instructions to the parents of an infant with a ventriculoperitoneal shunt. The nurse plans to include which of the following instructions?

1. "Call the health care provider if the infant is fussy." 2. "Position the infant on the side of the shunt when the infant is put to bed." 3. "Expect an increased urine output from the shunt." *4. "Call the health care provider if the infant has a high-pitched cry."* *Rationale:* If the shunt is broken or malfunctioning, the fluid from the ventricular part of the brain will not be diverted to the peritoneal cavity. The cerebrospinal fluid will build up in the cranial area. The result is increased intracranial pressure, which then causes a high-pitched cry in the infant. The infant should not be positioned on the side of the shunt because this will cause pressure on the shunt and skin breakdown. This type of shunt affects the gastrointestinal system, not the genitourinary system, and an increased urinary output is not expected. Option 1 is a concern only if other signs indicative of a complication are occurring.

A nurse is collecting data on a child recently diagnosed with glomerulonephritis. Which of the following questions to the mother would elicit information about the cause of this disease?

1. "Did your child sustain any injuries to the kidney area?" *2. "Did your child recently complain of a sore throat?"* 3. "Has your child had any diarrhea?" 4. "Have you noticed any rashes on your child?" *Rationale:* Group A beta hemolytic streptococcal infection is a cause of glomerulonephritis. Often the child becomes ill with streptococcal infection of the upper respiratory tract and then develops symptoms of acute poststreptococcal glomerulonephritis after an interval of 1 to 2 weeks. The questions to the mother in options 1, 3, and 4 are unrelated to a diagnosis of glomerulonephritis.

A nurse is preparing a 2-year-old child with suspected nephrotic syndrome for a renal biopsy to confirm the diagnosis. The mother asks the nurse, "Will my child ever look thin again?" The nurse appropriately responds by saying:

1. "Do you feel guilty because about your child's weight gain?" *2. "In most cases, medication and diet will control fluid retention."* 3. "Wearing loose-fitting clothing should help conceal the extra weight." 4. "When children are little, it's expected that they'll look a little chubby." *Rationale:* It is important to give the mother information that addresses the issue that is the parent's concern. Most children experience remission with treatment. Options 1 and 3 are nontherapeutic and may add to the mother's guilt. Option 4 does not acknowledge the concern and is a stereotypical response.

A nurse is providing information to the mother of a child with nephrotic syndrome regarding the edematous appearance of the child. Which of the following statements should the nurse make to the mother?

1. "Dress the child in loose-fitting clothing to hide the extra weight." 2. "Children always look a little bit fat, so don't be concerned." *3. "The fluid retention should be controlled by medication and diet."* 4. "The child will always have this appearance, and preparing the child for the body image change is important." *Rationale:* Most children experience remission with treatment and corticosteroids. Diuretics also may be a component of the treatment plan, and a restricted sodium diet is recommended. It is important to give the parent information in a matter-of-fact manner and address the issue that is the parent's concern. Options 1, 2, and 4 are inaccurate and inappropriate statements to the mother.

A nurse reinforces home-care instructions to the parents of a child with hepatitis regarding the care of the child and the prevention of the transmission of the virus. Which statement by a parent indicates a need for further instruction?

1. "Frequent handwashing is important." *2. "I need to provide a well-balanced, high-fat diet to my child."* 3. "I need to clean contaminated household surfaces with bleach." 4. "Diapers should not be changed near any surfaces that are used to prepare food." *Rationale:* The child with hepatitis should consume a well-balanced, low-fat diet to allow the liver to rest. Options 1, 3, and 4 are components of the home-care instructions to the family of a child with hepatitis.

A nurse is told that a child with rheumatic fever (RF) will be arriving to the nursing unit for admission. Which question should the nurse ask the family to elicit information specific to the development of RF?

1. "Has the child complained of back pain?" 2. "Has the child complained of headaches?" 3. "Has the child had any nausea or vomiting?" *4. "Did the child have a sore throat or an unexplained fever within the past 2 months?"* *Rationale:* Rheumatic fever (RF) characteristically presents 2 to 6 weeks after an untreated or partially treated group A β-hemolytic streptococcal infection of the upper respiratory tract. Initially, the nurse determines if the child has had a sore throat or an unexplained fever within the past 2 months. Options 1, 2, and 3 are unrelated to RF.

A nurse is collecting data on a child with a diagnosis of rheumatic fever. Which of the following questions would the nurse initially ask the mother of the child?

1. "Has the child had any diarrhea?" 2. "Has the child been vomiting?" 3. "Does the child complain of chest pain?" *4. "Has the child complained of a sore throat within the past few months?"* *Rationale:* Rheumatic fever characteristically presents 2 to 6 weeks following an untreated or partially treated group A β-hemolytic streptococcal infection of the upper respiratory tract. Initially, the nurse determines whether any family members have had a sore throat or unexplained fever within the past 2 months. Although options 1, 2, and 3 may be asked during data collection, they would not be the initial concerns for a child with rheumatic fever.

A mother with human immunodeficiency virus (HIV) infection brings her 10-month-old infant to the clinic for a routine checkup. A health care provider has documented that the infant is asymptomatic for HIV infection. After the checkup, the mother tells the nurse that she is so pleased that the infant will not get HIV. Which response by the nurse is appropriate?

1. "I am also so pleased that everything has turned out fine." 2. "Since symptoms have not developed, it is unlikely that the infant will develop HIV infection." 3. "Everything looks great, but be sure that you return with your infant next month for the scheduled visit." *4. "Most children infected with HIV develop symptoms within the first 9 months of life, and some become symptomatic at some point before the age of 3 years."* *Rationale:* Most children who are infected with HIV develop symptoms within the first 9 months of life. The remainder of these infected children become symptomatic sometime before the age of 3 years. Children, with their immature immune systems, have a much shorter incubation period than adults. Options 1, 2, and 3 are incorrect responses.

The mother of an infant newly diagnosed with cystic fibrosis is being taught proper nutritional needs for the infant. The nurse determines that the mother understands nutritional needs when the mother replies:

1. "I know that my infant needs to drink predigested formula until she has her stool pattern developed." 2. "When I begin feeding my infant cereal, I will make sure to warm the cereal and administer the pancreatic enzyme mixed in." 3. "I will make sure that I give my infant fat-free milk as a supplement to her predigested formula, because she is not able to digest fat." *4. "I know I need to monitor my infant's stools and if there are more than four stools a day, I will increase the pancreatic enzyme."* *Rationale:* Cystic fibrosis requires a high-calorie, high-protein diet with pancreatic enzyme replacement therapy. The infant needs to remain on the predigested formula until 1 year of age, when formula can be discontinued and then fat-free milk consumed. The pancreatic enzyme should not be mixed with warmed foods because this inactivates the enzyme. Stools must be monitored, and pancreatic enzymes are administered based on the stool pattern.

A nurse is providing instructions to the parents of a child with scoliosis regarding the use of a brace. Which statement by a parent indicates the need for further instruction?

1. "I need to have my child wear a soft fabric under the brace." *2. "I will apply lotion under the brace to prevent skin breakdown."* 3. "I need to encourage my child to perform the prescribed exercises." 4. "I need to avoid applying powder under the brace, because it will cake." *Rationale:* The use of either lotions or powders should be avoided, because they can become sticky or cake under the brace, thus causing irritation. Options 1, 3, and 4 are appropriate statements regarding the care of a child with a brace.

Griseofulvin (Gris-PEG) is prescribed for a child with tinea capitis. The nurse provides instructions to the family regarding administration of the medication. Which statement by the mother indicates a need for further instructions?

1. "I need to keep my child out of the sun." 2. "I need to continue the therapy as long as it is prescribed." *3. "I need to administer the medication 2 hours before meals."* 4. "I need to shake the oral suspension before preparing the dose." *Rationale:* Gris-PEG is given with or after meals to avoid gastrointestinal (GI) irritation and to increase absorption. Oral suspensions should be shaken well. Parents are instructed to continue therapy as prescribed and not to miss a dose. Exposure to the sun is avoided during treatment.

A nurse provides instructions to the parents of an infant with hip dysplasia regarding care of the Pavlik harness. Which statement, if made by one of the parents, indicates an understanding of the use of the harness?

1. "I need to remove the harness to feed my infant." 2. "I need to remove the harness to change the diaper." 3. "My infant needs to remain in the harness at all times." *4. "I can remove the harness to bathe my infant."* *Rationale:* The harness should be worn 23 hours a day and should be removed only to check the skin and for bathing. The hips and buttocks should be supported carefully when the infant is out of the harness. The harness does not need to be removed for diaper changes or feedings. Option 3 is incorrect.

The nurse is reinforcing instructions to an adolescent with type 1 diabetes mellitus regarding insulin administration and rotation sites. Which statement, if made by the adolescent, would indicate an understanding of the instructions?

1. "I need to use a different site for each insulin injection." 2. "I should use only my stomach and my thighs for injections." 3. "I need to use the same site for 1 month before rotating to another site." *4. "I need to use one major site for the morning injection and another major site for the evening injection for 2 to 3 weeks before changing major sites."* *Rationale:* To help decrease variations in absorption from day to day, the child should use one location within a major site for the morning injection. The child should then rotate to another site for the evening injection, and a third site for the bedtime injection. The child should follow this pattern for a period of 2 to 3 weeks before changing major sites.

A nurse provides home care instructions to the parents of an infant following surgical intervention for imperforate anus and tells the parents about the procedure for anal dilation. Which statement by the parents indicates the need for further instructions?

1. "I need to use only dilators supplied by the health care provider." 2. "I need to use a water-soluble lubricant." 3. "I will insert the dilator no more than 1 to 2 cm into the anus." *4. "I will insert a glycerin suppository before the dilation."* *Rationale:* Following this surgery, anal dilation at home by the parents is necessary to achieve and maintain bowel patency. Inserting a glycerin suppository before dilation is not a component of this procedure. Options 1, 2, and 3 are accurate instructions and will prevent damage to the rectal mucosa.

A nurse provides instruction to the parents of a child with leukemia regarding measures related to monitoring for infection. Which statement by the parents indicates the need for further instructions?

1. "I need to use proper handwashing techniques." *2. "I need to take my child's rectal temperature daily."* 3. "I need to inspect my child's skin daily for redness." 4. "I need to inspect my child's mouth daily for lesions." *Rationale:* The risk of injury to the fragile mucous membranes is so great in the child with leukemia that only oral, axillary, or temporal or tympanic temperatures should be taken. Rectal abscesses can easily occur in damaged rectal tissue, so no rectal temperatures should be taken. In addition, oral temperatures should be avoided if the child has oral ulcers. Options 1, 3, and 4 are appropriate teaching measures.

A nurse has provided instructions to the mother of a child who has been diagnosed with bacterial conjunctivitis. Which of the following, if stated by the mother, would indicate the need for further instructions?

1. "I need to wash my hands frequently." 2. "I need to clean the eye, as prescribed." *3. "It is OK to share towels and washcloths."* 4. "I need to give the eyedrops, as prescribed." *Rationale:* Bacterial conjunctivitis is highly contagious, and infection control measures should be taught; these include frequent handwashing and not sharing towels and washcloths. Options 1, 2, and 4 are correct measures.

A nursing student is assigned to help administer immunizations to children in a clinic. The nursing instructor asks the student about the contraindications to receiving an immunization. Immunization is contraindicated in the presence of which condition?

1. A cold 2. Otitis media 3. Mild diarrhea *4. A severe febrile illness* *Rationale:* A severe febrile illness is a reason to delay immunization, but only until the child has recovered from the acute stage of the illness. Minor illnesses such as a cold, otitis media, or mild diarrhea are not contraindications to immunization.

Isoniazid (INH) is prescribed for a 2-year-old child with a positive Mantoux test. The mother of the child asks the nurse how long the child will need to take the medication. The appropriate response is:

1. 6 months *2. 9 months* 3. 15 months 4. 18 months *Rationale:* Isoniazid is given to prevent tuberculosis (TB) infection from progressing to active disease. A chest x-ray film is obtained before the initiation of preventive therapy. In infants and children, the recommended duration of INH therapy is 9 months. For children with human immunodeficiency virus infection, a minimum of 12 months is recommended.

A mother of a child with cystic fibrosis asks the nurse when the postural drainage should be performed. The mother states that the child eats meals at 8:00 AM, 12 noon, and at 6:00 PM The nurse tells the mother that the postural drainage should be performed at:

1. 8:00 AM, 2:00 PM, and 6:00 PM 2. 9:00 AM, 1:00 PM and 6:00 PM 3. 8:00 AM, 12:00 noon, and 6:00 PM *4. 10:00 AM, 2:00 PM and 8:00 PM* *Rationale:* Respiratory treatments should be performed at least 1 hour before meals or 2 hours after meals to prevent vomiting. In some children with cystic fibrosis, treatments are prescribed every 2 hours, particularly if infection is present. It is also important to perform these treatments before bedtime to clear airways and facilitate rest.

A nurse is preparing to feed a 1-year-old hospitalized child. The nurse prepares the amount of formula to be given to this child, knowing that generally a 1-year-old consumes approximately:

1. 90 mL per feeding 2. 100 mL per feeding *3. 175 mL per feeding* 4. 380 mL per feeding *Rationale:* A 1-year-old child consumes approximately 175 mL (6 ounces) of formula per feeding. Options 1, 2, and 4 are incorrect.

A nurse provides instructions to the parents of an infant with gastroesophageal reflux (GER) regarding proper positioning to manage reflux. The nurse tells the parents that the infant should be maintained in:

1. A 30-degree angle when supine 2. A 60-degree angle when prone *3. An upright angle 24 hours a day* 4. A 20-degree angle when side-lying *Rationale:* Proper positioning is an important component of reflux management. Ideally the goal is to maintain the infant in an upright angle 24 hours a day, at a 60-degree angle when supine, and at a 30-degree angle when prone. This position is maintained until the infant remains asymptomatic for 6 weeks.

A nurse in a newborn nursery is told that a newborn with spina bifida (myelomeningocele type) will be transported from the delivery room. The nurse is asked to prepare for the arrival of the newborn. The nurse places which of the following priority items at the newborn's bedside?

1. A blood pressure cuff 2. A rectal thermometer 3. A specific gravity urinometer *4. A bottle of sterile normal saline* *Rationale:* The newborn with spina bifida is at risk for infection before the closure of the gibbus. A sterile normal-saline dressing is placed over the gibbus to maintain moisture of the gibbus and its contents. This prevents tearing or breakdown of the skin integrity at the site. Blood pressure is difficult to assess during the newborn period and is not the best indicator of infection. Urine concentration is not well developed in the newborn stage of development. A thermometer will be needed to assess the temperature, but in this newborn the priority is to maintain sterile normal-saline dressings over the gibbus.

A nursing student is presenting a clinical conference and discusses the causative factors related to beta-thalassemia. Which group is at greatest risk of developing this disorder?

1. A child of Mexican descent *2. A child of Mediterranean descent* 3. A child whose intake of iron is extremely poor 4. A child breast-fed by a mother with chronic anemia *Rationale:* Beta-thalassemia is an autosomal recessive disorder. This disorder is found primarily in individuals of Mediterranean descent. The disease also has been reported in Asian and African populations. Options 1, 3, and 4 are not risk factors for this disorder.

A nurse reviews the record of a 1-year-old child seen in the clinic and notes that the health care provider has documented a diagnosis of celiac crisis. Which of the following symptoms would the nurse expect to note in this condition?

1. Anorexia 2. Joint pain *3. Profuse, watery diarrhea* 4. Constipation *Rationale:* Clinical manifestations associated with celiac crisis include profuse, watery diarrhea and vomiting that quickly lead to severe dehydration and metabolic acidosis. The cause of the crisis is usually infection or hidden sources of gluten. The child may require intravenous fluids to correct fluid and acid-base imbalances, albumin to treat shock, and corticosteroids to decrease severe mucosal inflammation.

A nurse is assigned to care for a child after a spinal fusion for the treatment of scoliosis. The child complains of abdominal discomfort and begins to have episodes of vomiting. On data collection, the nurse notes abdominal distention. Which action should the nurse take?

1. Administer an antiemetic. 2. Increase the intravenous fluids. *3. Notify the registered nurse (RN).* 4. Place the child in a side-lying Sims' position. *Rationale:* A complication after the surgical treatment of scoliosis is superior mesenteric artery syndrome. This disorder is caused by mechanical changes in the position of the child's abdominal contents that result from the lengthening of the child's body. It results in a syndrome of emesis and abdominal distention that is similar to that which occurs with intestinal obstruction or paralytic ileus. Postoperative vomiting among children with body casts or among those who have undergone spinal fusion warrants attention because of the possibility of superior mesenteric artery syndrome. Therefore, the remaining options are incorrect.

Choose the interventions for a child older than 2 years of age with type 1 diabetes mellitus who has a blood glucose level of 60 mg/dL. *Select all that apply.*

1. Administer regular insulin. 2. Encourage the child to ambulate. *3. Give the child a teaspoon of honey.* 4. Provide electrolyte replacement therapy intravenously. 5. Wait 30 minutes and confirm the blood glucose reading. *6. Prepare to administer glucagon subcutaneously if unconsciousness occurs.* *Rationale:* Hypoglycemia is defined as a blood glucose level less than 70 mg/dL. Hypoglycemia occurs as a result of too much insulin, not enough food, or excessive activity. If able, the nurse should confirm hypoglycemia with a blood glucose reading. Glucose is administered orally immediately; the rapid-releasing sugar (such as honey) is followed by a complex carbohydrate and protein, such as a slice of bread or a peanut butter cracker. An extra snack is given if the next meal is not planned for more than 30 minutes or if activity is planned. If the child becomes unconscious, cake frosting or glucose paste can be squeezed onto the gums, and the blood glucose level is retested. If the child does not improve within 15 minutes, administration of glucagon may be necessary, and the nurse should be prepared for this intervention. In the hospital setting the nurse should be prepared to administer dextrose intravenously. Encouraging the child to ambulate and administering regular insulin will result in a lowered blood glucose level. Providing electrolyte replacement therapy intravenously is an intervention to treat diabetic ketoacidosis. Waiting 30 minutes to confirm the blood glucose level delays necessary intervention.

A nurse is reviewing the health care provider's prescriptions for a child with rheumatic fever who is suspected of having a viral infection. The nurse notes that acetylsalicylic acid (aspirin) is prescribed for the child. Which of the following nursing actions is appropriate?

1. Administer the aspirin if the child's temperature is elevated. 2. Administer the aspirin if the child experiences any joint pain. *3. Consult with the registered nurse to verify the prescription.* 4. Administer acetaminophen (Tylenol) instead of the aspirin for temperature elevation. *Rationale:* Anti-inflammatory agents, including aspirin, may be prescribed for the child with rheumatic fever. Aspirin should not be given to a child who has chickenpox or other viral infections. The nurse would not administer acetaminophen without specific health care provider prescriptions. Options 1 and 2 are not appropriate actions.

A 1-year-old child is diagnosed with intussusception. The mother of the child asks the nurse to describe the disorder. The nurse tells the mother that this disorder is:

1. An acute bowel obstruction *2. A condition in which a proximal segment of the bowel prolapses into a distal segment of the bowel* 3. A condition in which a distal segment of the bowel prolapses into a proximal segment of the bowel 4. A condition that causes an acute inflammatory process in the bowel *Rationale:* Intussusception occurs when a proximal segment of the bowel prolapses into a distal segment of the bowel. It is a common cause of acute bowel obstruction in infants and young children. It is not an inflammatory process.

A nurse is caring for an infant who has been diagnosed with primary hypothyroidism. The nurse is reviewing the results of the laboratory tests and would expect to note which of the following?

1. An elevated T4 level *2. An elevated thyroid-stimulating hormone (TSH) level* 3. A decreased TSH level 4. A normal T4 level *Rationale:* Diagnostic findings in primary hypothyroidism include a low T4 level and a high TSH level. Options 1, 3, and 4 are not diagnostic findings in this condition.

Laboratory studies are performed on a child suspected of iron deficiency anemia. The nurse reviews the laboratory results, knowing that which of the following would indicate this type of anemia?

1. An elevated hemoglobin level with a low hematocrit level 2. A decreased reticulocyte count 3. An elevated red blood cell (RBC) count *4. RBCs that are microcytic and hypo chromic* *Rationale:* The results of a complete blood cell count in children with iron deficiency anemia will show low hemoglobin levels and microcytic and hypochromic RBCs. The reticulocyte count is usually normal or slightly elevated.

A nurse is caring for a child with a fracture who is placed in skeletal traction. The nurse monitors for the most serious complication associated with this type of traction by checking for:

1. An increase in the blood pressure 2. A decrease in the urinary output 3. A lack of appetite *4. An elevated temperature* *Rationale:* The most serious complication associated with skeletal traction is osteomyelitis, an infection involving the bone. Organisms gain access to the bone systemically or through the opening created by the metal pins or wires used with the traction. Osteomyelitis may occur with any open fracture. Clinical manifestations include complaints of localized pain, swelling, warmth, tenderness, an unusual odor from the fracture site, and an elevated temperature.

A nurse is caring for a child recently diagnosed with cerebral palsy. The parents of the child ask the nurse about the disorder. The nurse bases the response to the parents on the understanding that cerebral palsy is:

1. An infectious disease of the central nervous system 2. An inflammation of the brain as a result of a viral illness 3. A congenital condition that results in moderate to severe retardation *4. A chronic disability characterized by a difficulty in controlling the muscles* *Rationale:* Cerebral palsy is a chronic disability characterized by difficulty in controlling the muscles as a result of an abnormality in the extrapyramidal or pyramidal motor system. Meningitis is an infectious process of the central nervous system. Encephalitis is an inflammation of the brain that occurs as a result of viral illness or central nervous system infection. Down syndrome is an example of a congenital condition that results in moderate to severe retardation.

A mother brings her 5-month-old daughter into the pediatrician's office with complaints that the child has been vomiting during feedings. The mother also states that the child is sometimes very fussy. The nurse's initial action would be to:

1. Assess the child's growth status. *2. Obtain a complete history of the child's feeding habits.* 3. Assess whether any other children in the family have had the same problem. 4. Explain to the mother that the health care provider will prescribe a barium swallow and upper gastrointestinal (GI) series. *Rationale:* In most situations, a complete history and physical examination of the child is the initial step in diagnosing gastroesophageal reflux disease. The child's feeding habits will give the nurse an indicator of the growth status. The child is weighed and measured after the initial interview is completed with the parent. Hereditary factors are not the priority. Further diagnostic studies may be ordered but only after a complete history is obtained.

A nurse is reviewing the health care record of an infant suspected of having unilateral hip dysplasia. Which assessment finding would the nurse expect to note documented in the infant's record regarding this condition?

1. Asymmetric adduction of the affected hip when placed supine with the knees and hips flexed *2. Asymmetry of the gluteal skin folds when the infant is placed prone and the legs are extended against the examining table * 3. An apparent short femur on the unaffected side 4. Full range of motion in the affected hip *Rationale:* Asymmetry of the gluteal skin folds when the infant is placed prone and the legs are extended against the examining table is noted in hip dysplasia. Asymmetrical abduction of the affected hip, when an infant is placed supine with the knees and hips flexed, also would be an assessment finding in hip dysplasia in infants beyond the newborn period. An apparent short femur on the affected side is noted, as well as limited range of motion.

The nurse in the newborn nursery is preparing to feed a newborn the first feeding of sterile water. During the feeding, the newborn suddenly begins to cough, choke, and become cyanotic. Based on these symptoms, the nurse might suspect that the newborn has which of the following conditions?

1. Atrial septal defect *2. Tracheoesophageal fistula* 3. Bronchopulmonary dysplasia 4. Respiratory distress syndrome *Rationale:* The first feeding a newborn receives is sterile water to assess whether the newborn might have one of the tracheoesophageal (TE) conditions. Although sterile water is more easily absorbed and causes less aspiration than formula, the newborn with a suspected TE fistula condition will cough and choke during feedings. These symptoms are not associated with the conditions noted in options 1, 3, or 4.

A nurse is reviewing a chart of a child with a head injury. The nurse notes that the level of consciousness has been documented as obtunded. Which of the following would the nurse expect to note on data collection of the child?

1. Awake, alert, interacting with the environment 2. The ability to think clearly and rapidly is majorly impaired. 3. The ability to recognize place or person is severely affected. *4. Sleeps unless aroused and, once aroused, interacts poorly with the environment* *Rationale:* Obtunded indicates that the child sleeps unless aroused and, once aroused, has limited interaction with the environment. Full consciousness indicates that the child is alert, awake, orientated, and interacts with the environment. Confusion indicates that the ability to think clearly and rapidly is lost, and disorientation indicates that the ability to recognize a place or person is lost.

A nurse assists in monitoring for early signs of meningitis in a child and assists with attempting to elicit Kernig's sign. The appropriate procedure to elicit Kernig's sign is to:

1. Bend the head toward the knees and hips and check for pain. 2. Tap the facial nerve and check for spasm. 3. Compress the upper arm and check for tetany. *4. Extend the leg and knee and check for pain.* *Rationale:* Kernig's sign is pain that occurs with extension of the leg and knee. Brudzinski's sign occurs when flexion of the head causes flexion of the hips and knees. Chvostek's sign, seen in tetany, is a spasm of the facial muscles elicited by tapping the facial nerve in the region of the parotid gland. Trousseau's sign is a sign for tetany in which carpal spasm can be elicited by compressing the upper arm and causing ischemia to the nerves distally.

A nurse is preparing to care for a child who received an allogenic bone marrow transplant (BMT). The nurse understands that which of the following is the priority concern?

1. Bleeding *2. Infection* 3. Sensory alterations 4. Social isolation *Rationale:* Once the marrow is infused, nursing care focuses on preventing immunocompromised children from developing a life-threatening infection until they engraft and produce their own white blood cells to fight infections. Although options 1, 3, and 4 are considerations in the plan of care, the potential for infection is the priority for a child following a BMT.

A nurse is caring for a newborn with spina bifida (myelomeningocele type) who is scheduled for the removal of the gibbus (sac on the back filled with cerebrospinal fluid, meninges, and some of the spinal cord). In the preoperative period, the priority nursing action is to monitor:

1. Blood pressure *2. Moisture of the normal saline dressing on the gibbous area* 3. Specific gravity of the urine 4. Anterior fontanel for depression *Rationale:* The newborn is at risk for infection before closure of the gibbus. A sterile normal saline dressing is placed over the gibbus to maintain moisture of the gibbus and its contents. This prevents tearing or breakdown of the skin integrity at the site. Blood pressure is difficult to determine during the newborn period and is not the best indicator of infection. Urine concentration is not well developed in the newborn stage of development. Depression of the anterior fontanel is a sign of dehydration. With spina bifida, an increase in intracranial pressure is more of a priority. A complication of spina bifida would demonstrate a bulging or taut anterior fontanel.

A 4-year-old child is diagnosed with otitis media, and the mother asks the nurse about the causes of this illness. The nurse responds, knowing that which of the following is an unassociated risk factor related to otitis media?

1. Bottle-feeding 2. Household smoking 3. A history of urinary tract infections 4. Exposure to illness in other children *Rationale:* Factors that increase the risk of otitis media include exposure to illness, household smoking, bottle-feeding, and congenital conditions such as Down syndrome and cleft palate. The use of a pacifier beyond age 6 months has also been identified as a risk factor. Allergies are also thought to precipitate otitis media. Urinary tract infections are not with a risk factor for otitis media.

A nurse is caring for an infant with a diagnosis of tetralogy of Fallot. The infant suddenly becomes cyanotic and the oxygen saturation reading drops to 60%. Choose the interventions that the nurse should perform. *Select all that apply.*

1. Call a code blue. *2. Notify the registered nurse.* 3. Place the infant in a prone position. *4. Prepare to administer morphine sulfate.* *5. Prepare to administer intravenous fluids.* *6. Prepare to administer 100% oxygen by face mask.* *Rationale:* The child who is cyanotic with oxygen saturations dropping to 60% is having a hypercyanotic episode. Hypercyanotic episodes often occur among infants with tetralogy of Fallot, and they may occur among infants whose heart defect includes the obstruction of pulmonary blood flow and communication between the ventricles. If a hypercyanotic episode occurs, the infant is placed in a knee-chest position immediately. The registered nurse is notified, who will then contact the health care provider. The knee-chest position improves systemic arterial oxygen saturation by decreasing venous return so that smaller amounts of highly saturated blood reach the heart. Toddlers and children squat to get into this position and relieve chronic hypoxia. There is no reason to call a code blue unless respirations cease. Additional interventions include administering 100% oxygen by face mask, morphine sulfate, and intravenous fluids, as prescribed.

A nurse is assigned to care for an infant with tetralogy of Fallot. The mother of the infant calls the nurse to the room because the infant suddenly seems to be having difficulty breathing. The nurse enters the room and notes that the infant is experiencing a hypercyanotic episode. The initial nursing action is to:

1. Call a code. 2. Contact the respiratory therapy department. 3. Place the infant in a prone position. *4. Place the infant in a knee-chest position.* *Rationale:* If a hypercyanotic episode occurs, the infant is placed in a knee-chest position. The nurse would contact the registered nurse, who would then contact the health care provider. The knee-chest position is thought to increase pulmonary blood flow by increasing systemic vascular resistance. This position also improves systemic arterial oxygen saturation by decreasing venous return so that smaller amounts of highly saturated blood reach the heart. Toddlers and children squat to obtain this position and relieve chronic hypoxia.

An adolescent with diabetes mellitus is attending gym class and suddenly becomes flushed and complains of dizziness and a headache. The gym teacher quickly takes the adolescent to the school nurse's office. The nurse obtains a blood glucose level, and the results indicate a level of 65 mg/dL. The appropriate initial nursing intervention is to:

1. Call the child's mother for permission to treat the child. 2. Call the school health care provider immediately. 3. Let the child rest until the blood glucose has an opportunity to rise. *4. Give the child 6 oz of a regular cola drink.* *Rationale:* A blood glucose level below 70 mg/dL indicates hypoglycemia. The child is participating in an activity that requires more energy than that of the normal routine at school. Insulin and food requirements change with situations that require more energy. When signs of hypoglycemia occur, the child needs an immediate source of glucose. Options 1, 2, and 3 do not address the hypoglycemic state immediately and delay required treatment.

A nurse is caring for an infant. A urinalysis has been prescribed, and the nurse plans to collect the specimen. The nurse implements which appropriate method to collect the specimen?

1. Catheterizes the infant, using a No. 5 French Foley *2. Attaches a urinary collection device to the infant's perineum* 3. Obtains the specimen from the diaper, using a syringe, after the infant voids 4. Monitors the urinary patterns and prepares to collect the specimen into a cup when the infant voids *Rationale:* Although many methods have been used to collect urine from an infant, the most reliable method is the urine collection device. This device is a plastic bag that has an opening lined with adhesive so that it may be attached to the perineum. Urine for certain tests, such as specific gravity, may be obtained from a diaper. Urinary catheterization is not to be done unless specifically prescribed because of the risk of infection. It is not reasonable to monitor urinary patterns and attempt to collect the specimen in a cup when the infant voids.

A nurse is monitoring a child with a cast on the forearm for signs of compartment syndrome. The nurse understands that which data collection technique is unlikely to provide information about this complication?

1. Checking the quality of the radial pulse 2. Checking the child's ability to extend the fingers 3. Checking for effectiveness of analgesics administered for pain *4. Checking the child's ability to perform range of motion to the shoulder area of the affected extremity* *Rationale:* Compartment syndrome occurs when swelling causes pressure to rise within a compartment (sheath of inelastic fascia). The increased pressure compromises circulation to the muscles and nerves within the compartment and can result in paralysis and necrosis of tissues. Signs of compartment syndrome include severe pain, often unrelieved by analgesics, and signs of neurovascular impairment. Compartment syndrome is not uncommon in fractures of the forearms; therefore the quality of the radial pulse and the ability to extend the fingers should be assessed. If extension of the fingers produces pain, the health care provider should be notified. Option 4 is unlikely to provide information about compartment syndrome.

A nurse is caring for a newborn with a diagnosis of spina bifida (myelomeningocele). To monitor for a major symptom associated with this disorder, the nurse:

1. Checks the capillary refill on the nail beds of the upper extremities 2. Tests the urine for blood 3. Palpates the abdomen for masses *4. Checks for responses to painful stimuli from the torso downward* *Rationale:* Newborns with spina bifida (myelomeningocele type) demonstrate lack of nerve innervation from below the site of the gibbus (sac containing the meninges and spinal cord). They therefore show diminished or no responses to painful stimuli in the areas below the gibbus. Options 1, 2, and 3 are incorrect because the area above the gibbus is not affected. The capillary refill would be normal. The urine would not have blood present. If the kidneys are affected, proteinuria could be present but this is not generally noted in the newborn period. No masses are present besides the gibbus on the back area, externally protruding from the vertebral deformity.

The health care provider prescribes laboratory studies for an infant of a woman positive for human immunodeficiency virus (HIV) to determine the presence of HIV antigen in the infant. The nurse anticipates that which laboratory study will be prescribed for the infant?

1. Chest x-ray 2. Western blot 3. CD4+ cell count *4. p24 antigen assay* *Rationale:* The detection of HIV in infants is confirmed by a p24 antigen assay, virus culture of HIV, or polymerase chain reaction. A chest x-ray evaluates the presence of other manifestations of HIV infection, such as pneumonia. A Western blot test confirms the presence of HIV antibodies. The CD4+ cell count indicates how well the immune system is working.

A mother brings her 4-month-old infant to the well-baby clinic for immunizations. Which immunizations should be administered to this infant?

1. DTaP (diphtheria, tetanus, acellular pertussis), MMR (measles, mumps, rubella), IPV (inactivated poliovirus vaccine) 2. MMR, Hib (Haemophilus influenzae type b), DTaP *3. DTaP, Hib, IPV, pneumococcal vaccine (PCV)* 4. Varicella and hepatitis B vaccines *Rationale:* DTaP, Hib, IPV, and PCV are administered at 4 months of age. DTaP is administered at 2 months, 4 months, 6 months, between 12 and 18 months, and between 4 and 6 years of age. Hib is administered at 2 months, 4 months, 6 months, and between 12 and 15 months of age. IPV is administered at 2 months, 4 months, 6 months, and between 4 and 6 years of age. The first dose of MMR is administered between 12 and 15 months of age; the second dose is administered at 4 to 6 years of age (if the second dose was not given by 4 to 6 years of age, it should be given at the next visit). The first dose of hepatitis B vaccine is administered between birth and 2 months, the second dose is administered between 1 and 4 months, and the third dose is administered between 6 and 18 months of age. Varicella zoster vaccine is administered between 12 and 18 months of age. PCV is administered at 2, 4, and 6 months of age and between 12 and 15 months of age.

A nurse is preparing to care for a child with a head injury. On review of the records, the nurse notes that the health care provider has documented decorticate posturing. The nurse plans care, knowing that this type of posturing indicates which of the following?

1. Damage to the midbrain 2. Dysfunction of the pons 3. Damage to the diencephalon *4. Dysfunction in the cerebral hemisphere* *Rationale:* Decorticate posturing indicates a lesion in the cerebral hemisphere or disruption of the corticospinal tracts. Decerebrate posturing indicates damage in the diencephalon, midbrain, or pons.

A nurse gives fentanyl 50 mcg IV for a patient in labor. Which fetal heart rate pattern should the nurse anticipate? 1. Decreased variability 2. Variable decelerations 3. Late decelerations 4. Early decelerations

1. Decreased variability Rationale: Fentanyl may cause decreased fluctuations in the baseline fetal heart rate; Fentanyl crosses the placenta and is a CNS depressant. Early decelerations have an onset before the peak of the contraction and result from fetal head compression.

A nurse is developing goals for a school-age child with a knowledge deficit related to the use of inhalers and peak flowmeters. The nurse identifies which of the following as an appropriate goal for this child?

1. Denies shortness of breath or difficulty breathing 2. Has regular respirations at a rate of 18 to 22 breaths per minute *3. Expresses feelings of mastery and competence with breathing devices* 4. Watches the educational video and reads printed information provided *Rationale:* School-age children strive for mastery and competence to achieve the developmental task of industry and accomplishment. Options 1 and 2 do not relate to the knowledge deficit, which is the subject of the question. Option 4 is an intervention rather than a goal.

A nurse is assisting a health care provider (HCP) during the examination of an infant with hip dysplasia. The HCP performs the Ortolani maneuver. Which of the following best describes the action/purpose of the Ortolani maneuver?

1. Determining the extent of range of motion 2. Checking for asymmetry on the affected side 3. Pushing the unstable femoral head out of the acetabulum *4. Reducing the dislocated femoral head back into the acetabulum* *Rationale:* With the Ortolani maneuver, the examiner reduces the dislocated femoral head back into the acetabulum. A positive Ortolani maneuver is a palpable clunk as the femoral head moves over the acetabular ring. Options 1 and 2 are data collection techniques for the identification of the clinical manifestations of hip dysplasia, but they do not describe the Ortolani maneuver. When performing the Barlow maneuver, the examiner pushes the unstable femoral head out of the acetabulum.

An emergency department nurse is caring for a child brought to the emergency department following the ingestion of approximately one half bottle of acetylsalicylic acid (aspirin). The nurse anticipates that the likely initial treatment will be:

1. Dialysis 2. The administration of vitamin K *3. The administration of activated charcoal* 4. The administration of sodium bicarbonate *Rationale:* Initial treatment of salicylate overdose includes administration of activated charcoal to decrease absorption of the aspirin. Intravenous (IV) fluids and inducing emesis may be prescribed to enhance excretion but would not be the initial treatment. Dialysis is used in extreme cases if the child is unresponsive to therapy. Vitamin K is the antidote for warfarin (Coumadin) overdose.

A nurse reviews the record of a 3-week-old infant and notes that the health care provider has documented a diagnosis of suspected Hirschsprung's disease. The nurse understands that which of the following symptoms led the mother to seek health care for the infant?

1. Diarrhea 2. Projectile vomiting 3. The regurgitation of feedings *4. Foul-smelling, ribbon-like stools* *Rationale:* Chronic constipation that begins during the first month of life and that results in foul-smelling, ribbon-like or pellet-like stools is a clinical manifestation of Hirschsprung's disease. The delayed passage or absence of meconium stool during the neonatal period is a characteristic sign. Bowel obstruction (especially during the neonatal period), abdominal pain and distention, and failure to thrive are also clinical manifestations. Options 1, 2, and 3 are incorrect.

A child is scheduled for a tonsillectomy. Which of the following would present the highest risk of aspiration during surgery?

1. Difficulty swallowing 2. Bleeding during surgery 3. Exudate in the throat area *4. The presence of loose teeth* *Rationale:* In the preoperative period, the child should be observed for the presence of loose teeth to decrease the risk of aspiration during surgery. Options 1 and 3 are incorrect. Bleeding during surgery will be controlled via packing and suction as needed.

A nursing student is assigned to care for a child with hemophilia. The nursing instructor reviews the plan of care with the student and asks the student to describe the characteristics of this disorder. Which statement by the student indicates a need for further research?

1. Hemophilia is inherited in a recessive manner via a genetic defect on the X chromosome. *2. Males inherit hemophilia from their fathers.* 3. Females inherit the carrier status from their fathers. 4. Hemophilia A results from deficiency of factor VIII. *Rationale:* Males inherit hemophilia from their mothers, and females inherit the carrier status from their fathers. Some females who are carriers have an increased tendency to bleed, and, although it is rare, females can have hemophilia if their fathers have the disorder and their mothers are carriers of the genetic disorder. Hemophilia is inherited in a recessive manner via a genetic defect on the X chromosome. Hemophilia A results from a deficiency of factor VIII. Hemophilia B (Christmas disease) is a deficiency of factor IX.

Which of the following assessment findings may indicate that a child had a tonic-clonic seizure during the night?

1. High-pitched cry 2. Blanched toenails *3. Blood on the pillow* 4. Migraine headaches *Rationale:* The complications associated with seizures include airway compromise, extremity and teeth injuries, and tongue lacerations. Night seizures can cause the child to bite down on the tongue. Seizures do not cause a high-pitched cry unless a tumor or intracranial pressure is the cause of the seizure diagnosis. Cyanosis can occur during the tonic-clonic part of the seizure activity, but blanching does not occur. Migraine headaches are not common in children with seizures.

A health care provider has told the mother of a newborn diagnosed with strabismus that surgery will be necessary to realign the weakened eye muscles. The mother asks the nurse when the surgery might be performed. The nurse responds by telling the mother that surgery will probably be performed:

1. Immediately *2. Before the child is 3 years old* 3. Shortly before the child starts school 4. Just before the child begins to learn to read *Rationale:* In a child diagnosed with strabismus, surgery may be indicated to realign the weakened muscles. It is most often indicated when amblyopia (decreased vision in the deviated eye) is present. The surgery should be performed before the child is 3 years old.

A nurse is caring for a child with a suspected diagnosis of rheumatic fever (RF). The nurse reviews the laboratory results. Which laboratory study would assist in confirming the diagnosis of RF?

1. Immunoglobulin 2. Red blood cell count *3. Antistreptolysin O titer* 4. White blood cell count *Rationale:* A diagnosis of RF is confirmed by the presence of two major manifestations or one major and two minor manifestations from the Jones criteria. In addition, evidence of a recent streptococcal infection is confirmed by a positive antistreptolysin O titer, streptozyme, or an anti-DNase B assay. Options 1, 2, and 4 will not assist in confirming the diagnosis of RF.

A nurse is developing a plan of care for a child with autism. The nurse identifies which of the following as the priority problem for this child?

1. Inability to interact socially *2. Risk for injury* 3. Troubling thought processes 4. Inability to verbally communicate *Rationale:* Risk for injury related to an inability to anticipate danger, a tendency for self-mutilation, and sensory perceptual deficits is the priority concern. Inability to interact socially, troubling thought processes, and inability to verbally communicate are also appropriate problems for the child with autism, but the priority is the risk for injury.

A nurse reviews the record of an infant who is seen in the clinic. The nurse notes that a diagnosis of esophageal atresia with tracheoesophageal fistula (TEF) is suspected. The nurse expects to note which most likely clinical manifestation of this condition in the medical record?

1. Incessant crying 2. Coughing at nighttime *3. Choking with feedings* 4. Severe projectile vomiting *Rationale:* Any child who exhibits the "3 Cs"—coughing and choking during feedings and unexplained cyanosis—should be suspected of having TEF. Options 1, 2, and 4 are not specifically associated with TEF.

A mother of a child being treated for right lower lobe pneumonia at home calls the clinic nurse. The mother tells the nurse that the child has discomfort on the right side and that the acetaminophen (Tylenol) is not very effective. The appropriate suggestion by the nurse would be to:

1. Increase the dose of the acetaminophen. 2. Encourage the child to lie on the left side. *3. Encourage the child to lie on the right side.* 4. Increase the frequency of the acetaminophen. *Rationale:* Splinting of the affected side by lying on that side may decrease discomfort. It is inappropriate to advise the mother to increase the dose or frequency of the acetaminophen. Lying on the left side will not be helpful in alleviating discomfort.

A child suspected of sickle cell disease is seen in the clinic, and laboratory studies are performed. The nurse reviews the results of the laboratory studies and expects to note which of the following that is a characteristic of this disease?

1. Increased hematocrit count 2. Increased platelet count *3. Increased reticulocyte count* 4. Increased hemoglobin count *Rationale:* A laboratory diagnosis is established on the basis of a complete blood cell count, examination for sickled red blood cells (RBCs) on the peripheral smear, and hemoglobin electrophoresis. Laboratory studies will show decreased hemoglobin, hematocrit, and platelet count, increased reticulocyte count, and the presence of nucleated red blood cells. Elevated reticulocyte counts occur in children with sickle cell disease because the life span of their sickled RBCs is shortened.

A child is admitted to the pediatric unit with a diagnosis of coarctation of the aorta (COA). The health care provider prescribes that the child's blood pressure be taken every 4 hours in the legs and arms. The nurse would expect the blood pressure in the child's legs and arms to be:

1. Increased in the arms and the legs 2. Decreased in the arms and the legs *3. Decreased in the legs and increased in the arms* 4. Increased in the legs and decreased in the arms *Rationale:* Coarctation indicates a narrowing in the aorta. This would indicate an increased pressure proximal to the defect and a decreased pressure distal to the defect. This would result in a lower blood pressure in the legs and a higher blood pressure in the arms, which is indicated in option 3. Therefore options 1, 2, and 4 are incorrect.

A 4-year-old child is being transported to the trauma center from a local community hospital for treatment of a burn injury that is estimated as covering over 40% of the body. The burns are partial- and full-thickness burns. The nurse is asked to prepare for the arrival of the child and gathers supplies anticipating that which of the following will be prescribed initially?

1. Insertion of a nasogastric tube *2. Insertion of a Foley catheter* 3. Administration of an anesthetic agent for sedation 4. Application of an antimicrobial agent to the burns *Rationale:* A Foley catheter is inserted into the child's bladder so that urine output can be measured accurately on an hourly basis. Although pain medication may be required, the child would not receive an anesthetic agent and should not be sedated. The burn wounds would be cleansed and treated after assessment, but this would not be the initial action. Intravenous fluids are administered at a rate sufficient to keep the child's urine output at 1 mL/kg of body weight per hour, thus reflecting adequate tissue perfusion. A nasogastric tube may or may not be required but would not be the priority intervention.

A mother brings her child to the clinic because the child has developed a rash on the trunk and scalp. The child is diagnosed with varicella. The mother inquires about the infectious period associated with varicella, and the nurse tells the mother that the infectious period:

1. Is unknown *2. Is 1 to 2 days before the onset of the rash to 5 days after the onset of lesions and the crusting of lesions* 3. Is 10 days before the onset of symptoms to 15 days after the rash appears 4. Ranges from 2 weeks or less up to several months *Rationale:* Varicella is known as chickenpox. The infectious period for varicella is 1 to 2 days before the onset of the rash to 5 days after the onset of lesions and the crusting of lesions. In roseola, the infectious period is unknown. Option 3 describes rubella. Option 4 describes diphtheria.

A nurse is caring for a child with a diagnosis of Kawasaki disease. The mother of the child asks the nurse about the disorder. Which statement most accurately describes Kawasaki disease?

1. It is an acquired cell-mediated immunodeficiency disorder. *2. It is also called mucocutaneous lymph node syndrome and is a febrile generalized vasculitis of unknown cause.* 3. It is a chronic multi-system autoimmune disease characterized by the inflammation of connective tissue. 4. It is an inflammatory autoimmune disease that affects the connective tissue of the heart, joints, and subcutaneous tissues. *Rationale:* Kawasaki disease, also called mucocutaneous lymph node syndrome, is a febrile generalized vasculitis of unknown etiology. Option 1 describes human immunodeficiency virus (HIV) infection. Option 3 describes systemic lupus erythematosus. Option 4 describes rheumatic fever.

A child with cerebral palsy (CP) is working to achieve maximum potential for locomotion, self-care, and socialization in school. The nurse would work with the child to meet these goals by:

1. Keeping the child in a special education classroom with other children with similar disabilities 2. Laying the child in the supine position with a 30-degree elevation of the head to facilitate feeding 3. Removing ankle-foot orthoses and braces once the child arrives at school *4. Placing the child on a wheeled scooter board* *Rationale:* Option 4 provides the child with maximum potential in locomotion, self-care, and socialization. The child can move around independently on the abdomen anywhere the child wants to go and can interact with others as desired. Orthoses must be used all the time to aid locomotion (option 3). Option 1 does not provide for maximum socialization and normalization; rather, children with CP need to be mainstreamed as much as cognitively able. Not all children with CP are intellectually challenged. Option 2 does not provide for normalization in self-care. Just as children without CP sit up and use assistive devices when eating, so should children with CP.

A nurse is assigned to care for a child who is in skeletal traction. The nurse avoids which of the following when caring for the child?

1. Keeping the weights hanging freely 2. Ensuring that the ropes are in the pulleys *3. Placing the bed linens on the traction ropes* 4. Ensuring that the weights are out of the child's reach *Rationale:* Bed linens should not be placed on the traction ropes because of the risk of disrupting the traction apparatus. Options 1, 2, and 4 are appropriate measures when caring for a child who is in skeletal traction.

An 18-month-old child is being discharged after surgical repair of hypospadias. Which postoperative nursing care measure should the nurse stress to the parents as they prepare to take this child home?

1. Leave diapers off to allow the site to heal. *2. Avoid tub baths until the stent has been removed.* 3. Encourage toilet training to ensure that the flow of urine is normal. 4. Restrict the fluid intake to reduce urinary output for the first few days. *Rationale:* After hypospadias repair, the parents are instructed to avoid giving the child a tub bath until the stent has been removed to prevent infection. Diapers are placed on the child to prevent the contamination of the surgical site. Toilet training should not be an issue during this stressful period. Fluids should be encouraged to maintain hydration.

An infant is seen in a clinic and is diagnosed with unilateral hip dysplasia. Which finding is associated with this condition?

1. Limited range of motion in the unaffected hip 2. An apparent short femur on the unaffected side 3. Adduction of the affected hip when placed supine with the knees and hips flexed *4. Asymmetry of the gluteal skin folds when the infant is placed prone and the legs are extended against the examining table* *Rationale:* Asymmetry of the gluteal folds when the infant is placed prone would be a finding in hip dysplasia in infants beyond the newborn period. Options 1, 2, and 3 are inaccurate assessment findings in this disorder.

A child with leukemia is experiencing nausea related to medication therapy. The nurse, concerned about the child's nutritional status, should offer which of the following during this episode of nausea?

1. Low-calorie foods *2. Cool, clear liquids* 3. Low-protein foods 4. The child's favorite foods *Rationale:* When the child is nauseated, it is best to offer frequent intake of cool, clear liquids in small amounts because small portions are usually better tolerated. Cool, clear fluids are also soothing and better tolerated when a client is nauseated. It is best not to offer favorite foods when the child is nauseated because foods eaten during times of nausea will be associated with being sick. It is best to offer small, frequent meals of high-protein and high-calorie content once the nausea has been controlled with medication or has subsided.

A nurse is caring for a hospitalized infant and is monitoring for increased intracranial pressure (ICP). The nurse notes that the anterior fontanel bulges when the infant cries. Based on this finding, which of the following actions would the nurse take?

1. Lower the head of the bed. *2. Document the findings.* 3. Place the infant on nothing-by-mouth (NPO) status. 4. Ask the registered nurse to notify the health care provider immediately. *Rationale:* The anterior fontanel is diamond shaped and located on the top of the head. It should be soft and flat in a normal infant, and it normally closes by 18 to 24 months of age. The posterior fontanel closes by 2 to 3 months of age. A bulging or tense fontanel may result from crying or increased ICP. If the nurse notes a bulging fontanel when the infant cries, this is a normal finding that should be documented and monitored. It is not necessary to notify the health care provider for this finding. Options 1 and 3 are inappropriate actions.

A nurse is reinforcing instructions to the mother of a preschool child who was recently diagnosed with pediculosis capitis (head lice). Which item should be included in discussions to prevent a reinfestation?

1. Machine wash all of the child's clothing, towels, and bed linens, and place in a warm dryer for at least 20 minutes. 2. Shave the child's hair if pediculicide and nit-removal combs prove ineffective. 3. Spray the home's furniture and beds with insecticide. *4. Seal nonwashable items in a plastic bag for 2 to 3 weeks in a warm place if they cannot be vacuumed or dry cleaned.* *Rationale:* The adult louse can survive up to 48 hours away from a host, although nits can hatch in 7 to 10 days if they are shed into the environment. Thus, 2 weeks represents a safe interval of time that prevents reinfestation from occurring. Hot water and hot air should be used in the washer and dryer. Shaving the hair is unnecessary with proper treatment and would have an adverse psychological effect on the child. Insecticides can endanger children and animals and should not be sprayed on furniture and beds.

A child is diagnosed with infectious mononucleosis. The nurse provides home-care instructions to the parents about the care of the child. Which information given by the nurse is accurate?

1. Maintain the child on bedrest for 2 weeks. 2. Maintain respiratory precautions for 1 week. 3. Notify the health care provider if the child develops a fever. *4. Notify the HCP if the child develops abdominal or left shoulder pain.* *Rationale:* The parents need to be instructed to notify the HCP if abdominal pain (especially in the left upper quadrant) or it left shoulder pain occurs, because this may indicate splenic rupture. Children with enlarged spleens are also instructed to avoid contact sports until the splenomegaly resolves. Bedrest is not necessary, and children usually self-limit their activity. Respiratory precautions are not required, although transmission can occur via direct intimate contact or contact with infected blood. Fever is treated with acetaminophen (Tylenol).

A 3-year-old child is brought to the emergency department. The mother states that the child has had flulike symptoms with vomiting and diarrhea for the past 2 days. On data collection the nurse finds that the child's heart rate is slightly elevated and the blood pressure is normal. The child is irritable and crying only a few tears. The mother states that the child's weight before the illness was 33 pounds. The nurse finds the current weight to be 31 pounds. The nurse correctly interprets this as what level of dehydration?

1. Mild dehydration 2. Severe dehydration 3. Very mild dehydration *4. Moderate dehydration* *Rationale:* Moderate dehydration demonstrates itself with a weight loss in children of 6% to 8% of weight. Mild dehydration would not present with these symptoms. In severe dehydration, additional findings would include lethargy and listlessness. The symptoms listed are all characteristics of moderate dehydration. Very mild dehydration is not a term used to describe dehydration.

A 6-month-old infant receives a diphtheria, tetanus, and acellular pertussis (DTaP) immunization at the well-baby clinic. The mother returns home and calls the clinic to report that the infant has developed swelling and redness at the site of injection. Which instruction by the nurse is appropriate?

1. Monitor the infant for a fever. 2. Bring the infant back to the clinic. *3. Apply an ice pack to the injection site.* 4. Leave the injection site alone, because this always occurs. *Rationale:* Occasionally tenderness, redness, or swelling may occur at the site of the injection. This can be relieved with cool packs for the first 24 hours and followed by warm or cool compresses if the inflammation persists. It is not necessary to bring the infant back to the clinic. Option 1 may be an appropriate intervention, but it is not specific to the subject of the question.

Following tonsillectomy, which of the health care provider's prescriptions would the nurse question?

1. Monitor vital signs. 2. Monitor for bleeding. *3. Allow ice cream when awake.* 4. Offer clear, cool liquids when awake. *Rationale:* Clear, cool liquids are encouraged. Milk and milk products are avoided initially because they coat the throat, which causes the child to clear the throat, increasing the risk of bleeding. Options 1 and 2 are important nursing interventions following any type of surgery.

The nurse assists with preparing a nursing care plan for a child who has Reye's syndrome. Which of the following is the priority nursing intervention?

1. Monitoring the output 2. Checking for hearing loss 3. Changing the body position every 2 hours *4. Providing a quiet atmosphere with dimmed lights* *Rationale:* The major elements of care for a child who has Reye's syndrome are to maintain effective cerebral perfusion and to control intracranial pressure. Decreasing stimuli in the environment would decrease the stress on the cerebral tissue and the neuron responses. Cerebral edema is a progressive part of this disease process. Hearing loss and output are not affected. Changing the body position every 2 hours would not directly affect the cerebral edema and intracranial pressure. The child should be in a head-elevated position to decrease the progression of the cerebral edema and to promote the drainage of cerebrospinal fluid.

A nursing student is asked to discuss human immunodeficiency virus (HIV) during clinical conference. The nursing student includes which correct item in the discussion?

1. Most newborns of HIV-positive women test positive for HIV virus. 2. HIV primarily attacks the hematological system. 3. In HIV, the B cells are depleted and cannot signal T4 cells to form protective antibodies. *4. HIV virus attacks the immune system by destroying T lymphocytes.* *Rationale:* Children born to HIV-positive women test positive for HIV antibody, not HIV virus. This is actually a measure of maternal antibody and not indicative of true infection. T4 cells are depleted in number and cannot signal B cells to form protective antibodies to fight off the invading virus. The virus attacks the immune system by destroying T lymphocytes.

A 6-year-old child with leukemia is hospitalized and is receiving combination chemotherapy. Laboratory results indicate that the child is neutropenic, and the nurse prepares to implement protective isolation procedures. Which interventions would the nurse initiate? *Select all that apply.*

1. Restrict all visitors. *2. Place the child on a low-bacteria diet.* *3. Change dressings using sterile technique.* 4. Encourage the consumption of fresh fruits and vegetables. *5. Perform meticulous handwashing before caring for the child.* 6. Allow fresh-cut flowers in the room as long as they are kept in a vase with fresh water. *Rationale:* For the hospitalized neutropenic child, flowers or plants should not be kept in the room because standing water and damp soil harbor Aspergillus and Pseudomonas, to which these children are very susceptible. Fruits and vegetables not peeled before being eaten harbor molds and should be avoided until the white blood cell count rises. The child is placed on a low-bacteria diet. Dressings are always changed with sterile technique. Not all visitors need to be restricted, but anyone who is ill should not be allowed in the child's room. Meticulous handwashing is required before caring for the child. In addition, gloves, a mask, and a gown are worn (per agency policy).

A child has a basilar skull fracture. Which of the following health care provider's prescriptions should the nurse question?

1. Restrict fluid intake. 2. Insert an indwelling urinary catheter. 3. Keep an intravenous (IV) line patent. *4. Suction via the nasotracheal route as needed.* *Rationale:* Nasotracheal suctioning is contraindicated in a child with a basilar skull fracture. Because of the location of the injury, the suction catheter may be introduced into the brain. Fluids are restricted to prevent fluid overload. The child may require a urinary catheter for the accurate monitoring of intake and output. An IV line is maintained to administer fluids or medications, if necessary.

A nurse is assisting a health care provider with an assessment of a child with a diagnosis of suspected appendicitis. In assessing the intensity and progression of the pain, the health care provider palpates the child at McBurney's point. The nurse understands that McBurney's point is located midway between the:

1. Right anterior inferior iliac crest and the umbilicus 2. Left anterior superior iliac crest and the umbilicus *3. Right anterior superior iliac crest and the umbilicus* 4. Left anterior superior iliac crest and the umbilicus *Rationale:* McBurney's point is midway between the right anterior superior iliac crest and the umbilicus. It is usually the location of greatest pain in the child with appendicitis. Options 1, 2, and 4 are incorrect.

The school nurse notes that the child has a rash and suspects that it is caused by erythema infectiosum (fifth disease). The nurse bases this determination on the observation that the rash results in:

1. Rose-pink maculopapulars 2. Pruritic macule-to-papules 3. Pinkish red maculopapulars *4. A "slapped-face" appearance* *Rationale:* The classic rash of erythema infectiosum or fifth disease is the erythema on the face. The discrete rose-pink maculopapular rash is the rash of exanthema subitum (roseola). The highly pruritic profuse macule-to-papule rash is the rash of varicella (chickenpox). The discrete pinkish red maculopapular rash is the rash of rubella (German measles).

A mother brings her 15-month-old child to the health care provider's office with complaints that the child has suddenly developed a bright red rash on her cheeks. She has no other symptoms and has been playing and eating as usual. Based on the appearance of the child, the nurse might suspect that the child has:

1. Rubella 2. Roseola *3. Fifth disease* 4. Chickenpox *Rationale:* Fifth disease has the general appearance of "slapped cheeks." Many children do not have any symptoms prior to the appearance of the reddened cheeks. This characteristic is not associated with the communicable diseases identified in options 1, 2, or 4.

A nursing student is asked to discuss sudden infant death syndrome (SIDS) at the clinical conference being held at the end of the clinical day. The student plans to include which of the following in the discussion during the conference?

1. SIDS usually occurs during sleep and is more common in girls. *2. SIDS usually occurs during sleep and is more common in premature infants.* 3. SIDS usually occurs during sleep and is more common in high-birth-weight infants. 4. SIDS usually occurs during sleep and most frequently occurs between 8 and 10 months of age. *Rationale:* SIDS usually occurs during sleep. It most frequently occurs between the second and fourth months of life. It is more common in boys, low-birth-weight infants, and premature infants.

The nurse assists in planning care for a child who sustained a burn injury based on which of the following accurate statements?

1. Scarring is not as severe in a child as in an adult. 2. Children are at a lower risk of infection than adults because of their strong immune systems. *3. Lower burn temperatures and shorter exposure to heat can cause a more severe burn in a child than an adult because a child's skin is thinner.* 4. Infants and children are at decreased risk for protein and calorie deficiency because they have smaller muscle mass and less body fat than adults. *Rationale:* Lower burn temperatures and shorter exposure to heat can cause a more severe burn in a child than an adult because a child's skin is thinner. Scarring is more severe in a child; additionally, disturbed body image will be a distinct issue for a child or adolescent, especially as growth continues. An immature immune system presents an increased risk of infection for infants and young children. Infants and children are at increased risk for protein and calorie deficiency because they have smaller muscle mass and less body fat than adults.

A nurse is reinforcing home care instructions to the mother of a child with hemophilia. Which activity should the nurse suggest that the child can safely participate in with peers?

1. Soccer 2. Basketball *3. Swimming* 4. Field hockey *Rationale:* Children with hemophilia need to avoid contact sports and need to take precautions, such as wearing elbow and knee pads and helmets, when participating in other sports. The safest activity that will prevent injury is swimming.

A nurse working in the day care center is told that a child with autism will be attending the center. The nurse collaborates with the staff of the day care center and assists in planning activities that will meet the child's needs. The nurse understands that the priority consideration in planning activities for the child is to ensure:

1. Social interactions with other children in the same age group *2. Safety with activities* 3. Familiarity with all activities and providing orientation throughout the activities 4. Activities providing verbal stimulation *Rationale:* Safety with all activities is a priority in planning activities with the child. The child with autism is unable to anticipate danger, has a tendency for self-mutilation, and has sensory-perceptual deficits. Although providing social interactions, verbal communications, and familiarity and orientation are also appropriate interventions, the priority is safety.

A licensed practical nurse (LPN) is assigned to assist in caring for a hospitalized child who is receiving a continuous infusion of intravenous (IV) potassium for the treatment of dehydration. The LPN monitors the child closely and notifies the registered nurse if which of the following is noted?

1. Temperature of 100.8° F rectally 2. Weight increase of 0.5 kg *3. A decrease in urine output to 0.5 mL/kg/hr* 4. Blood pressure (BP) unchanged from baseline *Rationale:* The priority assessment is to monitor the status of urine output. Potassium should never be administered in the presence of oliguria or anuria. If urine output is less than 1 to 2 mL/kg/hr, potassium should not be administered. A slight elevation in temperature would be expected in a child with dehydration. A weight increase of 0.5 kg is relatively insignificant. A BP that is unchanged is a positive indicator unless the baseline was abnormal. However, there is no information in the question to support such data.

A nurse is collecting data about a child who has been admitted to the hospital with a diagnosis of seizures. The nurse checks for causes of the seizure activity by:

1. Testing the child's urine for specific gravity 2. Asking the child what happens during a seizure 3. Obtaining a family history of psychiatric illness *4. Obtaining a history regarding factors that may occur before the seizure activity* *Rationale:* Fever and infections increase the body's metabolic rate. This can cause seizure activity among children who are less than 5-years-old. Dehydration and electrolyte imbalance can also contribute to the occurrence of a seizure. Falls can cause head injuries, which would increase intracranial pressure or cerebral edema. Some medications could cause seizures. Specific gravity would not be a reliable test, because it varies, depending on the existing condition. Psychiatric illness has no impact on seizure occurrence or cause. Children do not remember what happened during the seizure itself.

A nurse is providing home care instructions to the mother of a child with bacterial conjunctivitis. The nurse should tell the mother:

1. That the child may attend school if antibiotics have been started 2. To save any unused eye medication in case a sibling gets the eye infection *3. That the child's towels and washcloths should not be used by other members of the household* 4. To wipe any crusted material from the eye with a cotton ball soaked in warm water, starting at the outer aspect of the eye and moving toward the inner aspect *Rationale:* Bacterial conjunctivitis is highly contagious, and infection control measures should be taught. These include good handwashing and not sharing towels and washcloths with others. The child should be kept home from school until 24 hours after antibiotics are started. Bottles of eye medication should never be shared with others. Crusted material may be wiped from the eye with a cotton ball soaked in warm water, starting at the inner aspect of the eye and moving toward the outer aspect.

A nurse is reviewing the health record of a child who has been recently diagnosed with glomerulonephritis. Which finding noted in the child's record is associated with the diagnosis of glomerulonephritis?

1. The child fell off a bike and onto the handlebars. 2. The child has had nausea and vomiting for the last 24 hours. 3. The child had urticaria and itching for 1 week before diagnosis. *4. The child had a streptococcal throat infection 2 weeks before diagnosis.* *Rationale:* Group A β-hemolytic streptococcal infection is a cause of glomerulonephritis. The child often becomes ill with streptococcal infection of the upper respiratory tract and then develops symptoms of acute poststreptococcal glomerulonephritis after an interval of 1 to 2 weeks. The data presented in options 1, 2, and 3 are unrelated to a diagnosis of glomerulonephritis.

A day care nurse is observing a 2-year-old child and suspects that the child may have strabismus. Which of the following observations may be indicative of this condition?

1. The child has difficulty hearing. 2. The child does not respond when spoken to. *3. The child consistently tilts his or her head to see.* 4. The child consistently turns his or her head to see. *Rationale:* The nurse may suspect strabismus in a child when the child complains of frequent headaches, squints, or tilts the head to see. Options 1, 2, and 4 are not indicative of this condition.

A 1-year-old child is seen in the health care provider's office with complaints of an elevated temperature that began the previous evening. When gathering subjective data from the mother, the nurse notices that which of the following would most likely indicate the child has acute otitis media?

1. The child is crying and irritable. 2. The temperature is 40° C (104° F). 3. The child is pulling at her ear and rolling her head from side to side. *4. The mother states the child had purulent discharge from the ear last night.* *Rationale:* Subjective data are what the mother tells the nurse. Therefore option 4 is correct because the mother is describing the child's ear drainage that occurred last night. The other options are considered objective data, which are observations that the nurse makes.

A child is brought to a clinic after developing a rash on the trunk and on the scalp. The parents report that the child has had a low-grade fever, has not felt like eating, and has been generally tired. The child is diagnosed with chickenpox. Which statement by the nurse is accurate regarding chickenpox?

1. The communicable period is unknown. 2. The communicable period ranges from 2 weeks or less up to several months. 3. The communicable period is 10 days before the onset of symptoms to 15 days after the rash appears. *4. The communicable period is 1 to 2 days before the onset of the rash to 6 days after the onset and crusting of lesions.* *Rationale:* The communicable period for chickenpox is 1 to 2 days before the onset of the rash to 6 days after the onset and crusting of lesions. In roseola the communicable period is unknown. Option 2 describes diphtheria. Option 3 describes rubella.

A nurse is assigned to care for a child with a compound (open) fracture of the arm that occurred as a result of a fall. The nurse plans care, knowing that this type of fracture involves:

1. The entire bone fractured straight across *2. A greater risk of infection than a simple fracture* 3. One side of the bone being broken and the other side being bent 4. The bone being fractured but not producing a break in the skin *Rationale:* In a compound (open) fracture, a wound in the skin leads to the broken bone, and there is an added danger of infection. Option 1 describes a transverse fracture. Option 3 describes a greenstick fracture. Option 4 describes a closed or simple fracture.

In a discussion about an 11-month-old patient's diet, which statement by her mother indicates a possible cause for iron-deficiency anemia? 1.) "Formula is so expensive. We switched to regular milk early on." 2.) "She almost never drinks water." 3.) "She doesn't really like peaches or pears, so we stick to bananas for fruit." 4.) "I give her a piece of bread now and then. She like to chew on it."

1.) "Formula is so expensive. We switched to regular milk early on." Pg. 983 -Foundations of Nursing

A lethargic, pale child is brought to the health care provider's office with symptoms of periorbital edema and reduced quantity of urine output. The urine is cloudy and smoky in color. The nurse asks the mother if the child has had any recent infections, to which the mother responds that the child had a very sore throat a few weeks ago. The health care provider suspects that the child might have acute poststreptococcal glomerulonephritis. Which of the following laboratory tests would rule out a past streptococcal infection in the child?

1. Urinalysis 2. Throat culture *3. Antistreptolysin titer* 4. Creatinine clearance *Rationale:* Option 3 is the only laboratory test that will determine if a streptococcal infection was present. The other options do not relate to a past streptococcal infection. Option 1 will determine if protein is present in the urine, which is present in glomerulonephritis. Option 2 will determine if a current throat infection is present. Option 4 will determine glomerular filtration rate.

An adolescent is admitted to the hospital with complaints of lower right abdominal pain. The health care provider prescribes laboratory tests to rule out ectopic pregnancy rather than appendicitis. Which of the following is most significant in ruling out an ectopic pregnancy?

1. Urinalysis 2. White blood count 3. C-reactive protein *4. Serum human chorionic gonadotropin* *Rationale:* The test to rule out an ectopic pregnancy is the serum human chorionic gonadotropin. The other tests may be prescribed to rule out appendicitis, but because the client is an adolescent it would be necessary to rule out an ectopic pregnancy as well. Urinalysis will rule out a urinary tract infection, and the white blood count and the C-reactive protein will rule out some other types of infection.

A nurse is providing home care instructions to the mother of an infant who has just been found to have hemophilia. The nurse tells the mother to:

1. Use aspirin for pain relief. *2. Pad crib rails and table corners.* 3. Use a soft toothbrush for dental hygiene. 4. Use a generous amount of lubricant when taking a temperature rectally. *Rationale:* Establishment of an age-appropriate safe environment is of paramount importance for hemophiliac clients. Providing a safe environment for an infant includes padding table corners and crib rails, providing extra "joint" padding on clothes, observing a mobile infant at all times, and keeping items that can be pulled down onto the infant out of reach. Use of a soft toothbrush is an appropriate measure for a child with hemophilia, but is not typically necessary for an infant. Rectal temperature measurements and the use of aspirin are contraindicated in hemophiliac individuals because of the risk of bleeding.

A nurse prepares a list of home care instructions for the parents of a child who has a plaster cast applied to the left forearm. Choose the instructions that would be included on the list. *Select all that apply.*

1. Use the fingertips to lift the cast while it is drying. *2. Keep small toys and sharp objects away from the cast.* 3. Use a padded ruler or another padded object to scratch the skin under the cast if it itches. 4. Place a heating pad on the lower end of the cast and over the fingers if the fingers feel cold. *5. Contact the health care provider if the child complains of numbness or tingling in the extremity.* *6. Elevate the extremity on pillows for the first 24 to 48 hours after casting to prevent swelling.* *Rationale:* While the cast is drying, the palms of the hands are used to lift the cast. If the fingertips are used, indentations in the cast could occur and cause constant pressure on the underlying skin. Small toys and sharp objects are kept away from the cast, and no objects (including padded objects) are placed inside of the cast because of the risk of altered skin integrity. A heating pad is not applied to the cast or fingers. Cold fingers could indicate neurovascular impairment, and the HCP should be notified. The extremity is elevated to prevent swelling, and the HCP is notified immediately if any signs of neurovascular impairment develop.

A nursing student is preparing to conduct a clinical conference, and the topic is hepatitis in children. The nursing instructor advises the student to further research the topic if the student plans to include which of the following in the discussion?

1. Vaccines are available to prevent hepatitis A (HAV) and hepatitis B (HBV). 2. Cases of hepatitis should be promptly reported to health care officials. *3. Enteric precautions are necessary for HBV but not for HAV.* 4. The child's stools will be pale and clay-colored. *Rationale:* Prevention of the spread of infection is an essential intervention for HAV. This should include enteric precautions for at least 1 week after the onset of jaundice and strict handwashing. Options 1, 2, and 4 are accurate regarding hepatitis.

The nurse is caring for a pediatric client in skin traction. To prevent skin breakdown, the best nursing intervention for this child is to:

1. Vigorously massage bony prominences every 4 hours. 2. Replace the elastic bandage on skin traction every 8 hours. *3. Stimulate circulation with gentle massage over pressure areas.* 4. Change the child's position at least every 4 hours to relieve pressure. *Rationale:* Nonadhesive straps and/or elastic bandage on skin traction are replaced when permitted and/or when absolutely necessary. Circulation should be stimulated with gentle, not vigorous, massage over pressure areas. The child's position should be changed at least every 2 hours to relieve pressure.

A 2-year-old child is admitted to the hospital with a diagnosis of nephrotic syndrome. In planning care for this child, which of the following nursing interventions would be of highest priority?

1. Weigh morning and afternoon. 2. Maintain a strict intake and output. *3. Dipstick the urine for protein every 4 hours.* 4. Take vital signs with blood pressure every 4 hours. *Rationale:* Continuous monitoring of fluid retention and excretion is an important nursing intervention in the care of the child with nephrotic syndrome. Although it is important to maintain a strict intake and output in monitoring fluid retention and excretion, the goal of treatment with this child is to decrease the amount of protein lost in the urine. Because this is the goal, option 3 has the highest priority. Although weight is monitored, it is not necessary to check the weight morning and evening. Taking vital signs with blood pressure is important but is not the priority in this situation.

A health care provider prescribes intravenous potassium for a child with hypertonic dehydration. The nurse assigned to assist in caring for the child would check which highest-priority item before administration of the potassium?

1. Weight *2. Urine output* 3. Temperature 4. Blood pressure *Rationale:* The priority assessment would be to check the status of urine output. Potassium should never be administered in the presence of oliguria or anuria. If urine output is less than 1 to 2 mL/kg/hr, it should not be administered. Although options 1, 3, and 4 may be a component of the data collected, they are not specifically related to the administration of this medication.

A health care provider has prescribed oxygen as needed for a 10-year-old child with congestive heart failure (CHF). In which situation would the nurse administer the oxygen to the child?

1. When the child is sleeping 2. When changing the child's diapers 3. When the mother is holding the child *4. When drawing blood for the measurement of electrolyte levels* *Rationale:* Oxygen administration may be prescribed for the infant with CHF for stressful periods, especially during bouts of crying or invasive procedures. Drawing blood is an invasive procedure that would likely cause the child to cry.

A nurse is caring for a hospitalized infant with a diagnosis of bronchiolitis. The nurse positions the infant:

1. With the head at a 60-degree angle with the neck slightly flexed 2. In a supine, side-lying position *3. With the head and chest at a 30-degree angle, with the neck slightly extended* 4. Prone, with the head of the bed elevated 15 degrees *Rationale:* The nurse should position the infant with the head and the chest at a 30- to 40-degree angle with the neck slightly extended to maintain an open airway and to decrease pressure on the diaphragm. Options 1, 2, and 4 do not achieve these goals.

With the administration of oxygen, a preterm infant's Pa02 level is monitored carefully. It is important to keep this level under which value to help prevent retinopathy of prematurity? a) 180 mm Hg b) 50 mm Hg c) 100 mm Hg d) 40 mm Hg

100 mm Hg Explanation: Retinal capillaries can be damaged by excessive oxygen levels. Keeping the Pa02 level under 100 mm Hg helps prevent this.

A 6 month old child developed kernicterus immediatedly after birth. Which of the following test should be done to determine whether or not this child has developed andy sequelae to the illness? 1. Blood urea nitrogen and serum creatinine 2. Alkaline phosphatase and bilirubin 3. Hearing testing and vision assessment 4. Peak expiratory flow and blood gass assessments

3. Because the central nervous system may have been damaged by the high bilirubin levels, testing of the senses as well as motor and cognitive assessment are aappropriate

The nurse is reviewing the results of a clotting study for a healthy 6-year-old. Which of the following would the nurse document as a normal prothrombin finding? a) 16.0 to 18.0 seconds b) 6.0 to 9.0 seconds c) 11.0 to 13.0 seconds d) 21.0 to 35.0 seconds

11.0 to 13.0 seconds Explanation: The nurse would identify a prothrombin time of 11.0 to 13.0 seconds as normal for a healthy child. A result of 21.0 to 35.0 seconds would be the expected range for partial thromboplastin time and activated partial thromboplastin time. Findings of 6.0 to 9.0 seconds and 16.0 to 18.0 seconds are outside the normal range.

A 1 day old neonate, 32 weeks gestation, is in an overhead warmer. The nurse assesses the morning axillary temperature as 96.9 F. Which of the following could explain this assessment finding? 1. This is a normal temperature for a preterm neonate 2. Axillary temperatures are not valid for preterm babies 3. The supply of brown adipose tissue is incomplete 4. Conduction heat loss is pronounced in the baby

3. Preterm babies are born with an insufficient supply of brown adipose tissue that is needed for thermogenesis, or heat generation

A baby, born at 3,199 grams, now weighs 2,746 grams. The baby is being monitored for dehydration because of the following percent weight loss? (calculate to the nearest hundreth) ____________________________%

14.16% The formula for percentage of weight loss is : Original weight minus current weight divided by original weight. The value is then multiplies by 100 to convert the number to percentage: 3199-2746=453 453/3199=0.1416*100=14.16%

The nurse is caring for a child in sickle cell crisis. To best promote hemodilution, the nurse would expect to administer how much fluid per day intravenously or orally? a) 110 mL/kg of fluids b) 120 mL/kg of fluids per day c) 150 mL/kg of fluids d) 130 mL/kg of fluids per day

150 mL/kg of fluids Explanation: To promote hemodilution in sickle cell crisis, the nurse would provide 150 mL/kg of fluids per day or as much as double maintenance, either orally or intravenously

The nurse is caring for an infant with congenital cardiac defect is monitoring the child for which of the following early signs of congestive heart failure? (Select all that apply) 1. Palpitation 2. Tachypnea 3. Tachcardia 4. Diaphoresis 5. Irritability

2,3,4 2. No matter whether a baby or an adult were developing CHF, the patient would be tachpneic 3. No matter whether a baby or an adult were developing CHF, the patient would be tachycardic 4. No matter whether a baby or an adult were developing CHF, the patient would be diaphoretic

A baby is suspected of having esophageal atresia. The nurse would expect to see which of the following signs/symptoms? ( Select all that apply) 1. Frequent vomiting 2. Excessive mucus 3. Ruddy complexion 4. Abdominal distention 5. Pigeon chest

2,4 2. Babies with esophageal atresia would be expected to expel large amounts of mucus from the mouth 4. Abdominal distention can be seen with esophageal atresia as air enters the stomach via the trachea

An 18 hour old baby is placed under the bili lights with an elevated bilirubin level. Which of the following is an expected nursing action in these circumstances? 1. Give the baby oral rehydration therapy after all feedings 2. Rotate the baby from side to back to side to front every two hours 3. Apply restraints to keep the baby under the light source 4. Administer intravenous fluids via pump per doctor orders

2. Rotating the baby's position maximizes the therapeutic response because the more skin surface that is exposed to the light source, the better the results are

A nurse makes the following obsevations when admitting a full term, breastfeeding baby, into the neonatal nursery: 9lb 2 oz, 21 inches long, TPR: 96.6 F, 158,62, jittery, pink body with bluish hands and feet, crying. Which of the following actions is of highest probability? 1. Swaddle the baby to provide warmth 2. Assess the glucose level of the baby 3. Take the baby to the mother for feeding 4. Administer the neonatal medications

2. The glucose level should be assessed to determine whether or not this baby is hypoglycemic

The nurse is caring for a client in labor and delivery with the following history: G2 P1000, 39 weeks gestation in transition phase, FH 135 with the early decelerations. The client states, I'm so scared. Please make sure the baby is OK!" Which of the following responses by the nurse is appropriate? 1. There is absolutely nothing to worry about 2. The fetal heart rate is within normal limits 3. How did your first baby die 4. Was your first baby preterm

2. This is the best reponse for the nurse to make. The nurse is providing the client with accurate, reassuring information without guaranteeing that there wil definitely be a positive outcome

A woman is admitted to the labor and delivery unit with active TB. She has not been under a physician's care and is not on medication. Which of the following actions should the nursery nurse perform when the neonate is delivered? 1. Isolate the baby from the other babies is a special care nursery 2. Keep the baby in the regular care nursery but separated from the mother 3. Isolate the baby with the mother in the mother's room 4. Obtain an order from the doctor for antituberculosis medications for the baby

2. This response is accurate. The baby can be cared for in the well baby nursery, but must be kept separated from its mother

Four babies in the well baby nursery were born with congenital defects. Which of the babies' complications developed as a result of the delivery method? 1. Clubfoot 2. Brachial palsy 3. Gastroschisis 4. Hydrocele

2. Brachial palsy can result from either a traumatic vertex or breech delivery

The mother of a child with Down syndrome expresses concerns about her child's physical health. Which response by the nurse would be the most appropriate? 1.) "Children with Down syndrome are prone to upper respiratory infections." 2.) "Congenital heart defects are uncommon in children with Down syndrome." 3.) "Your child will probably need to be institutionalized before the age of 18." 4.) "Most children with Down syndrome develop leukemia."

2.) "Congenital heart defects are uncommon in children with Down syndrome." Pg. 1056 -Foundations of Nursing (#1 also a possible answer. Refer to the Foundations of Nursing book Ch. 31 page 1056)

The is assisting a patient who just delivered a healthy baby boy weighing 7 pounds. Upon cord traction of placenta, she notices a sudden gushing of a large amount of blood and the fundus is no longer palpable in the abdomen. What are useful nursing interventions if uterine inversion is suspected? 1. Administering oxytocic 2. Assess vital signs 3. Discontinue uterotonic drugs 4. Do not attempt to remove the placenta 5. Establish IV access and fluids

2. Assess vital signs 3. Discontinue uterotonic drugs 4. Do not attempt to remove the placenta 5. Establish IV access and fluids Rationale: Never attempt to remove the placenta if it is still attached, because this will only create a larger surface area for bleeding. When an inversion occurs a large amount of blood suddenly gushes from the vagina. The fundus is not palpable in the abdomen. If the loss of blood continues unchecked, the woman will immediately show signs of blood loss. Uterine inversion may occur after the birth if traction is applied to the umbilical cord too soon or if the pressure is applied to the uterine fundus when the uterus is not contracted. Administering an oxytocic drug only compounds the inversion. Uterotonic drugs should be discontinued to allow uterine relaxation for replacement. IV fluids should be commenced to support blood pressure.

Which is the normal lochial appearance in the first 24 hours after birth? 1. Large clots or tissue 2. Bright red blood 3. A foul odor 4. Absence of lochia

2. Bright red blood Rationale: Normal lochia in the first 24 hours after birth consists of bright red blood.

A woman is in the 16th week of pregnancy, and she asks the nurse why she is to be given RhoGAM. Which is the best response by the nurse? 1. It prevents your blood from mixing with the blood of baby. 2. It prevents your body from producing antibodies to the baby's blood type. 3. It prevents the body from developing jaundice because of blood incompatibility. 4. It binds to your blood.

2. It prevents your body from producing antibodies to the baby's blood type. Rationale: RhoGAM prevents the mother from forming antibodies to the fetus's Rh+ blood. Sensitization may occur during pregnancy, birth, abortion, and amniocentesis. Iso- immunization occurs when a Rh negative mother becomes pregnant with a Rh-positive infant, resulting in Rh incompatibility. If mother forms antibodies against Rh, the antibodies could cause hemolysis and jaundice in the next fetus. Jaundice within the first 24 hours of life is a sign of hemolytic disease of the newborn. Physiologic jaundice normally occurs after 24 hours.

A patient has meconium-stained amniotic fluid. Fetal scalp sampling indicates a blood pH of 7.12 and fetal bradycardia is present. Based on these findings, the nurse should take which action? 1. Administer amnioinfusion. 2. Prepare for cesarean section. 3. Reposition the patient. 4. Start IV as prescribed.

2. Prepare for cesarean section. Rationale: Infants with meconium-stained amniotic fluid may have respiratory difficulties and bradycardia at birth. Based on this assessment, fetal metabolic acidosis is present. These findings pose a great threat to the newborn's well-being. A cesarean section is required. Amnioinfusion is an infusion of sterile isotonic solution into the uterine cavity during labor to reduce umbilical cord compression. This is also done to dilute meconium in the amniotic fluid, reducing the risk that the infant will aspirate thick meconium at birth. The procedure is not sufficient in this scenario neither is the IV.

Which behavior indicates that the delivery is imminent and that the physician should be summoned immediately? 1. Decreased duration of contractions 2. Sense of rectal pressure 3. Increase in fetal heart rate 4. Episodes of nausea and vomiting

2. Sense of rectal pressure Rationale: Rectal pressure indicates that the patient is moving into the second stage of labor.

A baby has been admitted to the neonatal nursery whose mother is hepatitis B surface antigen positive. Which of the following actions by the nurse should be taken at this time? 1 Monitor the baby for signs of hepatitis B 2. Place the baby on contact isolation 3. Obtain an order for the hepatitis B vaccine and the immune globulin 4. Advise the mother that breast feeding is absolutely contraindicated

3. Babies exposed to hepatitis B in utero should recieve the first dose of hepatitis B vaccine as well as hepatitis B immune globulin (HBIG) within 12 hours of delivery to reduce transmission of the virus

A 42 week gestational client is receiving an intravenous infusion of oxytocin(Pitocin) to augment early labor. The nurse should discontinue the oxytocin infusion for with pattern of contractions? A. transition labor with contractions every 2 mins, lasting 90 seconds each. B. early labor with contractions every 5 min, lasting 40 seconds each. C. Active labor with contractions every 31 mins, lasting 60 seconds each. D. Active labor with contraction every 2 to 3 mins, lasting 70 to 80 seconds each.

A. transition labor with contractions every 2 mins, lasting 90 seconds each.

The nurse is caring for a client, 37 weeks gestaton, who was just told that she is group B strep positive. The client states, "How could that happen? I only have sex with my husband. Will my baby be OK?" Based on this information, which of the following should the nurse communicate to the client? 1. The client's partner must have acquired the bacteria during a sexual encounter 2. The bacteria do not injure babies, but they could cause the client to have a bad sore throat 3. The client is high risk for developing pelvic inflammatory disease from the bacteria 4. Antibiotics will be administered during labor to prevent vertical transmission of the bacteria

4. This statement is accurate. Antibiotics will be administered to the mother during labor and delivery to prevent verticle transmission.

A newborn admitted to the nursery has a positive direct Coombs' test. Which of the following is an appropriate action by the nurse? 1. Monitor the baby for jitters 2. Assess the blood glucose level 3. Assess the rectal temperature 4. Monitor the baby for jaundice

4. When the neonatal blood stream contains antibodies, hemolysis of the red blood cells occurs and jaundice develops

A 28-year-old client in active labor complains of cramps in her leg. What intervention should the nurse implement? A. Massage the calf and foot. B. Extend the leg and dorsiflex the foot C. Lower the leg off the side of the bed. D. Elevate the leg above the heart.

B. Extend the leg and dorsiflex the foot

The nurse is aware that visual acuity evaluation in a child is best assessed after the age of _____ years

6 A child's refraction does not reach 20/20 until about the age of 6. p. 942

28 year old client in active labor complains of cramps in her leg.What intervention should the nurse implement. A. massage the calf and foot B. extend the leg and dorsiflex the foot C. lower the leg off the side of the bed D. elevate the leg above the heart.

B. Extend the leg and dorsiflex the foot.

The nurse is preparing to give an enema to a laboring client. Which client requires the most caution when carrying out this procedure?

A 40-week primigravida who is at 6 cm cervical dilatation and the presenting part is not engaged. When the presenting part is ballottable (D), it is floating out of the pelvis. In such a situation, the cord can descend before the fetus causing a prolapsed cord, which is an emergency situation.

In evaluating the respiratory effort of a one-hour-old infant using the Silverman-Anderson Index, the nurse determines the infant has synchronized chest and abdominal movement, just visible lower chest retractions, just visible xiphoid retractions, minimal and transient nasal flaring, and an expiratory grunt heard only on auscultation. What Silverman-Anderson score should the nurse assign to this infant? (Enter numeral value only.)

A Silverman-Anderson Index has five categories with scores of 0, 1, or 2. The total score ranges from 0 to 10. Four of the these assessment findings should receive a score of 1, and the 5th finding (synchronized chest and abdominal movement) receives a score of 0. Therefore, the total score is 4. A total score of 0 means the infant has no dyspnea, a total score of 10 indicates maximum respiratory distress.

What prophylactic measures are used for a child w/ ITP after a splenectomy?

A child receives penicillin prophylactically after splenectomy. The length of the prophylactic therapy is controversial but the usual recommendation is for a minimum of 3yrs

A woman who thinks she could be pregnant calls her neighbor, a nurse, to ask when she could use a home pregnancy test to diagnose pregnancy. Which response is best?

A home pregnancy test can be used right after your first missed period.

While assessing a full-term neonate, which symptom would cause the nurse to suspect a neurologic impairment? a) A positive Babinski's reflex b) A positive rooting reflex c) A weak sucking reflex d) Startle reflex in response to a loud noise

A weak sucking reflex Correct Explanation: Normal neonates have a strong, vigorous sucking reflex. The rooting reflex is present at birth and disappears when the infant is between ages 3 and 4 months. A positive Babinski's reflex is present at birth and disappears by the time the infant is age 2. The startle reflex is present at birth and disappears when the infant is approximately age 4 months.

A maternity nurse is providing instruction to a new mother regarding the psychosocial development of the newborn infant. Using Erikson's psychosocial development theory, the nurse would instruct the mother to A) Allow the newborn infant to signal a need B) Anticipate all of the needs of the newborn infant C) Avoid the newborn infant during the first 10 minutes of crying D) Attend to the newborn infant immediately when crying

A) Allow the newborn infant to signal a need. Trust vs Mistrust stage-This will allow the infant opportunity to gain trust.

A pregnant woman reports that she is still playing tennis at 32 weeks of gestation. The nurse would be most concerned regarding what this woman consumes during and after tennis matches. Which is the MOST important? A. Several glasses of fluid B. Extra protein sources, such as peanut butter C. Salty foods to replace lost sodium D. Easily digested sources of carbohydrate

A. Several glasses of fluid If no medical or obstetric problems contraindicate physical activity, pregnant women should get 30 minutes of moderate physical exercise daily. Liberal amounts of fluid should be consumed before, during, and after exercise, because dehydration can trigger premature labor. Also the woman's calorie intake should be sufficient to meet the increased needs of pregnancy and the demands of exercise. All pregnant women should consume the necessary amount of protein in their diet, regardless of level of activity. Many pregnant women of this gestation tend to retain fluid. This may contribute to hypertension and swelling. An adequate fluid intake prior to and after exercise should be sufficient. The woman's calorie and carbohydrate intake should be sufficient to meet the increased needs of pregnancy and the demands of exercise

A multigravida client arrives at the L&D unit and tells the nurse that her bag of water has broken. The nurse identifies the presence of meconium fluid on the perineum and determines the fetal HR is between 140 and 150 beats/min. What action should the nurse implement next? A. complete sterile vag exam B. take maternal temp every 2 hrs C. Prepare for an immediate cesarean bitrh D. Obtain sterile suction equipment

A. complete sterile vag exam

A healthcare provider informs the charge nurse of a labor and delivery unit that a client is coming to the unit suspected abruptio placentae. What findings should the charge nurse expect the client to demonstrate. A. dark,red vaginal bleeding B. lower back pain C. premature rupture of membranes D. increased uterine irritability E. bilateral pitting edema F. Rigid abdomen

A. dark,red vaginal bleeding D. increased uterine irritability F. Rigid abdomen

The healthcare provider prescribes terbutaline for a client in preterm labor. Before initating this prescription, it is most important for the nurse to assess the client for which of condition. A. gestational diabetes B. Elevated BP C. UTI D> Swelling in lower extremities

A. gestational diabetes

When planning a diet with a pregnant woman, the nurse's FIRST action would be to: A. review the woman's current dietary intake. B. teach the woman about the food pyramid. C. caution the woman to avoid large doses of vitamins, especially those that are fat-soluble. D. instruct the woman to limit the intake of fatty foods.

A. review the woman's current dietary intake. Reviewing the woman's dietary intake as the first step will help to establish if she has a balanced diet or if changes in the diet are required. These are correct actions on the part of the nurse, but the first action should be to assess the patient's current dietary pattern and practices since instruction should be geared to what she already knows and does.

An adolescent boy tells the nurse that he has recently had homosexual feelings. The nurse's response should be based on knowledge that: a. This indicates that the adolescent is homosexual. b. This indicates that the adolescent will become homosexual as an adult. c. The adolescent should be referred for psychotherapy. d. The adolescent should be encouraged to share his feelings and experiences.

ANS: D These adolescents are at increased risk for health-damaging behaviors, not because of the sexual behavior itself, but because of society's reaction to the behavior. The nurse's first priority is to give the young man permission to discuss his feelings about this topic, knowing that the nurse will maintain confidentially, appreciate his feelings, and remain sensitive to his need to talk it. In recent studies among self-identified gay, lesbian, and bisexual adolescents, many of the adolescents report changing their self-labels one or more times during their adolescence.

The school nurse has been asked to begin teaching sex education in the 5th grade. The nurse should recognize that: a. Children in 5th grade are too young for sex education. b. Children should be discouraged from asking too many questions. c. Correct terminology should be reserved for children who are older. d. Sex can be presented as a normal part of growth and development.

ANS: D When sex information is presented to school-age children, sex should be treated as a normal part of growth and development. Fifth graders are usually 10 to 11 years old. This age is not too young to speak about physiologic changes in their bodies. They should be encouraged to ask questions. Preadolescents need precise and concrete information.

Which of the following is a typical feature of a small for gestational age (SGA) newborn that differentiates it from a preterm baby with a low birth weight? a) Decreased muscle mass b) Face is angular and pinched c) Decreased body temperature d) Ability to tolerate early oral feeding

Ability to tolerate early oral feeding Explanation: Unlike preterm babies with low birth weights, a small for gestational age baby can safely tolerate early oral feeding. It usually has a coordinated sucking and swallowing reflex. Decreased muscle mass, decreased body temperature and an angular and pinched face are features common to both an SGA and a preterm baby.

Acquired Heart Disease (AHD)

Abnormalities that occur after birth that compromise the hearts function

A newborn with high serum bilirubin is receiving phototherapy. Which of the following is the most appropriate nursing intervention for this client? a) Gentle shaking of the baby b) Placing light 6 inches above the newborn's bassinet c) Application of eye dressings to the infant d) Delay of feeding until bilirubin levels are normal

Application of eye dressings to the infant Correct Explanation: Continuous exposure to bright lights by phototherapy may be harmful to a newborn's retina, so the infant's eyes must always be covered while under bilirubin lights. Eye dressings or cotton balls can be firmly secured in place by an infant mask. The lights are placed 12 to 30 inches above the newborn's bassinet or incubator. Bilirubin is removed from the body by being incorporated into feces. Therefore, the sooner bowel elimination begins, the sooner bilirubin removal begins. Early feeding (either breast milk or formula), therefore, stimulates bowel peristalsis and helps to accomplish this. Gently shaking the infant is a method of stimulating breathing in an infant experiencing apnea.

S/Sx of Tetralogy of Fallot

Cyanosis and hypoxia (blue spells), systolic ejection murmur, clubbing of the nail beds, dyspnea, squatting, mental slowness, poor growth, syncope (fainting), stroke

A pregnant woman with a body mass index (BMI) of 22 asks the nurse how she should be gaining weight during pregnancy. The nurse's BEST response would be to tell the woman that her pattern of weight gain should be approximately: A. a pound a week throughout pregnancy. B. 2 to 5 lbs during the first trimester, then a pound each week until the end of pregnancy. Correct C. a pound a week during the first two trimesters, then 2 lbs per week during the third trimester. D. a total of 25 to 35 lbs.

B. 2 to 5 lbs during the first trimester, then a pound each week until the end of pregnancy. A pound a week is not the correct guideline during pregnancy. A BMI of 22 represents a normal weight. Therefore, a total weight gain for pregnancy would be about 25 to 35 lbs or about 2 to 5 lbs in the first trimester and about 1 lb/wk during the second and third trimesters. These are not accurate guidelines for weight gain during pregnancy. The total is correct, but the pattern needs to be explained.

The nurse caring for a laboring client encourages her to void at least q2h, and records each time the client empties her bladder. What is the primary reason for implementing this nursing intervention? A. Emptying the bladder during delivery is difficult because of the position of the presenting fetal part. B. An over-distended bladder could be traumatized during labor as well as prolong the progress of labor. C.Urine specimens for glucose and protein must be obtained at certain intervals throughout labor. D. Frequent voiding minimizes the need for catheterization which increases the chance of bladder infection.

B. An over-distended bladder could be traumatized during labor as well as prolong the progress of labor.

A clinic nurse assesses the communication patterns of a 5-month-old infant. The nurse determines that the infant is demonstrating the highest level of developmental achievement expected if the infant: A) Uses simple words such as "mama" B) Uses monosyllabic babbling C) Links syllables together D) Coos when comforted

B. Using monosyllabic babbling occurs between 3 and 6 months of age. Using simple words such as "mama" occurs between 9 and 12 months. Linking syllables together when communicating occurs between 6 and 9 months. Cooing begins at birth and continues until 2 months.

With regard to nutritional needs during lactation, a maternity nurse should be aware that: A. the mother's intake of vitamin C, zinc, and protein now can be lower than during pregnancy. B. caffeine consumed by the mother accumulates in the infant, who therefore may be unusually active and wakeful. C. critical iron and folic acid levels must be maintained. D. lactating women can go back to their prepregnant calorie intake.

B. caffeine consumed by the mother accumulates in the infant, who therefore may be unusually active and wakeful. Vitamin C, zinc, and protein levels need to be moderately higher during lactation than during pregnancy. A lactating woman needs to avoid consuming too much caffeine. The recommendations for iron and folic acid are somewhat lower during lactation. Lactating women should consume about 500 kcal more than their prepregnancy intake, at least 1800 kcal daily overall.

A pregnant woman at 7 weeks of gestation complains to her nurse midwife about frequent episodes of nausea during the day with occasional vomiting. She asks what she can do to feel better. The nurse midwife could suggest that the woman: A. drink warm fluids with each of her meals. B. eat a high-protein snack before going to bed. Correct C. keep crackers and peanut butter at her bedside to eat in the morning before getting out of bed. D. schedule three meals and one midafternoon snack a day

B. eat a high-protein snack before going to bed. Fluids should be taken between (not with) meals to provide for maximum nutrient uptake in the small intestine. A bedtime snack of slowly digested protein is especially important to prevent the occurrence of hypoglycemia during the night that would contribute to nausea. Dry carbohydrates such as plain toast or crackers are recommended before getting out of bed. Eating small, frequent meals (about five or six each day) with snacks helps to avoid a distended or empty stomach, both of which contribute to the development of nausea and vomiting.

The nurse is caring for an 18-month-old with suspected iron deficiency anemia. Which of the following lab results confirms the diagnosis? a) Increased serum iron and ferritin levels: decreased FEP level, microcytosis and hypochromia b) Decreased hemoglobin and hematocrit, decreased reticulocyte count, microcytosis, and hypochromia, decreased serum iron and ferritin levels and increase FEP level c) Increased hemoglobin and hematocrit, increased reticulocyte count, microcytosis, and hypochromia d) Increased hemoglobin and hematocrit, increased reticulocyte, microcytosis and hypochromia, increased serum iron and ferritin levels and decreased FEP level

Decreased hemoglobin and hematocrit, decreased reticulocyte count, microcytosis, and hypochromia, decreased serum iron and ferritin levels and increase FEP level Explanation: Laboratory evaluation will reveal decreased hemoglobin and hematocrit, decreased reticulocyte count, microcytosis and hypochromia, decreased serum iron and ferritin levels, and increase FEP level. The other findings do not point to iron deficiency anemia.

A nurse assisting in a birth notices that the amniotic fluid is stained greenish black as the baby is being delivered. Which of the following interventions should the nurse implement as a result of this finding? a) Administration of oxygen via a bag and mask b) Intubation and suctioning of the trachea c) Gently shaking the infant d) Flicking the sole of the infant's foot

Intubation and suctioning of the trachea Correct Explanation: Although there is some dispute regarding whether all infants with meconium staining need intubation, those with severe staining are usually intubated and meconium is suctioned from their trachea and bronchi. Do not administer oxygen under pressure (bag and mask) until a meconium stained infant has been intubated and suctioned, so the pressure of the oxygen does not drive small plugs of meconium farther down into the lungs, worsening the irritation and obstruction. Gently shaking the infant and flicking the sole of his foot are methods of stimulating breathing in an infant experiencing apnea.

A client who is attending antepartum classes asks the nurse why her healthcare provider has prescribed iron tablets. The nurse's response is based on what knowledge?

It is difficult to consume 18 mg of additional iron by diet alone.

The nurse should explain to a 30-year-old gravid client that alpha fetoprotein testing is recommended for which purpose?

Screen for neural tube defects

Twenty-four hours after admission to the newborn nursery, a full-term male infant develops localized edema on the right side of his head. The nurse knows that, in the newborn, an accumulation of blood between the periosteum and skull which does not cross the suture line is a newborn variation known as

a cephalhematoma, caused by forceps trauma and may last up to 8 weeks. Cephalhematoma (A), a slight abnormal variation of the newborn, usually arises within the first 24 hours after delivery. Trauma from delivery causes capillary bleeding between the periosteum and the skull

A 14-year-old girl who is a vegetarian has recently developed anemia. Blood smear results show large, fragile, immature erythrocytes. She claims to take an iron supplement regularly and is surprised to learn that she is anemic, as she is otherwise healthy. The nurse recognizes that which of the following is the likely cause of this type of anemia? a) Sickle-cell disorder b) Vitamin B12 deficiency c) Iron deficiency d) Acute blood loss

Vitamin B12 deficiency Explanation: Vitamin B12 is necessary for the maturation of RBCs. Pernicious anemia results from deficiency or inability to use the vitamin, resulting in RBCs that appear abnormally large and are immature megaloblasts (nucleated immature red cells). Thus, pernicious anemia is one of the megaloblastic anemias. In children, the cause is more often lack of ingestion of vitamin B12 rather than poor absorption. Adolescents may be deficient in vitamin B12 if they are ingesting a long-term, poorly formulated vegetarian diet as the vitamin is found primarily in foods of animal origin

A woman who has given birth to a postterm newborn asks the nurse why her baby looks so thin, with so little muscle. The nurse responds based on the understanding about which of the following? a) The newborn was exposed to an infection while in utero. b) The newborn aspirated meconium, causing the wasted appearance. c) A postterm newborn has begun to break down red blood cells more quickly. d) With postterm birth, the fetus uses stored nutrients to stay alive, and wasting occurs.

With postterm birth, the fetus uses stored nutrients to stay alive, and wasting occurs. Explanation: After 42 weeks' gestation, the placenta loses its ability to provide adequate oxygen and nutrients to the fetus, causing the fetus to use stored nutrients to stay alive. This leads to wasting. Meconium aspiration can occur with postterm newborns, but this is not the reason for the baby's wasted appearance. Hyperbilirubinemia occurs with the increased breakdown of red blood cells, but this too would not account for the wasted appearance. Exposure to an intrauterine infection is unrelated to the wasted appearance.

14. After you complete your nutritional counseling for a pregnant woman, you ask her to repeat your instructions so you can assess her understanding of the instructions given. Which statement indicates that she understands the role of protein in her pregnancy? a. "Protein will help my baby grow." b. "Eating protein will prevent me from becoming anemic." c. "Eating protein will make my baby have strong teeth after he is born." d. "Eating protein will prevent me from being diabetic."

a. "Protein will help my baby grow."

19. Which nutritional recommendation about fluids is accurate? a. A woman's daily intake should be eight to ten glasses (2.3 L) of water, milk, or juice. b. Coffee should be limited to no more than two cups, but tea and cocoa can be consumed without worry. c. Of the artificial sweeteners, only aspartame has not been associated with any maternity health concerns. d. Water with fluoride is especially encouraged because it reduces the child's risk of tooth decay.

a. A woman's daily intake should be eight to ten glasses (2.3 L) of water, milk, or juice.

The nurse is teaching a woman how to use her basal body temperature (BBT) pattern as a tool to assist her in conceiving a child. Which temperature pattern indicates the occurrence of ovulation, and therefor, the best time for intercourse to ensure conception? a. Between the time the temperature falls and rises b. Between 36 and 48 hours after the temperature rises c. When the temperature falls and remains low for 36 hours d. Within 72 hours before the temperature falls

a. Between the time the temperature falls and rises In most women, the BBT drops slightly 24 to 36 hours before ovulation and rises 24 to 72 hours after ovulation, when the corpus luteum of the ruptured ovary produces progesterone. Therefore, intercourse between the time of the temperature fall and rise (A) is the best time for conception.

A multigravida client arrives at the labor and delivery unit and tells the nurse that her bag of water has broken. The nurse identifies the presence of meconium fluid on the perineum and determines the fetal heart rate is between 140 to 150 beats/minute. What action should the nurse implement next? a. Ccmplete a sterile vaginal exam b. Take maternal temperature every 2 hours c. Prepare for an immediate cesarean birth d. Obtain sterile suction equipment

a. Complete a sterile vaginal exam A vaginal exam (A) should be performed after the rupture of membranes to determine the presence of a prolapsed cord.

The nurse is teaching the parents of a 4-year-old girl with thalassemia about sound nutritional choices. The nurse asks the mother about good snack choices to send to preschool. Which response by the mother would indicate a need for further teaching? a) "She likes string cheese and saltine crackers." b) "She can bring graham crackers and peanut butter." c) "I can send apple slices with yogurt dip." d) "Yogurt and granola is a good choice."

b. "She can bring graham crackers and peanut butter." Explanation: Children with thalassemia should avoid foods that are high in iron. Peanut butter is high in iron and should be avoided. Yogurt, granola, string cheese, saltine crackers, and apples are appropriate choices.

An 8-year-old child asks how a blood pressure is taken. What would be the most appropriate response? a. "This small machine will measure your systolic and diastolic pressure." b. "The armband will hug your arm and tell me how well your blood is going through your arm." c. "The armband will cut off your circulation for a while and then we can hear when it comes back." d. "When you are ill we need to know if your blood is still moving in your body."

b. "The armband will hug your arm and tell me how well your blood is going through your arm." Because children are upset by unfamiliar procedures, it is best to explain each step in simple terms. It is best not to mention anything that may increase anxiety. p. 941

Following a lumbar puncture of a 2-year-old, what should the nurse do? a. Keep the child flat for several hours b. Allow the child to play quietly at will c. Hold the child in a flexed position for 5 minutes d. Stand the child upright immediately

b. Allow the child to play quietly at will Children younger than 3 years of age are usually not affected by postlumbar headache. These children are allowed to play at will following a lumbar puncture. p. 962

27. Three servings of milk, yogurt, or cheese plus two servings of meat, poultry, or fish adequately supply the recommended amount of protein for a pregnant woman. Many patients are concerned about the increased levels of mercury in fish and may be afraid to include this source of nutrients in their diet. Sound advice by the nurse to assist the client in determining which fish is safe to consume would include: a. Canned white tuna is a preferred choice. b. Avoid shark, swordfish, and mackerel. c. Fish caught in local waterways are the safest. d. Salmon and shrimp contain high levels of mercury.

b. Avoid shark, swordfish, and mackerel.

The parents of a 6-year-old male with idiopathic thrombocytopenic purpura (ITP) ask the nurse conducting an assessment of the child what causes the disease. What is the nurse's best response? a) "ITP is a serious bleeding disorder characterized by a decreased, absent, or dysfunctional procoagulant factor." b) "ITP occurs when the body's iron stores are depleted due to rapid physical growth, inadequate iron intake, inadequate iron absorption, or loss of blood." c) "ITP is primarily an autoimmune disease in that the immune system attacks and destroys the body's own platelets, for an unknown reason." d) "ITP is characterized by the loss of surface area on the red blood cell membrane."

c. "ITP is primarily an autoimmune disease in that the immune system attacks and destroys the body's own platelets, for an unknown reason." Explanation: Idiopathic thrombocytopenic purpura (ITP) is primarily an autoimmune disease, which is an acquired, self-limiting disorder of hemostasis characterized by destruction and decreased numbers of circulating platelets. Hemophilia A and hemophilia B are distinguished by the particular procoagulant factor that is decreased, absent, or dysfunctional. Iron deficiency anemia occurs when the body's iron stores are depleted. Hereditary spherocytosis (HS) is characterized by loss of surface area on the red blood cell membrane

The nurse is teaching an inservice program to a group of nurses on the topic of children diagnosed with sickle cell anemia. The nurses in the group make the following statements. Which statement is most accurate regarding sickle cell anemia? a) "The disease is most often seen in individuals of Asian decent." b) "The trait or the disease is seen in one generation and skips the next generation." c) "If the trait is inherited from both parents the child will have the disease." d) "Males are much more likely to have the disease than females."

c. "If the trait is inherited from both parents the child will have the disease." Explanation: When the trait is inherited from both parents (homozygous state), the child has sickle cell disease, and anemia develops. The trait does not skip generations. The trait occurs most commonly in African Americans. Either sex can have the trait and disease

In discussing the causes of iron deficiency anemia in children with a group of nurses, the following statements are made. Which of these statements is a misconception related to iron deficiency anemia? a) "Caregivers sometimes don't understand the importance of iron and proper nutrition." b) "Children have a hard time getting enough iron from food during their first few years." c) "Milk is a perfect food, and babies should be able to have all the milk they want." d) "A family's economic problems are often a cause of malnutrition."

c. "Milk is a perfect food, and babies should be able to have all the milk they want." Explanation: Babies with an inordinate fondness for milk can take in an astonishing amount and, with their appetites satisfied, may show little interest in solid foods. These babies are prime candidates for iron deficiency anemia. Many children with iron deficiency anemia, however, are undernourished because of the family's economic problems. A caregiver knowledge deficit about nutrition is often present. Because only 10 percent of dietary iron is absorbed, a diet containing 8 to 10 mg of iron is needed for good health. During the first years of life, obtaining this quantity of iron from food is often difficult for a child. If the diet is inadequate, anemia quickly results

The nurse is preparing a child for discharge following a sickle cell crisis. The mother makes the following statements to the nurse. Which statement by the mother indicates a need for further teaching? a) "She loves popsicles, so I'll let her have them as a snack or for dessert." b) "I put her legs up on pillows when her knees start to hurt." c) "She has been down, but playing in soccer camp will cheer her up." d) "I bought the medication to give to her when she complains of pain."

c. "She has been down, but playing in soccer camp will cheer her up." Explanation: Following a sickle cell crisis the child should avoid extremely strenuous activities that may cause oxygen depletion. Fluids are encouraged, pain management will be needed, and the child's legs may be elevated to relieve discomfort, so these are all statements that indicate an understanding of caring for the child who has had a sickle cell crisis.

When the newly admitted 2-year-old who was potty-trained before admission begins to wet the bed, the mother is frightened. What statement by the nurse will be most helpful to the mother? a. "Don't be concerned. Accidents happen." b. "Let's put a diaper on your child until this gets better." c. "The stress of hospitalization makes children regress a little." d. "Your child will relearn 'potty-training' if you are patient."

c. "The stress of hospitalization makes children regress a little." It is not unusual for children to regress when hospitalized. Explaining that regression is normal during hospitalization will help allay the mother's anxiety. p.s 952-953

9. A 27-year-old pregnant woman had a preconceptual body mass index (BMI) of 18.0. The nurse knows that this woman's total recommended weight gain during pregnancy should be at least: a. 20 kg (44 lb). b. 16 kg (35 lb). c. 12.5 kg (27.5 lb). d. 10 kg (22 lb).

c. 12.5 kg (27.5 lb).

What should the nurse do to minimize an unpleasant-tasting drug? a. Pour the drug over ice b. Squirt the drug in the mouth with a syringe c. Administer the drug through a straw d. Enlist the parent's assistance

c. Administer the drug through a straw Administering the drug through a straw will diminish an unpleasant taste. Having the child hold the nose is helpful, as bad taste is associated with the smell of the drug. Pouring the drug over ice may result in the child not getting the entire amount of the drug. Squirting the drug into the mouth with a syringe will still allow the child to taste the medication. The parent's assistance should be enlisted, but will not minimize the taste of the drug. p. 966

What is the correct way to assess for the presence of jaundice in an African American child? a. Examine the sclera b. Press the edge of the pinna c. Apply pressure to the gum d. Compare the color on the soles of the feet

c. Apply pressure to the gum The gums in individuals with dark complexions can be used to assess jaundice by pressing the gums about the teeth. p. 941

Epogen

given to prevent or treat anemia, stimulates erythropoietin production, resulting in an increase in RBCs. Since the body has not had to compensate for anemia with an increased heart rate, changes in heart rate from high to normal

appropriate for gestational age (AGA) newborn

should fall between the measurement ranges of weight, 6-9 pounds (2700-4000 grams);; length, 19-21 inches (48-53 cm);; FOC, 13-14 inches (33-35 cm).

Terbutaline sulfate (Brethine), a beta-sympathomimetic drug,

stimulates beta- adrenergic receptors in the uterine muscle to stop contractions


Related study sets

8.3 Beyond the Mississippi: Louisiana Purchase and the Expedition of Lewis and Clark

View Set

Chapter 1: What is Plant Biology?

View Set

Chapter 12 Quiz, Chapter 13 Quiz, Chapter 14 Quiz, Chapter 15 Quiz, Chapter 16 Quiz, Chapter 17 Quiz, Chapter 18 Quiz, Part III Test

View Set

Principle of Accountant II - Final Exam

View Set

Amino Acids Structure, Name Function

View Set